0% found this document useful (0 votes)
327 views551 pages

Otvori Pediatric Surgical Diseases A Radiologic Surgical Case Study Approach

Uploaded by

slatka
Copyright
© © All Rights Reserved
We take content rights seriously. If you suspect this is your content, claim it here.
Available Formats
Download as PDF, TXT or read online on Scribd
0% found this document useful (0 votes)
327 views551 pages

Otvori Pediatric Surgical Diseases A Radiologic Surgical Case Study Approach

Uploaded by

slatka
Copyright
© © All Rights Reserved
We take content rights seriously. If you suspect this is your content, claim it here.
Available Formats
Download as PDF, TXT or read online on Scribd
You are on page 1/ 551

C. Esposito · G. Esposito (Eds.

)
Pediatric Surgical Diseases
A Radiologic Surgical Case Study Approach
C. Esposito · G. Esposito (Eds.)

C. T. Albanese · M. Fujioka · G. MacKinlay · N. Rollins


F. Schier (Ass. Eds.)

Pediatric
Surgical Diseases
A Radiologic Surgical Case
Study Approach

123
Ciro Esposito, MD, PhD Giovanni Esposito, MD, PhD
Associate Professor of Pediatric Surgery Emeritus Professor of Pediatric Surgery
Department of Pediatrics Federico II University of Naples
Chair of Pediatric Surgery School of Medicine
Federico II University of Naples Via S. Pansini 5
School of Medicine 80131 Naples
Via S. Pansini 5 Italy
80131 Naples
Italy

ISBN 978-3-540-71515-3 e-ISBN 978-3-540-71516-0

DOI 10.1007/978-3-540-71516-0

Library of Congress Control Number: 2008939686

© 2009 Springer-Verlag Berlin Heidelberg

This work is subject to copyright. All rights are reserved, wether the whole or part of the material is con-
cerned, specifically the rights of translation, reprinting, reuse of illustrations, recitation, broadcasting, re-
production on microfilm or any other way, and storage in data banks. Duplication of this publication or
parts thereof is permitted only under the provisions of the German Copyright Law of September 9, 1965,
in it current version, and permission for use must always be obtained from Springer. Violations are liable to
prosecution under the German Copyright Law.

The use of general descriptive names, registered names, trademarks, etc. in this publication does not imply,
even in the absence of a specific statement, that such names are exempt from the relevant protective laws and
regulations and therefore free for general use.

Product liability: the publishers cannot guarantee the accuracy of any information about dosage and applica-
tion contained in this book. In every individual case the user must check such information by consulting the
relevant literature.

Cover design: Frido Steinen-Broo, eStudio Calamar, Spain

Printed on acid-free paper

9 8 7 6 5 4 3 2 1

springer.com
To my mentors, Ettore Ruggieri, Giuseppe Zannini and François
D’Allaines, who showed me the way to be a part of the medical world.
With gratitude,
Giovanni Esposito

To my father, the best pediatric surgeon I have ever known,


and the man who instilled in me the thrill, challenge and satisfaction
of lifelong learning.
I thank him for his continuous teaching, and for transferring to me
his love for the children we care for every day.
With gratitude,
Ciro Esposito
Preface

The idea to write this book was born about 3 years ago eases, and we are extremely grateful to them for their
when we were changing the seat at the Unit of Pediatric contribution and for devoting their time to producing
Surgery of the University of Naples “Federico II.” such outstanding reviews.
We realized that during the 30-year activity of our Thus, with their precious help, we have created an ed-
unit we had collected a large number of rare and inter- ucational text, focusing on more than 200 case reports of
esting cases of pediatric surgical diseases. pediatric surgical diseases, which will certainly be very
The difficulties met in the diagnosis of many of these useful to all professionals working in pediatrics who
cases gave us the idea to publish some of the most im- need to prepare themselves when approaching a variety
portant ones, each with a collection of images accom- of diagnostic and therapeutic problems in conditions af-
panied by text providing a practical guide to reach the fecting infants and children.
final diagnosis. We would like to thank in particular our associate
Since our idea was welcomed by numerous col- editors, Craig T. Albanese, Masayuki Fujioka, Gordon
leagues and friends, we proposed to Springer to publish MacKinlay, Nancy Rollins, and Felix Schier, whose great
this book. competence and extensive experience in this field helped
All the authors who accepted to participate in our us in writing, collecting, and organizing this material.
project (pediatricians, pediatric surgeons, pediatric and
adult radiologists) are considered leading world experts Ciro Esposito, Giovanni Esposito
in the diagnosis and treatment of pediatric surgical dis- Editors
Contents

1 Head and Neck . . . . . . . . . . . . . .  5 Cardiovascular Disorders . . . . 

Case 1–13 Case 152–167

2 Thorax . . . . . . . . . . . . . . . . . . . . .  6 Musculoskeletal Disorder . . . . 

Case 14–44 Case 168–179

3 Abdomen . . . . . . . . . . . . . . . . . .  7 Emergency and Trauma . . . . . . 

Case 45–122 Case 180–221

4 Genitourinary Disorders . . . . .  8 Oncology . . . . . . . . . . . . . . . . . . 

Case 123–151 Case 222–249

Index . . . . . . . . . . . . . . . . . . . . . . . . 
Associated Editors

Craig T. Albanese, Prof. Gordon MacKinlay, Prof.


Chief of Pediatric Surgery Department of Pediatric Surgery
Department of Surgery University of Edinburgh
Stanford University Medical Center Scotland
Division of Pediatric Surgery UK
Lucile Packard Children’s Hospital and
780 Welch Road, Suite 206 Royal Hospital for Sick Children
Stanford, CA 94305-5733 9 Sciennes Road
USA Edinburgh
Scotland
Masayuki Fujioka, Prof. UK
Stroke Center
Neurology and Neurosurgery Nancy Rollins, Prof.
Helios General Hospital Aue Department of Pediatric Radiology
Dresden University of Technology Children’s Medical Center Dallas
Gartenstr. 6 University of Texas
08280 Aue Southwestern Medical Center Dallas
Germany and Holder of the Charles Cameron Sprague Chair
and in Medical Science
Kansai Medical University 1935 Motor Street
2-3-1 Shinmachi Dallas, TX 75235
Hirakata City USA
573-1191 Osaka
Japan Felix Schier, Prof.
Department of Pediatric Surgery
University Medical Center Mainz
Langenbeckstr. 1
551101 Mainz
Germany
List of Contributors

Yves Aigrain, Prof. Giuseppe Amici, Prof.


Department of Pediatric Surgery and Urology Chair of Pediatric Surgery
Hôpital Necker Enfants Malades University of Ancona
149 rue de Sèvres Via Corridoni 11
75015 Paris 60123 Ancona
France Italy

Craig T. Albanese, Prof. Giuseppe Ascione, Dr.


Chief of Pediatric Surgery Chair of Pediatric Surgery
Department of Surgery University of Naples “Federico II”
Stanford University Medical Center Via Pansini 5
Division of Pediatric Surgery 80131 Naples
Lucile Packard Children’s Hospital Italy
780 Welch Road, Suite 206
Stanford, CA 94305-5733 Richard A. Barth, Prof.
USA Chair of Radiology, Department of Radiology
Lucile Packard Children’s Hospital at Stanford
Francesca Alicchio, Dr. Stanford University School of Medicine
Chair of Pediatric Surgery 725 Welch Road
University of Naples “Federico II” Stanford, CA 94305-5654
Via Pansini 5 USA
80131 Naples
Italy François Becmeur, Prof.
Service de Chirurgie Infantile
Michele Ametrano, Dr. Hôpital de Hautepierre
Department of Pediatrics 67091 Strasbourg Cedex
University of Naples “Federico II” France
Via Pansini 5
80131 Naples Sophie Branchereau, Dr.
Italy Service de Chirurgie Pediatrique
Hôpital de Bicêtre
Brice Antao 78 Rue de General Leclerc
Department of Paediatric Surgery 94275 Le Kremlin Bicêtre Cedex
Leeds Teaching Hospitals NHS Trust France
Beckett Street
Leeds LS9 7TF Hervé Brisse, Dr.
UK Department of Radiology
Curie Institute
26 rue d’Ulm
75248 Paris Cedex 05
France
XIV List of Contributors

Luigi Califano, Prof. Alba Cruccetti, Dr.


Department of Maxillofacial Surgery Pediatric Surgery Unit
University of Naples “Federico II” Salesi Children’s Hospital
Via Pansini 5 Via Corridoni 11
80131 Naples 60123 Ancona
Italy Italy

Antonella Centonze, Dr. Anne Dariel, Dr.


Pediatric Surgery Unit Pediatric Surgery Department
Pugliese Hospital Hopital Mère-Enfant
Via Pio X 111 7 Quai Moncousu
88100 Catanzaro 44000 Nantes
Italy France

Christophe Chardot, Prof. Carolina De Fazio, Dr.


University of Geneva Children’s Hospital Gabriele D’Annunzio University
Paediatric Surgery Unit School of Medicine
Rue Willi Donzè 6 Via Vestini 150
CH 1205 Geneve 66100 Chieti
Switzerland Italy

Aurelie Chiappinelli, Dr. Luis de la Torre, Prof.


Chair of Pediatric Surgery Hospital Angeles Puebla
University of Naples „Federico II“ Unidad de Pediatria
Via Pansini 5 Av. Kepler no. 2143
80131 Naples Col. Unidad Territorial Atlixcayotl
Italy Puebla CP 72810
Mexico
Priscilla Chiu, Dr.
Staff Surgeon, Scientist-Track Investigator Concetta De Luca, Dr.
Division of General Surgery Chair of Pediatric Surgery
University of Toronto University of Naples “Federico II”
The Hospital for Sick Children Via Pansini 5
555 University Avenue 80131 Naples
Toronto Italy
Ontario M5G 1X8
Canada Ugo de Luca, Dr.
Pediatric Surgery Unit
Fabrizio Cigala, Prof. Santobono Children’s Hospital
Department of Orthopedics and Traumatology Via Mario Fiore 15
School of Medicine 80129 Naples
University of Naples “Federico II” Italy
Via Pansini 5
80131 Naples Marianna De Marco, Dr.
Italy Chair of Pediatric Surgery
University of Naples “Federico II”
Bruno Cigliano, Dr. Via Pansini 5
Chair of Pediatric Surgery 80131 Naples
University of Naples “Federico II” Italy
Via Pansini 5
80131 Naples
Italy
List of Contributors XV

Vincenzo Di Benedetto, Prof. Vincenzo Farina, Prof.


Department of Pediatric Surgery Department of Pediatrics
University of Catania University of Naples “Federico II”
95100 Catania Via Pansini 5
Italy 80131 Naples
Italy
Hana Dolezalova, Dr.
Chair of Pediatric Radiology Amedeo Fiorillo, Prof. Dr.
University of Naples “Federico II” University of Naples Federico II
Via Pansini 5 Department of Pediatrics
80131 Naples Via S. Pansini 5
Italy 80131 Naples
Italy
Sanjeev Dutta, Prof.
Division of Pediatric Surgery Donald Frush, Prof.
Department of Surgery Duke Medical Center
Lucile Packard Children’s Hospital Department of Radiology
Stanford Medical Center 1905 McGovern-Davison Children’s Health Center
Stanford, CA 94305-5733 Box 3808 DUMC
USA Durham, NC 27710
USA
Alaa El-Ghoneimi, Prof.
Department of Pediatric Surgery and Urology Masayuki Fujioka, Prof.
Hôpital Robert Debré Stroke Center
48 Boulevard Sérurier Neurology and Neurosurgery
75935 Paris Cedex 19 Helios General Hospital Aue
France Dresden University of Technology
Gartenstr. 6
Ciro Esposito, Prof. 08280 Aue
Associate Professor of Pediatric Surgery Germany
Department of Pediatrics and
Chair of Pediatric Surgery Kansai Medical University
Federico II University of Naples 2-3-1 Shinmachi
School of Medicine Hirakata City
Via S. Pansini 5 573-1191 Osaka
80131 Naples Japan
Italy
Frédéric Gauthier, Prof.
Giovanni Esposito, Prof. Service de Chirurgie Pediatrique
Emeritus Professor of Pediatric Surgery Hôpital de Bicêtre
Chair of Pediatric Surgery 78 Rue de General Leclerc
Federico II University of Naples 94275 Le Kremlin Bicêtre Cedex
School of Medicine France
Via S. Pansini 5
80131 Naples Valerio Gentilino, Dr.
Italy Department of Pediatric Surgery
Gaslini Children Hospital
Paola Esposito, Dr. University of Genoa
Ss. Annunziata Hospital Largo Giannina Gaslini 5
Via dei Vestini 150 16100 Genoa
66100 Chieti Italy
Italy
XVI List of Contributors

Emanuela Giordano, Dr. Korgun Koral, Prof.


Chair of Pediatric Surgery Department of Radiology
University of Naples “Federico II” University of Texas Southwestern Medical
Via Pansini 5 Center at Dallas and Children’s Medical Center
80131 Naples 1935 Motor Street
Italy Dallas, TX 75235
USA
Chiara Grimaldi, Dr.
Service de Chirurgie Pediatrique Jacob C. Langer, Prof.
Hôpital de Bicêtre Department of Surgery
78 Rue de General Leclerc University of Toronto
94275 Le Kremlin Bicêtre The Hospital for Sick Children
France 555 University Avenue
Toronto
Sylviane Hanquinet, Prof. Ontario M5G 1X8
University of Geneva Children‘s Hospital Canada
Paediatric Radiology Unit
Rue Willi Donzé 6 Marc-David Leclair, Dr.
CH 1205 Genève Pediatric Surgery Department
Switzerland Hopital Mère-Enfant
7 Quai Moncousu
Yves Heloury, Prof. 44000 Nantes
Department of Pediatric Surgery France
Hopital Mère-Enfant
7 Quai Moncousu Selvaggia Lenta, Dr.
44000 Nantes Department of Pediatrics
France University of Naples “Federico II”
Via Pansini 5
Salvatore Iacobelli, Dr. 80131 Naples
Chair of Pediatric Surgery Italy
University of Naples “Federico II”
Via Pansini 5 Mario Lima, Prof.
80131 Naples Chair of Pediatric Surgery
Italy University of Bologna
Department of Pediatric Surgery
Vincenzo Jasonni, Prof. Policlinico S. Orsola
Department of Pediatric Surgery Via Massarenti 11
Gaslini Children Hospital 40138 Bologna
University of Genoa Italy
Largo Giannina Gaslini 5
16100 Genoa Francesco Longo, Dr.
Italy Department of Maxillofacial and ENT Surgery
National Cancer Institute of Naples
Nadia Khan, Dr. “Fondazione Pascale”
Department of Neurosurgery Via Semmola
University of Zurich 80131 Naples
Frauenklinikstrasse 10 Italy
8091 Zurich
Switzerland François I. Luks, Prof.
Division of Pediatric Surgery
Alpert Medical School of Brown University
Hasbro Children‘s Hospital
2, Dudley Street, Suite 180
Providence, RI 02905
USA
List of Contributors XVII

Marcelo Martinez-Ferro, Prof. Carl Muroi, Dr.


Professor of Surgery and Pediatrics Department of Neurosurgery
Deparment of Surgery University of Zurich
“Fundacion Hospitalaria” Private Children’s Hospital Frauenklinikstrasse 10
Cramer 4601, 3rd Floor 8091 Zurich
C1429AKK Switzerland
Buenos Aires
Argentina Azad Najmaldin, Prof.
Department of Paediatric Surgery
Gordon A. MacKinlay, Prof. Leeds Teaching Hospital NHS Trust
The University of Edinburgh St. James’s University Hospital
The Royal Hospital for Sick Children Beckett Street
Sciennes Road Leeds LS9 7TF
Edinburgh EH9 1LF UK
Scotland
UK Deepika Nehra, Dr.
Division of Pediatric Surgery
Luigi Mansi, Prof. Department of Surgery
Department of Radiology Lucile Packard Children’s Hospital
University of Naples “SUN” Stanford Medical Center
Via Pansini 5 Stanford, CA 94305-5733
80131 Naples USA
Italy
Abdellatif Nouri, Prof.
Antonio Marte, Prof. Service de Chirurgie Pédiatrique
Associate Professor of Pediatric Surgery University of Monastir
Second University of Naples CHU Monastir 5000
Via Pansini 5 Tunisia
80131 Naples
Italy Francesco Onorati, Dr.
Cardiac Surgery Unit
Luciano Mastroianni, Dr. Policlinico di Germaneto
Pediatric Surgery Unit Università Magna Grecia
Salesi Children Hospital Via Europa
Via Corridoni 11 Germaneto
60123 Ancona 88100 Catanzaro
Italy Italy

Girolamo Mattioli, Prof. Daniel Orbach, Dr.


Department of Pediatric Surgery Department of Pediatric Oncology
Gaslini Children Hospital Curie Institute
University of Genoa 26 rue d’Ulm
Largo Giannina Gaslini 5 75248 Paris Cedex 05
16100 Genoa France
Italy
Alfonso Papparella, Prof.
Mongi Mekki, Prof. Second University of Naples
Service de Chirurgie Pédiatrique Pediatric Surgery, Department of Pediatrics
University of Monastir Via Pansini 5
CHU Monastir 5000 80131 Naples
Tunisia Italy
XVIII List of Contributors

Pio Parmeggiani, Prof. Attilio Renzulli, Prof.


Professor of Pediatric Surgery and Chief Cardiac Surgery Unit
Second University of Naples Policlinico di Germaneto
Pediatric Surgery, Department of Pediatrics Università Magna Grecia
Via Pansini 5 Via Europa
80131 Naples Germaneto
Italy 88100 Catanzaro
Italy
Annalisa Passariello, Dr.
Department of Pediatrics Samuel Rice-Townsend, Dr.
Federico II University of Naples Division of Pediatric Surgery
Via S. Pansini 5 Department of Surgery
80129 Naples Lucile Packard Children’s Hospital
Italy Stanford Medical Center
Stanford, CA 94305-5733
Flavio Perricone, Dr. USA
Chair of Pediatric Surgery
University of Naples “Federico II” Corrado Rispoli, Dr.
Via Pansini 5 General Surgery Unit
80131 Naples Ascalesi Hospital
Italy 80143 Naples
Italy
Pascale Philippe-Chomette, Dr.
Department of Pediatric Surgery and Urology Nancy Rollins, Prof.
Hôpital Robert Debré Department of Pediatric Radiology
48 Boulevard Sérurier Children’s Medical Center Dallas
75935 Paris Cedex 19 University of Texas
France Southwestern Medical Center Dallas
and Holder of the Charles Cameron Sprague Chair
Alessio Pini Prato, Dr. in Medical Science
Department of Pediatric Surgery 1935 Motor Street
Gaslini Children Hospital Dallas, TX 75235
Largo Giannina Gaslini 5 USA
16100 Genoa
Italy Mercedes Romano, Dr.
Second University of Naples
Guillaume Podevin, Dr. Pediatric Surgery, Department of Pediatrics
Pediatric Surgery Department Via Pansini 5
Hopital Mère-Enfant 80131 Naples
7 Quai Moncousu Italy
44000 Nantes
France Carmelo Romeo, Prof.
Department of Pediatric Surgery
Pier Francesco Rambaldi, Dr. University of Messina
Department of Radiology and Nuclear Medicine Viale Gazzi Padiglione NI
Second University of Naples 98124 Messina
Piazza Miraglia 3 Italy
80138 Naples
Italy
List of Contributors XIX

Giovanni Ruggeri, Dr. Jürgen Schleef, Dr.


Chair of Pediatric Surgery Department of Paediatric Surgery
University of Bologna IRCCS Burlo Garofolo
Policlinico S. Orsola Via dell´Istria 65/1
Via Massarenti 11 34100 Trieste
40138 Bologna Italy
Italy
Françoise Schmitt, Dr.
Maria Domenica Sabatino, Dr. Department of Pediatric Surgery
Chair of Pediatric Surgery 44000 Nantes
Second University of Naples France
Via Pansini 5
80131 Naples Reinhard Schumacher, Prof.
Italy Department of Pediatric Radiology
University Medical Center Mainz
Francesco Sadile, Prof. Langenbeckstr. 1
Department of Orthopedics and Traumatology 551101 Mainz
School of Medicine Germany
University of Naples “Federico II”
Via Pansini 5 Luigi Scippa, Dr.
80131 Naples Department of Pediatrics
Italy University of Naples “Federico II”
Via Pansini 5
Sabine Sarnacki, Prof. 80131 Naples
Department of Pediatric Surgery and U 793 Italy
Hôpital Necker Enfants Malades
149 rue de Sèvres Alessandro Settimi, Prof.
75015 Paris Chair of Pediatric Surgery
France University of Naples “Federico II”
Via Pansini 5
Antonio Savanelli, Dr. 80131 Naples
Chair of Pediatric Surgery Italy
University of Naples “Federico II”
Via Pansini 5 Carla Settimi, Dr.
80131 Naples Department of Pediatrics
Italy University of Naples “Federico II”
Via Pansini 5
Cristina Savanelli, Dr. 80129 Naples
Department of Endocrinology Italy
University of Naples “Federico II”
Via Pansini 5 Giacomo Sica, Dr.
80129 Naples Cardiac Surgery Unit
Italy Policlinico di Germaneto
Università Magna Grecia
Felix Schier, Prof. Via Europa
Department of Pediatric Surgery Germaneto
University Medical Center Mainz 88100 Catanzaro
Langenbeckstr. 1 Italy
551101 Mainz
Germany Etienne Suply, Dr.
Department of Pediatric Surgery
44000 Nantes
France
XX List of Contributors

Gianluca Terrin, Dr. Gianfranco Vallone, Dr.


Department of Pediatrics Department of Radiology
University of Naples “Federico II” University of Naples “Federico II”
Via Pansini 5 Via Pansini 5
80131 Naples 80131 Naples
Italy Italy

Claudia Tiano, Dr. Isabelle Vidal, Dr.


Department of Pediatrics Pediatric Surgery Department
University of Naples “Federico II” Hôpital Mère-Enfant
Via Pansini 5 7 Quai Moncousu
80131 Naples 44000 Nantes
Italy France

Juan A. Tovar, Prof. Dr. Salam Yazbeck, Prof.


Departamento de Cirugía Pediátrica Department of Pediatric Surgery
Hospital Universitario La Paz San Justine Children Hospital
P. Castellana 261 Montreal
28046 Madrid Canada
Spain
Yasuhiro Yonekawa, Prof. emeritus
Antonino Tramontano, Dr. Dept. of Neurosurgery
Pediatric Surgery Unit University of Zurich
Santobono Children Hospital Haldenbachstrasse 12
Via Mario Fiore 15 8091 Zurich
80129 Naples Switzerland
Italy

Jean Stephane Valla, Prof.


Department of Pediatric Surgery and Urology
Fondation Hôpital Lenval
Avenue de Californie 55
06200 Nice
France
Abbreviations

AP Anteroposterior GI Gastrointestinal
ALT Alanine aminotransferase HCG Human chorionic gonadotropin
AST Aspartate aminotransferase ICU Intensive care unit
CE Contrast enhanced IV Intravenous
CRP C-reactive protein IVU Intravenous urogram
CT Computed tomography MCDK Multicystic dysplastic kidney
DMSA Dimercaptosuccinic acid MIBG Metaiodobenzylguanidine
ECG Electrocardiogram MR Magnetic resonance
ERCP Endoscopic retrograde NMR Nuclear magnetic resonance
cholangiopancreatography NPO Nil per os
EXIT Ex utero intrapartum treatment PET Positron emission tomography
FDG [(18)F]-fluorodeoxyglucose SIOP International Society of Pediatric
FLAIR Fluid attenuated inversion recovery Oncology
FNAB Fine-needle aspiration biopsy STIR Short tau inversion recovery
Gd Gadolinium US Ultrasonography
Gd-DTPA Gadolinium-diethylenetriamine UTI Urinary tract infection
pentaacetic acid VATS Video-assisted thoracoscopic surgery
GER Gastroesophageal reflux VCE Video capsule endoscopy
VUR Vesicoureteral reflux
Introduction

We think that a quick introduction is necessary to ex- ing on a specific case. Accompanying the figures is the
plain how this book is organized and how it should be clinical history of the patient along with some questions
read. on the interpretation of the images for the diagnosis and
The book is divided into eight sections, six of them management of the case presented.
focusing on different parts of the human body and two The A page contains the interpretation of the images
sections dedicated to emergency, trauma, and tumors. shown on the Q page, and possibly other figures of di-
The book is easy to read and includes more than agnostic procedures performed in the same case in or-
200 case reports of different pathological conditions. der to obtain a clear diagnosis. The A page also includes
Each radiological surgical case comprises no more than information about the particular condition affecting the
two printed pages. patient and the management of the case shown, includ-
The question page (Q) is a right-hand page; the an- ing therapy and follow-up. Most case reports end with a
swer page (A) is found overleaf. short “Suggested Reading” list.
The Q page contains radiological images (radiogra-
phy, CT, MRI, scintigraphy, ultrasonography etc.), focus-
1
Head and Neck

Case 1–13
2 Head and Neck

Introduction

The patient of pediatric neurosurgery ranges from the formed for the diagnosis of these pathological condi-
preterm infant to the adolescent. The diseases addressed tions are computed tomography, magnetic resonance
in pediatric neurosurgery include congenital disorders imaging, cerebral angiography, and single photon emis-
such as hydrocephalus, spina bifida, and craniosynos- sion computed tomography. The brain tumors discussed
tosis, particular types of tumors of the central nervous in this section are craniopharyngioma, ganglioglioma,
system, cerebrovascular diseases including vascular mal- medulloblastoma, choroid plexus papilloma, pilocytic
formations and moyamoya disease, intractable epilepsy, astrocytoma, dysembryoplastic neuroepithelial tumor,
and traumatic brain injury or acquired disorders. Pedi- anaplastic astrocytoma, and atypical meningioma. Ce-
atric neurosurgery requires team work that involves pe- rebral arterial aneurysm, moyamoya disease, and moy-
diatric specialists from various fields including pediatric amoya syndrome are also presented.
anesthesia, neurology, neuro-oncology, plastic surgery, All patients underwent neurosurgical operations and
psychology, and neuroradiology. had a good recovery after the surgery. We hope the read-
In this section, 13 pediatric cases of head or neck ers will enjoy learning the characteristics of diagnostic
diseases are presented. These include brain tumors imaging in these surgical cases.
and cerebrovascular diseases. The imaging studies per-
1 Head and Neck 3

Q1
Nancy Rollins

Fig. 1 Fig. 2

A 4-year-old boy presented with a soft, slowly enlarging,


right cervical soft-tissue mass (Fig. 1).
• What is the differential diagnosis?
• What is the best imaging strategy?
• What does the MR image show (Fig. 2)?
• Should the lesion be biopsied or resected?
• Is there a nonsurgical alternative for treatment?
4 Head and Neck

A1

Fig. 3
Fig. 4 Fig. 5

The differential diagnosis includes a low-flow vascular pouches of fluid may be large (macrocystic) or micro-
malformation, neuroblastoma or nerve-sheath tumor, or cystic. Lymphatic malformations are often admixed with
possibly a rhabdomyosarcoma. Contrast-enhanced MR a venous malformation, e. g., venolymphatic malforma-
imaging is used to define the internal architecture of the tions. Macrocystic lymphatic malformations and mixed
lesion, e.g., solid vs. cystic, slow flow vs. high flow, and venolymphatic malformations are amenable to sclero-
the extent of the lesion proximity to vital structures such therapy, whereas microcystic lesions and microcystic
as the carotid sheath. Lymphatic malformations are usu- components of lymphatic malformations are usually not.
ally low signal on T1 images; however, the signal inten- Sclerotherapy is usually performed under fluoroscopic
sity on the T1 images may be similar to or higher than guidance, although ultrasonography is useful in punc-
that of regional muscle if bleeding has occurred into the turing nonpalpable lesions. The cyst should be emptied
lymphatic malformation or if the fluid has a high pro- of fluid as much as possible. Contrast medium is in-
tein content. The presence of fluid-fluid levels is almost jected to document correct intralesional positioning of
pathognomonic of a lymphatic malformation. On T2 the needle and lack of extravasation of contrast and of
images and STIR sequences (Fig. 2), the fluid shows very the sclerosing agent (Fig. 4). Effective sclerosing agents
bright signal although blood products may decrease in include OK-432, absolute alcohol, and doxycycline. OK
signal intensity. Ultrasonography may show a cyst(s) 432 (picibinal) is a lyophilized mixture of a low-viru-
with absence of echoes or medium levels of echoes in lence strain of Streptococcus pyogenes mixed with ben-
high proteinaceous or hemorrhagic fluid (Fig. 3). The zypenicillin. Intralesional hemorrhage may complicate
MR image shows a large, fluid-filled, unilocular cyst ex- sclerotherapy, which is seen as an abrupt increase in the
tending from the skull base to the supraclavicular region size of the lymphatic malformation and change from a
anterior to the sternocleidomastoid muscle. The imag- soft spongy lesion to a tense slightly painful one. Intral-
ing findings are classic for a macrocystic lymphatic mal- esional hemorrhage does not, as a rule, require drainage
formation and biopsy is not indicated. The lesion is best since the hemorrhage will slowly resolve. Figure 5 shows
treated with sclerotherapy. the patient 2 weeks after sclerotherapy.
Lymphatic malformations are composed of dysplas-
tic vesicles or pouches filled with lymphatic fluid. The
1 Head and Neck 5

Q2
Nancy Rollins

Fig. 1 Fig. 2

A 3-year-old girl presented with a large disfiguring fa- • What is the differential diagnosis?
cial mass that failed to involute with high-dose pulsed • Should this lesion be biopsied?
steroids and alpha interferon. Figure 1 shows the patient • What are the options for medical management?
at presentation. Figure 2 is a cross-sectional image of the • What does the MR imaging show?
face.
6 Head and Neck

A2

MR imaging (Fig. 2) shows a large nonlipomatous mass


which enhances and which has extensive involvement of
the parotid glands and muscles of mastication as well as
the infratemporal fossa. Branches of the external carotid
arteries are dilated as are the internal jugular veins indi-
cating a high-flow lesion.
The patient underwent sequential arterial embolizations
using particles and coils with considerable decrease in
the size of the lesion. Figure 3 shows the patient 1 year
later. There is residual facial deformity due to residual
fibro-adipose tissue that will be corrected surgically. La-
ser therapy will be used to treat the remaining cutaneous
component.
Hemangiomas usually appear within 2 weeks after
birth as a small red blemish or bump, which grows rap-
idly. The lesion may spontaneously regress, usually be- Fig. 4
tween 12–18 months of age. Complete regression results
in the lesion being inapparent by age 3–5 years of age,
with no or only minor residual scaring. In other patients,
involution may take longer; 50% will involute by age 5,
70% by age 7, and 90% by the age of 9. Lesions which observed within 1 month of treatment, although a slow
regress slowly are often associated with scaring, atro- protracted response may also occur.
phoderma, stria, and cutaneous discoloration. Heman- Superficial hemangiomas may be treated with pulsed
giomas that require early aggressive treatment include dye laser, but deeper lesions are not treatable with this
those that are cosmetically deforming, growing rapidly, modality as the depth of laser penetration is only 1–
or obstructing vision, hearing, breathing, eating or, any 2 mm. For large multicompartmental facial lesions, arte-
other body function. rial embolization is usually effective at accelerating the
Systemic corticosteroids 2–3 mg/kg, given for 4- regression of the hemangiomas. The procedure involves
8 weeks comprise the first-line therapy for complicated superselective catherization of branches of the exter-
hemangiomas; regression rates of up to 90% have been nal carotid arteries and occlusion of arteries supplying
reported. Intralesional corticosteroid injections may be the hemangiomas using particulate material and small
used for lesions that are smaller than 3 cm in diameter endovascular coils. The internal carotid arteries should
and well-defined and for lesions that show ulceration. also be studied to assess what, if any, contributions to
Three to five intralesional injections are usually given at the hemangiomas arise from the internal carotid arteries
6-week intervals; each dose should not exceed 3 mg/kg. and to exclude carotid stenosis in patients with PHACE
Hemangioma not responsive to corticosteroid ther- syndrome (posterior fossa brain malformations, heman-
apy may be treated with both alpha and the 2a form of giomas, arterial anomalies, coarctation of the aorta and
alpha interferon. However, treatment with interferon is cardiac defects, and eye abnormalities).
associated with the development of irreversible spastic Sequential embolizations are needed to devascular-
diplegia in about 20% of children. Vincristine is now ize the lesions because arterial collaterals form rapidly.
recommended for hemangiomas with airway, eyelid, Potential complications of embolization include inad-
and orbital involvement, disseminated neonatal heman- vertent embolization of the central retinal artery causing
giomatosis of the skin, liver, kidney, and cardiac failure. blindness as well as stroke and damage to the femoral
A weekly dosage of 1 mg/m(2) is injected intravenously. arteries resulting in leg length discrepancy. If surgical
The dose is tapered depending on the clinical response. removal or reconstruction is needed, preoperative su-
The reported range of injections is 5–25 with a length of perselective embolization is recommended to minimize
treatment of 1.5–8 months. Dramatic response may be intraoperative blood loss.
1 Head and Neck 7

Q3
François Luks

Fig. 1

On routine prenatal ultrasound at 22 weeks, a complex • What is the most likely diagnosis, and what is the dif-
cystic mass was found in the cervical region of an other- ferential diagnosis?
wise normal-appearing fetus. • How should the expecting couple be counseled?
On subsequent examinations at 24 and 26 weeks, the • Is intervention before birth indicated?
mass was seen to increase dramatically in size. At • How should the pregnancy be further monitored,
26 weeks, moderate polyhydramnios was noted. The re- and what might prompt early intervention? Should
mainder of the examination was normal. time, place, and/or mode of delivery be altered?
MR imaging was performed to better characterize the • Is neonatal intervention required? If so, how soon af-
mass (Fig. 1). At that time, the total size of the mass was ter delivery?
larger than the fetal head. Again, polyhydramnios was • What is the prognosis for a fetus with this condition?
noted.
8 Head and Neck

A3

Fig. 2 Fig. 3 Fig. 4

The size of the lesion and its complex, cystic/solid and Once the airway is secured, the cord can be clamped
heterogeneous appearance are typical of a cervical tera- and the infant delivered. This approach requires a very
toma. If predominantly cystic, the only other possible high level of control and collaboration between mater-
diagnosis would be a cystic hygroma. nal-fetal medicine specialists (perinatologists), maternal
Head and neck teratomas are far less frequent than anesthesiologists, pediatric surgical specialists, and neo-
pelvic and sacrococcygeal ones, and are less likely than natologists. Obtaining an airway can range from simple
sacrococcygeal teratomas to cause significant vascular orotracheal intubation to rigid bronchoscopy as a tem-
steal, fetal hydrops, or mirror syndrome (concomitant porary airway and tracheostomy or even (partial) resec-
maternal preeclampsia). However, the size and location tion of the obstructing mass. EXIT procedures of up to
of this lesion are likely to cause some degree of respira- 60–90 min have been described, although the average
tory obstruction at birth. The presence of polyhydram- duration of this procedure is about 20 min.
nios suggests that fetal swallowing is impaired, causing Because of the polyhydramnios and the risk of pre-
further concern about neonatal respiratory distress. term labor, it is important to choose the time of delivery
In the past, the mortality rate of large cervical tera- by EXIT carefully: in the present case, the mother expe-
tomas exceeded 50% because of airway obstruction at rienced some contractions at 29 weeks, and an EXIT was
birth. In addition, the presence of polyhydramnios in- performed at 32 weeks. Several days before the planned
creases the risk of premature rupture of membranes and procedure, glucocorticoids were administered to the
preterm delivery. mother to accelerate lung maturation.
Because of the rapid growth of the lesion, several At delivery, the diagnosis of cervical teratoma was
multidisciplinary meetings were held to plan an EXIT confirmed (Fig. 2). Intubation proved impossible, and a
procedure: ex-utero, intrapartum treatment of the upper tracheostomy was performed.
airway obstruction. This approach requires a planned MR imaging was performed in the ensuing days
and controlled C-section whereby uterine contractions (Fig. 3), and semi-elective resection of the entire mass
are suppressed, preventing separation of the placenta. was performed at 8 days of life. Despite the massive dis-
Only the head and neck of the fetus are delivered, leav- tortion of normal structures, these lesions are not inva-
ing the umbilical cord in utero. Thus, an airway can be sive, and symmetry is usually restored postoperatively
obtained while the infant remains on placental support. (Fig. 4).
1 Head and Neck 9

Q4
Giovanni Esposito and Ciro Esposito

Fig. 1

A 4-year-old child without previous medical problems, • What does the ultrasound demonstrate?
apart from an episode of bilateral parotitis at the age of • What other examinations were performed?
3 months, was admitted to hospital after the mother no- • What was the suspected diagnosis?
ticed a swelling on the child’s left cheek. After objective • What was the surgical treatment?
examination a painless mass was found with indistinct • What was the definitive diagnosis?
margins in the left parotid region. An ultrasound study • What was the follow-up?
(Fig. 1) and other examinations were performed, indi-
cating the need for surgical intervention.
10 Head and Neck

A4

The ultrasound shows a small hypoechogenic lesion in


the left parotid region.
A fine-needle aspiration biopsy (FNAB) was carried out,
which demonstrated the presence of adipocytes mixed
with acinic cells at histologic examination.
The diagnosis was a lipoma of the parotid gland. The
surgical treatment comprised superficial parotidectomy
with conservation of the facial nerve. On the basis of a
histologic examination of the removed specimen (Fig. 2),
demonstrating the presence of mature adipocytes with
abnormal mature and multilobular adipose tissue com-
bined with inflammatory cells (Fig. 3), the definitive di-
agnosis was that of parotid lipomatosis.
The postoperative course was uneventful and the
child was discharged from hospital 6 days after the op- Fig. 2
eration. At the 5-year follow-up, the child had recovered
completely.

Fig. 3

Suggested Reading
1. Al-Arfaj AA, Arora RK, El Hassan AY, Al-Metwalli RR. 4. Som PM, Scherl MP, Rao VM, Biller HF. Rare presentations
Lipomatosis of the parotid gland in children. Saudi Med J of ordinary lipomas of the head and neck: a review. AJNR
2003 Aug; 24(8):898–900 Am J Neuroradiol 1986 Jul–Aug; 7(4):657–64
2. Holland AJ, Baron-Hay GS, Brennan BA. Parotid lipomato- 5. Walts AE, Perzik SL. Lipomatous lesions of the parotid
sis. J Pediatr Surg 1996 Oct; 31(10):1422–3 area. Arch Otolaryngol 1976 Apr; 102(4):230–2
3. Sinha DD, Joshi M, Sharma C, Chaturvedi V. Infantile con-
genital parotid lipomatosis: a rare case report. J Pediatr
Surg 2005 Sep; 40(9):e15–6
1 Head and Neck 11

Q5
Luigi Califano and Francesco Longo

A 13-year-old girl presented with a history of parotid


swelling from 15 months, which was not related to eat-
ing. During this period she did not experience fever or
pain.
When the girl presented to our department, she had al-
ready undergone many cycles of antibiotic therapy, with-
out any result. The results of blood examinations were
normal.
The surgeon asked for a CT scan with iodine medium.
• What does Fig. 1 show?
• Why did the surgeon perform this examination?
• Are there further examinations to be performed in
this case?
• What pathological condition is affecting this girl?
• What is the best way to manage this condition?

Fig. 1
12 Head and Neck

A5

This girl was affected by a parotid acinic cell neoplasm.


The history and clinical examination already pointed to
a parotid tumor. A CT scan with iodine medium allows
one to gain more information about the precise location
of the neoplasm and to define its borders and its rela-
tionships with closer structures.
Figure 1 shows a roundish, inhomogeneous lesion of
10-mm diameter, without peripheral anomalies compat-
ible with pleomorphic adenoma or noninfiltrating neo-
plasms, localized in the superficial part of the parotid
gland. There was no involvement of the neck lymph
nodes.
For a preoperative diagnosis to be made, it is neces-
sary to perform fine-needle aspiration biopsy (FNAB), Fig. 2
which has very good accuracy with very little discomfort
for the patient. Obtaining an almost-certain diagnosis
allows one to plan the intervention with better accuracy
and to prepare the patient for the surgical procedure.
The FNAB in this case showed a solid proliferation of
acinic cells arranged in solid blocks, suggesting an acinic
cell tumor.
The patient underwent total parotidectomy under
general anesthesia. A face lift approach was used on this
young patient to minimize the effect of scars (Fig. 2).
Moreover, in an attempt to avoid the risk of Frey syn-
drome, a temporalis fascia flap was used to cover the
nerve and to reduce the gap left by the parotid gland ex-
cision (Figs. 3–4). No sign of facial nerve damage was
evident after intervention. Histological examination of Fig. 3
the surgical specimen confirmed the preoperative diag-
nosis of acinic cell tumor.
The patient was followed up every 3 months with
ultrasonography and twice a year with CT in the first
year, and then with ultrasonography twice a year and CT
once a year. Four years after the intervention, the patient
was free of disease.

Suggested Reading
1. Mathew S, Ali SZ. Parotid fine-needle aspiration: a cyto-
logic study of pediatric lesions. Diagn Cytopathol 1997;
17:8–13
2. Orvidas LJ, Kasperbauer JL, Lewis JE, Olsen KD, Lesnick
TG. Pediatric parotid masses. Arch Otolaryngol Head Neck
Surg 2000; 126:177–84 Fig. 4
1 Head and Neck 13

Q6
Luigi Califano, Paola Esposito, and Francesco Longo

Fig. 1 Fig. 2

A 3-year-old girl presented with mandibular swelling • What do Figs. 1 and 2 show?
that had appeared 6 months earlier. • What other diagnostic examinations can be per-
The girl had growth hormone deficit and psychomo- formed?
tor retardation, facial dysmorphism (telecantus), and • What is the diagnosis?
an arachnoid cyst in the adenohypophyseal region. The • What is the appropriate treatment?
child had the clinical picture of an atypical facial-cardio- • What is the prognosis for this young patient?
cutaneous syndrome.
After the girl’s mother noted swelling of the mandible,
a pediatrician performed CT (Figs. 1, 2) and referred the
child to a maxillofacial surgeon.
14 Head and Neck

A6

This child is affected by fibrous dysplasia of the mandible,


a condition in which normal medullary bone is gradually
replaced by abnormal fibrous connective proliferation.
This disease is localized in the head in less than 10% of
cases and produces monostotic or polyostotic lesions, iso-
lated or in association with McCune-Albright syndrome.
This disease most commonly presents as an asymp-
tomatic, slow enlargement of the involved bone. The
monostotic form is the most common (80%) and more
frequently affects the ribs, long bones, pelvis, jaws, and
skull. Maxillary lesions occur more frequently than
mandibular ones.
Fibrous dysplasia characteristically occurs during the
first and second decade of life and becomes stable after Fig. 3
puberty.
In the classic presentation, as shown in Figs. 1 and
2, the radiographic appearance is described as homoge-
neous radio-opacity with numerous trabeculae of woven
bone imparting a ground glass appearance. Lesions of
fibrous dysplasia may also present as unilocular or mul-
tilocular radiolucencies. There are no alterations in labo-
ratory values.
A three-dimensional CT scan can be very useful in
showing the complete extension of the disease and in
planning preoperatively the margins of surgical inter-
vention.
Typical histological features are represented by pro-
liferation of fibrous connective tissue with trabeculae of
immature bone tissue. A histological differential diag- Fig. 4
nosis with osteoma, fibroma, or Paget’s disease can be
difficult to make without the correct interpretation of
clinical and radiological findings.
Once a jaw lesion is diagnosed, the extent of skeletal functional or cosmetic disability (Figs. 3, 4). En bloc
involvement should be investigated, with plain radio- resection is unnecessary considering the lesion’s slow-
graphs, total-body CT, or scintigraphy. In our case, there growing and non-neoplastic nature.
was only one lesion. The incidence of malignant transformation is very
Because of the stabilization of growth after puberty, rare, less than 1%, and is related to cases in which pa-
the treatment consists only of biopsy for confirmation tients were treated with radiotherapy, which should al-
of the diagnosis and periodic follow-up for small lesions ways be avoided considering the benign nature and lim-
and bone recontouring via a transoral approach to treat ited growth of the lesion.
1 Head and Neck 15

Q7
Luigi Califano and Francesco Longo

Fig. 1

An 11-year-old girl presented with a history of facial


asymmetry that had started 18 months previously with
dental crowding.
The patient was in general good health and had men-
arche when she was 10 years old. She did not complain
of any pain or discomfort.
When the girl presented to our department, she had a Fig. 2
deviation of the chin and a cross-bite and had already
started orthodontic treatment without any result.
The surgeon asked for a panoramic plain radiograph fol-
lowed by CT with three-dimensional reconstruction.
• What do Figs. 1–3 show?
• Should further examinations be performed in this
case? If so, why?
• What condition is affecting this girl?
• What is the best way to manage this pathological
condition?

Fig. 3
16 Head and Neck

A7

Fig. 4 Fig. 5

This girl was affected by hemimandibular hyperplasia. To manage correctly these patients, it is very impor-
Hemimandibular hyperplasia is a developing anomaly tant to collect all clinical data. In fact, a history of rapid
of the mandible which appears at puberty and often con- changes suggests an active growth, while a history show-
tinues in the following years. Its features are asymmetry ing an old presentation and slow changes indicates inac-
of the lower third of the face, deviation of the mandible tivity or reduced activity of the condylar process. Bone
to the opposite side of the affected hemimandible, and scans are very useful for confirming an active growth,
secondary growth anomaly of the maxilla. but there is a high risk of false-positive findings.
A panoramic plain radiograph (Fig. 1) shows an In cases of active growth, as in our case, condylec-
elongation and enlargement of one side of the mandi- tomy is the treatment of choice. The patient needs close
ble, i.e., enlargement of the condyle, condylar neck and follow-up and orthodontic treatment to prepare for a
ascending and horizontal rami; the head of the condyle two-jaw surgery if the maxilla is also affected.
can be normal in cases of slow growth or enlarged in Our patient underwent condylectomy via a rhitidec-
cases of rapid evolution. A cephalometric study shows tomy approach to minimize scars and avoid any risk of
variable dental compensation in the sagittal and verti- facial nerve damage (Fig. 4). An intraoperative view of
cal planes. CT with three-dimensional reconstruction the condyle is presented in Fig. 5.
(Figs. 2, 3) clearly shows the hemimandibular enlarge- The patient was followed up for 3 years, with progres-
ment. sive improvement of the occlusion and facial symmetry.

Suggested Reading
1. Chen YR, Bendor-Samuel RL, Huang CS. Hemimandibular 3. Marchetti C, Cocchi R, Gentile L, Bianchi A. Hemiman-
hyperplasia. Plast Reconstr Surg 1996; 97:730–7 dibular hyperplasia: treatment strategies. J Craniofac Surg
2. Ferguson JW. Definitive surgical correction of the de- 2000; 11:46–53
formity resulting from hemimandibular hyperplasia.
J Craniomaxillofac Surg 2005; 33:150–7
1 Head and Neck 17

Q8
Luigi Califano and Francesco Longo

Fig. 1

Fig. 2

A 14-year-old boy presented with facial asymmetry, To evaluate the airway status, the surgeon asked for
nasal obstruction, and exophthalmus that had started a CT scan.
1 year previously. • What do Figs. 1 and 2 show?
The patient was in general good health. He did not • What is the diagnosis?
complain of any pain, but reported diplopia. • What treatment plan should be followed?
18 Head and Neck

A8

The patient had juvenile aggressive ossifying fibroma.


Juvenile aggressive ossifying fibroma occurs in young
patients generally before the age of 15 years. The most
common sites of occurrence include the maxilla, frontal
bones, ethmoid bones, and paranasal sinus.
At clinical presentation, facial asymmetry, nasal ob-
struction, and exophthalmus with visual disturbances
are generally present. The tumor presents a progressive
and often rapid enlargement with thinning and erosion
of adjacent bone.
The CT studies show an expansile, destructive, ra-
diopaque, or hyperintense nonhomogeneous lesion
(Figs. 1, 2).
The approach to juvenile aggressive ossifying fi- Fig. 3
broma is surgical. Depending on the size, location, and
extension of the tumor, the goal of treatment should be
complete excision, which in many cases requires an ag-
gressive surgical procedure. En bloc resection should be
evaluated in cases of large or recurrent lesions and can
cause significant deformities especially in the case of or-
bital or cranial involvement.
Close follow-up is mandatory, since the reported re-
currence rate for this tumor is between 30% and 60%;
however, no case of metastasis has been reported.
Our patient underwent submental intubation to
avoid the presence of an anesthesia tube in the operative
field. The lesion was reached and removed via a bipartite
Le Fort 1 fracture (Fig. 3). The fracture lines were fixed
with biodegradable plates to avoid any alteration of the
residual growth of the maxilla (Fig. 4). Fig. 4
The patient was followed-up for 3 years with clinical
examinations and CT every 4 months, and at the time of
writing was free of disease (Fig. 5).

Suggested Reading
1. Bertrand B, Eloy P, Cornelis JP, Gosseye S, Clotuche J, Gil-
liard C. Juvenile aggressive cemento-ossifying fibroma: case
report and review of the literature. Laryngoscope 1993;
103:1385–90
2. Brannon RB, Fowler CB. Benign fibro-osseous lesions: a re-
view of current concepts. Adv Anat Pathol 2001; 8:126–43
3. Marvel JB, Marsh MA, Catlin FI. Ossifying fibroma of the
mid–face and paranasal sinuses: diagnostic and thera-
peutic considerations. Otolaryngol Head Neck Surg 1991;
Fig. 5
104:803–8
1 Head and Neck 19

Q9
Craig T. Albanese

A routine screening ultrasound at 20 weeks’ gestation


was performed (Fig. 1) revealing a large mass (arrows)
protruding from the fetal oral cavity (open arrow). The
exact origin could not be delineated by this examination.
There were calcifications (arrowhead) in the mass.
MR imaging (Fig. 2) was performed, which con-
firmed the diagnosis of an oral cavity-based mass (ar-
row). There was polyhydramnios (asterisk) secondary to
the obstructing mass in the oral cavity.
• What is the diagnosis?

Fig. 1

Fig. 2
20 Head and Neck

A9

This large mass with calcifications is most consistent


with an oral teratoma (epignathus).
Because of the risk of airway compromise, the fetus
was delivered using the EXIT (ex utero intrapartum
treatment) strategy. During the procedure, the uterus
is opened and the mass is removed while the fetus is
still connected to the placenta. After resecting the mass
(Fig. 2) that was emanating from the area of the hard
palate, orotracheal intubation was performed, the um-
bilical cord cut, and the baby delivered. The child recov-
ered uneventfully.

Suggested Reading
1. Abendstein B, Auer A, Pumpel R, Mark E, Desch B, Tscharf
J. Epignathus: prenatal diagnosis by sonography and
magnetic resonance imaging Ultraschall Med 1999 Oct;
20(5):207–11
2. Chattopadhyay A, Patra R, Vijaykumar. Oral tumors in
newborn. Indian J Pediatr 2003 Jul; 70(7):587–8
3. Ozeren S, Yuksel A, Altinok T, Yazgan A, Bilgic R. Prenatal
ultrasound diagnosis of a large epignathus. J Obstet Gynae-
col 1999 Nov; 19(6):660–1
1 Head and Neck 21

Q 10
Masayuki Fujioka, Carl Muroi, Nadia Khan, and Yasuhiro Yonekawa

Fig. 1 Fig. 2 Fig. 3

A 6-year-old girl suffered from headaches, poor appetite,


general fatigue, and projectile vomiting for 3 days.
According to her father, the girl had recently been
saying that it was dark even during the day. The pres-
ence of increased intracranial pressure and visual distur-
bance was suspected.
On admission to hospital, the bilateral light reflex
(direct and indirect) was weak. CT scanning of the head
was performed (Fig. 1).
She had decreased skin turgor suggestive of dehydra-
tion associated with polyuria (low specific gravity). Her
blood glucose level was normal and the plasma natrium
level was moderately increased. She also presented with
growth failure and short statue.
Fig. 4
• What does Fig. 1 show?
• What pathophysiology should be suspected?
• What does the MR imaging study (Gd-T1-weighted) • What diagnosis can be considered?
of the head show (Figs. 2–4)? • What treatment should be performed?
22 Head and Neck

A 10

Fig. 5 Fig. 6 Fig. 7

The CT scan (Fig. 1) and T1-weighted MR images with transrostrum corporis callosi and lamina terminals ap-
administration of gadolinium-diethylenetriamine pen- proach with the patient in the supine position.
taacetic acid (Gd-DTPA; Figs. 2–4) reveal a cystic tumor The postoperative CT scan and MR images (Figs. 5–
extending from the sella turcica through the suprasellar 7) demonstrate the total removal of the tumor. The
cistern to the third ventricle. The CT scan shows calcifi- histological diagnosis is adamantinomatous craniopha-
cation in some portions of the cyst wall. The tumor cyst ryngioma. The characteristic features include peripheral
wall is enhanced on the Gd-T1-weighted MR image. The palisading of nuclei, loose arrangements of squamous
MR images show dilatation of the bilateral lateral ven- cells, and nodules of keratin and calcification.
tricle due to the obstruction of the foramen of Monro In general, the features of craniopharyngioma on CT
(obstructive hydrocephalus). images include cyst formation with calcification and en-
The differential diagnosis for brain tumor in this re- hancement effects of the cyst wall and solid portion. The
gion includes craniopharyngioma, germ cell tumor, pi- signal behavior of craniopharyngioma on T1-weighted
tuitary adenoma, glioma, and meningioma. images varies according to its cystic contents (hypo-,
The simultaneous presence of dehydration and exces- iso-, and hyper-intensity). The increased intensity on T1-
sive urination of low specific gravity suggests diabetes weighted images seems to result partly from the high con-
insipidus. In addition, the patient had growth failure centration of liquid cholesterol. On T2-weighted images,
suggestive of pituitary dwarfism. The endocrinological craniopharyngiomas are commonly hyperintense. The
study showed hypopituitarism with decreased blood solid portions are heterogeneously enhanced and the cyst
levels of growth hormone, insulin-like growth factor-1 wall is strongly enhanced on Gd-T1-weighted images.
(somatomedin-C), and thyroid-stimulating hormone. Postoperative hormonal replacement therapy was
Craniopharyngioma is the most probable diagnosis initiated with administration of adrenal corticosteroid,
for a suprasellar cystic tumor with partially calcified wall, thyroid hormone, and antidiuretic hormone. After
leading to hypopituitarism in this 6-year-old girl. The 6 months, the patient had improved visual acuity (right
tumor occupying the third ventricle was totally removed, 0.2 and left 0.6) and no apparent recurrence of cranio-
without additional deficits, via an interhemispheric pharyngioma.
1 Head and Neck 23

Q 11
Giovanni Esposito, Aurelie Chiappinelli, Gianfranco Vallone,
and Ciro Esposito

An infant weighing 3,350 g, born at term after a normal


pregnancy and delivery, had a mass in the submandibu-
lar left region, the size of a walnut, in front of the sterno-
cleidomastoid muscle. On US the mass appeared to be
a characteristic lymphangioma, and therefore it was de-
cided not to treat the lesion but to follow its evolution.
Because the mass increased in size, hospitalization
was decided when the child was 3 months old.
On admission, the child was in good condition, with-
out any local or general anomalies except for the pres-
ence of the mass.
The left submandibular region was deformed by an
irregular ovoidal swelling with a maximum diameter of
8 cm, showing partly cystic and partly solid consistency.
Laboratory test results were normal. Fig. 1
After another US study, an intervention was decided
on (Fig. 1).
• What does Fig. 1 show?
• What was the diagnosis?
• What was found during the intervention and what
was performed?
• What was the definitive diagnosis?
• What was the follow-up?
24 Head and Neck

A 11

Fig. 2 Fig. 3

The US shows a mass with a complex structure charac- On the basis of the histologic examination, which
terized by multiple eccentric fluid areas around a central showed embryogenic tissues including mainly mature
solid irregular area. thyroid tissue, the definitive diagnosis was that of thy-
The diagnosis was cervical teratoma. roid teratoma (Fig. 3).
At surgery, the multicystic mass was reached via a The postoperative course was uneventful, and at fol-
transversal cut along its maximum diameter. The mass low-up after 2 years the boy was in good condition with-
was excised with some difficulties because it extended to out any relapse.
the back of the trachea reaching the paravertebral space.
The mass, measuring 6×4×2.5 cm, showed multiple
small cysts around a central solid area (Fig. 2).
1 Head and Neck 25

Q 12
Ugo De Luca

A 9-year-old boy had an asymptomatic, midline anterior


cervical mass initially interpreted by his parents as the
Adam’s apple (Figs. 1, 2). The mass was mobile during
swallowing, firm, and it had well-defined margins. The
growth had been slow and no infections occurred.
The patient underwent ultrasonography (Fig. 3),
which revealed a hypoechoic cystic mass very close to
the hyoid bone.
• What is the diagnosis?
• What is the differential diagnosis?
• What is the treatment? Fig. 1
• What complications can be expected?

Fig. 2

Fig. 3
26 Head and Neck

A 12

Fig. 4 Fig. 5

The midline cervical mass was a large thyroglossal duct


cyst (TDC). It did not become infected and slowly in-
creased in volume.
The differential diagnosis must include lymph node
hyperplasia and dermoid cyst.
The treatment of choice for TDC is surgery via the
Sistrunk procedure. Once the surgeon has isolated the
cyst through a transverse cervical incision, the hyoid
bone body (1–2 cm) must be resected together with the
cyst, and the dissection should be continued until the
foramen cecum at the base of the tongue (Fig. 4).
Accurate hemostasis must be achieved and closure in
layers realized. No drainage is necessary and the child
can be discharged on the same day of surgery. Fig. 6
Complications of unoperated TDC are recurrent in-
fections, fistulization (Figs. 5, 6) to the skin with conse-
quent retracting scar, and possible, although rare, can-
cerization. Suggested Reading
Complications of operated cases are postoperative 1. C. Boglino, A. Inserra, A. Silvano et al. La chirurgia del
infection and recurrence of the cyst, which is very high dotto tireoglosso in età pediatrica. Minerva Chir 1993;
if the Sistrunk procedure is not radical; the recurrence 48:393–402
rate is 4% in complete Sistrunk operations and rises to 2. Bratu, J.M. Laberge. Day Surgery for thyroglossal duct cyst
20% if several cyst infections precede the radical opera- excision : a safe alternative. Pediatr Surg Int 2004; 20:675–
tion. 678
Moreover, attention must be paid to the distal por- 3. J. Maddalozzo, T.K. Venkatesan, P. Gupta. Complications
tion (toward the pyramidal lobe of the thyroid gland) associated with the Sistrunk Procedure. Laryngoscope
2001; 111: 119–123
of the thyroglossal duct, if present; if this is ignored, it
could be responsible for relapse.
1 Head and Neck 27

Q 13
Nancy Rollins

A 13-month-old girl presented with a slow-growing, soft,


purplish mass involving the left buccal surface and lip
(Fig. 1). The lesion was seen at birth as a small bluish
region. The parents were told the lesion represented a
small hemangioma which would involute with time.
• What is the differential diagnosis?
• What is the best imaging strategy?
• Should the lesion be biopsied or resected?
• Is there a nonsurgical alternative for treatment?

Fig. 1
28 Head and Neck

A 13

The purplish lesion is a venous malformation. The pa-


tient underwent MR imaging using gadolinium-DTPA.
An axial short tau inversion recovery (STIR) sequence
(Fig. 2) and a post-contrast T1 spin echo (Fig. 3) se-
quence through the lesion were performed. The lesion is
well-circumscribed, nonlipomatous, with moderately in-
tense enhancement and no dilated feeding arteries. The
lesion represents a congenital venous malformation. The
patient underwent sclerotherapy with absolute alcohol Fig. 2 Fig. 3
repeated four times 4–6 weeks apart (Figs. 4, 5). Figure 6
shows the final result. Residual cutaneous discoloration
was treated with laser.
The venous malformation should not be confused
with a hemangioma. Attempts at surgical resection
would be disfiguring and unlikely to affect a complete
cure. Biopsy is contraindicated. The patient was therefore
referred for sclerotherapy. Prior to sclerotherapy, cross-
sectional imaging is mandatory and is best performed
with MR imaging. MR with contrast is needed to define
the composition of the lesion; i.e., venous, lymphatic, or Fig. 4 Fig. 5
mixed, as well as to assess the extent of the lesion and
the proximity to neurovascular structures. Lesions that
have a sharp abrupt transition from the surrounding tis-
sue respond better to sclerotherapy than do lesions with
ill-defined margins that cross tissue and fascial planes.
Sclerotherapy is the injection of aqueous or oleic solu-
tions into abnormally dilated vascular or lymphatic
channels inducing damage to the endothelial lining and
resulting in thrombosis, fibrosis, stenosis, and local scar-
ring. Residual cosmetic deformity due to formation of
scar tissue within the venous malformation is treated
with surgery. Blood loss is minimized by the sclerother-
apy and complete resection of the lesion is more readily
accomplished. For deeper venous malformations that
present with pain or functional impairment in the ab-
sence of cosmetic deformity, subsequent surgery is not Fig. 6
usually needed. Large multi-compartmental venous mal-
formations of the face and extremities often have phlebo-
liths, calcified blood clots that are diagnostic of venous
malformations and which are not seen in hemangiomas. nolamine oleate), absolute alcohol, and doxycycline.
These large lesions are usually not curable and treatment Bleomycin is a chemotherapeutic agent known to cause
in the form of sequential sclerotherapy is designed to pulmonary fibrosis and should be avoided for nonmalig-
decrease pain and promote full functionality. Venous nant disease in children. Absolute alcohol denudes the
malformations usually demonstrate significant growth endothelial cells upon direct contact with the vessel wall.
with the onset of puberty, especially in female subjects. Permanent damage to the vessel wall results in complete
There are multiple agents used for sclerotherapy in- obliteration of the vessel lumen, which prevents recanali-
cluding bleomycin, sodium tetradecyl, ethibloc (etha- zation. Sclerosing agents are usually mixed with nonionic
1 Head and Neck 29

contrast and the installation of the sclerosant should be respiratory depression, cardiac arrhythmias, seizures,
performed under fluoroscopic guidance to limit com- rhabdomyolysis, hypoglycemia, and death. The treated
plications resulting from extravasation of the sclerosing site undergoes marked often very painful swelling.
agent into the regional soft tissue or draining veins. Doxycycline is a tetracycline derivative commonly
Complications of sclerotherapy using absolute alco- used for malignant pleural effusions via direct intrapleu-
hol include skin necrosis, neuropathy, muscle atrophy, ral injection. Animal studies have shown that doxycycline
and cardiovascular collapse. Absolute alcohol is par- can induce a marked decrement in neural function when
ticularly dangerous, and installation of absolute alcohol applied to the subepineural layers of the sciatic nerve in
into venous structures should be performed with careful the rat, and phrenic nerve paralysis after intrapleural
monitoring of the patient’s vital signs by the anesthesi- installation of doxycycline has been reported. Caution
ologist. No more than 1.0 cc/kg ethanol should be given should be used therefore when a facial venous malfor-
during sclerotherapy. Reported complications include mation involves the distribution of the facial nerve.

Suggested Reading
1. Berenguer B, Burrows PE, Zurakowski D, Mulliken JB. 3. Goyal M, Causer PA, Armstrong D. Venous Vascular Mal-
Sclerotherapy of craniofacial venous malformations: com- formations in Pediatric Patients: Comparison of Results of
plications and results. Plast Reconstr Surg 1999; 104:1–11 Alcohol Sclerotherapy with Proposed MR Imaging Classifi-
2. Esterly NB. Cutaneous hemangiomas, vascular stains and cation. Radiology 2002 223:639–644
malformations, and associated syndromes. Curr Probl Pe- 4. Mason KP et al. Serum Ethanol Levels in Children and
diatr 1996; 26(1):3–39 Adults after Ethanol Embolization or Sclerotherapy for
Vascular Anomalies. Radiology 2000; 217:127–132
2
Thorax

Case 14–44
32 Thorax

Introduction

Pediatric surgical pathology in the chest can occur in pathology, air or fluid in the pleural space, and medi-
the heart, the lung, the mediastinum, or the chest wall. astinal masses. More recently, the use of computerized
Mechanisms of disease can range from congenital anom- tomography has provided additional information and
alies, infectious or inflammatory conditions, trauma, sensitivity over plain chest radiography, but at the ex-
neoplasia, or disorders of organ function. This section pense of significantly higher radiation dose, which must
addresses a wide variety of thoracic problems encoun- be kept in mind when using this modality. Ultrasound
tered by the pediatric surgeon, and gives examples of has had limited utility in the chest, because the air in
how these problems can be diagnosed and managed ef- the lungs interferes with the sound waves. However, for
fectively. evaluation of the heart, blood flow in vessels, and pleu-
The diagnosis of thoracic surgical problems starts ral fluid collections, ultrasound offers excellent imaging
with a careful history and physical examination. For without radiation. Magnetic resonance imaging also has
many pulmonary, tracheal, and mediastinal problems, the advantage of providing excellent images without ra-
attention to the subtleties of breathing patterns, stridor, diation, but it has been underutilized in the chest. This is
chest wall movement and symmetry, and auscultation an area for future development.
will permit the surgeon to narrow down the diagnostic More invasive diagnostic modalities include bron-
alternatives before any imaging is done. Similarly, it is choscopy, with or without bronchoalveolar lavage, im-
on the basis of the clinical evaluation of the patient that age-guided needle and core biopsies, and video-assisted
the clinician will be able to prioritize interventions. The thoracoscopic surgery, which of course is increasingly
“ABCs” (rapid evaluation of airway, breathing, and cir- being used for therapeutic purposes as well.
culation) are more important in thoracic diseases than Finally, the widespread use of prenatal diagnostic
in any other part of the body. techniques, ranging from the maternal alpha-fetal pro-
There are several levels of imaging commonly used tein test to sophisticated ultrasound, has resulted in
for the evaluation of thoracic disease. Chest radiography identification of many thoracic anomalies before birth.
is one of the oldest and most commonly used modalities, This has provided the opportunity for counseling, alter-
and will often reveal everything the surgeon needs to ing the location or mode of delivery, and in some cases
know. In many cases the chest radiograph may be more performing fetal interventions for life-threatening con-
sensitive than physical examination for identifying lung ditions.
2 Thorax 33

Q 14
Jürgen Schleef

Fig. 1

A 4-year-old girl had a history of recurrent pulmonary The doctor in the emergency department decided, on
infections from the age of 20 months, which were usu- the basis of the history provided by the mother, to ask
ally treated with antibiotics. for a CT scan.
The mother informed the pediatrician that the girl • What does Fig. 1 show?
had a prenatal diagnosis of a small lung malformation, • Why did the surgeon in the emergency department
but that a plain radiograph of the chest at the age of decide to perform a CT scan?
1 year was normal. No further investigations were per- • What condition is affecting this child?
formed. • How should this condition be treated?
The girl was brought to our hospital again with fever
and a cough. The white blood cell count was pathologic
at 22,000; the CRP value was 8.2.
34 Thorax

A 14

Figure 1 shows a CT scan of the thorax. In the inferior


part, a pulmonary malformation (congenital cystic ade-
nomatoid malformation; CCAM) is visible. The radiolo-
gist described this malformation and ruled out extrapul-
monary sequestration, since no separate blood supply
from the inferior aorta could be identified.
The history of the child is typical of a connatal condi-
tion (pulmonary malformation). In most of these cases,
a chest radiograph is not indicated to rule out a postna-
tal persisting malformation. A CT scan (or MR imaging)
is mandatory in every case.
CCAM is a condition that is frequently found at pre-
natal ultrasound examination. If this pathological condi-
tion is not treated, persistent pulmonary infections re- Fig. 2
sistant to antibiotic treatment are very frequent. In rare
cases, an associated malignancy has been described in
the literature.
The therapy is usually surgical. In extrapulmonary
sequestration, an embolization of the supplying vessel is
proposed by some authors as an alternative treatment.
In this case an atypical resection of the malformation
was performed by thoracoscopy. Figure 2 shows the tho-
racoscopic view, while Fig. 3 shows the postoperative CT
scan.

Fig. 3

Suggested Reading
1. Davenport M, Warne SA, Cacciaguerra S, Patel S, Gree- 3. Illanes S, Hunter A, Evans M, Cusick E, Soothill P. Prenatal
nough A, Nicolaides K. Current outcome of antenatally di- diagnosis of echogenic lung: evolution and outcome. Ultra-
agnosed cystic lung disease. J Pediatr Surg 2004; 39(4):549– sound Obstet Gynecol 2005; 26(2):145–9
56 4. Pai S, Eng HL, Lee SY, Hsiao CC, Huang WT, Huang SC
2. Gornall AS, Budd JL, Draper ES, Konje JC, Kurinczuk JJ. Rhabdomyosarcoma arising within congenital cystic ade-
Congenital cystic adenomatoid malformation: accuracy of nomatoid malformation. Pediatr Blood Cancer 2005; 45(6):
prenatal diagnosis, prevalence and outcome in a general 841–5
population. Prenat Diagn 2003; 15; 23(12):997–1002 5. Shanmugam G, MacArthur K, Pollock JC. Congenital lung
malformations–antenatal and postnatal evaluation and
management. Eur J Cardiothorac Surg 2005; 27(1):45–52
2 Thorax 35

Q 15
Jürgen Schleef

An 11-year-old girl was brought to our hospital with a


chest wall deformity. The mother reported that the girl
was growing normally, but the deformity, which had
been present since birth, was getting worse and the pec-
tus had become “deeper” in the last year.
The girl was very shy, avoided sport activities, and
told the surgeon that she sometimes felt short of breath.
The surgeon decided, on the basis of the history sup-
plied by the mother, to ask for a CT scan of the thorax,
an electrocardiogram (ECG), echocardiography, and a
pulmonary function test.
• What does Fig. 1 show?
• Why did the surgeon ask for these examinations?
• What pathological condition is affecting this child?
• How should this condition be treated?

Fig. 1
36 Thorax

A 15

Figure 1 is a picture of the chest of an 11-year-old girl.


The deformity is a typical pectus excavatum. Note the
retraction of the sternum.
Figure 2 is a CT slice of the thorax as requested by
the surgeon. It shows the typical configuration of a pec-
tus excavatum with a left side shift of the heart. The ante-
rior thoracic wall is retracted.
The surgeon asked for the examination to rule out
further associated diseases. In most cases, the heart or
the lungs are not affected.
The treatment of this condition can be surgical. Differ-
ent techniques are described in the literature. The most Fig. 2
popular technique is a thoracoscopic-assisted approach
with retrosternal bar implantation, as described by Don-
ald Nuss. Figure 3 shows the clinical result 2 years after
surgery on this patient.

Suggested Reading
1. Croitoru DP, Kelly RE Jr, Goretsky MJ, Gustin T, Keever
R, Nuss D. The minimally invasive Nuss technique for re-
current or failed pectus excavatum repair in 50 patients.
J Pediatr Surg 2005; 40(1):181–6; discussion 186–7
2. Goretsky MJ, Kelly RE Jr, Croitoru D, Nuss D. Chest wall
anomalies: pectus excavatum and pectus carinatum. Ado-
lesc Med Clin 2004; 15(3):455–71
3. Krasopoulos G, Dusmet M, Ladas G, Goldstraw P. Nuss
procedure improves the quality of life in young male adults
with pectus excavatum deformity. Eur J Cardiothorac Surg
2006 Jan; 29(1):1–5. Epub 2005 Dec 5. PMID: 16337131
[PubMed – indexed for MEDLINE]
4. Lawson ML, Mellins RB, Tabangin M, Kelly RE Jr, Croitoru
DP, Goretsky MJ, Nuss D. Impact of pectus excavatum on
pulmonary function before and after repair with the Nuss
procedure. J Pediatr Surg 2005; 40(1):174–80

Fig. 3
2 Thorax 37

Q 16
Nancy Rollins and Korgun Koral

A 2-day-old infant presented with respiratory distress.


• What does the chest radiograph show (Fig. 1)?
• What are the CT findings (Figs. 2, 3)?
• What is the differential diagnosis?
• What is the diagnosis?
• What are the types and prognostic implications?
• How is this condition treated?

Fig. 1

Fig. 2 Fig. 3
38 Thorax

A 16

There is a rightward shift of the mediastinal structures


with expansion of the left hemithorax. A small air cyst
can be appreciated in the left lung.
A shift of the mediastinum to the right is seen with a
small right lung. There are multiple small cysts in the left
lung with areas of solid tissue.
The differential diagnosis includes cystic congenital
adenomatoid malformation (CCAM), pulmonary se-
questration, congenital diaphragmatic hernia, and cavi-
tary necrosis complicating pneumonia.
The diagnosis is CCAM, type 2. Type 1 consists of
one or more large cysts. In type 2, numerous small cysts
of uniform size are present. Type 3 appears solid on im-
aging, but has microcysts. Different types of CCAM do
not have different clinical implications.
Treatment is surgical resection.

Suggested Reading
1. Donnelly LF. Pocket Radiologist: Pediatrics. Amirsys Ed.
2002.
2. Kuhn JP, Slovis TL, Haller JO. Caffey’s Pediatric Diagnostic
Imaging. Elsevier Ed. 2004.
2 Thorax 39

Q 17
Nancy Rollins and Korgun Koral

Fig. 1 Fig. 2

Fig. 3 Fig. 4

A 6-year-old boy presented with multiple episodes of left • What do the CT scans of the chest show (Figs. 3, 4)?
lower lobe pneumonia. • What is the differential diagnosis?
• What are the findings on the posteroanterior and lat- • What are the types of this abnormality?
eral chest radiographs (Figs. 1, 2)? • How is it treated?
40 Thorax

A 17

There is a well-defined air-space opacity in the posterior


segment of the left lower lobe. On the lateral chest ra-
diograph, the lucency of the lungs should increase from
the apices to bases; in this case the lung bases are more
opaque.
The contrast-enhanced chest CT shows an air-space
opacity in the posterior segment of the left lower lobe.
The surrounding lung parenchyma appears normal.
There is a large vessel within the air-space opacity; how-
ever, no direct communication with the descending
aorta can be displayed.
Pulmonary sequestration refers to a congenital area of
abnormal lung which does not connect to the bronchial
tree or pulmonary arteries. It is categorized as extralobar
and intralobar. It is not possible to make this distinction Fig. 5
on imaging. Extralobar sequestration has its own pleu-
ral covering, the intralobar type does not. Pulmonary
sequestration usually affects the lower lobe of the left
lung. The systemic artery entering into the sequestration
is characteristic (different patient; Figs. 5, 6; arrow). Se-
questration does not contain air, unless infected (Figs. 5,
6; Ao, aorta). Treatment is surgical resection.

Suggested Reading
1. Donnelly LF. Pocket Radiologist: Pediatrics. Amirsys Ed.
2002.
2. Siegel MJ, Coley BD. Pediatric Imaging. Lippincott Wil-
liams & Wilkins Ed. 2006.

Fig. 6
2 Thorax 41

Q 18
Abdellatif Nouri and Mongi Mekki

Fig. 1 Fig. 2

A 20-day-old full-term male infant at birth presented


with respiratory tachypnea with progressive apparition
of respiratory distress and cyanosis.
On physical examination, his temperature was 37°C,
breath sounds were diminished on the left hemithorax
with hyper-resonance on percussion and shift of the me-
diastinum.
An anteroposterior chest radiograph was obtained
(Fig. 1).
• What does Fig. 1 show?
• What are the three diagnoses that can be suggested?
The distress was not severe and a CT scan was performed
(Figs. 2, 3).
• What do Figs. 2 and 3 show?
• Which diagnosis do you retain? What are the two dif- Fig. 3
ferential diagnoses and how do you eliminate them?
• How do you manage this pathological condition?
42 Thorax

A 18

Fig. 4 Fig. 5

The chest radiograph (Fig. 1) shows increased translu- lobar emphysema by mucus plugging, granuloma or
cency of the left hemithorax with herniation of the ex- lymph node compression of the lobar bronchi. In this
panded lung across the midline to the right side. The case, corticotherapy may induce the patient’s recovery.
lower part of the left lung is markedly compressed. The This may explain spontaneous improvement of some lo-
heart and the trachea are displaced to the right and there bar emphysema cases. The second differential diagnosis
is a widening of the rib spaces and a depression of the is compensatory emphysema which counterbalances a
left diaphragm. A chest radiograph is usually sufficient ventilation defect of the other lobes. If the distress is not
for the diagnosis. severe, pulmonary perfusion scintigraphy can be help-
A left pneumothorax can be suspected, but it cannot ful. The perfusion of the emphysematous lobe is normal
explain the compression of the lower part of the left lung in the case of compensatory emphysema and very de-
and there are bronchovascular markings in the hyper- creased in the case of CLE. For our patient, pulmonary
lucent chest. The second diagnosis is a huge lung cyst scintigraphy (Fig. 4) showed the absence of perfusion of
distended by air, but normally it takes an oval shape and the left superior lobe and confirmed CLE.
is well delimited. The most likely diagnosis is lobar em- CLE presenting with distress in neonates requires
physema. lobectomy by thoracotomy or thoracoscopy. Figure 5
The CT scan (Fig. 2) confirms the emphysema of the shows an operative view of the emphysematous superior
left superior lobe with herniation of this hyperinflated left lobe.
lobe to the right side and a mediastinal shift. The right After a 4-year follow-up, the child was free of symp-
superior lobe is markedly collapsed (Fig. 3). toms and his chest radiograph was normal because of
The most likely diagnosis is congenital lobar emphy- the compensatory growth of the remaining lung.
sema (CLE). The first differential diagnosis is acquired

Suggested Reading
1. Koontz CS, Olivia V, Gow KW, Wulkan ML. Video assisted 3. Ozcelik U, Gocmen A, Kiper N, Dogru D, Dilber E, Yal-
thoracoscopic surgical excision of cystic lung disease in cin EG. Congenital lobar emphysema: evaluation and long
children. J Pediatr Surg 2005; 40:835–7 term follow up of thirty cases at a single center. Pediatr Pul-
2. Mei-Zahav M, Konen O, Manson D, Langer JC. Is con- monol 2003; 35:384–91
genital lobar emphysema a surgical disease? J Pediatr Surg
2006; 41:1058–61
2 Thorax 43

Q 19
Abdellatif Nouri and Mongi Mekki

Fig. 4

Fig. 1 Fig. 3

Fig. 2

A 2-month-old boy had a history of recurrent broncho- Medical treatment was prescribed without improvement.
pulmonary infection, cough, cyanosis, and choking on A barium meal study was performed (Figs. 3, 4).
feeding with alimentary vomiting since birth. • What do Figs. 3 and 4 show?
Apart from inspiratory crackles and marked abdomi- The medical treatment of this pathological condition
nal distension, the physical examination was normal. was reinforced with the prescription of proton pump in-
A thoracoabdominal radiograph (Fig. 1) was ob- hibitors without improvement of symptomatology.
tained. • What diagnosis should be advocated?
• What does Fig. 1 show? • How do you confirm this diagnosis?
• What is the second exploration that may be helpful
The pediatrician performed a 24-h pH-metry (Fig. 2). for the diagnosis and the treatment?
• What does Fig. 2 show? • How do you prepare this patient for treatment?
• What treatment do you recommend?
44 Thorax

A 19

The thoracoabdominal radiograph (Fig. 1) shows a right


pneumopathy with bronchial congestion and gaseous
distension of the bowel.
The pH-tracing (Fig. 2) shows clusters of numerous
episodes of gastroesophageal reflux (GER).
The barium meal study (Fig. 3) shows GER with
opacification of the airway by the contrast substance.
The history of cough, cyanosis, and choking on feed-
ing, in addition to the opacification of the airway dur-
ing the barium meal, allowed the diagnosis of tracheo-
esophageal fistula (TEF) to be made.
H-type TEF can be visualized by a tube injection
esophagogram performed with the patient in the prone
position after the introduction of the tube into the mid-
esophagus. Figure 4 shows an H-type TEF above the Fig. 5
third thoracic vertebra. Video recording is helpful be-
cause the fistula may open transiently. Recently, there
has been a great interest in the use of three-dimensional
CT for the diagnosis of TEF. The treatment of TEF is surgical, although there are
A tracheoscopy is the second helpful exploration for some reports of endoscopic techniques consisting in the
both diagnosis and treatment. It shows the opening of destruction of the mucosa of the fistula by diathery or
the fistula on the posterior wall of the trachea (Fig. 5) laser and its occlusion by injecting glue.
and specifies the distance between the fistula and the A cervicotomy is the best approach for all fistulae
carina. located above the third thoracic vertebra (70%). Below
It is important to look for any associated malforma- this level, the fistula opening is at the carina and a thora-
tions, which are observed in about 50% of cases and cotomy or a thoracoscopy is needed.
correspond to abnormalities of esophageal atresia. The This patient was treated by right cervicotomy. The
patient’s preparation includes medical treatment of GER, patient’s evolution was uneventful and the GER disap-
respiratory physiotherapy, and replacing oral feeding by peared after 13 months of medical treatment. At the 3-
nasogastric tube feeding or parenteral nutrition. year follow-up, the child was free of symptoms.

Suggested Reading
1. Allal H, Montes-Tapia F, Andina G, Bigorre M, Lopez M, 3. Le SD, Lam WW, Tam PK, Cheng W, Chan FL. H-type tra-
Galifer RB. Thoracoscopic repair of H-type tracheoesopha- cheooesophageal fistula: appearance of three dimensional
geal fistula in the newborn: a technical case report. J Pedi- computed tomography and virtual bronchoscopy. Pediatr
atr Surg 2004; 39:1568–70 Surg Int 2001; 17:642–3
2. Bhatnagar V, Lal R, Sriniwas M, Agarwala S, Mitra DK.
Endoscopic treatment of tracheooesophageal fistula using
electrocautery and the Nd YAG Laser. J Pediatr Surg 1999;
34:464–7
2 Thorax 45

Q 20
Abdellatif Nouri and Mongi Mekki

Fig. 1

Fig. 2

Fig. 3 Fig. 4

A 7-year-old girl without a significant previous medi- • What do the recumbent frontal (Fig. 1) and lateral
cal history presented with an epigastric and retrosternal (Fig. 2) chest radiographs show?
pain, which was noticed 1 month previously, and with • What do the thoracic and abdominal CT scans
recent fever. (Figs. 3, 4) show?
The physical examination revealed a temperature of • Which diagnoses can be suggested?
38°C, polypnea, and a decrease of the breath sounds at • What is the most helpful exploration for the diagno-
the low part of the left hemithorax, which was dull to sis and the surgical approach?
percussion. • How do you manage this pathological condition?
The pediatrician performed the examinations shown
in Figs. 1–4.
46 Thorax

A 20

The frontal and lateral chest radiographs (Figs. 1, 2)


show bibasilar densities silhouetting the right and left
borders of the heart and the left hemidiaphragm. These
densities are anterior to the heart and do not have the
typical appearance of pneumonia.
The thoracic CT scan (Fig. 3) shows a multilocular
cystic mass casting the anterior and lateral sides of the
heart. The septae of the cyst are vascularized and there is
no evidence of calcification. This cyst involves the infe-
rior and medium parts of the anterior mediastinum.
The abdominal CT scan (Fig. 4) shows an extension
of the cystic mass to the abdomen, probably through a
retrosternal defect. This cyst causes a dent on the ante- Fig. 5
rior side of the left liver.
A multilocular cyst of the anterior mediastinum may
correspond to a teratoma or a cystic lymphangioma.
The absence of calcification, ossification, and fat tissue
stands against teratoma.
The ultrasonographic evaluation of intrathoracic cys-
tic lymphangioma has been reported, but mediastinal
lesions are better evaluated by CT scan. At present, MR
imaging is the most specific exploration that permits
one to identify the infiltration by the lymphangioma of
the vascular, visceral, and neural structures.
Spontaneous regression of mediastinal lymphan-
gioma has been rarely observed. Rapid enlargement of
the lymphangioma, as a result of either hemorrhage or
infection, can lead to compression of vital structures Fig. 6
with life-threatening risks. Injection of sclerosing agents
or radiation therapy has little success and many poten-
tial complications. A complete surgical excision of the
lymphangioma is the best treatment.
In this case, after the resolution of fever and polyp-
nea by administering antibiotics for 10 days, laparos-
copy was performed. A multilocular cyst was discovered, Suggested Reading
probably corresponding to a multicystic lymphangioma 1. Alqahtani A, Nguyen LT, Flageole H, Shaw K, Laberge JM.
extending to the abdomen through a retrosternal hernia 25 years’ experience with lymphangiomas in children. J Pe-
(Fig. 5). Dissection through this defect allowed complete diatr Surg 1999; 34:1164–8
resection of the mediastinal cysts without any complica- 2. Jeung MY, Gasser B, Gangi A, Bogorin A, Charneau D, Wi-
tions (Fig. 6). The retrosternal defect was repaired with- hlm JM, Dietemann JL, Roy C. Imaging of cystic masses of
out any drainage. Pathological examination confirmed the mediastinum. Radiographics 2002; 22:S79–93
the diagnosis of cystic lymphangioma. 3. Wu MP, Wu RC, Lee JS, Yao WJ, Kuo PL. Spontaneous reso-
After a 15-month follow-up, the child was well and lution of fetal mediastinal cystic hygroma. Int J Gynaecol
Obstet 1995; 48:295–8
free of symptoms. CT scans showed no recurrence.
2 Thorax 47

Q 21
François Luks

Fig. 1 Fig. 2

A 14-year-old girl had been complaining of intermit-


tent epigastric and chest pain for more than 1 year. On
further questioning, she reported significant dysphagia
with any type of solid food.
She was otherwise healthy, had not lost weight, did
not report nausea or vomiting, and had an otherwise
normal medical and surgical history.
She was referred to a pediatric gastroenterologist,
who ordered an upper gastrointestinal contrast series
(Fig. 1).
Following this, CT of the thorax (Fig. 2) and upper
endoscopy (Fig. 3) were performed.
She was then referred to a pediatric surgeon, who Fig. 3
proposed operative treatment of this condition.
• What does the upper gastrointestinal (GI) series
(Fig. 1) show?
• What is the contribution of CT (Fig. 2) and endos-
copy (Fig. 3) to the diagnosis?
• What is the optimal treatment for this condition?
Which approaches can be offered?
• Is any other test necessary before proceeding with the
treatment?
48 Thorax

A 21

Symptoms in this adolescent are vague, and the differ-


ential diagnosis list is extensive. Dysphagia, epigastric
pain, and chest pain suggest esophageal pathology, but
can be manifestations of gastritis and peptic ulcer dis-
ease, in addition to gastroesophageal reflux, achalasia,
diffuse esophageal spasms, scleroderma, (chronically
lodged) esophageal foreign body, or Mallory-Weiss tears,
to name only a few.
The upper GI series (Fig. 1) clearly shows a filling de-
fect in the distal third of an otherwise normal appearing
esophagus. Not demonstrated in this image, but noted
on the full examination, is an absence of gastroesopha-
geal reflux and normal esophageal peristalsis.
An esophageal filling defect may represent an intrin-
sic esophageal mass, an intramural lesion, or an extrinsic Fig. 4
compression. A classical example of extrinsic compres-
sion of the esophagus is dysphagia lusoria caused by a
vascular ring. These filling defects are typically located in
the upper esophagus. Tumors of the esophagus are very
rare in children; leiomyomas and inflammatory pseu-
dotumors are usually submucosal, while the (extremely
rare) leiomyosarcoma may invade superficial layers.
Intramural lesions in children are most commonly du-
plication cysts, whereby the duplication has a separate
mucosa and submucosa, but shares a common muscu-
laris with the native esophagus. The endoscopic image
(Fig. 3) suggests an intramural or extrinsic process, as
the mucosa is pushed in, but is not involved by the pro-
cess. The CT scan (Fig. 4) is less useful, but demonstrates Fig. 5
compression of the esophageal lumen to the right, and a
mass effect that appears to be round and cystic.
With a diagnosis of esophageal duplication, resection Suggested Reading
of the lesion was offered. A left thoracoscopic approach 1. Accadia M, Ascione L, De Michele M, et al. Esophageal
was used, although thoracotomy has traditionally been duplication cyst: a challenging diagnosis of a paracardiac
the approach of choice. In general, a left thoracic ap- mass. Echocardiography. 2004 Aug; 21(6):551–4
proach is preferable for the distal third of the esopha- 2. Cheynel N, Rat P, Couailler JF, et al. Tubular duplication of
gus, whereas the middle esophagus is best approached the esophagus. Contribution of magnetic resonance imag-
through the right thorax. ing in anatomical analysis before surgery. Surg Radiol Anat
Excision of the duplication cyst was performed after 2000; 22(5–6):289–91
opening the mediastinal pleura (Fig. 5). Dissection of 3. Herbella FA, Tedesco P, Muthusamy R, Patti MG. Thora-
coscopic resection of esophageal duplication cysts. Dis
the cyst off the native esophagus was greatly facilitated
Esophagus 2006; 19(2):132–4
by concomitant esophageal endoscopy, as the common
4. Joyce AM, Zhang PJ, Kochman ML. Complete endoscopic
wall between the two is very thin. Following removal of
resection of an esophageal duplication cyst (with video).
the cyst, the esophageal muscularis was closed with ab-
Gastrointest Endosc 2006 Aug; 64(2):288–9
sorbable sutures.
5. Michel JL, Revillon Y, Montupet P, et al. Thoracoscopic
The patient was discharged 4 days after surgery and treatment of mediastinal cysts in children. J Pediatr Surg
has remained symptom-free since then. 1998 Dec; 33(12):1745–8
2 Thorax 49

Q 22
François Luks

Fig. 1 Fig. 2

At 21 weeks of gestation, a fetus was found to have a On subsequent ultrasound examinations, at 25, 28, and
right-sided chest lesion on routine ultrasonography. 32 weeks, the lesion became less obvious, although its
The mother was referred for level-2 ultrasonography size remained stable. The previously noted mediastinal
(detailed examination) and MR imaging of the fetus shift (toward the left) gradually resolved.
(Fig. 1). At birth (38 weeks’ gestation), the infant had no re-
• What is the differential diagnosis of a chest lesion in a spiratory distress and had normal APGAR scores. A
fetus? chest radiograph within hours of delivery was essentially
• What are the recommendations to the expecting cou- unremarkable.
ple? At 3 weeks of life, the infant developed increasing
• Which antenatal interventions, if any, might be con- dyspnea and tachypnea. A chest radiograph was ob-
sidered? tained (Fig. 2). The patient underwent an urgent surgical
• What is the predicted outcome of a chest lesion in the procedure.
fetus? • What does the radiograph in Fig. 2 demonstrate?
• What is the diagnosis, based on this radiograph and
an antenatal history of a right-sided chest lesion?
50 Thorax

A 22

Fig. 3 Fig. 4

Fetal chest lesions represent a variety of conditions of unilocular cysts can be drained (double pigtail catheter).
seemingly different origins. However, they have more in Solid or complex lesions may require open fetal surgery.
common than previously realized. The differential diagno- However, most lesions, even large ones, will start to re-
sis of a congenital cystic lung lesion is bronchogenic cyst, gress around 26–28 weeks, and may hardly be detectable
intralobar and extralobar pulmonary sequestration, and at birth. In CLE, spontaneous resolution of the bronchial
congenital cystic adenomatoid malformation (CCAM). obstruction and drainage of lung fluid may lead to reso-
In addition, congenital diaphragmatic hernia may some- lution of the ultrasound findings (as was the case here),
times be mistaken for a primary lesion of the chest. but the weakness of the bronchi and bronchioles persists,
Congenital lobar emphysema (CLE) is not strictly leading to a risk of air trapping and sudden respiratory
speaking a cystic lesion of the lung, but is often part of distress after birth.
the differential diagnosis. This condition usually occurs Although the infant was asymptomatic at birth, pro-
in the right middle lobe, presumably as a result of a tem- gressive air trapping in the right middle lobe led to
porary and/or incomplete occlusion of the right middle respiratory distress at 3 weeks of life. The repeat chest
lobe bronchus. As a result, fluid trapping distal to the radiograph (Fig. 2) shows a dramatically overinflated
stenosis or obstruction causes dilatation and attenuation right lung and significant shift of the mediastinum to
of the distal bronchi and an emphysematous appearance the left.
of the affected lobe. In the present case, the MR image This confirmed the diagnosis of CLE, and the patient
clearly shows an enlarged right middle lobe, with pres- underwent a right thoracotomy and resection of a se-
ervation of the lobar vascular anatomy. This, therefore, is verely enlarged and emphysematous right middle lobe
congenital lobar emphysema. (Figs. 3, 4). Figure 3 shows the protrusion through the
Most congenital cystic lung lesions do not cause fetal wound of an extremely hyperinflated right middle lobe.
distress. If they become very large, they may cause medi- Figure 4 shows the emphysematous and floppy disten-
astinal shift and hydrops, presumably from impaired sys- sion of the right middle lobe.
temic venous return. In utero intervention should only The patient recovered well, was discharged on post-
be considered in cases of (impending) hydrops. Large, operative day 5, and has been well since then.

Suggested Reading
1. Truitt AK, Carr SR, Cassese J, et al. Perinatal management 2. Tander B, Yalcin M, Yilmaz B, et al. Congenital lobar em-
of congenital cystic lung lesions in the age of minimally in- physema: a clinicopathologic evaluation of 14 cases. Eur J
vasive surgery. J Pediatr Surg 2006 May; 41(5):893–6 Pediatr Surg 2003 Apr; 13(2):108–11
3. Williams HJ, Johnson KJ. Imaging of congenital cystic lung
lesions. Paediatr Respir Rev 2002 Jun; 3(2):120–7
2 Thorax 51

Q 23
Mario Lima and Giovanni Ruggeri

Fig. 1

A malnourished 2 year-old boy, coming from a develop- • What is the diagnosis?


ing country (Mali, Africa), presented with serious dys- • What should be done in similar cases to reach a diag-
phagia. A barium swallow was performed (Fig. 1). nosis?
• What does Fig. 1 show? • What is the treatment?
• What other examinations could be helpful for the di- • What did the surgeon do in this particular case?
agnosis? • What is the outcome of the native esophagus?
52 Thorax

A 23

Fig. 2 Fig. 3

Fig. 4 Fig. 5

The young boy had accidentally ingested some house- stomach may also be examined. A contrast esophago-
hold cleaner he had found in a bottle. gram (with water-soluble contrast) should be obtained
The barium swallow shows a dilation of the proximal after 10–14 days; it shows the nature of the esophageal
part of the esophagus and a considerable narrowing of motility, which has predictive value for severe injury and
the medial part, making it difficult for the contrast me- extensive stricture formation. The study can be repeated
dium to reach the stomach. over time to gain more information.
The surgeon performed an esophagoscopy (Fig. 2): it The goal of treatment is to avoid stricture formation
shows esophagitis caused by the caustic ingestion, which and prevent sepsis. Patients are treated with systemic an-
has caused stenosis of the medial third of the esophagus. tibiotics for 7–14 days; prednisone (2–4 mg/kg per day)
Caustic ingestion is frequent in children under is often administered for 3–6 weeks. An oral liquid diet
5 years of age; 90% of cases are a result of alkali burns followed by soft solids after 2–3 days may be adminis-
and 10% are related to acid ingestion. The most com- tered. In cases of severe injury, it is best to insert a feed-
mon agents involved are potassium hydroxide, sodium ing tube both for enteral nutrition and to maintain ac-
hydroxide, and sulfuric acid. Solid granular forms of cess to the esophagus for later bougienage. Concomitant
caustic cleansers are usually irritating and more difficult use of acid-secreting inhibitors is widespread but their
to swallow than liquid forms, and therefore severe inju- efficacy has not been proven.
ries are usually associated with ingestion of liquid agents. Generally, an aggressive and frequent cycle of dila-
With all substances, the volume ingested and the time tions allows for recovery of the native esophagus. In our
the mucosa is exposed to the agent determine the extent case, this was not possible, and therefore an esophageal
and depth of the injury. replacement was necessary. In our department, we pre-
The management of corrosive injuries is well codified: fer to use the colon.
esophagoscopy is mandatory whenever caustic ingestion The esophageal remnant is at high long-term risk
is suspected, because it determines whether the esopha- for neoplastic degeneration, and so we proceeded with
gus has been injured. It should be performed within the esophagectomy, which was performed thoracoscopically
first 24–48 h for diagnosis to be made. With a flexible (Figs. 3–5). The child returned to his country and was
scope, the entire esophagus may be visualized and the reported to be doing well.
2 Thorax 53

Q 24
Mario Lima and Giovanni Ruggeri

Fig. 1 Fig. 2 Fig. 3

A 7-day-old newborn was brought to our department The surgeon then decided to obtain an esophagogram
with respiratory distress, dysphagia, excessive weight (see Fig. 2).
loss, sialorrhea, and frequent regurgitations. • Why did the surgeon order an esophagogram (Fig. 2)?
A chest radiograph was acquired (Fig. 1). What does it show?
• What do you see in Fig. 1? • What did the surgeon do in this case?
• Which examination would you perform next? What • What is the follow-up of this patient?
does it show?
• What is the differential diagnosis?
54 Thorax

A 24

The newborn was affected by an esophageal duplication.


These duplications are found in the posterior mediasti-
num and consist of a smooth muscle-walled structure
containing mucosal epithelium including gastric hetero-
topic tissue. The fluid content is ordinarily clear, color-
less, and thinly mucoid. Most duplications are 2–4 cm,
round, and cystic and can reach huge sizes that may fill
the entire thorax.
The chest radiograph shows a large cystic mass, with
a clean and regular outline, which occupies a big part of
the right hemithorax.
MR imaging was performed. This examination is
important so as to also evaluate potential communica-
tion with the neural canal. It shows a 4-cm mass in the
posterior mediastinum, adjacent to the esophagus, with
compressive effect on the right bronchial system (Fig. 3,
red circle); another 3-cm mass is visible in the right sub-
diaphragmatic space (yellow circle). Both masses have
a fluid content. It is not uncommon (10%–15%) for a
patient with a mediastinal enteric cyst to have an intra-
abdominal intestinal duplication.
In the differential diagnosis of mediastinal masses, Fig. 4
many pathologies must be considered. They can be di-
vided into two groups: mediastinal cysts and tumors.
The former includes thymic, enteric, pericardial, der- Suggested Reading
moid and bronchogenic cysts and cystic hygromas; the 1. Bratu I et al. Foregut duplications: is there an advantage to
latter includes lymphomas (Hodgkin and non-Hodgkin), thoracoscopic resection? J Pediatr Surg 2005; 40:138–41
neurogenic tumors (ganglioneuroma, neurofibroma, 2. Michel JL et al. Thoracoscopic treatment of mediastinal
neuroblastoma), thymomas, teratomas, and other rare cysts in children. J Pediatr Surg 1998; 33:1745–8
tumors (hemangiomas, lipomas, rhabdomyosarcomas). 3. Soares-Oliveira M et al. Intestinal duplications. A survey of
In 20% of the cases the cyst may communicate with 18 cases. An Esp Pediatr 2002; 56:430–3
the esophagus. This is why the surgeon decided to per-
form the barium swallow (Fig. 2). In our case there was
no communication with the esophagus, which was, in-
stead, compressed, as can be seen by the narrowing in
the medial part.
The esophageal duplication was excised thoracoscop-
ically, while the subdiaphragmatic one, which proved to
be a jejunal duplication, was removed laparoscopically.
One month after surgery, another barium swal-
low was performed (see Fig. 4), which showed normal
esophageal canalization and gastroesophageal reflux,
not related to surgery; for this reason, the young girl un-
dergoes periodic endoscopic checkups.
2 Thorax 55

Q 25
Mario Lima and Giovanni Ruggeri

Fig. 1 Fig. 2 Fig. 3

Fig. 4 Fig. 5

A 5-year-old boy was brought to our department suffer- The surgeon subsequently decided to perform esopha-
ing from progressive dysphagia of recent onset. goscopy (see Fig. 5).
A barium swallow was performed (see Figs. 1, 2). • Why did the surgeon decide to perform esophagos-
• What do you see in Figs. 1 and 2? copy? What does it show?
• What did the surgeon do in this case?
More examinations were performed to reach a diagnosis
(see Figs. 3, 4).
• What do Figs. 3 and 4 show?
• What is the diagnosis?
56 Thorax

A 25

The boy was affected by an esophageal duplication.


The barium swallow (Figs. 1, 2) study shows a pos-
teriorly displaced esophagus, with an ab extrinsic com-
pression in the distal part, along the anterior profile.
MR imaging (Fig. 3) was performed as a first-line
examination: it shows a large expansive formation, with
round margins, 3 cm in diameter, in the medium me-
diastinum. Since the differential diagnosis included a
bronchogenic cyst, a CT scan was performed (Fig. 4):
the lesion is strictly adherent to the esophagus, without
a clear cleavage. In some points it seems like the wall of
the esophagus and that of the cyst are in continuity but
without a real communication between the two. No rela-
tionships with the respiratory tree are seen.
The main differential diagnosis is between a bron- Fig. 6
chogenic cyst and an esophageal duplication, the latter
seeming the most likely disease.
If esophageal duplication is suspected, it is useful to
know preoperatively whether a communication between
the cyst and the esophagus is present; which is why the
surgeon decided to perform esophagoscopy (Fig. 5). No
communication with the esophagus was seen in this case,
but a bulge was visible along the esophageal wall, caused
by the compression of the intestinal duplication.
The surgeon decided to excise the lesion thoraco-
scopically (Fig. 6). Since the duplication was very large,
a mini-thoracotomy was necessary (Fig. 7). The fluid
content of the cyst was clear, colorless, and thinly mu-
coid. Some reinforcement stitches were made along the
esophageal wall (Fig. 8).
At the time of writing, the boy was well and feeding Fig. 7
without any problems.

Suggested Reading
1. Bratu I et al. Foregut duplications: is there an advantage to
thoracoscopic resection? J Pediatr Surg 2005; 40:138–41
2. Michel JL et al. Thoracoscopic treatment of mediastinal
cysts in children. J Pediatr Surg 1998; 33:1745–8
3. Soares-Oliveira M et al. Intestinal duplications. A survey of
18 cases. An Esp Pediatr 2002; 56:430–3
Fig. 8
2 Thorax 57

Q 26
Jacob C. Langer and Priscilla Chiu

Fig. 1 Fig. 2

A 1-year-old boy presented to the local emergency room


with cough and fever. His history was otherwise unre-
markable and he had no significant birth history.
On clinical examination, the child appeared very well
other than having the cough. He had no signs of respira-
tory distress, displayed no evidence of dyspnea, and his
oxygen saturation on room air was normal.
On auscultation, he had diminished air entry to his
right chest compared to his left chest. There were no
wheezes or crackles. His heart sounds were difficult to
auscultate in the right chest. Fig. 3
The child was sent for a chest radiograph, shown in
Figs. 1 and 2.
• What are the findings in Figs. 1 and 2?
The surgical team was contacted and a chest CT scan
was ordered, shown in Figs. 3 and 4.
• What does the CT scan show?
• What is the differential diagnosis for this lesion based
on the features identified on the CT scan?
• What other investigations would you order for this
child?
• What is your management plan?
• How would you surgically approach this lesion? Fig. 4
58 Thorax

A 26

Fig. 5 Fig. 6

The chest radiograph clearly shows a large, anterior me- phatic malformation, and teratoma. The most likely di-
diastinal lesion extending into the right chest causing agnosis is teratoma based on the CT scan findings.
marked left-sided displacement of the heart and atelec- Teratomas are tumors containing at least two of three
tasis at the right lung base. germ cell layers. Malignant teratomas include immature
The cough may be secondary to superimposed airway teratoma and germ cell tumors that express tumor mark-
inflammation unrelated to this lesion or the lesion may ers (α-feto-protein, β-human choriogonadotropin) and
be compressing the phrenic nerve. should be assessed preoperatively with a broad radiolog-
The minimal symptoms may be explained by the ical metastatic survey. The adequacy of cardiac function
gradual increase in the size of the lesion, despite its large should be assessed preoperatively by echocardiogram.
size. Postoperative surveillance of malignant tumors includes
Chest radiography is a very effective screening test chest radiographs and serum tumor markers.
for chest lesions and should be the first test of choice. This patient was negative for serum tumor markers
However, a chest radiograph may fail to detect a central and his echocardiogram showed good function.
mediastinal lesion or one that overlaps the cardiac sil- The definitive management of anterior mediastinal
houette. teratoma is primary resection. After the patient’s fever
CT can distinguish solid from cystic lesions as well resolved, the tumor was resected through a median ster-
as the relationship to adjacent structures with three-di- notomy (Fig. 5). The pathology report indicated a be-
mensional reconstruction software (Fig. 4). nign, mature teratoma. Postoperative CT showed return
The boy’s CT scan showed a heterogeneous solid me- of the mediastinal structures to normal position and no
diastinal mass containing fat and calcifications. recurrence of tumor (Fig. 6).
The differential diagnosis for an anterior mediastinal
mass includes thymic cyst, thymoma, lymphoma, lym-

Suggested Reading
1. Billmire D, Vincouir C, Rescorla F, Rescorla F, Colombani 2. Borecky N, Gudinchet F, Laurini R, Duvoisin B, Hohlfeld J,
P, Cushing B, Hawkins E, London WB, Giller R, Lauer S. Schnyder P. Imaging of cervico-thoracic lymphangiomas in
Malignant mediastinal germ cell tumors: an Intergroup children. Pediatr Radiol 1995; 25:127–130
study. J Pediatr Surg 2001; 36:18–24 3. Bower R, Kiesewetter W. Mediastinal masses in infants and
children. Arch Surg 1977; 112:1003–1009
2 Thorax 59

Q 27
Jacob C. Langer and Priscilla Chiu

A 9-year-old boy presented to the local emergency room


with left-sided chest pain and shortness of breath. His
past medical history was significant for asthma, for
which he received inhaled salbutamol treatment. He had
not had any recent history of trauma.
On physical examination, his vital signs were stable
and his oxygen saturation on room air was 90%. There
was a shift of the trachea toward the patient’s right side
and there was distension of the neck veins.
On auscultation, there was minimal air entry in his
left chest. Air entry as well as heart sounds were audible
in his right chest.
His initial chest radiograph in the emergency room
is shown in Fig. 1.
• What are the significant findings shown in Fig. 1?
• What is your diagnosis?
• What are the immediate management issues for this
patient?
Fig. 1
It was revealed that this was the second episode for this
patient and the surgical team was consulted.
A chest CT scan was then ordered.
• What findings are demonstrated in this patient’s CT
scan, as seen in Fig. 2?
• What are the options for definitive management of
this condition?
• What associated disorders increase the risk of devel-
oping this condition?

Fig. 2
60 Thorax

A 27
This patient had a significant left tension pneumothorax
as indicated by the findings of tracheal deviation, dis-
tended neck veins, and shortness of breath. The decrease
or absence of breath sounds in the left chest distinguishes
tension pneumothorax from other causes of acute he-
modynamic compromise and respiratory distress.
The differential diagnosis for distended neck veins and
shortness of breath includes tension hemothorax, peri-
cardial tamponade, and massive pulmonary embolism.
The immediate management of a tension pneumo-
thorax includes application of supplemental oxygen and
needle decompression by placing a large bore needle at
the second intercostal space in the mid-clavicular line.
The definitive management is tube thoracostomy. An
appropriately sized chest tube (Table 1) is placed in the
fifth intercostal space in the anterior axillary line. The tube
is connected to an underwater seal chamber with suction
applied. In girls, care should be taken to avoid placing
the tube through breast tissue as this can lead to abnor- Fig. 3
mal breast development and disfigurement in the future.
In small children, the chest tube should be positioned
properly and not curled up in the chest cavity, a situation
that can lead to obstruction of the chest tube and re-ac-
cumulation of a tension pneumothorax (Fig. 3).
The chest tube should be removed once the pneu-
mothorax is completely evacuated, with no recurrence
when the tube is taken off suction, and no further air
leaks are detected with maximal inspiratory effort.
Recurrent pneumothoraces in the ipsilateral side sug-
gest underlying pulmonary pathology such as blebs and
bullous lung cysts. A second episode of spontaneous
pneumothorax warrants further investigation following
acute management.
The gold standard for the investigation of recurrent Fig. 4
pneumothorax is CT scan to detect underlying cystic
lung lesions, typically in the apical segments of the lobes.
This imaging should be performed when the pneumo- Table 1 Suggested chest tube size by patient weight
thorax has completely resolved, as collapsed lung tissue Weight (kg) Chest tube size (Fr)
may mask the presence of small cysts. 3–5 10–12
The presence of lung cysts combined with patient fea- 6–9 12–16
tures such as tall stature and a family history of pneumo-
10–11 16–20
thorax or vascular disease suggest possible underlying
connective tissue disorders such as Marfan’s syndrome 12–14 20–22
or Ehlers-Danlos syndrome. 15–18 22–24
Surgical management of lung blebs or cysts in- 19–22 24–28
volves thoracoscopic or open apical bullectomy (Fig. 4). 23–30 24–32
Pleurodesis should be used as part of the operation to
>30 28–40
obliterate the potential space and prevent future pneu-
mothoraces.
2 Thorax 61

Q 28
Jacob C. Langer and Priscilla Chiu

Fig. 1 Fig. 2

A 16-year-old boy presented to the local emergency Serial chest radiographs were acquired. Figure 2
room with a 2-week history of cough, fever, and progres- shows the chest radiographs of this patient following the
sive neurological deficits, requiring emergency intuba- development of subcutaneous emphysema.
tion for respiratory failure. The intubation was techni- • What does Fig. 2 show?
cally difficult and several attempts were made which • What is the differential diagnosis?
resulted in esophageal intubation rather than endotra- • What other investigations would you order for this
cheal intubation. patient?
He was admitted to the intensive care unit. High-dose
steroids and antibiotics were started for this patient’s The surgical team was contacted after a significant in-
pneumonia and acute demyelinating disease. His initial crease in subcutaneous emphysema.
chest radiograph is shown in Fig. 1. • What is your management plan?
Shortly after intubation, subcutaneous emphysema • What are the complications associated with this con-
was noted during the bedside clinical examination. On dition?
auscultation, he had fine crackles with air entry to his
chest. However, he did not experience any significant
deterioration in ventilatory status.
62 Thorax

A 28

This patient developed pneumomediastinum and subcu-


taneous emphysema following traumatic endotracheal
intubation and mechanical ventilation.
Most commonly, pneumomediastinum is secondary
to pneumothorax with leakage of intrapleural air into
the mediastinum. In the absence of associated pneumo-
thorax, pneumomediastinum most commonly results
from traumatic injury to the mediastinal aerodigestive
tract (i.e., esophagus, trachea, main stem, and proximal
bronchi).
Esophageal perforation may be associated with signs
such as mediastinitis (fever, pain, respiratory distress)
and pleural effusion. In small children, foreign body in-
gestion with esophageal perforation must be considered
in the differential diagnosis.
Fig. 3
The treatment of small esophageal perforation is anti-
biotic administration and drainage of any para-esopha-
geal fluid collections. Large esophageal perforations
should be managed by early and aggressive surgical re-
pair, as mediastinitis can lead to fatal sepsis.
Airway injury represents the second most common
cause of isolated pneumomediastinum. For this patient,
additional factors such as mechanical ventilation, un-
derlying infection, and steroid use may predispose to
airway injury.
Treatment of airway perforation may be difficult, es-
pecially among the mechanically ventilated patient pop-
ulation. Minimizing airway pressures, treatment with
antibiotics for proven infection, and tapering of steroid
dosage facilitate spontaneous closure of small airway
perforations.
Management of traumatic airway injuries requires
early control of the airway with endotracheal intubation
beyond the level of injury if possible (e.g., endobronchial
intubation). Surgical repair is indicated if the air leak is
Fig. 4
uncontrolled with endotracheal intubation or fails to re-
solve with conservative treatment.
Investigations for isolated pneumomediastinum
should include an upper gastrointestinal contrast study detection of a pneumothorax on plain film radiographs
or esophagoscopy and bronchoscopy to identify the site may be difficult in the setting of extensive subcutaneous
of injury. emphysema.
This patient’s contrast study (Fig. 3) was normal, but Options for surgical repair of airway injuries include
his bronchoscopy revealed a small ulcer at the proximal primary repair, patch repair using pericardial or pleural
right mainstem bronchus below the carina, and his CT autograft, resection and primary anastomosis, and lo-
scan demonstrated impressive pneumomediastinum bectomy (for bronchial injuries). Complications include
(Figs. 4). The CT scan also confirmed the absence of any persistent air leak or bronchopleural fistula, airway
associated pneumothoraces; this is important, since the granulation tissue, and stenosis.
2 Thorax 63

Q 29
Jacob C. Langer and Priscilla Chiu

Fig. 2

Fig. 1

Fig. 3

A 9-month-old boy was taken to the family doctor after The child was sent for a chest radiograph, shown in
his mother noticed progressively noisy breathing. The Fig. 1.
child was otherwise active and alert. • What does Fig. 1 show?
The patient was afebrile and had no significant past
medical history. This child came to our attention after chest radiography
The family physician found that the child had sig- was performed. The surgeon requested additional exam-
nificant indrawing with his respirations, but his oxygen inations, shown in Figs. 2 and 3.
saturations were normal on room air and there was air • What do Figs. 2 and 3 show?
entry bilaterally. There were no crackles on auscultation. • What is the differential diagnosis for this lesion?
The remainder of his physical examination was nor- • How would you manage this child?
mal. On further questioning, the patient’s mother also
reported progressive feeding intolerance with vomiting
shortly after feeding.
64 Thorax

A 29

This child showed signs of respiratory distress suggesting


airway obstruction. His chest radiograph showed a mass
in the upper mediastinum with deviation and compres-
sion of the trachea.
Mediastinal lesions in infants and children are best
imaged by CT scan. CT imaging can determine whether
the lesion is solid or cystic and the location and extent of
the lesion in relation to adjacent structures.
This infant’s CT scan of the chest clearly showed a
large upper and posterior mediastinal cystic lesion caus-
ing compression and rightward deviation of the trachea
with near complete obliteration of the esophageal lumen.
There was no cystic extension into the spine.
The differential diagnosis includes foregut duplica-
tion cyst, mediastinal abscess, and lymphatic malforma-
tion. The CT scan showed that the cyst was localized to
the mediastinum without septations or evidence of ex- Fig. 4
tension into the neck, suggesting it was unlikely to be a
cervicomediastinal lymphatic malformation.
Foregut duplication cysts are malformations of fore-
gut development that result in cystic enteric (intestinal) notochord, representing a different embryological deri-
or bronchial remnants within the mediastinum. There vation from foregut duplications. Inflammation or cyst
may be multiple duplications along the intestinal tract, enlargement can result in spinal cord compression and
as duplication cysts can arise in any part of the bowel. paraplegia. The presence of neurological deficits man-
Children with foregut duplication cysts may have associ- dates emergency laminectomy to decompress the spinal
ated VACTERL anomalies: vertebral, anal, cardiac, tra- cord.
cheoesophageal, renal and limb. Large foregut duplication cysts can cause airway or
Foregut duplication cysts may be spherical or tubular esophageal obstruction and are sometimes identified
in shape. The cysts may communicate with the foregut lu- prenatally by antenatal ultrasound. The perinatal man-
men or may be completely separated by the cyst wall. The agement of such large cysts may warrant immediate con-
cysts may harbor ectopic tissues, including gastric mu- trol of the airway at birth.
cosa, which may result in acid production, ulcer forma- The definitive management of foregut duplication
tion, and bleeding. Occasionally, these cysts can become cysts is surgical resection.
infected, fistulize to adjacent structures, or rupture. This child underwent thoracoscopic resection of the
In contrast, neurenteric cysts are foregut duplications lesion (Fig. 4). The cyst contained mucus, which was as-
with spinal extension thought to arise from the dorsal pirated before the cyst was removed from the chest.

Suggested Reading
1. Azzie G, Beasley S. Diagnosis and treatment of foregut du- 3. Carachi R, Azmy A. Foregut duplications. Pediatr Surg Int
plications. Semin Pediatr Surg 2003; 12:46–54 2002; 18:371–4
2. Bratu I, Laberge JM, Glageole H, Bouchard S. Foregut du- 4. Superina R, Ein S, Humphreys R. Cystic duplications of
plications: is there an advantage to thoracoscopic resection? the esophagus and neurenteric cysts. J Pediatr Surg 1984;
J Pediatr Surg 2005; 40:138–41 19:527–530
2 Thorax 65

Q 30
Giovanni Esposito and Ciro Esposito

Fig. 1

A 6-year-old boy without any anamnestic findings of • What do the thoracic radiographs show?
disease suffered thoracic trauma after a car accident. He • Which other procedures should be performed to
presented with severe respiratory distress and significant reach the diagnosis?
dyspnea. On auscultation, hyperphonesis of the right • What was the diagnosis?
hemithorax was found, and therefore chest radiography • What was the treatment?
was performed (Fig. 1). After this, surgery was carried • What was the follow-up?
out.
66 Thorax

A 30

The chest radiographs show a severe right pneumotho-


rax with a collapsed ipsilateral lung.
An explorative puncture was performed at the level
of the fifth intercostal space along the mid-axillary line,
from which 20 ml of blood came out.
Based on the clinical and radiological features, the di-
agnosis of hemothorax due to a lung rupture was made.
The treatment consisted in introducing a Petzer tube
in the pleural space by minimal incision, connected with
an aspiration system (Fig. 2).
During the follow-up, the radiograph of the thorax
revealed complete lung expansion; the drainage tube
was therefore removed 10 days later and the child was Fig. 2
discharged (Fig. 3).

Fig. 3

Suggested Reading
1. Adegboye VO, Ladipo JK, Adebo OA, Brimmo AI. Dia- 4. Soundappan SV, Holland AJ, Cass DT, Farrow GB. Blunt
phragmatic injuries. Afr J Med Med Sci 2002 Jun; 31(2): traumatic diaphragmatic injuries in children. Injury 2005
149–53 Jan; 36(1):51–4
2. Karnak I, Senocak ME, Tanyel FC, Buyukpamukcu N. Dia- 5. Voeller GR, Reisser JR, Fabian TC, Kudsk K, Mangiante
phragmatic injuries in childhood. Surg Today 2001; 31(1): EC. Blunt diaphragm injuries. A five-year experience. Am
5–11 Surg 1990 Jan; 56(1):28–31
3. Shehata SM, Shabaan BS. Diaphragmatic injuries in chil-
dren after blunt abdominal trauma. J Pediatr Surg 2006
Oct; 41(10):1727–31
2 Thorax 67

Q 31
Felix Schier

Fig. 1 Fig. 2

Fig. 3

A 2-year-old boy presented to our department suffering


from a recurrent cough. Two episodes of pneumonia
had been recorded. There was a slight failure to thrive.
All laboratory values were normal. On auscultation there Fig. 4
were slightly reduced breath sounds on the right side.
The patient underwent imaging, shown in Figs. 1–4.
• What is the most likely diagnosis?
68 Thorax

A 31

The chest radiograph in Fig. 1 shows a moderate me- Symptoms vary accordingly. Usually, children with
diastinal shift to the left. The right lung is of increased symptomatic bronchogenic cysts suffer from recurrent
transparency. The right diaphragm is lowered indicating pneumonia, dysphagia, and failure to thrive.
a bronchial valve mechanism. Surgical excision is curative in these cases, and it is rec-
Figure 2 is an esophagogram. The esophagus is dis- ommended if a symptomatic cystic lung lesion does not
located in a curve to the right and to the back, resulting respond to medical treatment. The possible complications
from a mass at the level of the tracheal bifurcation. of bleeding, ulceration, infection, and obstruction of the
Figures 3 and 4 are axial and sagittal MR images, re- esophagus or airway should generally lead to resection.
spectively. A well-delineated high-intensity mass can Thoracoscopy is advantageous for isolated intratho-
be identified due to the high protein content of the cyst. racic bronchogenic cysts. Depending on the location,
The diagnosis is tracheogenic bronchogenic cyst. bronchogenic cysts may also be removed laparoscopi-
Foregut duplications may present in a variety of ways cally.
and locations. The histological similarity and anatomic
proximity of “bronchogenic cysts” and of intramural
“esophageal duplications” support a common origin.

Suggested Reading
1. Bratu I, Laberge JM, Flageole H, Bouchard S. Foregut dupli- 3. Nobuhara KK, Gorski YC, LaQuaglia MP, Shamberger RC.
cations: is there an advantage to thoracoscopic resection? J Bronchogenic cysts and esophageal duplications: common
Pediatr Surg 2005; 40:138–141 origins and treatment. J Pediatr Surg 1997; 32:1408–1413
2. Koontz CS, Oliva V, Gow KW, Wulkan ML. Video-assisted 4. Parikh D, Samuel M. Congenital cystic lung lesions: is sur-
thoracoscopic surgical excision of cystic lung disease in gical resection essential? Pediatr Pulmonol 2005; 40:533–
children. J Pediatr Surg 2005; 40:835–837 537
5. Schier F, Waldschmidt J. Thoracoscopy in children. J Pedi-
atr Surg 1996; 31:1640–1643
2 Thorax 69

Q 32
Brice Antao and Azad Najmaldin

Fig. 1 Fig. 2

A male infant was born by normal vaginal delivery at


36 weeks’ gestation. He weighed 2,845 g and had good
APGAR scores. Soon after birth, he was noted to have
increased oral secretions and developed choking, brady-
cardia, and apnea with feeds.
• What features are seen in Figs. 1 and 2, suggestive of
the diagnosis?
• Why was the examination shown in Fig. 3 per-
formed?
• What other investigations may prove helpful?
• Which is the best approach for treatment of this ab-
normality?
• What is the outcome of this condition?

Fig. 3
70 Thorax

A 32

This neonate had an H-type tracheoesophageal fistula. allows placement of a catheter through the fistula and
The classical triad of symptoms of H-type fistula are precise localization at the time of surgical repair.
paroxysm of severe choking and cough precipitated by Esophagoscopy may also help in identifying an H-
feeds, abdominal distension due to passage of air from type fistula. Although bronchoscopy is a better diag-
the trachea to the esophagus, and recurrent attacks of nostic measure than esophagoscopy, usually both pro-
pneumonia due to aspiration. cedures are performed in order to diagnose and localize
Figures 1 and 2 are anterior and lateral views, respec- an H-type fistula.
tively, of a tube esophagogram. It clearly demonstrates The fistula can be repaired via a cervical or thoracic
the fistula. Contrast medium can be seen in the tracheo- route. As more than 70% of the H-type fistulas are above
bronchial tree. However, it is important to recognize that the second thoracic vertebra, the majority of surgeons
contrast medium in the trachea may be a feature of aspi- advocate repair via the cervical route. This case was re-
ration alone. A routine upper gastrointestinal (GI) con- paired through a cervical incision following a broncho-
trast study often fails to demonstrate an H-type fistula. scopic localization of the fistula. Thoracoscopic repair is
A tube esophagogram facilitates contrast medium being an alternative approach.
injected under high pressure at different levels, and has This child recovered well and was thriving at the
a better diagnostic yield. It is important to start the tube 2-year follow-up. Recurrence is a rare complication. On
esophagography high in the cervical esophagus in order rare occasions more than one H-type fistula may exist
to minimize the risk of missing a proximal (high) fistula. and missed during investigations and exploration. En-
Figure 3 is a bronchoscopic view of the thoracic end doscopic evaluation prior to surgery increases the sensi-
of the H-fistula. It usually confirms the diagnosis and tivity of identifying such cases.

Suggested Reading
1. Benjamin B, Pham T. Diagnosis of H-Type tracheoesopha- 3. Ng J, Antao B, Bartram J, Raghavan A, Shawis R. Diagnos-
geal fistula. J Pediatr Surg 1991; 26:667–671 tic difficulties in the management of H type tracheoesopha-
2. Crabbe DC, Kiely EM, Drake DP, Spitz L. Management of geal fistula. Acta Radiologica 2006; 8:801–805
the isolated congenital tracheo-oesophageal fistula. Eur J
Pediatr Surg 1996; 6:67–9
2 Thorax 71

Q 33
Craig T. Albanese

Fig. 1 Fig. 2

Fig. 3 Fig. 4

A large echogenic solid mass (calipers) was noted in the chest. There was normal lung tissue (asterisks). A trans-
lower left hemithorax of the fetal chest on a 22-week verse MR image through the lower fetal thorax (Fig. 4)
surveillance ultrasound study (Fig. 1). The fetal heart showed the lung mass (asterisk) and lower thoracic
(arrowheads) was displaced into the right chest (fetal aorta (arrowhead) giving rise to a systemic feeding ves-
spine, arrow). sel to the mass.
Color flow Doppler (Fig. 2) revealed a systemic ar- • What is the anomaly here?
tery (arrow) arising from the aorta (arrowheads) feed- • Which lobes are usually affected?
ing this mass. A coronal MR image of the fetus (Fig. 3) • Does this mass ever shrink in size prenatally?
showed the bright mass (arrowheads) in the left lower • What is the treatment?
72 Thorax

A 33

This fetus has a pulmonary sequestration. It can be ei-


ther extralobar (retains its own pleural investment) or
intralobar (within the pleural investment of the adjacent
lobes)—this distinction is difficult to make prenatally,
unless the sequestered lobe is below the diaphragm (al-
ways extralobar in this case). By definition, a sequestra-
tion (as opposed to the closely related cystic adenoma-
toid malformation lesion) has an aberrant blood supply,
derived chiefly from the infradiaphragmatic aorta.
Sequestrations are almost always found in the lower
lobes. They can diminish in size prenatally but are always
noted postnatally on CT scans, even when the plain chest
radiograph is normal and the patient is asymptomatic.
A newborn chest radiograph (Fig. 5) shows a left lower
lobe mass (arrows). A chest CT scan (Fig. 6) shows left
lower lobe sequestration with systemic arterial feeders
(arrows) originating from the lower thoracic aorta.
This baby was asymptomatic at birth. The lesion is
removed urgently when there are respiratory symptoms.
For asymptomatic babies, elective excision is performed
since there is a risk of recurrent pulmonary infections
and malignant transformation after several decades of
life. This child had an intralobar sequestration that was
removed thoracoscopically. Fig. 5

Suggested Reading
1. de Lagausie P, Bonnard A, Berrebi D, Petit P, Dorgeret S,
Guys JM.Video-assisted thoracoscopic surgery for pulmo-
nary sequestration in children. Ann Thorac Surg 2005 Oct;
80(4):1266–9
2. Odaka A, Honda N, Baba K, Tanimizu T, Takahashi S,
Ohno Y, Satomi A, Hashimoto D. Pulmonary sequestra-
tion. J Pediatr Surg 2006 Dec; 41(12):2096–7
3. Torres LF, Jacob GV, de Noronha L, Seade M, Artigas JL.
Congenital pulmonary extralobar intra-abdominal seques-
tration J Pediatr (Rio J) 1997 Jan-Feb; 73(1):51–3
Fig. 6
2 Thorax 73

Q 34
Craig T. Albanese

Fig. 1 Fig. 2

Fig. 3 Fig. 4

A pregnant woman was found to have polyhydramnios lungs was seen. Subsequent fetal MR images (Figs. 3, 4)
clinically. Fetal ultrasound (Figs. 1, 2; axial and sagittal demonstrated a dilated trachea (arrow) terminating at
views) demonstrated distended pulmonary parenchyma the level of the fetal larynx (arrowhead), distended lungs
(small asterisks), fluid-filled airways, pleural effusion, as- (small asterisks), fetal ascites (large asterisks), and poly-
cites (large asterisk), and inverted diaphragms (arrows). hydramnios.
A compressed heart (circle) secondary to hyperinflated • What is your diagnosis?
74 Thorax

A 34

The newborn chest radiograph (Fig. 5) demonstrates hy-


perinflated lungs and inverted diaphragms.
This neonate has congenital high airway obstruction
syndrome (CHAOS). This can be due to any number
of complete obstructions at the level of the larynx or
trachea. Most commonly, it is due to laryngeal atresia,
which is what this patient had.
The baby was delivered using the EXIT strategy and
underwent a tracheostomy to secure the airway. A “pig-
tail” drainage catheter can be noted in the left upper ab-
domen in Fig. 5, placed to drain the tense ascites.

Fig. 5

Suggested Reading
1. Adzick NS. Management of fetal lung lesions. Clin Perina- 3. Marwan A, Crombleholme TM. The EXIT procedure: prin-
tol 2003 Sep; 30(3):481–92 ciples, pitfalls, and progress. Semin Pediatr Surg 2006 May;
2. Lim FY, Crombleholme TM, Hedrick HL, Flake AW, John- 15(2):107–15
son MP, Howell LJ, Adzick NS. Congenital high airway
obstruction syndrome: natural history and management. J
Pediatr Surg 2003 Jun; 38(6):940–5
2 Thorax 75

Q 35
Ciro Esposito and Giovanni Esposito

Fig. 1

A newborn requiring vigorous resuscitation because of • What does the radiograph show?
severe asphyxia at birth presented with respiratory dis- • What was the treatment?
tress a few hours later, which suddenly became worse. • What was the follow-up?
The findings from a plain radiograph (Fig. 1) of the
chest suggested urgent treatment.
76 Thorax

A 35

The standard radiograph of the chest shows a large pneu-


mothorax on the right site, with a collapse of the lung
and a contralateral shift of the mediastinum (Fig. 1).
The pneumothorax was treated by aspiration using
a needle, inserted through the second intercostal space
along the mid-clavicular line, initially connected with a
10-ml syringe and over the following days to an underwa-
ter seal drain without any suction. After 5 days, the lung
had expanded and therefore the needle was removed.
After 10 days, the follow-up radiograph of the chest
confirmed the complete re-expansion of the lung and
the neonate was discharged from hospital (Fig. 2).

Fig. 2

Suggested Reading
1. Katar S, Devecioglu C, Kervancioglu M, Ulku R. Symptom- 4. van den Boom J, Battin M.Chest radiographs after removal
atic spontaneous pneumothorax in term newborns. Pediatr of chest drains in neonates: clinical benefit or common
Surg Int 2006 Sep; 22(9):755–8 practice? Arch Dis Child Fetal Neonatal Ed 2007 Jan; 92(1):
2. Kirby C, Trotter C. Pneumothorax in the neonate: as- F46–8
sessment and diagnosis. Neonatal Netw 2005 Sep-Oct; 5. Wenzel V, Russo S, Arntz HR, Bahr J, Baubin MA, Bot-
24(5):49–55 tiger BW, Dirks B, Dorges V, Eich C, Fischer M, Wolcke B,
3. Margau R, Amaral JG, Chait PG, Cohen J. Percutane- Schwab S, Voelckel WG, Gervais HW; European Resusci-
ous thoracic drainage in neonates: catheter drainage ver- tation Council. The new 2005 resuscitation guidelines of
sus treatment with aspiration alone. Radiology 2006 Oct; the European Resuscitation Council: comments and sup-
241(1):223–7 plements. Anaesthesist 2006 Sep; 55(9):958–66, 968–72,
974–9
2 Thorax 77

Q 36
François Becmeur

Fig. 1 Fig. 2

The majority of the class members of this 9-year-old boy


had suffered from gastroenteritis for a few days. This boy
presented with vomiting and a fever of about 38°C.
The parents called the doctor on the following eve-
ning. After clinical examination, the doctor was able to
exclude a diagnosis of appendicitis and suspected gas-
troenteritis.
Over the next 4 days, the clinical evolution of the boy
was variable: he suffered from episodes of septic fever
associated with abdominal painful crises.
Five days later, his temperature rose to 40°C. The
boy suffered from abdominal pains and especially from
severe dyspnea. A clinical examination suggested right
pleuritis.
Fig. 3
The first radiographic study of the thorax was car-
ried out with the child in bed because he seemed tired
(Fig. 1). • What do you observe in Fig. 2 and what is your hy-
• What do you observe in Fig. 1 and which assump- pothesis?
tions can you make?
A thoracoabdominal CT scan was required (Fig. 3). An
The findings of the first radiograph suggested an infec- interhepatic diaphragmatic abscess is observed with a
tious pleural pathology. A radiograph of the thorax, with dense image, probably partially calcified, in the center.
the patient upright, was then obtained (Fig. 2). • What is the principal diagnosis and what is your
treatment plan?
78 Thorax

A 36

Initially, the diagnosis of food poisoning or of gastroen- A pleural empyema was initially proposed by the
teritis is indeed the most probable diagnosis. A symp- doctors. Pleural empyema generally occurs secondary to
tomatic treatment (analgesics and antisecretory medica- an infectious pneumopathy. However, at no time during
tion) accompanied by a liquid diet is recommended. the week did the patient have respiratory symptoms.
The majority of gastroenteritis cases are of viral ori- The anteroposterior upright radiograph shows the
gin. The duration of viral gastroenteritis is between 3 and existence of a fluid–fluid level. The thoracoabdominal
5 days with a typical chronology of symptoms: vomiting, scan confirmed the presence of a subdiaphragmatic ab-
fever, abdominal pains, and diarrhea. In this patient, the scess with a fluid–fluid level testifying to the presence
evolution of symptoms over 7 days (from Saturday to of anaerobic germs. The hyperdense and calcified image
Friday), the absence of diarrhea, and the septic fever ac- within this abscess in the right under the hepatic region
companied by abdominal pains suggest a deep suppura- suggests the existence of stercoral colitis of appendicular
tion. No signs of urinary or hepatobiliary involvement origin. The most probable diagnosis is of a deep abscess
were found. with pleural reaction due to subhepatic appendicitis.

Suggested Reading
1. Chandesris MO, Schleinitz N, Gayet S, Bernit E, Crebassa 3. Mac Erlean DP, Gibney RG. Radiological management of
C, Veit V, Harle JR, Kaplanski G. Anaerobic deep abscesses abdominal abscess. J R Soc Med 1983 Apr; 76(4):256–61
with unusual location: report of 5 cases. Rev Med Interne 4. Shuler FW, Newman CN, Angood PB, Tucker JG, Lucas
2005 Jul; 26(7):534–40 GW. Nonoperative management for intra-abdominal ab-
2. Fernandez M, Ortega D, Darras A, Gallardo S, Yarmuch J. scesses. Am Surg 1996 Mar; 62(3):218–22
Percutaneous drainage of abdominal abscesses Rev Med 5. Useche E, Salazar S, Vetencourt R, Monzon R. Non-surgi-
Chil 1990 Jul; 118(7):772–6 cal drainage of intra-abdominal abscesses. G E N 1991 Jan-
Mar; 45(1):9–13
2 Thorax 79

Q 37
François Becmeur

Fig. 2

Fig. 1

A 10-year-old boy presented to the emergency depart-


ment with pain in his left leg that had started 3 weeks
earlier without any incident of trauma.
On clinical examination, he was afebrile, there were
no hematomas, and the articulations were painless. The
pulses were well perceived, and the calf was flexible. A
neurological examination revealed pain in the posterior
face of the left thigh, extending to the external face of
the leg, which involved a limp on walking, without any
functional deficit. All osteotendinous reflexes were pres-
ent except for the left patellar reflex. The cutaneoplantar
reflexes were indifferent. A discrete hypoesthesia of the Fig. 3
perineum was noted.
A radiological assessment of the rachis was made A reduction in the vesicular murmurs on the left lung
(Fig. 1). was noted during a pulmonary examination. Imaging of
the thorax was subsequently performed (Figs. 2, 3).
• Describe Figs. 2, 3.
• What is your diagnosis?
80 Thorax

A 37

All the results lead to the diagnosis of a thoracic tumor A CT scan of the thorax highlights a very bulky mass
with metastases. In this age, the most frequent thoracic in the left mediastinum, measuring 10 cm in diameter,
tumors are lymphomas, neuroblastomas, and germinal pushing back the mediastinum toward the midline, com-
tumors. pressing the left bronchus at the back. This mass is het-
The diagnosis of germinal tumor was confirmed his- erogeneous and contains some small calcifications. After
tologically and chemotherapy was started immediately injection of contrast medium, a moderate amount of con-
because of the neurological signs. After a few days, the trast medium is present on the reformatted images, with
patient’s symptoms improved. total vascularization in the form of thread-like vessels.
The radiograph of the thorax, in inspiration, high- To assess the germinal tumor, laboratory examina-
lights an opacity in the upper-left part of the thorax. This tions should measure the levels of alpha-fetoprotein and
is responsible for the deviation of the trachea. A metas- B-human chorionic gonadotropin, which will be in-
tasis in the right parenchyma is noted. There is no pleu- creased. Biopsy of the mass and histological assessment
ral effusion. are mandatory.

Suggested Reading
1. Adzick NS. Management of fetal lung lesions. Clin Perina- 3. Gathwala G, Rattan KN. Posterior mediastinal mass: missed
tol 2003 Sep; 30(3):481–92 diagnosis. Indian J Pediatr 1994 Sep–Oct; 61(5):577–8
2. Freud E, Ben-Ari J, Schonfeld T, Blumenfeld A, Steinberg 4. Glick RD, La Quaglia MP. Lymphomas of the anterior me-
R, Dlugy E, Yaniv I, Katz J, Schwartz M, Zer M. Mediastinal diastinum. Semin Pediatr Surg 1999 May; 8(2):69–77
tumors in children: a single institution experience. Clin Pe- 5. Tansel T, Onursal E, Dayloglu E, Basaran M, Sungur Z,
diatr (Phila) 2002 May; 41(4):219–23 Qamci E, Yilmazbayhan D, Eker R, Ertugrul T. Childhood
mediastinal masses in infants and children. Turk J Pediatr
2006 Jan–Mar; 48(1):8–12
2 Thorax 81

Q 38
Giovanni Esposito and Ciro Esposito

Fig. 1

Fig. 2

A 2-year-old child, without a significant previous medi- medical report showed no lung sounds in the lung base
cal history, was admitted to our hospital because of the and a diffused tympanic sound. Radiographs of the tho-
sudden onset of a cough and high fever. On objective rax (Figs. 1, 2) were acquired.
examination, hypophonesis of the thorax and wet noises • What do the radiographs show?
were reported. The child was treated with broad-range • What was the diagnosis?
antibiotics and the symptoms improved remarkably • What was the evolution of the disease?
with a reduction of the fever. Two days later, the child • What was the treatment?
presented with respiratory distress along with intense • What was the follow-up?
dyspnea and diffuse cyanosis. This time the physical
82 Thorax

A 38

The radiographs (Figs. 1, 2) demonstrated the presence


of an air–fluid level in the right hemithorax with air in
the pleural space and a collapse of the lung with a medi-
astinal contralateral shift.
The initial diagnosis was tension hydropneumotho-
rax and subsequently after the treatment the diagnosis
was tension pyopneumothorax.
The treatment consisted in a minimal pleurotomy of
the fifth intercostal space on the axillary midline and in-
troduction of a drainage tube (Petzer probe no. 8) in the
pleural space, which allows draining of the pus and air
under strong pressure.
The patient was followed up with periodic radiologi-
cal studies, which demonstrated progressive reduction
of the pleural effusion along with progressive expansion Fig. 3
of the lung. In fact, as early as the first follow-up 10 days
after the water drainage (Fig. 3) there was reduction of
the pneumothorax, which completely reverted in suc-
cessive follow-ups. After 25 days, the water-drainage
tube was removed. After 1 month, a radiograph of the
thorax showed complete lung expansion with a light
pleural air-frame.

Suggested Reading
1. Abramzon OM, Kurlaev PP. Prediction of the course of 4. Hsieh CF, Lin HJ, Foo NP, Lae JC. Tension pyopneumotho-
acute purulent diseases of the lung and pleura. Probl Tu- rax. Resuscitation 2007; 1: 35–38
berk Bolezn Legk 2006; (7):8–11 5. Tsai WK, Chen W, Lee JC, Cheng WE, Chen CH, Hsu WH,
2. Chinnan NK, Rathore A, Shabaan AI, Al Samman W. The Shih CM. Pigtail catheters vs large-bore chest tubes for
“forbidden” chest x-ray: tension pyopneumothorax. Am J management of secondary spontaneous pneumothoraces
Emerg Med 2007 Feb; 25(2):200–1 in adults. Am J Emerg Med 2006 Nov; 24(7):795–800
3. Eller P, Theurl I, Koppelstaetter F, Weiss G. Pyopneumo-
thorax due to Streptococcus milleri. Wien Klin Wochen-
schr 2006 May; 118(7-8):207
2 Thorax 83

Q 39
Giovanni Esposito and Ciro Esposito

Fig. 1

An 8-year-old child was hospitalized for cryptorchidism


treatment, where the incidental radiological finding of
thoracic opacity was made. He was transferred to our
department, where other examinations were performed
that also indicated the need for a thoracic intervention.
• What does Fig. 1 show?
• Which other examinations were performed and what
were the results?
• What was the suspected diagnosis?
• What was found during intervention and what was
the treatment?
• What was the definitive diagnosis?
• What was the follow-up?
84 Thorax

A 39

The radiography study demonstrates the presence of me-


diastinal opacity on the left side.
MR imaging was subsequently performed along with
measurement of urinary vanillylmandelic acid (VMA)
and homovanillic acid (HVA) levels.
The MR image (Fig. 1) showed an opacity on the left
side of the thoracic spine; the VMA and MVA levels
were lower than normal. The suspected diagnosis was of
a neurogenic tumor.
At intervention, a mass adherent to the vertebral col-
umn was found and was easily excised.
At histologic examination, the tumor proved to be a
ganglioneuroblastoma.
The postoperative course was uneventful and the
child was discharged on the seventh day after the opera-
tion. At the 5-year follow-up, the child was doing well.

Suggested Reading
1. Domanski HA Fine-needle aspiration of ganglioneuroma. 4. Nio M, Nakamura M, Yoshida S, Ishii T, Amae S, Hayashi Y.
Diagn Cytopathol 2005 Jun; 32(6):363–6 Thoracoscopic removal of neurogenic mediastinal tumors
2. Geraci AP, de Csepel J, Shlasko E, Wallace SA. Ganglioneu- in children. J Laparoendosc Adv Surg Tech A 2005 Feb;
roblastoma and ganglioneuroma in association with neu- 15(1):80–3
rofibromatosis type I: report of three cases. J Child Neurol 5. Tang KT, Lee HC, Liang DC, Chen SH, Liu HC, Sheu JC.
1998 Jul; 13(7):356–8 Neural-crest tumor presenting with chronic diarrhea:
3. Milovic I, Scekic M, Vujic D, Djurisic S, Djokic D. The char- a report of three cases. J Formos Med Assoc 2002 Dec;
acteristics of mediastinal neuroblastoma and perspectives 101(12):864–7
on surgical excision Acta Chir Iugosl 2003; 50(4):103–7
2 Thorax 85

Q 40
Giovanni Esposito, Carmelo Romeo, and Ciro Esposito

Fig. 1

A 5-year-old girl was admitted to our institution because • What does Fig. 1 show?
of a thoracic opacity detected on a chest radiograph • Which other procedures were performed and what
which had been acquired to examine a persistent cough was their result?
after a respiratory infection. • What was the diagnosis?
On physical examination, no pathological findings • What was the treatment?
were revealed. Because of the localization of the opacity, • What was the follow-up?
other procedures were performed. Subsequently, a sur-
gical intervention was scheduled.
86 Thorax

A 40

Fig. 2

The thoracic radiograph (Fig. 1) showed an opacity oc- A posterolateral thoracotomy along the fifth intercos-
cupying the superior and medial part of the right hemi- tal space was performed, and a paravertebral tumor was
thorax, probably of mediastinal origin. excised without many difficulties.
After a CT scan that confirmed the mediastinal site of After histologic examination, the diagnosis of neuro-
the opacity, an MIBG scan was obtained and the vanil- blastoma was confirmed.
lylmandelic acid (VMA) and homovanillic acid (HVA) The postoperative course was uneventful and there-
levels were measured. fore the child was discharged on the seventh day after the
The MIBG (Fig. 2) test demonstrated an accumulation operation. The patient was transferred to a pediatric on-
of the radioisotope in the region of the opacity. The lev- cology unit for complementary treatment of the tumor.
els of both HVA and of VMA were found to be elevated. At the 2-year follow-up, the girl was doing well with-
A diagnosis of thoracic neuroblastoma was made. out any relapse of the tumor.

Suggested Reading
1. DeCou JM, Schlatter MG, Mitchell DS, Abrams RS. Pri- 4. Kang CH, Kim YT, Jeon SH, Sung SW, Kim JH. Surgical
mary thoracoscopic gross total resection of neuroblastoma. treatment of malignant mediastinal neurogenic tumors in
J Laparoendosc Adv Surg Tech A 2005 Oct; 15(5):470–3 children. Eur J Cardiothorac Surg 2007; 2:321–325
2. Diez Jimenez L, Mitjavila Casanovas M. MIBG scintigra- 5. Petty JK, Bensard DD, Partrick DA, Hendrickson RJ, Al-
phy in neuroblastoma: something more than an image. Rev bano EA, Karrer FM. Resection of neurogenic tumors in
Esp Med Nucl 2006 Mar–Apr; 25(2):118–43 children: is thoracoscopy superior to thoracotomy? J Am
3. Escobar MA, Grosfeld JL, Powell RL, West KW, Scherer LR Coll Surg 2006 Nov; 203(5):699–703
3rd, Fallon RJ, Rescorla FJ. Long-term outcomes in patients
with stage IV neuroblastoma. J Pediatr Surg 2006 Feb;
41(2):377–81
2 Thorax 87

Q 41
Giovanni Esposito, Michele Ametrano, and Ciro Esposito

Fig. 1

A 5-year-old boy had a persistent cough that had started • What does the MR image show?
a few months earlier. A chest radiograph demonstrated a • What is the opacity suggestive of?
round opacity in the visceral compartment of the medi- • What was found at surgery?
astinum. To define the precise nature of the opacity, MR • What was the definitive diagnosis and treatment?
imaging was performed (Fig. 1), which indicated the • What was the follow-up?
need for surgical intervention.
88 Thorax

A 41

Fig. 2

The MR image (Fig. 1) shows the presence of a cystic Histological examination confirmed the diagnosis of
area with a high signal on T1- and T2-weighted images bronchogenic cyst.
that is characteristic of a cyst containing liquid with a The postoperative course was uneventful and the
high concentration of proteins. child was discharged 7 days after surgery. Follow-up
Because of the connection of the cyst with the main consisted in a plain radiograph at 6 and 12 months post-
left bronchus, the diagnostic suspicion was of a bron- operatively, which was normal.
chogenic cyst.
At surgery, performed via left posterolateral thora-
cotomy along the fifth intercostal space, a cyst adherent
to the bronchus was found and excised (Fig. 2).

Suggested Reading
1. Fukasawa C, Ohkusu K, Sanayama Y, Yasufuku K, Ishiwada 3. Kunisaki SM, Fauza DO, Barnewolt CE, Estroff JA, Myers
N, Ezaki T, Kohno Y. A mixed bacterial infection of a bron- LB, Bulich LA, Wong G, Levine D, Wilkins-Haug LE, Ben-
chogenic lung cyst diagnosed by PCR. J Med Microbiol son CB, Jennings RW. Ex utero intrapartum treatment with
2006 Jun; 55(Pt 6):791–4 placement on extracorporeal membrane oxygenation for
2. Inzani F, Recusani F, Agozzino M, Cavallero A, De Siena fetal thoracic masses. J Pediatr Surg 2007 Feb; 42(2):420–5
PM, D’Armini A, Vigano M, Arbustini E. Bronchogenic 4. Rubinas TC, Manera R, Newman B, Picken MM. Pneumo-
cyst: unexpected finding in a large aneurysm of the pars thorax and pulmonary cyst in a 2-year-old child. Pleuropul-
membranacea septi. J Thorac Cardiovasc Surg 2006 Oct; monary blastoma. Arch Pathol Lab Med 2006 Apr; 130(4)
132(4):972–4 5. Sauvat F, Fusaro F, Jaubert F, Galifer B, Revillon Y. Parae-
sophageal bronchogenic cyst: first case reports in pediatric.
Pediatr Surg Int 2006 Oct; 22(10):849–51
2 Thorax 89

Q 42
Giovanni Esposito and Ciro Esposito

Fig. 1

A 4-year-old boy, without a significant medical history, After other examinations, surgery was scheduled.
presented to our department with a respiratory infection • What does the chest radiograph show?
accompanied by fever, cough, and light dyspnea that had • What other examinations were performed and what
started 1 week earlier. were their results?
The fever disappeared after treatment with antibiotics, • What was the diagnostic suspicion?
whereas the cough and dyspnea continued. After stan- • What was found at surgery and what was the treat-
dard radiography of the thorax was carried out (Fig. 1), ment?
the child was hospitalized at our institution. • What was the definitive diagnosis?
On admission, physical examination of the thorax • What was the follow-up?
revealed an anterior hypophonesis in the superior part
of the left hemithorax where diffuse humid noises were
found.
90 Thorax

A 42

The chest radiograph of the thorax (Fig. 1) shows an


opacity of the superior left hemithorax that resides ante-
riorly in the lateral position.
MR angiography was performed, which demon-
strated that the opacity was located in the anterior me-
diastinum and presented some cystic areas impregnated
with contrast medium.
The diagnosis was of mediastinal lymphangioma.
At surgery, performed via anterolateral thoracotomy
at the eighth intercostal space, a lymphangioma was
found. The lymphangioma was strongly adherent to the
surrounding vascular and nerve structures, and it was
isolated after many difficulties. For these reasons, we
performed a subtotal excision of the mass. The postop-
erative course was uneventful and the thoracic drain was
removed after 48 h. The child was discharged on postop-
erative day 8. At the 6-month and 1-year follow-ups, no
relapse was observed (Fig. 2).

Fig. 2

Suggested Reading
1. Al-Salem AH. Lymphangiomas in infancy and childhood. 4. Okazaki T, Iwatani S, Yanai T, Kobayashi H, Kato Y, Maru-
Saudi Med J 2004 Apr; 25(4):466–9 sasa T, Lane GJ, Yamataka A. Treatment of lymphangioma
2. Handa R, Kale R, Upadhyay KK. Isolated mediastinal in children: our experience of 128 cases. J Pediatr Surg 2007
lymphangioma herniating through the intercostal space. Feb; 42(2):386–9
Asian J Surg 2004 Jul; 27(3):241–2 5. Tamay Z, Saribeyoglu E, Ones U, Anak S, Guler N, Bilgic B,
3. Nouri H, Raji A, Rochdi Y, Elhattab Y, M’Barek BA. Cer- Yilmazbayhan D, Gun F. Diffuse thoracic lymphangiomato-
vical cystic lymphangioma in children. Rev Laryngol Otol sis with disseminated intravascular coagulation in a child. J
Rhinol (Bord) 2006; 127(4):263–6 Pediatr Hematol Oncol 2005 Dec; 27(12):685–7
2 Thorax 91

Q 43
Giovanni Esposito and Ciro Esposito

Fig. 1

Fig. 2

A 5-year-old child, who was born at term after an un- mal except for a marked eosinophilia. On the basis of
complicated delivery, was in good health until 1 month the radiological report, other tests were performed be-
before admission to our hospital when he had a bron- fore making the diagnosis and deciding on the type of
chopulmonary infection that was treated with antibiotics. intervention.
The fever disappeared, but many bouts of nonproductive • What do Figs. 1 and 2 show?
coughing persisted, and therefore standard radiography • What other examinations were performed and what
of the chest was performed. The radiography findings were their results?
indicated the need for surgical intervention. On admis- • What was the diagnosis?
sion, the child, who was in a good general condition, • What was the treatment?
presented many respiratory sounds on auscultation of • What was the follow-up?
the left hemithorax. The laboratory test results were nor-
92 Thorax

A 43

The radiographs show a single large pulmonary nodule


in the superior part of the left hemithorax in the lateral
projection.
The other tests performed were: an indirect hemoag-
glutination assay with the result of 1:152; and an immu-
noelectrophoresis assay that was positive. Complement
fixation test results were both positive. Furthermore, an
abdominal ultrasound was performed that was negative
for the possible presence of the same lesion.
The diagnosis was of echinococcus cyst of the lung.
Treatment consisted in a posterolateral thoracotomy
at the fifth intercostal space and a small pneumotomy of
the superior lobe, through which an echinococcus cyst, Fig. 3
the size of an orange, was extracted (Fig. 3).
The postoperative course was uneventful and the
child was discharged 7 days after the intervention with
a prescription of albendazole for 2 weeks.
At the 1-year follow-up, the child was in good con-
dition.

Suggested Reading
1. Dincer SI, Demir A, Sayar A, Gunluoglu MZ, Kara HV, 3. Erdem CZ, Erdem LO. Radiological characteristics of pul-
Gurses A. Surgical treatment of pulmonary hydatid disease: monary hydatid disease in children: less common radiolog-
a comparison of children and adults. J Pediatr Surg 2006 ical appearances. Eur J Radiol 2003 Feb; 45(2):123–8
Jul; 41(7):1230–6 4. Mallick MS, Al-Qahtani A, Al-Saadi MM, Al-Boukai AA.
2. Durakbasa CU, Sander S, Sehiralti V, Tireli GA, Tosyali AN, Thoracoscopic treatment of pulmonary hydatid cyst in a
Mutus M. Pulmonary hydatid disease in children: outcome child. J Pediatr Surg 2005 Dec
of surgical treatment combined with perioperative alben- 5. Turkyilmaz Z, Sonmez K, Karabulut R, Demirogullari
dazole therapy. Pediatr Surg Int 2006 Feb; 22(2):173–8 B, Gol H, Basaklar AC, Kale N. Conservative surgery for
treatment of hydatid cysts in children. World J Surg 2004
Jun; 28(6):597–601
2 Thorax 93

Q 44
Giovanni Esposito and Ciro Esposito

Fig. 1

A 9-year-old boy, born at term after a normal pregnancy tory examination showed an arterial hypoxia with poly-
and delivery and with no family history of disease, was cythemia.
in good health until the age of 4 years, when he showed After other diagnostic procedures, surgery was sched-
dyspnea and cyanosis with some exercise intolerance. uled.
Because the symptoms became worse, a radiograph of • What does the radiograph show (Fig. 1)?
the chest was acquired after which hospitalization was • What other procedures were performed and what
indicated. was their result?
On admission, the physical examination revealed • What was the diagnosis?
mild cyanosis on the lips and nail beds with marked ex- • What was the intervention?
ercise intolerance. • What was the follow-up?
On auscultation of the left hemithorax, many respira-
tory murmurs were noted especially inferiorly. Labora-
94 Thorax

A 44

A standard radiograph of the chest showed a normal car-


diac silhouette with a shadow in the inferior part of the
left lung that was connected with a spindle-shaped band
to the hilum and was interpreted as an enlarged pulmo-
nary artery and vein.
Angioscintigraphy and cardiac catheterization with
pulmonary angiography were performed. The angios-
cintigraphy showed a hypervascularized pulmonary le-
sion with an arteriovenous shunt, while the cardiac cath-
eterization with pulmonary angiogram showed a large
localized arteriovenous fistula of the left lower lobe of
the lung (Fig. 2).
The degree of right-to-left shunt was calculated by
placing a catheter into the right pulmonary vein (via
a retrograde left atrium approach) and measuring the Fig. 2
oxygen saturation in the right pulmonary vein, in the
left atrium, and in the aorta during basal conditions
and after balloon occlusion of the arterial branches of
the arteriovenous fistula. The oxygen saturation values
were: pulmonary artery 44%, left atrium 58%, right pul-
monary vein 86%, aorta 55%, and aorta after occlusion
of the fistula 78%.
On the basis of all these procedures the diagnosis was
congenital pulmonary arteriovenous fistula.
During surgery, on the surface of the lower lobe many
varicose vessels were found. An inferior lobectomy was
performed.
The postoperative course showed no complications
with prompt relief of symptoms. At 1-year follow-up,
the child was in good condition.

Suggested Reading
1. Berthezene Y, Howarth NR, Revel D. Pulmonary arteriove- 3. Seth RK, Seth S, Kabra SK. Micropulmonary arteriove-
nous fistula: detection with magnetic resonance angiogra- nous fistula causing cyanosis. Indian J Pediatr 2003 Sep;
phy Eur Radiol 1998; 8(8):1403–4 70(9):747–9
2. Motsch K, Porstmann W. Arteriovenous pulmonary fistula.
The differential diagnosis of cyanosis in children. Dtsch
Gesundheitsw 1968 Sep 12; 23(37):1750–5
3
Abdomen

Case 45–122
96 Abdomen

Introduction

Clinical expression of abdominal surgical problems in ture and aiming for cost-effective results. Nowadays,
children comprises a spectrum of symptoms, many of for example, examinations that could be used to study
them shared by different diseases. Consequently, a com- a neonate or older child with an upper gastrointestinal
plete and well-oriented medical history and physical problem include, among others, barium esophagogas-
examination are the main tools for establishing the diag- trogram, ultrasonography, CT scan, MR imaging, PET
nosis and the guidelines to request radiological studies. scan, gammagraphy, endoscopy with or without biopsies,
Sometimes, clinical evaluation is enough to initi- manometry, pH-metry, and electrophysiologic studies.
ate a medical treatment or indicate a surgical therapy, The approach to an abdominal problem depends on
for example, when a patient has vomiting secondary to the possible etiology. In this way, for example, “intestinal
gastroesophageal reflux or pyloric hypertrophy. In spite obstruction syndrome” in neonates and children with no
of good medical practice, frequently the etiology is not previous laparotomy should be considered a congeni-
clear and complementary laboratory and/or radiological tal malformation. This syndrome can be approached as
studies are needed. “high” or “low” as well as “mechanical” or “functional.”
There are many studies available to help in the diag- The age of presentation is also an important issue when
nosis of an abdominal problem in childhood. Occasion- exploring a possible etiology. The radiological study of
ally, a plain film radiograph alone is sufficient to con- this syndrome includes in all cases a plain film radio-
firm a diagnosis (appendicolith in acute appendicitis), graph; other studies such as a barium enema, barium
while in others an expensive and complex examination esophagogram, intestinal transit, ultrasonography, or
is needed (MR imaging in cases of tethered cord syn- CT scan depend on the initial clinical and radiological
drome). Furthermore, in other cases, radiological stud- findings.
ies are not helpful in confirming the diagnosis and an This section offers over 75 case studies of common
invasive procedure is indicated: pH-metry in a vomiting congenital or acquired abdominal surgical diseases
patient or rectal biopsy in a baby with constipation to found in the pediatric population, including abdominal
rule out a gastroesophageal reflux or Hirschsprung’s dis- wall, gastrointestinal, hepatic, biliary, pancreatic, and
ease, respectively. diaphragmatic problems. We hope that these cases will
Medical technology has grown quickly in the last few be helpful to the practitioner in the study of their pedi-
years and pediatricians and pediatric surgeons need to atric patients.
decide on the best option according to the clinical pic-
3 Abdomen 97

Q 45
Salam Yazbeck

Fig. 2

Fig. 1

A previously healthy 6-month-old boy was brought to • What are your diagnoses?
the emergency room for bilious vomiting, abdominal • What is your interpretation of the radiograph in
distention, and passage of bright red blood per rectum. Fig. 1?
On physical examination the abdomen was tender and • Why did the emergency room physician ask for an
reddish. The abdominal radiograph is shown in Fig. 1. abdominal ultrasound and what did it show (Fig. 2)?
The emergency room physician asked for an abdomi- • What would you do next?
nal ultrasound (Fig. 2).
98 Abdomen

A 45

Fig. 3 Fig. 4

The diagnosis is any acute bowel obstruction with ne- bowel that persisted even after reduction of this volvulus
crosis. and soaking of the bowel loops in warm saline. Only 10–
A gasless abdomen, as that shown in Fig. 1, in a child 12 cm of the proximal bowel was viable (Fig. 3). It was
with bilious vomiting and passage of red blood per rec- decided to proceed with the surgical cure of the malrota-
tum should make you suspect a midgut volvulus compli- tion (Ladd’s procedure) and to close the abdomen and
cating an intestinal malrotation. perform a second-look laparotomy the next day. The
The abdominal ultrasound was requested in order to same findings were noted again and the patient was al-
check the position of the mesenteric vessels. It showed lowed to die.
an inversion of these vessels with the mesenteric vein to Had this patient been brought to our attention ear-
the left of the superior mesenteric artery, confirming the lier, the operative findings could have been like those
diagnosis of intestinal malrotation. in Fig. 4, showing a non-ischemic midgut volvulus that
An urgent laparotomy is the next step. could be untwisted in a counter-clockwise direction
In this case, the consultant pediatric surgeon decided with the entire bowel intact and ready to undergo the
to proceed with an immediate laparotomy. There was Ladd’s procedure.
a midgut volvulus with severe ischemia of the whole
3 Abdomen 99

Q 46
Salam Yazbeck

Fig. 1 Fig. 2

A 3,000-g male neonate, born in a university hospital, The abdominal radiograph is shown in Fig. 1.
presented with severe abdominal distension, bilious The consultant pediatric surgeon ordered a barium
vomiting, and failure to pass meconium in the first 24 h enema (Fig. 2).
of life. A prenatal ultrasound had shown an important • What is your differential diagnosis?
bowel dilatation that lasted for the last trimester. • What does Fig. 1 show?
On physical examination, dilated bowel loops could • Why did the surgeon order a barium enema?
be recognized. On abdominal inspection the abdomen • How would you manage this patient?
was tender, and the anus was small but patent. • What is the most likely underlying cause?
100 Abdomen

A 46

This is a case of bowel obstruction that could be If not, laparotomy is mandatory in order to empty the
caused by many intrinsic or extrinsic factors (atresias, bowel from its very thick and sticky meconium with or
Hirschsprung’s disease, meconium-related diseases, etc.). without a temporary ileostomy.
Figure 1 is a simple abdominal radiograph showing More than 99% of meconium ileus cases are associ-
many dilated bowel loops. It could be associated with ated with cystic fibrosis of the pancreas. The definitive
any cause of neonatal bowel obstruction. diagnosis is made with a sweat test that shows a high
The contrast enema shows a micro-colon contracted Cl– concentration. However, this test may yield normal
on inspissated meconium pellets. The same aspect could results in the first month of life, but a genetic mutation
be observed in the terminal ileum. of the cystic fibrosis transmembrane regulator protein
If there is no perforation, a non-ionic hydrosoluble (CFTR) can be identified on chromosomal studies be-
contrast enema could be therapeutic and relieve the fore birth.
obstruction on the first attempt or after many attempts.

Suggested Reading
1. Del Pin CA, Czyrko C, Ziegler MM, et al. Management and 3. Noblett HR. Treatment of uncomplicated meconium ileus
survival of meconium ileus. Ann Surg 1992; 215:179 by gastrograffin enema, a preliminary report. J Pediatr Surg
2. Escobar MA, Grosfeld JL, Burdick JJ, et al. Surgical consid- 1969; 4:190–197
erations in cystic fibrosis: a 32-year evaluation of outcomes. 4. Rescola F, Grosfeld JL, West KJ, et al. Changing patterns of
Surgery 2005; 138:560–571 treatment and survival in neonates with meconium ileus.
Arch Surg 1989; 124:837–840
3 Abdomen 101

Q 47
Salam Yazbeck

Fig. 1

A 1-month-old boy presented with intermittent vomit- The treating pediatrician ordered a barium meal that
ing, sometimes of bilious nature. His mother said that he was normal and an abdominal ultrasound (Fig. 1).
also had 15–30-min episodes of abdominal pain once or • Describe the findings in Fig. 1.
twice per day. • What is your differential diagnosis?
Physical examination of the patient was unremark- • Would you order any other test?
able. • How would you manage this patient?
102 Abdomen

A 47

Fig. 3

Fig. 2

Fig. 4

Figure 1 is an abdominal ultrasound scan that shows a the mesenteric side and sharing the same vascular sup-
cystic lesion lined by a mucosal inner layer. This is typi- ply with the adjacent bowel.
cal of a duplication cyst. Gastrointestinal duplications can be found every-
An intra-abdominal cystic lesion could be a mesen- where from the mouth to the anus. Resection of the du-
teric cyst, a cystic lymphangioma, a Meckel’s diverticu- plication with the adjacent gastrointestinal segment is not
lum, or a cystic duplication. possible if the duplication cyst is located in the duodenal
A pertechnetate scan could be obtained in order to loop or if the duplication is of the tubular type (Fig. 3)
identify the ectopic gastric mucosa that is frequently and if it extends along a very long segment of the bowel.
present in duplications. In these cases marsupialization of the duplication in the
The best treatment is resection of the duplication adjacent bowel is preferable. If gastric mucosa is present
with its adjacent bowel segment. Figure 2 shows a cystic (Fig. 4) within the duplication, it should be resected.
duplication of the terminal ileum, typically located on
3 Abdomen 103

Q 48
Ugo De Luca and Antonino Tramontano

Fig. 1 Fig. 2 Fig. 3

Fig. 4 Fig. 5 Fig. 6

A 3,050-g (39 weeks) newborn had severe abdominal Three months postoperatively, the child had another
distension, bile-stained vomiting, and no stool passage radiograph (Fig. 3) and a second operation was neces-
at birth. sary (Figs. 4–6).
A plain abdominal radiograph (Fig. 1) was obtained • What does Fig. 1 show?
and following this investigation the newborn was oper- • What is the intraoperative diagnosis?
ated on. • What are the surgical options?
The surgical specimen is shown in Fig. 2. At surgery, • Was postoperative complicated course an expression
an end-to-end intestinal anastomosis was realized. of which condition?
The postoperative period was characterized by fail- • What are the surgical options in these reoperations?
ure to thrive, abdominal distension, persistent vomiting,
and poor stool passage.
104 Abdomen

A 48

Fig. 7

The radiograph shows neonatal intestinal obstruction. The postoperative course was characterized by good
The operative diagnosis is intestinal atresia with a large passage of stool, weight gain, and eventually complete
mesenteric defect (IIIA type). The end-to-end anasto- recovery. A postoperative barium meal study (Fig. 7)
mosis without tapering of the large preatresic bowel in demonstrated a normal appearance of the small intes-
the first operation was responsible for the disturbed in- tine and a regular transit time.
testinal transit (DIT) in the postoperative period.
Reoperation, without large intestinal resection in or-
der to avoid small bowel syndrome, was considered with
a long antimesenteric tapering of the dilated bowel.

Suggested Reading
1. Bianchi A. Longitudinal intestinal lengthening and tailor- 4. Thomas CG Jr, Carter JM. Small intestinal atresia: the criti-
ing: results in 20 children. J R Soc Med 1997 Aug; 90(8): cal role of a functioning anastomosis. Ann Surg 1974 May;
429–432 179(5):663–670
2. Bueno J., Guiterrez J, Mazariegos GV et al. Analysis of pa- 5. Thompson JS, Langnas AN, Pinch LW, Kaufman S, Quig-
tients with longitudinal intestinal lengthening procedure ley EM, Vanderhoof JA. Surgical approach to short-bowel
referred for intestinal transplantation. J. Ped. Surg 2001; 36 syndrome. Experience in a population of 160 patients. Ann
(1):178–183 Surg 1995 Oct; 222(4):600–607
3. Miller RC. Complicated Intestinal Atresias. Ann Surg 1979
May; 189(5):607–610
3 Abdomen 105

Q 49
Ugo De Luca

Fig. 1

A 20-day-old preterm female was transferred to our de-


partment from another hospital. Initially, the newborn
was on total parenteral nutrition for sepsis. When oral
feeding was started on the 17th day, the baby presented
with vomiting of milk and gastric juice, which was not
bile stained. The vomiting soon became projectile. The
contrast meal study is shown in Fig. 1.
• What is the diagnosis?
• What is the treatment?
106 Abdomen

A 49

This is a rare case of pyloric atresia with a late presen-


tation of symptoms because of the initial suspension of
alimentation.
The treatment is surgical and consists in a Mikulicz
pyloroplasty (longitudinal incision and transverse su-
ture; Figs. 2, 3).
The postoperative course was uneventful and alimen-
tation was started on postoperative day 3.
Pyloric atresia is a rare congenital malformation,
representing about 1% of all atresias of the gastrointes-
tinal tract. Epidermolysis bullosa is the most common
associated abnormality. Familial occurrence of pyloric
atresia has been reported. The sex distribution is equal
and there is a high proportion of low-birth-weight new-
borns. Polyhydramnios is encountered in about 50% of Fig. 2
all cases of pyloric atresia. The main symptom is vom-
iting, often projectile and nonbilious. The diagnosis is
achieved mainly radiologically by a barium meal study.
The particularity of the case presented here is the late di-
agnosis due to the suspension of alimentation. In other
cases the diagnosis may be delayed because the atresia is
not complete but represented by a holed web.

Fig. 3

Suggested Reading
1. Bar-Maor JA, Nissan S, Nevo S Pyloric atresia. J Med Genet 4. Muller M, Morger R, Engert J Pyloric atresia. report of
9, 70–72, 1972 four cases and review of the literature. Pediatr Surg Int 5,
2. Ducharme JC, Bensoussan AL Pyloric atresia. J Pediatr 276–279, 1990
Surg 10, 149–150, 1975 5. Raffensperger JG, Pyloric and duodenal obstruction. In
3. Grunebaum M, Kornreich L, Ziv N et al. The imaging diag- Swenson’s Pediatric Surgery 5th edn, Appleton & Lange,
nosis of pyloric atresia. Z Kinderchir 40, 308–311, 1985 Norwalk, Conn, 509–516, 1990
3 Abdomen 107

Q 50
Jürgen Schleef

Fig. 1

An 18-month-old boy was brought to our hospital with • What does Fig. 1 show?
fever and signs of pulmonary infection. • Why did the surgeon ask for a barium swallow?
The mother reported that the boy was growing nor- • What condition is affecting this child?
mally, but had symptoms of vomiting associated with • How do you treat this condition?
coughing.
The surgeon decided, on the basis of the history re-
ported by the mother, to ask for a barium swallow to
rule out gastroesophageal reflux.
108 Abdomen

A 50

Fig. 2 Fig. 3

Figure 1 shows a radiograph of the chest of an 18-month-


old boy. The barium swallow showed no evidence of gas-
troesophageal reflux (the stomach is filled with barium).
The small bowel and colon are projected in the thorax,
suggestive of a diaphragmatic hernia.
The treatment of this pathological condition is sur-
gical. During laparoscopy, it was noted that the defect
in the diaphragm had repositioned the bowel into the
abdomen (Fig. 2). The defect was closed at laparoscopy.
The final result is shown in Fig. 3: a closed Bochdalek
hernia. The child was discharged from hospital 2 days
after surgery.

Suggested Reading
1. Eren S, Ciris F. Diaphragmatic hernia: diagnostic ap- 3. Mei-Zahav M, Solomon M, Trachsel D, Langer JC. Boch-
proaches with review of the literature. Eur J Radiol 2005; dalek diaphragmatic hernia: not only a neonatal disease.
54(3):448–59 Arch Dis Child 2003; 88(6):532–5
2. Langer JC. Congenital diaphragmatic hernia. Chest Surg 4. Mullins ME, Saini S. Imaging of incidental Bochdalek her-
Clin N Am 1998; 8(2):295–314 nia. Semin Ultrasound CT MR 2005; 26(1):28–36
3 Abdomen 109

Q 51
Nancy Rollins and Korgun Koral

Fig. 1 Fig. 2

Fig. 3 Fig. 4

A 12-year-old boy presented to the emergency room After symptoms subsided, a barium enema (Fig. 3) and
with right upper quadrant pain. A CT scan of the abdo- small-bowel follow-through (Fig. 4) were performed
men and pelvis was obtained. with an interval of 1 week.
• What are the findings on the CT scan (Figs. 1, 2)? • What are the findings?
• What is the differential diagnosis at this point? • What is the differential diagnosis?
• What is the diagnosis?
• What are the complications?
110 Abdomen

A 51

There is thickening of the walls (arrow, Fig. 2) of the ce-


cum and terminal ileum. The terminal ileum is dilated
as are the distal small bowel loops. Inflammatory strand-
ing of mesenteric fat is present. No free fluid or abscess
is seen.
Wall thickening of the terminal ileum and cecum
can be seen in Crohn’s disease, ulcerative colitis, infec-
tion (Yersinia, tuberculosis), lymphoma, and mesenteric
adenitis.
There is luminal narrowing involving the terminal
ileum (string sign). The cecum is narrowed with irreg-
ular mucosal contours (arrow, Fig. 4). The diagnosis is
Crohn’s disease. Crohn’s disease affects the bowel wall in
its entirety and results in sinus tracts, fissures, and fistu-
las; it can present with intra-abdominal abscesses. Skip
lesions are characteristic, whereas in ulcerative colitis
involvement is continuous. In ulcerative colitis there is
only mucosal involvement. In a quarter of patients with
ulcerative colitis there is terminal ileum involvement
(backwash ileitis).

Suggested Reading
1. Federle MP. Diagnostic Imaging: Abdomen. Amirsys Ed.
2005
3 Abdomen 111

Q 52
Nancy Rollins and Korgun Koral

A 1-month-old boy presented with constipation since


birth and failure to thrive.
• What test was performed? How is this test done?
• Which contrast material was used? Why?
• What are the findings?
• How is the definitive diagnosis made?
112 Abdomen

A 52

Fig. 1 Fig. 2

A contrast enema was performed (Figs. 1, 2). The test and descending colon are normal to mildly small in cali-
was performed without bowel preparation. An enema ber. There is a size discrepancy between the transverse
tip without a balloon was inserted into the distal rectal colon and descending colon. The transition zone ap-
vault. A balloon may obscure the transition zone. pears to be in the region of the splenic flexure.
Water-soluble contrast material was used in this in- The definitive diagnosis is made with colonic biopsy to
stance to also alleviate constipation. Usually barium pro- assess the presence of ganglion cells in the colonic wall.
vides better anatomical detail.
The transverse colon, hepatic flexure, and ascending
colon are markedly dilated. The rectum, sigmoid colon,

Suggested Reading
1. Kuhn JP, Slovis TL, Haller JO. Caffey’s Pediatric Diagnostic
Imaging. Elsevier Ed. 2004
2. Siegel MJ, Coley BD. Pediatric Imaging. Lippincott Wil-
liams & Wilkins Ed. 2006
3 Abdomen 113

Q 53
Nancy Rollins and Korgun Koral

A 2-month-old boy presented to the emergency room


with bilious vomiting.
• What test should be performed first?
• If a study with contrast medium is performed what
should be done? Upper gastrointestinal examination
or barium enema?
• What are the findings on the upper gastrointestinal
examination?
• Is this condition an emergency?
114 Abdomen

A 53

Fig. 1 Fig. 2 Fig. 3

Plain radiographs of the abdomen should be obtained ferent, normal subjects). In this case the duodenojejunal
first to assess for bowel dilatation and presence of free junction and proximal jejunum are to the right of the
intraperitoneal air. spine, consistent with a diagnosis of midgut malrotation.
In a child with bilious vomiting, the test of choice is This condition constitutes an emergency if the patient is
an upper gastrointestinal (UGI) examination with bar- symptomatic.
ium. Bilious vomiting indicates an obstruction distal to In malrotation, the attachment of bowel is abnormally
the duodenojejunal junction (ligament of Treitz). short, resulting in an abnormally positioned ligament of
A feeding tube is seen in the stomach and proximal Treitz. The peritoneal bands (Ladd bands) cause extrin-
duodenum. The ligament of Treitz is to the right of the sic compression and proximal small bowel obstruction.
spine. The proximal jejunum demonstrates the cork- Every effort should be made to obtain a picture of the
screw appearance (Figs. 1, 2). It is not unusual to place ligament of Treitz in true frontal projection. If the posi-
a feeding tube to perform a UGI examination. This pro- tion of the ligament of Treitz is equivocal, a small bowel
vides better control and a shorter procedure time. Ev- follow-through study can be performed; however, dem-
ery UGI examination should demonstrate the UGI tract, onstrating the normal position of the cecum in the right
which is normally to the left of the spine at (or about) lower quadrant does not necessarily ensure absence of
the level of the duodenal cap (arrows in Fig. 3; two dif- malrotation.

Suggested Reading
1. Siegel MJ, Coley BD. Pediatric Imaging. Lippincott Wil-
liams & Wilkins Ed. 2006
2. Swischuk LE. Imaging of the Newborn, Infant and Young
Child. Lippincott Williams & Wilkins Ed. 2004
3 Abdomen 115

Q 54
Abdellatif Nouri and Mongi Mekki

Fig. 3
Fig. 2

Fig. 1

A 3-day full-term male newborn was hospitalized for ab- At the end of the enema, the newborn passed meconium
dominal distension, bilious vomiting, and failure to pass and the abdominal distension disappeared. For 6 days,
meconium. the newborn had normal bowel movements and then
On examination, he weighed 3 800 g, the anus was presented again with abdominal distension.
normal, and withdrawal of the doctor’s finger did not • What is the most likely diagnosis and how can you
produce meconium. confirm it?
An anteroposterior erect abdominal radiograph was • Which two complications threaten this newborn?
obtained (Fig. 1). • What treatment do you recommend?
A barium enema was then performed (Figs. 2, 3).
• What does Fig. 1 show?
• What do Figs. 2 and 3 show?
• Which two diagnoses can be suggested on the basis
of these figures?
116 Abdomen

A 54

Figure 1 shows marked dilatation of the large and small


bowel loops with air–fluid levels.
The barium enema (Figs. 2, 3) shows significant dila-
tation of the transverse colon and a small left colon with
a transitional zone at the splenic flexure. The rectum
does not look narrow.
The first diagnosis is Hirschsprung’s disease extended
to the splenic flexure. The second diagnosis is small left
colon syndrome, which should be suspected when the
meconium is not eliminated, the transitional zone is at
the splenic flexure, and the rectum is not narrow.
Because of the recurrence of abdominal distension,
the most likely diagnosis is Hirschsprung’s disease. Ano-
rectal manometry has proved to be a reliable method for
diagnosing Hirschsprung’s disease in older children by
showing the absence of the recto-anal inhibitor reflex,
but there is considerable controversy regarding the diag-
nostic accuracy of manometry in neonates. The diagno-
sis of Hirschsprung’s disease is based on the histological
examination of rectal wall biopsy specimens.
The two serious complications that threaten this new-
born are enterocolitis, which remains the most common
cause of death in Hirschsprung’s disease patients, and
spontaneous perforation of the intestinal tract that oc-
curs particularly with long-segment aganglionosis. Fig. 4
After rectal irrigations for 5 days (Nursing), the ab-
dominal distension disappeared completely and the
newborn was operated on at the age of 14 days. The
procedure was a one-stage primary transanal Soave car-
ried out under laparoscopic assistance. Laparoscopy is
necessary to adequately mobilize the descending colon,
the splenic flexure and the transverse colon, to perform
safe ligations of the vessels, and to ensure that there is
no intra-abdominal twisting of the pull-through bowel. Fig. 5
Figure 4 is the operative view of the transanal Soave.
Biopsy of the frozen sections confirmed the presence
of ganglionic cells at the pull-through bowel. Figure 5
shows the surgical specimen, which was 31 cm long and Suggested Reading
showed a long narrow colon, the transitional zone, and 1. Lister J, Tam PKH (1990) Hirschsprung’s disease. In: neo-
the dilated colon. natal surgery (eds J Lister and IM Irving). Butterworths,
The child had an increased frequency of defecation London, 523–546
(eight times daily) with peri-anal irritation during the 2. Nunez R, Cabrera R, Moreno C, Agulla E, Vargas I, Blesa E.
first four postoperative months. After a 2-year follow-up, Usefulness of anorectal manometry in the neonatal diagno-
the child has two bowel movements daily and he is free sis of Hirschsprung disease. Cir Pediatr 2000; 13:16–9
of symptoms. 3. Puri P (1996) Hirschsprung’s disease. In: newborn surgery
(edited by P. Puri). Butterworths-Heinemann, Oxford,
363–378
3 Abdomen 117

Q 55
Abdellatif Nouri and Mongi Mekki

Fig. 1 Fig. 2

A 4-month-old boy had a history of alimentary vomiting • What does Fig. 1 show?
and recurrent bronchopneumopathy since birth. • Why did the pediatrician ask for the second chest ra-
The mother reported that a medical treatment for diograph?
supposed gastroesophageal reflux had been prescribed to • What does Fig. 2 show?
her infant without any improvement in his symptoms. • What diagnosis is suspected?
The physical examination revealed Down syndrome • Which investigation can confirm the diagnosis?
with a cutis excavatum and inspiratory crackles on pul- • How do you explain the alimentary vomiting?
monary auscultation. • Which other malformations can be associated with
The pediatrician obtained a postero-anterior recum- this pathological condition?
bent chest radiograph (Fig. 1) and completed the investi- • How do you manage this condition?
gation with a lateral recumbent chest radiograph (Fig. 2).
118 Abdomen

A 55

Figure 1 shows a gas-filled mediastinum which is con-


tinuous with abdominal gas.
The pediatrician obtained a lateral recumbent chest
radiograph (Fig. 2) to determine whether the air-filled
appearance is posterior, suggesting a hiatal hernia, ante-
rior suggesting a retrosternal hernia, or anterior and pos-
terior, generally suggesting a large posterolateral hernia.
Figure 2 shows that the abdominal gut loops are in
continuity, just behind the xiphoid process and the ster-
num, with the anterior air-filled mediastinum.
A retrosternal hernia is suspected.
This diagnosis can be confirmed by rectocolic opaci-
fication, because a hernia usually contains the transverse
colon. This is shown in Fig. 3. If the colon is not involved
with the hernia, the best radiological exploration is three-
dimensional CT.
Other malformations that can be associated with
retrosternal hernia are pericardium defect, cardiac mal-
formations, sternum malformations, epigastric diastases
or omphalocele (Cantrell’s pentalogy), and trisomy 21.
The alimentary vomiting was explored by a barium
meal. It showed no gastroesophageal reflux, no intesti-
nal malrotation, and no supravaterian duodenal stenosis.
Vomiting may be explained by the compression of the
herniated intestine. Fig. 3
Surgical repair is indicated because of the risk of in-
carceration. Operative repair is easily performed through
an upper abdominal incision or by laparoscopy.
Figure 4 is a laparoscopic view of the retrosternal
hernia. It shows the diaphragmatic defect after reduc-
tion of the transverse colon.
The herniated viscera are easily reduced, the sac
should be resected, and the defect closed.
After the 3-year follow-up, the child was well and free
of symptoms.

Suggested Reading
1. Ipek T, Altinli E, Yuceyar S. Laparoscopic repair of Morgani
Larrey hernia: report of three cases. Surgery Today 2002; Fig. 4
32:902–5
2. Sahnoun L, Ksia A, Jouini R, Maazoun K, Mekki M,
Krichene I, Belghith M, Nouri A. Pediatric retro sternal
hernia. Arch Pediatr 2006; 13:1316–9
3. Yamashita K, Tsunoda T. Three dimensional computer im-
ages of Morgani hernia. Am J Surg 2004; 187:109–110
3 Abdomen 119

Q 56
Abdellatif Nouri and Mongi Mekki

Fig. 2

Fig. 1

A 4-year-old girl presented with a 1-month history of um-


bilical pain and abdominal distension without vomiting.
The physical examination revealed a protuberant
nontender abdominal mass that was dull to percussion Fig. 3
and mobile at palpation.
The surgeon performed the examinations shown in
Figs. 1–4.
• What does the recumbent abdominal radiograph
(Fig. 1) show?
• What does the abdominal ultrasound (Fig. 2) show?
• What does the abdominal CT scan (Figs. 3, 4) show?
• Which diagnosis can be suggested?
• How do you manage this pathological condition?

Fig. 4
120 Abdomen

A 56

Fig. 5 Fig. 6

Figure 1 shows an abnormal bulbing and opaque abdo-


men with an intestinal gas pattern pushed to the right.
The abdominal ultrasound (Fig. 2) shows a multiloc-
ular pelvi-abdominal intraperitoneal cyst with multiple
hypoechoic zones separated by echogenic septa.
The abdominal CT scan (Fig. 3) confirms the pres-
ence of a multilocular intraperitoneal cyst which con-
tains vascularized septa and pushes the intestine back-
ward and to the right. This cyst contains an intestinal
structure (Fig. 4).
An intraperitoneal multicystic lymphangioma is sus-
pected, especially because the cyst contains an intestinal
structure and its septa are vascularized. The mobiliza-
tion of the cyst by palpation and the vascularization of
the septa refute the diagnosis of compartmentalized as-
cites. If the ultrasound and/or CT scan show(s) two nor- Fig. 7
mal ovaries, the diagnosis of ovarian cyst can be ruled
out. The aspect of a thin wall delimiting the cyst refutes
the diagnosis of intestinal duplication.
Ultrasound and CT are highly sensitive in the diag- (Fig. 6). Aspiration of the cyst permitted exteriorization
nosis and preoperative evaluation of an abdominal mul- of the lymphangioma and the ileal loop via an umbili-
tilocular cyst. For an abdominal localization in children, cal opening and complete removal of the lymphangioma
MRI is indicated only in the case of hemorrhagic com- and the ileal loop with intestinal anastomosis.
plications of the cyst. The surgical specimen (Fig. 7) shows a “butterfly as-
The most serious acute complication of this cyst is pect.” The pathology report confirmed the diagnosis of
intestinal volvulus. cystic lymphangioma.
Laparoscopy confirmed the diagnosis of mesenteric After a 2-year follow-up, the child is well and free of
multicystic lymphangioma (Fig. 5) holding an ileal loop symptoms.
3 Abdomen 121

Q 57
Abdellatif Nouri and Mongi Mekki

Fig. 1 Fig. 2, 3 Fig. 4

An 11-year-old girl presented with a 6-month history The surgeon performed the examinations shown in
of alimentary vomiting, abdominal pain, anorexia, and Figs. 1–4.
weight loss. • What do Figs. 1 and 2 show?
The mother has noticed epigastric distension, which • What do Figs. 3 and 4 show?
had gradually increased in size. Four days before consul- • What are the three important clinical signs that
tation, the patient experienced intolerance to food with should be sought systematically for the diagnosis?
bilious vomiting and abdominal pain. • Is there any other investigation that will help confirm
The physical examination revealed a nervous and pale the diagnosis?
patient with a large, firm, relatively mobile and tender • How can you explain the iron-deficiency anemia?
mass located in the left upper quadrant of the abdomen. • Are there any differential diagnoses?
Laboratory test results showed iron-deficiency ane- • How is this condition best managed?
mia (hemoglobin, 6 g/dl) and hyperleukocytosis (WBC,
15.6 103/mm3).
122 Abdomen

A 57

Fig. 7

Fig. 5

Fig. 6

This girl is affected by trichobezoar, which is a conglom- diagnoses are easily ruled out by ultrasound and barium
erate of swallowed hair because of a mental disturbance. meal study.
The abdominal plain radiograph (Fig. 1) shows the After correction of the hydro-electrolytic disorders
absence of gastric bubble and the presence of air–fluid and transfusion, laparotomy revealed a huge dilation
intestinal levels. In association with bilious vomiting, of the stomach containing a bezoar. The intestinal oc-
these levels confirm the intestinal occlusion. The ab- clusion was due to a prolongation of the bezoar to the
dominal ultrasonography (Fig. 2) shows an echogenic, duodenum and the proximal jejunum, which was di-
dense, solid mass in the stomach with posterior clean lated and necrotic because of a large hair ball (10 cm in
sonic shadowing. diameter). A longitudinal gastrotomy allowed removal
Figures 3 and 4 show a large bezoar outlined by bar- of the bezoar (Fig. 6). Resection of the necrotic proximal
ium, occupying the stomach and extending to the pylo- jejunum allowed removal of the hair ball with a 60-cm
rus and the duodenum. tail (Fig. 7). After jejunal anastomosis and closure of the
The three important clinical signs which should be gastrotomy, the evolution was uneventful.
sought systematically are a history of active trichopha- No attempt should be made to disrupt a large bezoar
gia, a patchy baldness from hair pulling, and an emo- endoscopically because of the risk of detaching frag-
tional disturbance. ments and forming satellite intestinal lesions. For the
When trichophagia is denied by the patient, gastros- same reasons, the prescription of pancreatic enzymes to
copy can show the trichobezoar and confirm the diag- dissolve a bezoar is not recommended. Psychiatric care
nosis (Fig. 5). is often indicated for the long term in the case of psycho-
The iron-deficiency anemia can be explained by mal- logical problems so as to avoid recurrence.
nutrition and chronic blood loss from erosions and ul- At the 8-year follow-up, the patient was well and free
cerations of the gastric mucosa. of symptoms; her psychiatrist still recommended psy-
The differential diagnosis includes gastric duplica- chotherapeutic measures.
tion, gastric tumor, and pancreatic pseudocyst. All these
3 Abdomen 123

Q 58
Luciano Mastroianni and Alba Cruccetti

Fig. 1 Fig. 2 Figs. 3, 4

An 8-year-old boy was brought to the emergency depart- The patient was transferred to the department of surgery
ment complaining of abdominal pain and high tempera- where he had loose stools and melena.
ture which had started a few hours earlier. • What would you do next?
On examination, the abdomen was not distended, a • What do Figs. 3 and 4 show?
palpable mass was present and it was movable and ten- • What pathological condition is affecting this child?
der. Rectal examination was normal. • What is the best way to manage this condition?
The first radiological investigations performed are
shown in Figs. 1 and 2.
• What do Figs. 1 and 2 show?
124 Abdomen

A 58

This child had an ileal intestinal duplication. Figure 1


is an abdominal ultrasound scan revealing a large
mass with an inhomogeneous structure measuring
18×10×7 cm and was located in the abdomen.
Figure 2 shows an abdominal radiograph with dis-
location of the bowel because of the presence of an ab-
dominal mass without calcification.
The MR imaging scan showed a large mass located
in the pelvis and in the central abdomen (Fig. 3) with
an inhomogeneous signal, a mixed mass with predomi-
nantly liquid structure, and the presence in the lesion of
an air–fluid level (Fig. 4).
The child underwent laparotomy. An ileal duplication
(Fig. 5) was found. Subsequently, resection of the dupli-
cation and the adjacent bowel with end-to-end anasto- Fig. 5
mosis of the remaining bowel was performed.
The postoperative period was uneventful and
9 months after surgery the child was doing well.
Intestinal duplication is a rare congenital anomaly
that may arise anywhere along the gastrointestinal tract.
Most duplications cause symptoms in infancy or early
childhood, but some present as an incidental finding.
The symptoms and signs of presentation depend on the
type and location of the duplications, which include ob-
struction, hemorrhage, infection, perforation, and an
asymptomatic mass. It is important to include intestinal
duplications in the differential diagnosis of gastrointesti-
nal bleeding, especially in children and when associated
with an abdominal mass.

Suggested Reading
1. Brown RL, Azizkhan RG. Gastrointestinal bleeding in in- 4. Pinter AB, Schubert W, Szemledy F, Gobel P, Schafer J,
fants and children: Meckel’s diverticulum and intestinal du- Kustos G. Alimentary tract duplications in infants and chil-
plication. Semin Pediatr Surg 1999; 8:202–209 dren. Eur J Pediatr Surg 1992; 2:8–12
2. Iyer CP, Mahour GH. Duplications of the alimentary tract 5. Wardell S, Vidican DE. Ileal duplication cyst causing massive
in infants and children. J Pediatr Surg 1995; 30:1267–70 bleeding in a child. J Clin Gastroenterol 1990; 12:681–4
3. Long L, Zhang JZ and Wang YX. Vascular classification for
small intestinal duplications: Experience with 80 cases. J
Pediatr Surg 1998; 33:1243–1245
3 Abdomen 125

Q 59
Luciano Mastroianni and Alba Cruccetti

Fig. 3
Fig. 1 Fig. 2

A 49-day-old boy without a significant medical history Eight hours after abdominal decompression with a na-
developed abdominal distension and vomiting. He was sogastric tube and a rectal probe, the patient showed no
a preterm infant of 30 weeks’ gestational age and a birth improvement in symptoms.
weight of 1,700 g. • What would you do next?
On examination, distended loops of gut were visible • Why did the surgeon perform the second examina-
and palpable, bowel sounds were present, and the abdo- tion?
men was slightly tender. • What do Figs. 2 and 3 show?
The first radiological investigation performed is • What pathological condition is affecting this child?
shown in Fig. 1. • What is the best way to manage this condition?
• What does Fig. 1 show?
126 Abdomen

A 59

This infant is affected by intestinal obstruction. In par-


ticular, he is affected by a distal intestinal obstruction;
the lower the obstruction, the greater the distended
loops of bowel observed.
Figure 1 is an abdominal radiograph showing with
several markedly distended loops of bowel and few air–
fluid levels.
Figures 2 and 3 show the second investigation per-
formed to distinguish between the different causes of
low intestinal obstruction: a contrast enema with Gas-
trografin, demonstrating a narrowing between the sig-
moid colon and the descending colon with distended
bowel proximal to the narrowed bowel, reflux of con-
trast through the ileocecal valve and into the terminal
ileum, filling several dilated loops. Fig. 4
The infant underwent laparotomy. A Meckel’s diver-
ticulum attached to the abdominal wall through a cord-
like fibrous remnant was found obstructing the sigmoid
colon where a stenosis was present (Figs. 4, 5). Diver-
ticulectomy and intestinal resection with anastomosis of
the stenotic bowel segment were performed.
The postoperative period was uneventful and 1 year
after surgery the child was well.
Meckel’s diverticulum should be considered in the
differential diagnosis of any previously healthy infant
without a history of surgery who develops a bowel ob-
struction.

Suggested Reading
1. Al-Zahem A, Arbuckle S, Cohen R. Combined ileal hetero- Fig. 5
topic pancreatic and gastric tissues causing ileocolic intus-
susception in an infant. Pediatr Surg Int 2006; 22:297–9
2. Ameh EA, Mshelbwala PM, Dauda MM, Sabiu L, Nmadu
PT. Symptomatic vitelline duct anomalies in children. S Afr 4. Moore TC. Omphalomesenteric duct malformations.
J Surg 2005; 43:84–5 Semin Pediatr Surg 1996; 5:116–23
3. Martinez Biarge M, Garcia-Alix A, Luisa del Hoyo M, Alar- 5. Sy ED, Shan YS, Tsai HM, Lin CH. Meckel’s diverticulum
con A, Saenz de Pipaon M, Hernandez F, Perez J, Quero associated with ileal volvulus in a neonate. Pediatr Surg Int
J. Intussusception in a preterm neonate; a very rare, major 2002; 18:529–31
intestinal problem—systematic review of cases. J Perinat
Med 2004; 32:190–4
3 Abdomen 127

Q 60
François Luks

Fig. 1 Fig. 2

A left adrenal mass was found on routine ultrasound in The mass remains unchanged on three further ultra-
a 24-week-old fetus. The remainder of the examination sound examinations, each 4 weeks apart.
was unremarkable (Fig. 1). The infant is delivered at term in a tertiary center. Ul-
A fetal MR imaging study was performed, which con- trasound on day 1 of life confirms the presence of the le-
firmed the presence of a left adrenal lesion of uncertain sion, which is unchanged from the prenatal images. MR
origin and of heterogeneous consistency (cystic ele- imaging at 2 weeks of life confirms the persistence of a
ments in a largely solid mass). lesion above the left kidney, possibly separate from the
• What is the differential diagnosis? adrenal gland (Fig. 2).
• What further diagnostic and therapeutic measures, if • What are the other diagnostic and therapeutic op-
any, would be indicated? tions after birth?
• If surgical intervention is indicated, what are the dif-
ferent approaches?
• Is there a place for nonoperative management of this
lesion?
128 Abdomen

A 60

Fig. 3 Fig. 4

The differential diagnosis of a fetal adrenal mass is rela- With a working diagnosis of neuroblastoma, two
tively narrow. The most common lesion of the fetal ad- therapeutic approaches are possible. Adrenalectomy in
renal gland is a neuroblastoma. Adrenal hemorrhage the newborn period is feasible and can be performed
can resemble a lesion, and a cause for the hemorrhage through a retroperitoneal approach (open or laparos-
is often not found. However, the appearance of an ad- copy). However, the “benign” course of most infant neu-
renal hemorrhage tends to change over time as the he- roblastomas has paved the way for a nonoperative ap-
matoma is reabsorbed, showing more cystic features as proach to congenital neuroblastoma, on the assumption
it decreases in size. The third and possibly most com- that these lesions will spontaneously regress.
mon lesion in that region is an intra-abdominal pul- If a conservative approach is used, it is important to
monary sequestration. This is a variant of an extralobar follow up the infant closely (ultrasound every 3–4 weeks
sequestration, typically found in the basal region of the and at least two MR imaging studies) and to measure
thoracic cavity. As the diaphragm closes, a sequestra- urinary catecholamines (VMA and HVA).
tion may become encased in the posterior mediasti- Intra-abdominal pulmonary sequestrations are obvi-
num or even become separated from the thoracic cavity ously benign lesions. However, up to 50% of sequestra-
altogether. tions contain elements of cystic adenomatoid malforma-
If there are no other fetal anomalies, further testing tion (CCAM), which may have a potential for malignant
is not strictly speaking necessary. However, close follow- degeneration. The exact incidence of malignancies devel-
up may be indicated: as mentioned, an adrenal hemor- oping into a CCAM is unknown and is likely very small,
rhage will tend to decrease in size through the remainder but real. Therefore, several authors advocate elective re-
of the pregnancy. Fetal neuroblastomas almost always section if the lesion does not spontaneously regress.
behave in a nonaggressive fashion (similar to neuroblas- This child was followed up for 6 months, at which
tomas in infants), but rapid growth and fetal or placental time an ultrasound showed persistence of the lesion.
metastases have been described. He underwent a retroperitoneal laparoscopic resection
A fetal MR imaging study may help further define the of the mass at 9 months of age (Figs. 3, 4). In Fig. 3, the
lesion, although cystic elements can be seen in all three relationship between the left kidney, the adrenal gland,
conditions (including cystic neuroblastoma). Finding a and the sequestration can be seen. The lesion was dis-
normal adrenal gland adjacent to the lesion essentially sected off the left adrenal gland, as shown in Fig. 4.
rules out adrenal hemorrhage of neuroblastoma, but an On pathological examination, a diagnosis of pulmo-
MR image is usually not very sensitive for small lesions nary sequestration with CCAM elements (hybrid lesion)
or glands. was confirmed.
3 Abdomen 129

Q 61
François Luks

Fig. 1

On a routine second-trimester ultrasound (Fig. 1), an


abdominal wall defect was noted.
• How can the main types of abdominal wall defects be
differentiated by prenatal examination?
• What additional testing would you recommend if the
lesion appears to be gastroschisis? And if the lesion is Fig. 2
an omphalocele?
• How would you counsel the future parents if the le-
sion is an omphalocele? • How does this information affect your counseling,
• What would you estimate the mortality rate to be, and what precautions would you recommend at the
and what degree of morbidity would you predict? time of delivery?

On subsequent ultrasound examination, the lesion is con- The infant is born at 36 weeks (Fig. 2).
firmed to be a giant omphalocele, and approximately 85% • What immediate and medium-term therapeutic mea-
of the liver mass is seen outside the abdominal cavity. sures would you recommend?
130 Abdomen

A 61

Fig. 3

With current ultrasound technology, it is possible to de-


termine whether the umbilical cord insertion is lateral to
the abdominal wall defect (in which case the diagnosis Fig. 4
is gastroschisis) or whether the cord is inserted at the
apex of the defect itself (omphalocele).
Typically, gastroschisis is an isolated lesion, and no
further work-up is required, except for a complete ul-
trasound assessment of the fetus. Omphalocele, how-
ever, is associated with other anomalies in about 70% of
cases. These anomalies can be chromosomal (trisomy
18 is the most common) or part of a complex syndrome
(Pentalogy of Cantrell, from a fusion defect in the up-
per, or epigastric, abdominal fold; cloacal exstrophy, a
lower abdominal folding defect). In more than a third
of omphaloceles, there is an associated cardiac mal-
formation. The prognosis of infants born with an om-
phalocele depends for the most part on the associated
anomalies.
If a large portion of the liver is exteriorized, these le- Fig. 5
sions are called “giant” omphaloceles. They are not asso-
ciated with a higher mortality, but difficulties in reducing
the viscera may lead to significant morbidity in the first 6– In the present case, the intact membrane offered a
18 months of life. Although a cesarean section is no lon- natural barrier. Wrapping of the abdomen with an elas-
ger mandatory for abdominal wall defects, the presence tic bandage offered graded pressure on the defect, to
of a large portion of the liver in the defect could increase start reduction of the viscera.
the risk of rupture and hemorrhage with vaginal delivery. At 2 weeks of life, a tissue expander was inserted in
Most centers will therefore recommend C-section. the pelvis through a small Pfannenstiel incision (Fig. 3).
Management of a giant omphalocele can be challeng- The expander was enlarged by 10 ml each day through
ing, and immediate reduction of the liver may lead to an external reservoir (Fig. 4). At 4 weeks of life, surgical
acute kinking of the hepatic veins and the vena cava. A reduction of the defect was achieved and the abdominal
more gradual approach is therefore necessary. wall fascia could be closed (Fig. 5).
3 Abdomen 131

Q 62
Mario Lima and Giovanni Ruggeri

Fig. 1 Fig. 2

An 8-year-old girl had a 6-month history of progressive A barium swallow was performed (Figs. 1, 2).
dysphagia, at first for liquids and then for solid, associ- • What do Figs. 1 and 2 show?
ated with frequent regurgitations, substernal pain, and • Which other examinations were performed to reach
weight loss. She did not have any respiratory symptoms. a diagnosis?
At another hospital medical, therapy was initiated to re- • What do these examinations show?
lieve the dysphagia, but without any improvement. • What is the differential diagnosis of this disease?
The girl came to our attention because her symptoms • What kind of therapy would be possible for this dis-
worsened. ease?
• How did we treat this girl?
132 Abdomen

A 62

This girl is affected by esophageal achalasia.


Achalasia consists in a failure of relaxation in the dis-
tal esophagus and esophagogastric junction. The cause
is unknown, but most studies point to a primary neu-
rogenic abnormality with lack of inhibitory innervation
and progressive degeneration or loss of ganglion cells in
the myenteric plexus.
The barium swallow (Figs. 1, 2) shows a markedly di-
lated esophagus with a tapering at the lower end, which
has the appearance of a “rat tail.” There is no evidence of
gastroesophageal reflux.
Esophagoscopy was performed (see Fig. 3), which
showed lots of retained undigested food; in the lower
part of the esophagus a stenosis was seen, but the instru- Fig. 3
ment could easily pass through. A congenital stenosis
could be excluded.
Esophageal manometry is the diagnostic examina-
tion of choice for this disease: it shows hypertension of
the lower esophageal sphincter (LES) and incomplete or
absent relaxation of the LES together with disordered,
weak, or absent peristaltic contractions in the body of
the esophagus after swallowing.
The differential diagnosis of achalasia includes Cha-
gas’ disease, a South American parasitic infection caused
by Trypanosoma cruzi..
Pharmacologic management (nifedipine) of achalasia
is almost always unsuccessful. Pneumatic dilation under
endoscopic guidance is an alternative to surgical treat-
ment; it is seldom successful and the risk of major compli-
cations (esophageal perforation, mediastinitis) is high. Fig. 4
We performed laparoscopic Heller myotomy (Fig. 4)
associated with a Dor anterior hemifundoplication. We
added pharmacological antireflux therapy (ranitidine) Suggested Reading
in the immediate postoperative period. 1. Boeckxstaens GE et al. Achalasia: from new insights in
After 1 year, we performed endoscopy, which showed pathophysiology to treatment. J Pediatr gastroenterol Nutr
a patent esophagus, no supracardial dilation, and no dif- 2005; 41:S36–7
ficulty in instrument insertion. At the time of writing, 2. Mattioli G et al. Results of the laparoscopic Heller-Dor
the girl was feeding regularly, without any problems, had procedure for pediatric esophageal achalasia. Surg Endosc
a good weight gain, and was well and free of symptoms. 2003; 17:1650–52
3. Smith CD et al. Endoscopic therapy for achalasia before
Heller myotomy results in worse outcomes than Heller my-
otomy alone.Ann Surg 2006; 243:579–84
3 Abdomen 133

Q 63
Mario Lima and Giovanni Ruggeri

Fig. 2

Fig. 1

A 1-year-old child suffered from dyspnea, cough, mu- • What does Fig. 1 show?
cous secretions, and regurgitations for 2 weeks. • What other diagnostic examinations could be use-
The pediatrician, suspecting a pulmonary infection, ful?
administered antibiotic therapy. Because the symptoms • What examinations are to be avoided and why?
persisted, the child was taken to the emergency depart- • What did the surgeon do?
ment where a chest radiograph was obtained (Fig. 1). • What are the risks of this procedure?
• What are the possible surgical alternatives?
• How can possible complications be managed?
134 Abdomen

A 63

Fig. 3

Fig. 4 Fig. 5 Fig. 6

The girl had ingested a spring. Her parents had not real- difficulties, because the spring was strongly embedded
ized that their daughter had ingested a part of a small toy in the esophageal mucosa and part of the inflamed mu-
she was playing with; in fact, this toy was later found to cosa stuck out through the loops of the spiral.
be broken and some parts were missing. The removal procedure is not free of possible com-
The radiograph (Fig. 1) clearly shows the spiral shape plications; the risk of causing an esophageal laceration is
of a radiopaque, ingested foreign body (a spring); espe- high, as well as of creating a fistula between the esopha-
cially in lateral view (Fig. 2), it is clear that the foreign gus and the aorta.
body is in the proximal part of the esophagus. If removal had not been possible, or in the case of a
Before attempting removal, it is important to study major complication, the surgeon would have been ready
the relationships between the foreign body and the other to perform a thoracotomy to open the esophagus, reach
structures of the mediastinum, in particular the aortic and remove the foreign body, and/or to repair any com-
arch. For this reason, a CT scan (see Fig. 3) was per- plication. Luckily, this was not our case.
formed. It shows the radiopaque body in the proximal In the postoperative period, the child developed an
part of the esophagus; it seems to lean on the aortic arch esophageal stricture, which was documented by a bar-
but there is an intact esophageal wall. Obviously, MR ium swallow (see Fig. 4). This stricture was managed
imaging, although less detrimental to health, could not with periodical dilations with Savary’s bougies (Fig. 5)
be performed because the radiopaque body was metallic, and corticosteroid therapy. The dilations were successful
and this is an absolute contraindication to MR imaging. (Fig. 6), and at the time of writing the child was feeding
The surgeon removed the foreign body endoscopically without any problems.
with a rigid endoscope. The procedure was not without
3 Abdomen 135

Q 64
Vincenzo Jasonni, Girolamo Mattioli, and Alessio Pini Prato

Fig. 1 Fig. 2

A 3-month-old girl had a history of chronic constipation • What investigation was performed? What do Figs. 1
dating back to birth and delayed meconium passage. and 2 show?
She was in a good general condition with a slightly • What should the first diagnostic step be in a case like
distended belly. Up to the time of referral, she was man- this one?
aged with daily enemas for regular bowel emptying. Her • What are the second-line investigations?
family history was unremarkable. • What are possible explanations of severe chronic
Figures 1 and 2 show an investigation performed on constipation? What is the disease?
this infant. • What other diagnostic investigations can be per-
formed in the work-up of such a patient?
• What is the suggested treatment and are there differ-
ent therapeutic options?
• What are the long-term implications of the disease?
136 Abdomen

A 64

This patient is affected by Hirschsprung’s disease (HSCR).


In particular, the infant is affected by a classic form of
HSCR extending up to the sigmoid colon, as confirmed
by the intraoperative histochemical study of seromuscu-
lar biopsy specimens taken during surgery. HSCR dis-
ease occurs in up to 1:5,000 live births and is due to an
abnormal development of the enteric nervous system
(absence of migration of ganglion cells in the Auerbach
and Meissner plexuses—aganglionosis). It can present
early after birth or later in childhood.
This infant underwent a barium enema study that
showed an unclear picture: slightly dilated sigmoid co-
lon and absence of stenosis or atresia. There are no clear Fig. 3
transition zones to allow a diagnosis of intestinal dys-
ganglionosis (Hirschsprung’s disease) to be made.
The first diagnostic step should have been a rectal suc-
tion biopsy that permits a reliable diagnosis to be made or for Ret mutations allows one to discriminate between
exclusion of intestinal dysganglionosis (Hirschsprung’s sporadic HSCR and inherited HSCR. This can be helpful
disease and intestinal neuronal dysplasia). Figure 3 is a in the counseling of parents who fear recurrence of the
typical histogram of HSCR, where the rectal suction bi- disease. Cardiac anomalies can be associated with HSCR
opsy specimen is stained in acetylcholinesterase (AChE; in up to 10% of patients and these anomalies should be
histochemical stains for parasympathetic fibers and gan- ruled out. HSCR will not resolve on its own. Therefore,
glion cells). A dramatic increase of AChE activity in the surgery is the only treatment. There are many available
lamina propria and in the submucosa along with the ab- pull-through procedures to resect the affected bowel
sence of ganglion cells permits one to diagnose HSCR. and to re-establish bowel continuity. Laparoscopic, lapa-
A barium enema should follow the rectal suction bi- rotomic, and transanorectal approaches are safe and ef-
opsy for any possible bowel involvement of agangliono- fective in experienced hands.
sis to be identified. Although HSCR has an excellent prognosis, parents
Severe chronic constipation in patients younger than should be aware of the possible occurrence of long-term
6 months of age can occur also in cases of cystic fibrosis, complications such as constipation, enterocolitis, and
hypothyroidism, electrolyte disturbance, anorectal mal- soiling. These complications (mainly soiling) usually
formations (rectoperineal fistula), and infant dischezia improve with time. Overall quality of life is very good
(functional disorder). in these patients.
In this case, there is no need for further investiga-
tions as the diagnosis is complete. However, searching

Suggested Reading
1. Martucciello G, Pini Prato A, Puri P, Holschneider AM, 2. Martucciello G, Ceccherini I, Lerone M, Jasonni V. Patho-
Meier-Ruge W, Jasonni V, Tovar JA, Grosfeld JA. Contro- genesis of Hirschsprung’s disease. J Ped Surg 2000; 35(7):
versies concerning diagnostic guidelines for anomalies of 1017–1025
the enteric nervous system: a report from the fourth Inter-
national Symposium on Hirschsprung’s disease and related
neurocristopathies. J Ped Surg 2005; 40(10):1527–1531
3 Abdomen 137

Q 65
Vincenzo Jasonni, Girolamo Mattioli, and Alessio Pini Prato

Fig. 2

Fig. 1

A 2-year-old girl had a history of chronic constipation • What was the first investigation performed by the pe-
dating back to birth. She was sent to a specialist pediat- diatric surgeon?
ric surgeon at the age of 2.5 years for a suspected anorec- • What does Fig. 1 show?
tal malformation with rectoperineal fistula. • Why did the surgeon perform the second investiga-
Her mother had an anterior meningocele that had tion?
been treated conservatively (observation and cesarean • What does Fig. 2 show?
section). A cousin (mother’s side) had undergone sur- • What other diagnostic examinations should be per-
gery for a mild form of anorectal malformation. Nothing formed in this specific case?
else was described in her family history. • What is the right timing for the treatment of this dis-
ease?
• What is the long-term outcome of these patients?
138 Abdomen

A 65

This girl is affected by Currarino syndrome (CS). CS is a terostomy. Barium enema is mainly required to exclude
congenital disease, frequently inherited, due to a muta- rectosigmoid dilatation and to assess colonic shape.
tion on the long arm of chromosome 7 (gene HLXB9). Figure 2 shows mild rectosigmoid dilatation and no
It is a peculiar form of caudal regression syndrome sign of Hirschsprung’s disease.
that involves the last lumbar vertebral bodies and the Rectal suction biopsy (exclusion of Hirschsprung’s
sacrum/coccyx. It is characterized by the association disease), spinal MR imaging (to look for anterior me-
of hemisacrum, anorectal malformation, and presacral ningocele), pelvic CT (presacral teratoma), neurophysi-
mass (anterior meningocele or teratoma). Patients with ological tests, cardiac US, thoracic radiography, and ab-
this disease often present with isolated severe chronic dominal US (to exclude associated anomalies) should be
constipation dating back to birth. Frequently, there is a performed to complete the diagnostic work-up.
family history of spinal dysraphism or anorectal malfor- In cases of associated anterior meningocele, neuro-
mations. The disease is extremely rare. surgical treatment should be performed first, with or
On the grounds of the suggestive family history and without protective colostomy. This patient had anterior
clinical examination, the pediatric surgeon acquired a meningocele, spinal tethering lipoma, and presacral ter-
sacral radiograph (two projections). atoma. She did not undergo colostomy, but underwent
Figure 1 shows the hypoplastic hemisacrum (com- closure of the meningocele and untethering first. After-
plete absence of right portion of sacral bone). The lat- ward, she underwent teratoma resection and posterior
eral projection is used to quantify the degree of sacral sagittal anorectoplasty.
hypoplasia in symmetric forms of caudal regression The long-term outcome of these patients is excellent,
syndrome. This investigation alone is sufficient for the provided the teratoma is completely removed and no
diagnosis of CS, since this feature is pathognomonic. malignancy is detected. Strict neurological follow-up is
CS can be associated (more than 50% of patients) with required, at least in the early postoperative period, for
Hirschsprung’s disease. Furthermore, the presence of an early detection of neurological deterioration. In cases of
associated anorectal malformation (rectoperineal fistula associated anorectal malformations, constipation is very
in this patient) makes the development of severe recto- common but it can be dealt with effectively.
sigmoid dilatation possible, which sometimes needs en-

Suggested Reading
1. Martucciello G, Torre M, Belloni E, Lerone M, Pini Prato 2. Merello E, De Marco P, Mascelli S, et al. HLXB9 homeobox
A, Cama A, Jasonni V. Currarino syndrome: proposal of a gene and caudal regression syndrome. Birth Defects Res A
diagnostic and therapeutic protocol. J Pediatr Surg 2004; Clin Mol Teratol 2006; 76(3):205–9
39(9):1305–11
3 Abdomen 139

Q 66
Vincenzo Jasonni, Girolamo Mattioli, and Alessio Pini Prato

Fig. 2

Fig. 1

An 8-year-old boy had a long-standing history of vomit- The pulmonologist performed the examination
ing, regurgitation, recurrent respiratory infections, and shown in Figs. 1 and 2.
failure to thrive. The onset of his symptoms dated back • What do Figs. 1 and 2 show?
to the age of 5 years. • The patient was subsequently referred to a surgeon.
The patient’s mother told the doctors that her child What is the second examination performed?
underwent repeated cycles of antireflux medical therapy • What should be included in the diagnostic work-up
(PPI and prokinetics) with partial and transient benefit. of this condition?
He was brought to the emergency department after • What are the main etiological considerations of this
an episode of acute respiratory distress. The pediatrician condition?
obtained a chest radiograph that showed a right lobar • How should this condition be managed?
pneumonia. The patient was treated accordingly and af- • Are there any therapeutic options?
terwards he was referred to a pulmonologist for his re-
current symptoms.
140 Abdomen

A 66

(narrowing of the distal esophagus at the esophagogastric


junction). Moreover, the esophagus is clearly depicted,
as demonstrated by the contrast level in the lumen.
When EA is suspected, one should immediately per-
form esophageal manometry in order to assess esopha-
geal motility. Figure 3 shows the typical manometric
findings in patients with EA: low-amplitude synchro-
nous esophageal contractions and absence of postdeglu-
tive LES relaxations.
To rule out GER, 24-h pH monitoring should be per-
formed. Moreover, these patients should always undergo
esophago-gastroduodenoscopy to rule out the presence
of stenosis or strictures and to confirm the diagnosis.
Fig. 3 Endoscopic assessment is typically unremarkable and
the tube easily passes from the esophagus to the stom-
ach. This is pathognomonic, as the cause of EA is func-
The patient is affected by esophageal achalasia (EA). This tional and not anatomic.
disease is characterized by abnormal synchronous mo- EA is a rare condition in childhood. Only 5% of the
tor activity of the esophageal body, associated with an patients suffering from this disease are younger than
absence of postdeglutive relaxation of the lower esopha- 15 years of age. Although EA is usually considered an
geal sphincter (LES). The achalasia of the LES deter- acquired esophageal motility disorder, several studies
mines the progressive dilatation of the esophageal body suggested that genetic background may play a role, at
and deterioration of the esophageal clearing capacity. least in children. The etiology and pathogenesis remain
This boy is affected by a severe form of AE, as the controversial; however, the abnormal esophageal motil-
symptoms date back to early childhood. The previous ity in EA seems to result from defects or an imbalance
episodes of respiratory symptoms are suggestive of the between the excitatory and inhibitory neuromuscular
severity of the disease. Gastroesophageal reflux (GER) is transmitters.
not the cause of this condition. Therefore, EA does not Different treatments have been proposed for EA:
respond to antireflux therapy. pharmacological treatments, pneumatic dilatations, re-
EA can present with dysphagia. Alternatively, respira- movable self-expanding metal stents, and injection of
tory symptoms can occur. Aspiration pneumonia usually Botulinum toxin. The results are transitory and repeated
involves the right lung due to the predisposing anatomy. treatments are frequently required. The only way to de-
In the case of suspected GER, the barium meal is one finitively relieve symptoms is surgery. Anterior extramu-
of the main examinations to be performed in order to cosal myectomy according to Heller, in association with
detect GER and to study the GE junction and any pos- a partial anterior fundoplication according to Dor, is
sible anatomical abnormalities. the most effective option. At present the laparoscopi-
Figures 1 and 2 show the barium meal test. The esoph- cally modified Heller–Dor procedure is the treatment of
agus is extremely dilated, with the typical mouse-tail sign choice for EA.

Suggested Reading
1. Mattioli G, Esposito C, Pini Prato A, Doldo P, Castagnetti 2. Mattioli G, Cagnazzo A, Barabino A, Caffarena PE, Ivani G,
M, Barabino A,Gandullia P, Staiano AM, Settimi A, Cucchi- Jasonni V. The surgical approach to esophageal achalasia.
ara S, Montobbio G, Jasonni V. Results of the laparoscopic Eur J Pediatr Surg 1997; 7:323–7
Heller-Dor procedure for pediatric esophageal achalasia.
Surg Endosc 2003; 17(10):1650–2
3 Abdomen 141

Q 67
Vincenzo Jasonni, Girolamo Mattioli, and Alessio Pini Prato

An 18-year-old girl had been operated on at the age of


10 years for total colonic Hirschsprung’s disease. She
had an uneventful course and grew properly up to the
time of presentation. Five moths before referral, she
experienced an episode of bowel obstruction related to
intestinal adhesions. She underwent laparotomy but did
not recover completely.
She presented to the emergency department with bil-
ious vomiting, soft and nondistended abdomen, and ab-
sence of bowel movement that had started 5 days earlier.
The pediatric surgeon at the emergency department
acquired an abdominal radiograph.
• What does Fig. 1 show?
• What was the next investigation?
• What are the possible diagnoses?
• What is a possible treatment?
• What was the course of this girl?

Fig. 1
142 Abdomen

A 67

Fig. 2 Fig. 3 Fig. 4

Figure 1 shows the patient, who had undergone a col- passage of contrast medium distally to the Treitz liga-
ectomy and ileal pull-through carrying an ileal loop ment. Figure 2 is an anterior–posterior projection and
stretched down to the lower rectum. During growth the Fig. 3 is a latero-lateral view of the same situation.
stretched loop and its vessels can press and fall onto the There are different possibilities to explain this situa-
duodenum, thus producing a clinical picture that resem- tion: proximal adhesions, previously asymptomatic web,
bles that of aortomesenteric compression. aortomesenteric compression, expanding lesion, and
This girl is affected by an upper gastrointestinal (GI) neurogenic abnormalities (the patient was previously
obstruction. Bilious vomiting without abdominal dis- operated on for Hirschsprung’s disease).
tension supports this suspicion. The previous laparoto- The child kept on vomiting and became dehydrated.
mies could also suggest the possibility of a proximal ad- The surgeon therefore decided to perform a laparotomy,
hesion. which confirmed the suspicion of aortomesenteric com-
An abdominal radiograph with the patient standing pression (normal progression of a tube through the
is the most useful investigation to be performed in cases duodenum—the obstruction was functional more that
of intestinal obstruction as it gives information regard- extrinsic or anatomic). A duodenojejunal anastomosis
ing the level and degree of obstruction. was effectively performed on the extremely dilated duo-
Figure 1 does not show any air-filled levels and looks denum. Alternatively, a gastrojejunal anastomosis could
almost normal. The only abnormality is the presence of have been performed.
poor air content. The rectum is empty and the stomach The girl recovered very slowly. Figure 4 shows the
contains a normal amount of air. passage of contrast medium down in the small bowel,
An upper GI barium meal was performed in order to distal to the anastomosis, and the significant improve-
better understand the level of the obstruction. Figures 2 ment of the duodenal dilatation.
and 3 show an enormous dilated duodenum with no
3 Abdomen 143

Q 68
Vincenzo Jasonni, Girolamo Mattioli, and Alessio Pini Prato

An 8-year-old boy had a recent onset of recurrent ab-


dominal pain. In the past, he had experienced some epi-
sodes of vomiting and regurgitation, but everything else
was normal. Growth was within the normal range. His
family and personal history was unremarkable.
He was brought to the pediatrician because of recur-
rent abdominal pain and constipation. The doctor asked
for an abdominal US.
• What do Figs. 1 and 2 show?
• What was done next?
• What does Fig. 3 show? Fig. 1
• What should be done next?
• What condition is affecting this child?
• What are the risks and implications of the disease?

Fig. 2

Fig. 3
144 Abdomen

A 68

This child is affected by intestinal malrotation. Intestinal A barium meal is the next diagnostic step to verify
malrotation is the result of a failure of the 270-degree the degree of malrotation.
anticlockwise rotation the usually occurs prenatally. If In the case, the barium meal was performed after US
this rotation does not occur, different forms of malro- in order to assess the degree of malrotation and to ex-
tation can develop (non-rotation, partial malrotation, clude torsion of the mesenteric vessels. Figure 3 shows
etc.). Malrotation can represent a dramatic acute prob- the typical radiological picture of a patient with malrota-
lem in the newborn period but also afterwards if midgut tion. The duodenum is malpositioned and fails to bend
volvulus occurs. Its occurrence is facilitated by the pre- on the left hand side. This situation can be associated to
disposing anatomy of the gut in this subset of patients. different anomalies of cecal fixation.
Moreover, malrotation is associated with the persistence Once the diagnosis is reached, surgery should be per-
of the Ladd’s band that can cause duodenal obstructive formed (also in asymptomatic patients) in order to avoid
symptoms and secondary GER. Therefore, malrotation and prevent the possible occurrence of midgut volvulus,
can produce chronic GER-like symptoms, recurrent ab- a potentially fatal complication (if not promptly recog-
dominal pain, or represent a dramatic emergency. nized and treated). The surgical treatment of malrotation
It is well known that asymptomatic intestinal abnor- involves division of Ladd’s band, appendectomy, widen-
malities occur in up to 10% of healthy subjects and that ing of the mesentery, and positioning of the colon on the
they are frequently incidental findings. This is one of the left hand side and ileum on the right hand side.
possible ways to diagnose malrotation. Midgut volvulus is rare but represents one of the most
The most frequent incidental diagnosis of malrota- serious and severe emergencies in abdominal surgery.
tion is made during a routine abdominal US (performed When misdiagnosed, it can be fatal or it results in short
for abdominal pain, trauma, or urinary infections). Ab- bowel syndrome. Although a percentage of patients with
dominal US and color-Doppler demonstrate the abnor- unknown malrotation will not experience such a com-
mal orientation of the mesenteric vein and artery, which plication, it is wise and safe to prevent it with proper sur-
is pathognomonic for the disease. gical treatment.
Figures 1 and 2 show the abnormal US findings in
this patient with malrotation.

Suggested Reading
1. Applegate KE, Anderson JM, Klatte EC. Intestinal malrota- 2. Palmas G, Maxia L, Fanos V. Volvulus and intestinal mal-
tion in children: a problem-solving approach to the upper rotation in the newborn. Pediatr Med Chir 2005; 27(1-
gastrointestinal series. Radiographics 2006; 26(5):1485– 2):62–6
500
3 Abdomen 145

Q 69
Frédéric Gauthier, Sophie Branchereau, and Chiara Grimaldi

Fig. 1 Fig. 2

A 5-month-old infant with no significant medical history • What do Figs. 1 and 2 show?
presented to the pediatric emergency department with • The jaundice and AP did not resolve in the following
abdominal pain, fever, jaundice, dark urine, and white days. What is the next step?
stools. Blood tests showed an aspartate aminotransfer- • What procedure would you perform?
ase level of 203 U/l, an alanine aminotransferase level of • Is there a surgical indication, and which procedure
142 U/l, a gamma-glutamyl transferase level of 608, and should be carried out?
increased levels of bilirubin.
146 Abdomen

A 69

This boy has cholestasis with increased conjugated bili-


rubin.
Abdominal US shows intra- and extrahepatic biliary
dilatation, with one stone in the gallbladder and one in
the main biliary duct.
The patient underwent cutaneous cholangiography,
which shows the persistence of lithiasis in the main bili-
ary duct (Fig. 3).
Biliary lavage was carried out and the choledocal li-
thiasis was eliminated. The stone in the gallbladder was
still present at the end of the lavage.
There was a risk of a second episode of stone migra-
tion, and therefore this child was scheduled to undergo
cholecystectomy.

Suggested Reading Fig. 3


1. Debray D, Pariente D, Gauthier F, Myara A, Bernard O.
Cholelithiasis in infancy: a study of 40 cases. J Pediatr 1993;
122(3):385–91
3 Abdomen 147

Q 70
Frédéric Gauthier, Sophie Branchereau, and Chiara Grimaldi

Fig. 1

A 2-year-old girl with a history of recurrent abdominal • What examination did the surgeon perform?
pain presented to the pediatric emergency department • What do you see on the US scan (Fig. 1)?
with acute abdominal pain with vomiting. There was no • What is your diagnosis and how can you confirm it?
jaundice and her hepatic enzyme levels were normal: • What is the surgical management?
amylase was 264 (2.5 N) and lipase 1,533 (25 N).
148 Abdomen

A 70

The diagnosis is acute pancreatitis. The US examination


performed in the ER is shown in Fig. 1.
The US image shows a cystic dilatation of the biliary
ducts with intrahepatic duct dilatation.
The US findings are suggestive of a congenital dilata-
tion of the main biliary duct. This diagnosis is confirmed
by cholangiography (Fig. 2) or MR cholangiography,
which shows a common biliopancreatic duct, and a
high level of amylase and lipase in the gallbladder bile
sample.
The surgical management consists of resection of the
gallbladder and of the cystic main bile duct.
In this case, a biliodigestive anastomosis with a Roux-
en-Y loop was performed.

Fig. 2

Suggested Reading
1. Branchereau S, Valayer J: Malformations kystiques de la 2. O’Neill J. Choledochal Cyst; In O’Neill J, Grosfeld J, Fonk-
voie biliaire chez l’enfant: dilatation congenitale de la voie alsrud E, Coran A, Caldamone A. Principles of Pediatric
biliaire principale. Traitement chirurgical; En Encyclopedie Surgery, Second Edition, Mosby 2003
Medico-Chirurgicale, Editions Scientifique et Medicales
Elsevier SAS, 2002
3 Abdomen 149

Q 71
Frédéric Gauthier, Sophie Branchereau, and Chiara Grimaldi

Fig. 1

A 19-day-old infant presented with a history of jaundice, • What is the differential diagnosis?
white stools, dark urine, and hepatomegaly. The results • Which examinations would you perform?
of a series of blood tests showed an aspartate amino- • The US study is shown in Fig. 1. What is your diagno-
transferase level of 237 U/l, an alanine aminotransferase sis?
level of 171 U/l, and a gamma-glutamyl transferase level • What should be done to confirm the diagnosis?
of 1,004 U/L. There were no signs of infection. • Which procedure would you perform?
150 Abdomen

A 71

In neonatal cholestasis, you must rule out maternofetal


infection, metabolic disease, hemolytic disease, toxicity,
certain genetic syndromes, Alagille’s syndrome, and bili-
ary atresia.
The following examinations should be performed:
hepatobiliary US, A1AT measurements, viral serology
for hepatitis, liver enzyme test, bilirubin test, sweat test,
and protein electrophoresis.
The US shows a small and irregular gallbladder (vol-
ume of 0.19 cc after 7 h of strict nil by mouth) and a
small cyst at the hepatic hilum. These findings are in fa-
vor of a diagnosis of biliary atresia.
The examination that confirms this diagnosis is an
explorative laparotomy. The ponction of the gallbladder
shows limpid bile. An operative cholangiography via a
gallbladder catheter shows nonopacification of the in-
trahepatic biliary ducts. In this case, cholangiography
shows a patent gallbladder, the duodenum is opacified
via the common bile duct, and the hepatic ducts and in-
trahepatic biliary ducts are not opacified (Fig. 2).
Since the gallbladder and the common hepatic ducts
are patent, a cholecystoportoenterostomy is done.
Fig. 2

Suggested Reading
1. Alagille D. Extrahepatic biliary atresia. Hepatology, Vol. 4, 3. Lykavieris P, Chardot C, Sokhn M, Gauthier F, Valayer J,
1984 Bernard O. Outcome in Adulthood of Biliary Atresia: a
2. Kasai M, Kimura S, Asakura Y, Suzuki Y, Taira Y . Surgical Study of 63 patients Who Survived for Over 20 Years With
treatment of biliary atresia. J Pediatr Surg 1968; 3:665–675 Their Native Liver. Hepatology, Vol. 41, No. 2, 2005
3 Abdomen 151

Q 72
Donald Frush

Fig. 2

Fig. 1

A 1-day-old male infant had an abdominal mass that Fig. 3


was identified on a prenatal sonogram. At the time of
imaging, the neonate was asymptomatic.
Initially, an abdominal radiograph was obtained fol-
lowed by an abdominal sonogram.
• What does the radiograph (Fig. 1) show?
• What do Figs. 2–4 show?
• What is the differential diagnosis?
• What other imaging, if any, should be performed?
• What is the management in this case?

Fig. 4
152 Abdomen

A 72

This neonate has a gastrointestinal duplication cyst, tion. In Fig. 3, the cyst (C) is displayed relative to the por-
which was suspected on prenatal sonography and con- tal vein (arrow). Figure 4 is a longitudinal sonographic
firmed by a sonographic evaluation in the immediate view showing the 5-cm extent of this cyst, beginning in
postnatal period. the sub-hepatic region (Liver).
The anteroposterior chest and abdomen radiograph In addition to gastrointestinal duplication, differen-
(“babygram”; Fig. 1) obtained to assess umbilical cath- tial considerations include a choledochal abnormality
eter positions shows the region of the mass (M) in the (including choledochal cyst and choledochocele), and
right upper quadrant displacing the hepatic flexure and omental cyst or mesenteric cyst (i.e., lymphatic malfor-
proximal transverse colon inferiorly and possibly dis- mation), loculated fluid such as ascitis or abscess, and
placing the umbilical venous catheter slightly to the left meconium pseudocyst; other less likely considerations
(arrows). No bowel obstruction is evident. would include abnormalities of regional organs such as a
The sonographic evaluations are shown in Figs. 2–4. mesenchymal hamartoma of the liver, duplication or other
Figure 2 is a transverse epigastric view showing the du- cystic abnormality of the kidney, or ectopic gallbladder.
plication cyst (C) adjacent to the liver, and containing With these sonographic findings, a duplication cyst is
dependent intraluminal debris. Note the normal neona- strongly suspected. Other cross-sectional imaging such
tal prominent limb of the adrenal gland (arrows). The as MR and CT could be performed as part of the preop-
bowel signature sign is evident consisting of echogenic erative evaluation but are not necessary.
mucosal lining and subjacent hypoechoic muscularis The surgical procedure is generally to remove these
(long arrows). Together these layers are virtually only lesions since there is a high frequency of symptoms
seen with hollow viscus and, when seen with a cystic caused by pain (due to secretion of enzymes into the
mass, are usually diagnostic of gastrointestinal duplica- cyst), bleeding, or mass effect.

Suggested Reading
1. Carachi R, Azmy A. Foregut duplications. Pediatr Surg Int 3. Puligandla PS, Nguyen LT, St-Vil D, Flageole H, Bensous-
2002; 18:371–374 san AL, Nguyen VH, Laberge JM. Gastrointestinal duplica-
2. Macpherson RI. Gastrointestinal tract duplications: clini- tions 2003; 38(5):740–744
cal, pathologic, etiologic, and radiologic considerations. 4. Tong SC, Pitman M, Anupindi SA. Ileocecal enteric dupli-
RadioGraphics 1993; 13:1063–1080 cation cyst: radiologic-pathologic correlation. RadioGraph-
ics 2002; 22:1217–1222
3 Abdomen 153

Q 73
Donald Frush

Fig. 1 Fig. 2

A 2-week-old neonate presented with bilious vomiting.


The neonate was brought to the radiology department to
rule out malrotation with midgut volvulus.
An abdominal radiographic series (Fig. 1) was ob-
tained and an upper gastrointestinal (GI) series was per-
formed (Fig. 2).
• What are the findings in Fig. 1?
• What are the findings in Fig. 2?
Subsequently, an enema (Fig. 3) was performed to assess
for possible additional anomalies of the colon.
• What is demonstrated in Fig. 3?
• What are the diagnostic considerations?
• What study, if any, should be performed next?

Fig. 3
154 Abdomen

A 73

This neonate has a proximal bowel obstruction. The ob-


struction is in the proximal jejunum and its appearance
is not the classic double bubble, but given the gas pattern
with gas seen in the proximal bowel on the radiograph,
this does represent a proximal obstruction.
The upper GI series demonstrated a normal duode-
nal jejunal junction but there was a complete obstruc-
tion of the proximal jejunum (arrows) with dilation of
the proximal bowel.
Figure 3 obtained early in the enema examination
does not show a microcolon, which would be present
with distal small bowel or colonic abnormalities. The re-
mainder of the enema was likewise normal.
Given the clinical history of bilious vomiting, and the
findings of proximal obstruction, operative intervention
is indicated. Further imaging would not be of additional
use.
At operation, several segments of proximal jejunal
atresia were encountered.
For proximal obstructions, such as those causing a Fig. 4
double bubble (Fig. 4; including midgut volvulus, duo-
denal atresia, duodenal web, duodenal stenosis, duode-
nal diaphragm, annular pancreas, pre-duodenal portal
vein, extrinsic mass causing proximal duodenal obstruc-
tion), if the radiographic examination demonstrates typ-
ical features of dilated duodenum with no bilious vom-
iting then a gastrointestinal series is not necessary; the
child undergoes surgical exploration, but not emergent
as with suspected volvulus.
For distal obstruction, with gas filling the small bowel,
initially, a limited upper GI series is performed to rule
out malrotation (since this can mimic a distal obstruc-
tion) with a subsequent contrast enema (usually water-
soluble contrast material) performed to assess for the
etiologies. The major etiologies consist of Hirschsprung’s
disease, immature colon syndrome (small left colon)—
which may be associated with meconium plug or meco-
nium plug may be an independent process—meconium
ileus, or ilea atresia.

Suggested Reading
1. Berrocal T, Lamas M, Gutierrez J, Torres I, Prieto C, del 2. Dalla Vecchia LK et al. Intestinal atresia and stenosis: a 25-
Hoyo ML. Congenital anomalies of the small intestine, co- year experience with 277 cases. Arch Surg 1998; 133:490–6
lon, and rectum. RadioGraphics 1999; 19:1219–1236 3. Hajivassiliou C. Intestinal obstruction in neonatal/pediat-
ric surgery. Semin Pediatr Surg 2003; 12:241–53
3 Abdomen 155

Q 74
Donald Frush

A 5-week-old male infant presented with nonbilious,


projectile vomiting.
The neonate was seen in the emergency department.
His past medical history was unremarkable.
• What does the abdominal radiograph in Fig. 1 dem-
onstrate?
• What do the sonographic images in Figs. 2 and 3
demonstrate?
• What other imaging, if any, is indicated in this situa-
tion?

Fig. 1

Fig. 2 Fig. 3
156 Abdomen

A 74

The diagnosis is hypertrophic pyloric stenosis.


The abdominal radiograph (Fig. 1) shows a markedly
gas-distended stomach with areas of slight constriction
due to active peristaltic contractions (arrows).
Longitudinal (Fig. 2) and transverse (Fig. 3) views
in the epigastric region show an elongated pylorus with
a thickened muscularis measuring over 3 mm on the
transverse (arrows) and longitudinal (arrowheads) views.
Note that this hypertrophied muscularis causes a “shoul-
der” effect, projecting into the gastric antrum (arrow in
Fig. 2).
On an upper gastrointestinal series performed in an-
other infant with hypertrophic pyloric stenosis (Fig. 4),
the pyloric channel is narrowed (arrows). A contrast up-
per gastrointestinal series for pyloric stenosis is no lon-
ger the primary imaging modality given the sensitivity
and specificity of sonography.
Pyloric stenosis is an entity of uncertain etiology. Dif-
ferential considerations include pyloric stenosis due to
other conditions (for example, pylorospasm, infiltrative
disorder such as eosinophilic gastritis, and use of prosta-
glandins), as well as extrinsic processes which may com-
press the pylorus and inflammatory changes such as in Fig. 4
severe gastritis.
While this is a self-limited disease, nonsurgical man-
agement is complicated. Pyloromyotomy is in general
recommended.
While palpation of an “olive” has traditionally been
the method of diagnosis on a physical examination, the
ability of sonography to accurately assess for the pres-
ence of hypertrophic pyloric stenosis has placed much
less emphasis on both physical findings and other imag-
ing evaluation (i.e., gastrointestinal series) in the appro-
priate clinical situation.

Suggested Reading
1. Hernanz-Schulman M. Infantile hypertrophic pyloric ste- 3. Hernanz-Schulman et al. In vivo visualization of pyloric
nosis. Radiology 2003; 227:319–331 mucosal hypertrophy in infants with hypertrophic pyloric
2. Hernanz-Schulman M et al. Hypertrophic pyloric stenosis stenosis: is there an etiologic role? AJR Am J Roentgenol
in infants: US evaluation of vascularity of the pyloric canal. 2001; 177:843–8
Radiology 2003; 229:389–93 4. Ito BS, Tamura K, Nagae I, Yagyu M, Tanabe Y, Aoki T,
Koyanagi Y. Ultrasonographic diagnosis criteria using
scoring for hypertrophic pyloric stenosis. J Pediatr Radiol
2000; 35(12):1714–1718
3 Abdomen 157

Q 75
Donald Frush

Fig. 2

Fig. 1

A 3-week-old infant presented with heme-positive stools


and abdomen distention. The infant was 31 weeks at ges-
tation age.
• What are the findings on the frontal abdominal ra-
diograph?
• What are the findings on the lateral abdominal radio-
graph?
• What is the most likely diagnosis in this case?
• What are the differential considerations?
• What other imaging, if any, should be performed?
Fig. 3

Fig. 4
158 Abdomen

A 75

The diagnosis is necrotizing enterocolitis. the interval should be based on clinical concern. When
The frontal abdominal radiograph (Fig. 1) demon- pneumatosis intestinalis is present, images are usually
strates multiple cystic lucencies representing pneumato- obtained in 6–12-hour intervals in order to assess for
sis intestinalis as well as portal venous gas (white arrows). perforation. Portal venous gas is a more ominous sign.
Note the abnormal lucency in the right upper quadrant When perforation with pneumoperitoneum is present,
due to pneumoperitoneum adjacent to the abdominal the cross-table lateral view is more sensitive than the
wall (arrowheads). In Fig. 2, portal venous gas (arrows) anteroposterior view. Small amounts of air may be evi-
is also seen. The pneumoperitoneum is more obvious on dent as triangular lucencies between bowel segments on
the lateral examination. the lateral view. When pneumoperitoneum is large, this
The differential considerations in this case are limited will be evident as a central lucency in the mid-abdomen,
given the pneumatosis intestinalis and portal venous gas, may outline both sides of the bowel wall (Rigler’s sign;
which are hallmarks of necrotizing enterocolitis. With- arrows in Fig. 4) and, when very large, pneumoperito-
out the portal venous gas, there are situations in which neum may constitute the “football” sign where gas fills
benign pneumatosis can be present, but virtually all the entire abdominal pelvic cavity (Fig. 3) also outlining
cases seen in the intensive care nursery in preterm in- the falciform ligament.
fants are not this benign variety. Mottled lucency from In the setting of necrotizing enterocolitis, radio-
stool or meconium can sometimes mimic pneumatosis graphic surveillance is all that is warranted. Given the
intestinalis; pneumatosis, however, is seen as cystic or disruption of mucosal integrity, contrast examinations
linear lucencies in the bowel wall. are not indicated. Sonography is more sensitive at de-
When there is a clinical suspicion of necrotizing en- tecting both portal venous gas (arrows in Fig. 4) and
terocolitis (for example, gastric residuals, heme-positive pneumatosis intestinalis.
stools, abdominal distention), radiographic evaluation In the setting of perforation, and in a relatively stable
is obtained. If a certain feature is present such as fixed, infant, a laparotomy is generally warranted. In extremely
featureless segments of bowel, or wall thickening, fur- sick preterm infants, catheter drainage can be performed
ther radiographic evaluation is generally warranted, but as an intermediate step.

Suggested Reading
1. Butter BY, Flageole H, Laberge JM. The changing face of 2. Kim WY, Kim WS, Kim IO, Kwon TH, Chang W, Lee EK.
surgical indications for necrotizing enterocolitis. J Pediatr Sonographic evaluation of neonates with early-stage necro-
Surg 2002; 37(3):496–499 tizing enterocolitis. Pediatr Radiol 2005; 35:1056–1061
3. Pierro A. The surgical management of necrotizing entero-
colitis. Early Human Development 2005; 81:79–85
3 Abdomen 159

Q 76
Donald Frush

Fig. 1

Fig. 2

A newborn female patient had abdominal distention and • What is the principal finding in Fig. 2?
there was failure to pass meconium. • What are the two major possible etiologies for this
• What does the anteroposterior abdominal radiograph finding?
in Fig. 1 show?
• What are the possible causes of the appearance of the
abdominal radiograph in Fig. 1?
160 Abdomen

A 76

The diagnosis is distal bowel obstruction due to imma-


ture colon syndrome (small left colon syndrome).
The anteroposterior radiograph shows multiple dis-
tended to mildly dilated segments of bowel which, given
the number of segments that are affected, indicate a dis-
tal obstruction.
When a distal obstruction is suspected, a contrast en-
ema is indicated. One should consider doing a limited
upper gastrointestinal series just to make sure that there
is no malrotation since this may present, rarely, as a dis-
tal bowel obstruction.
The contrast enema demonstrates a caliber change
between the descending colon (arrows) compared with
the more proximal colon at the splenic flexure (Fig. 2).
Note the gas-filled distended bowel proximal to this and
multiple filling defects indicating meconium in scattered
segments of the distal colon (arrowheads).
No further imaging evaluation is necessary when
these findings are encountered.
A distal obstruction could be due to a number of
etiologies including imperforate anus/anorectal mal-
formations (these could be diagnosed clinically), im-
mature colon syndrome (small left colon syndrome),
Hirschsprung’s disease, meconium ileus, or ileal atresia. Fig. 3
Given the change in caliber, the two major consider-
ations are long-segment Hirschsprung’s disease (Fig. 3,
different neonate) and small left colon syndrome. A
suction biopsy is the appropriate next step to assess
Hirschsprung’s disease.
Immature colon syndrome (small left colon) is as-
sociated with maternal diabetes. The etiology is unclear.
Hirschsprung’s disease must be ruled out. Immature
colon syndrome is self-limited and no further imaging
evaluation is generally indicated.

Suggested Reading
1. Burge D, Drewett M. Meconium plug obstruction. Pediatr
Surg Int 2004; 20:108–110
2. Dalla Vecchia LK et al. Intestinal atresia and stenosis: a 25-
year experience with 277 cases. Arch Surg 1998; 133:490–6
3. Hajivassiliou C. Intestinal obstruction in neonatal/pediat-
ric surgery. Semin Pediatr Surg 2003; 12:241–53
3 Abdomen 161

Q 77
Ciro Esposito, Carla Settimi, and Michele Ametrano

Fig. 1 Fig. 2

A 5-year-old boy had a history of vomiting, recurrent re- • What do Figs. 1 and 2 show?
spiratory infections from the age of 18 months, asthma, • Why did the surgeon perform the second examina-
and cough, especially during the night. tion?
The mother told the doctor that her child had under- • What other diagnostic examinations are necessary in
gone several types of medical therapy without any im- this case?
provement in his symptoms. The boy came to our atten- • What condition is affecting this child?
tion after a significant episode of apnea. • How should this condition be managed?
The surgeon in the emergency department performed
the examinations shown in Figs. 1, 2.
162 Abdomen

A 77

Fig. 3 Fig. 4

This child is affected by a gastroesophageal reflux (GER). The pH tracing (Fig. 5) shows a 24-h recording period.
In particular, this child is affected by a gastroesopha- Clusters of numerous episodes of GER are evident in the
geal reflux disease (GERD), considering that he is resis- post-prandial period (black vertical lines indicate feed-
tant to medical therapy, as reported by the mother. ing time). A long-lasting episode of GER is also recorded
Figures 1 and 2 show a barium esophagogram (BE) during the nocturnal period (recording between 1:00 and
that is considered, by pediatric surgeons, one of the main 5:00), which accounts for the child’s symptoms and the
examinations for the detection of GER and for studying episode of apnea, due to aspiration of gastric content.
the GE junction. Figures 3 and 4 show the results of the endoscopy. In
After the introduction of an adequate amount of Fig. 3 the presence of extensive esophagitis is remark-
barium (equal to a normal meal; Fig. 1), the BE dem- ably clear in the lower part of the esophagus.
onstrates GER and a sliding hernia with the esophago- Figure 4 shows a hiatal hernia after the U-turn of the
gastric junction above the diaphragm (Fig. 2). Consider- endoscope, already clear on the barium swallow study
ing that many patients with GERD have delayed gastric in Fig. 2. In view of the child’s symptoms, as well as the
emptying, the surgeon in the emergency department fact that he was refractory to the medical therapy with
asked for ultrasonography to be performed to check the proton pump inhibitors, he was referred to a surgeon for
gastric emptying. an antireflux procedure. One month later, he underwent
To complete the work-up of this patient, it is neces- a 360-degree laparoscopic fundoplication according to
sary to perform two more examinations: a 24-h pH-me- Nissen.
try and an esophago-gastroscopy (Figs. 3, 4). After 3 years of follow-up, the child was well and free
of symptoms.

Suggested Reading
1. Esposito C, Montupet Ph, Rothenberg S. Gastroesophageal 3. Esposito C, Langer J, Schaarschmidt K, Mattioli G, Cen-
Reflux in Infants and Children. Springer Ed. 2004 tonze A, Cigliano B, Settimi A, Jasonni V. Efficacy of lapa-
2. Esposito C, Montupet P, Reinberg O. Laparoscopic surgery roscopic antireflux procedures in the management of gas-
for gastroesophageal reflux disease during the first year of troesophageal reflux following esophageal atresia repair J
life. J Pediatr Surg 2001; 36:715–717 Ped Gastroenterol Nutr 2005; 40:349–351
3 Abdomen 163

Q 78
Ciro Esposito and Alaa El-Ghoneimi

Fig. 1

Fig. 2

A 9-year-old child without any previous illness came to • What kind of examination was performed?
our attention because of feeding-related vomiting that • What does this examination show?
had started when he was 7 years old. • What was the suspected diagnosis?
In the last 3 months the vomiting had become more • What is the diagnostic work-up in this pathological
frequent with regurgitation and the child had lost weight. condition?
The child was hospitalized and a radiological examina- • What is the management of this condition and what
tion was performed (Figs. 1, 2). other possible therapies are there?
At physical examination, the child had signs of mild • What is the follow-up?
malnutrition.
164 Abdomen

A 78

This child is affected by esophageal achalasia. The bar-


ium radiograph shows (Figs. 1, 2) a dilated large proxi-
mal esophagus without proper peristalsis and without
complete relaxation of the distal part of the esophagus,
and therefore the contrast material stagnates in its distal
part. In Figs. 1 and 2 it is possible to see a small amount
of contrast material passing in the stomach. This is a
typical radiological image of the so-called rat-tail sign in
esophageal achalasia.
To complete the diagnostic work-up for this pathol-
ogy, endoscopy could be performed to find any ana-
tomic anomalies and manometry (Fig. 3), which will
show disordered and weak peristaltic waves at the level
of the esophageal body with high pressure at the level of
the lower esophageal sphincter (LES) and no relaxation
of the latter.
Esophageal achalasia is a functional disorder char-
acterized by impaired motility of the distal esophagus
and failure of the lower sphincter to relax in response
to swallowing. The incidence of this pathological condi-
tion is about five cases per million people per year and
only 5% of these patients are children. In clinical terms,
the patient complains of rumination, substernal pain,
burning, and dysphagia. Pharmacological management
of achalasia in children is almost always unsuccessful, Fig. 3
pneumatic dilatation is rarely carried out in the pedi-
atric age, and treatment usually consists of surgery. The
surgical approach seems to be the treatment of choice
and most pediatric surgeons prefer to perform an esoph-
agomyotomy according to Heller’s procedure.
In this case, we performed a laparoscopic esophago-
myotomy according to Heller (Fig. 4). To reduce the risk
of postoperative GER, we performed an antireflux pro-
cedure according to Dor after the myotomy.
Long-term follow-up in these patients is essential, and
it includes manometric and barium upper GI studies.

Fig. 4

Suggested Reading
1. Esposito C, Cucchiara S, Borrelli O, Roblot-Maigret B, Des- 2. Esposito C, Mendoza-Sagaon M, Roblot-Maigret B, Amici
ruelle P, Montupet P. Laparoscopic esophagomyotomy for G, Desruelle P, Montupet P. Complications of laparoscopic
the treatment of achalasia in children: a preliminary report treatment of esophageal achalasia in children. J Pediatr
of eight cases. Surg Endosc 2000; 14(2):110–3 Surg 2000; 35(5):680–3
3. Tovar JA, Prieto G, Molina M, Arana J. Esophageal func-
tion in achalasia: preoperative and postoperative mano-
metric studies. J Pediatr Surg 1998; 33:834–838
3 Abdomen 165

Q 79
Ciro Esposito, Antonella Centonze, and Carolina De Fazio

Fig. 1

A 9-year-old boy had undergone surgery 1 year previ- • What is the suspected pathological condition affect-
ously at a different institution for GER with an open fun- ing this child?
doplication. • What does Fig. 1 show?
He came to our attention after a car accident because • Which other diagnostic examinations are necessary
he experienced vomiting, retrosternal pain, and difficul- in this case?
ties in swallowing. The surgeon at the emergency depart- • What is the best way to manage this condition?
ment performed the examination shown Fig. 1.
166 Abdomen

A 79

This child has a migration of the fundoplication into


the thorax. Figure 1, a plain film radiograph, shows
clearly that part of the stomach is dislocated above the
diaphragm. To confirm this diagnosis, you can give the
patient a small contrast meal (Fig. 2).
In the case of GER recurrence after surgery, symp-
toms are typical: substernal pain, vomiting, difficulty to
swallow; however, sometimes the presentation may be
variable according to the age of the patient and to other
factors such as the anatomic condition.
In this case no other diagnostic examinations are
necessary and the only therapeutic option is a redo sur-
gery that can be performed laparoscopically.
The first step of the laparoscopy consists in lysis of
the adhesions, which are particularly strong after an
open procedure, in particular between the epiploon
and the abdominal wall (Fig. 3) and between the liver
and the stomach. Thereafter, the old antireflux mecha-
nism is dismounted, the anatomic structures identified,
and a new fundoplication performed. In our case we
performed a new Nissen procedure after reducing the
stomach into the abdomen and after having performed a
hiatoplasty with two figure of “8” stitches of nonresorb-
able material.
The differential diagnosis in this case should include
failure of an antireflux procedure with recurrence of
symptoms but without migration of the valve. Recurrence
of GER after surgery can affect about 3%–7% of patients. Fig. 2

Suggested Reading
1. O’Reilly MJ, Mullins S, Reddick EJ. Laparoscopic manage-
ment of failed antireflux surgery. Surg Laparosc Endosc
Apr 1997; 7(2):90–93
2. Pointner R, Bammer T, Then P, Kamolz T. Laparoscopic re-
fundoplications after failed antireflux surgery. Am J Surg
Dec 1999; 178(6):541–544
3. Simpson B, Ricketts RR, Parker PM. Prosthetic patch sta-
bilization of crural repair in antireflux surgery in children.
Am Surg Jan 1998; 64(1):67–9; discussion 69–70

Fig. 3
3 Abdomen 167

Q 80
Ciro Esposito, Antonella Centonze, and Aurelie Chiappinelli

Fig. 1

A 6-year-old boy was operated on by our team 19 months • What is the suspected diagnosis?
previously for GERD with a laparoscopic Nissen fundo- • What does Fig. 1 show?
plication. • Which other diagnostic examinations are necessary
He presented with vomiting and retrosternal pain in this case?
that had started 1 week earlier. We performed the ex- • Which other examinations must be performed on a
amination shown in Fig. 1. child who has undergone laparoscopic fundoplica-
tion?
168 Abdomen

A 80

Figure 1 shows a radiographic barium swallow study,


which is absolutely normal. The radiograph clearly
shows the fundoplication profile under the diaphragm,
without signs of GER. To confirm that this child does
not have GER you can perform GI endoscopy (Fig. 2).
This examination can show in retroversion the aspect of
the cardia (perfectly closed, as Fig. 2 shows). Addition-
ally, you can assess the situation of the gastric mucosa
and perform gastric biopsy to search for Helicobacter
pylori (HP). In fact, this child had gastritis caused by HP.
He was referred to our pediatric gastroenterologist, who
treated him with proton pump inhibitors and antibiotics.
At the 1-year follow-up, the boy was free of symptoms.
The differential diagnosis in this case should include
failure of an antireflux procedure or migration of the
valve into the thorax. These events are reported in about
3%–7% of operated patients and can be due to recurrence
of GERD or to migration of the valve to the thorax. Fig. 2
Postoperative follow-up for a child who has under-
gone laparoscopic fundoplication comprises clinical
check-ups 7 days, 1, 3, and 6 months after surgery, and
then annual check-ups for the first 5 years. We also per-
form a barium swallow 1, 2, and 5 years postoperatively.
If there are more symptoms, other examinations can be
scheduled.

Suggested Reading
1. Esposito C, Montupet P, Amici G, Desruelle P. Complica- 3. Sydorak RM, Albanese CT. Laparoscopic antireflux proce-
tions of laparoscopic antireflux surgery in childhood. Surg dures in children: evaluating the evidence. Semin Laparosc
Endosc 2000; 622–624 Surg 2002; 9:133–138
2. Steyaert H, Al Mohaidly M, Lembo MA, Carfagna L, Tur- 4. van der Zee DC, Arends NJT, Bax NMA. The value of 24-h
sini S, Valla JS. Long-term outcome of laparoscopic Nissen Ph study in evaluating the results of laparoscopic antireflux
and Toupet fundoplication in normal and neurologically surgery in children. Surg Endosc 1999; 13:918–21
impaired children. Surg Endosc 2003; 543–546
3 Abdomen 169

Q 81
Deepika Nehra, Samuel Rice-Townsend, and Sanjeev Dutta

Fig. 1 Fig. 2

Fig. 3 Fig. 4

A 10-year-old boy with cystic fibrosis (CF) complicated • What possible etiologies could account for these find-
by severe obstructive lung disease, pancreatic insuffi- ings?
ciency, and sinusitis presented with a 2-day history of • What laboratory study would confirm the most likely
increasing abdominal pain localizing to the lower abdo- diagnosis in this particular case?
men associated with nausea, reduced oral intake, and • What radiographic findings are typical of this condi-
loose stools. He was on trimethoprim-sulfamethoxazole tion? What unique sign do the arrows indicate?
(TMP-SMX) for Pneumocystis carinii pneumonia (PCP)
prophylaxis. After 6 days of intravenous ciprofloxacin, metronidazole,
He was tachycardic and appeared acutely ill with gen- TMP-SMX, and oral vancomycin, the patient’s pain and
eralized abdominal tenderness and peritoneal signs. abdominal girth continued to worsen.
His white blood cell count was 39.4×103 cells/µl with • What are the indications for surgery in this condi-
>20% bands; the amylase level was 33 U/l and lipase tion?
11 U/l. • Why is this a surgical emergency?
CT was performed (Figs. 1–4). • What is the surgical procedure of choice?
• What are the salient findings of the CT scan? • How might this condition present differently in a CF
patient?
170 Abdomen

A 81

The CT scan shows a grossly dilated colon with bowel edematous folds or the “gas accordion sign” may be seen
wall thickening. The submucosal edema and enhance- (arrows). This sign is highly suggestive of pseudomem-
ment are consistent with colitis. branous colitis.
Potential etiologies include infectious causes (pseu- Surgery is indicated for lack of response to maximal
domembranous colitis with Clostridium difficile over- medical therapy and in cases of life-threatening compli-
growth, Salmonella, Shigella, certain viruses, etc.) and cations (peritonitis, progressing toxic megacolon, un-
inflammatory causes (ulcerative colitis, Crohn’s disease). controlled bleeding, and colonic perforation). Generally,
Ischemic colitis, uncommonly seen in children, is a re- 1% of all patients with C. difficile colitis require a colec-
sult of vasculopathy and usually displays segmental in- tomy.
volvement. The boy’s condition was refractory to medical man-
Based on the history of CF with long-term antibiotic agement and his preexisting CF significantly increased
use, the most likely etiology in this case is pseudomem- his mortality risk from this condition, therefore emer-
branous colitis. The diagnosis can be confirmed with a gent operation was performed
stool test for the C. difficile toxin B. A positive stool test The preferred surgical procedure for fulminant C.
is confirmatory for C. difficile colitis, but a negative re- difficile colitis is a subtotal colectomy with an ileostomy.
sult does not rule it out. Even in cases of fulminant coli- The colon may appear deceptively normal at surgery,
tis, the stool toxin test has a false-negative rate of 12.5%. tempting the surgeon to perform less than the required
The boy had a positive stool test, confirming the sus- subtotal colectomy. Patients with a segmental colectomy
pected diagnosis. have higher mortality and later often require subtotal
In a case of pseudomembranous colitis, a dilated co- colectomy.
lon with haustral thickening and a “thumbprinting” pat- Patients with CF are more likely to be colonized by C.
tern may be seen on plain radiographs. On a CT scan, difficile from repeated hospitalizations and frequent an-
colonic wall thickening and a “target sign” may be seen. tibiotic use. Diagnosis may be delayed as they may have
The “target sign” is a nonspecific finding that can be seen constipation instead of the characteristic diarrhea. The
on a contrast-enhanced CT scan, when a hyperemic mu- surgeon must have a low threshold for ordering imaging
cosa is surrounded by a relatively hypodense submucosa studies in a CF patient with abdominal complaints.
resulting in a bull’s eye or target appearance. In more ad- The patient had a quick and uneventful postoperative
vanced cases, trapping of oral contrast material between recovery.

Suggested Reading
1. Binkovitz LA, Allen E, Bloom D, Long F, Hammond S, 4. Dallal RM, Harbrecht BG, Boujoukas AJ, Sirio CA, Farkas
Buonomo C, Donnelly LF. Atypical presentation of Clos- L, Lee KK, Simmons RL. Fulminant Clostridium difficile:
tridium difficile colitis in patients with cystic fibrosis. AJR an underappreciated and increasing cause of death and
1999; 172(2):517–521 complications. Ann Surg 2002; 235:363–372
2. Bradbury AW, Barrett S. Surgical aspects of Clostridium 5. Klingler PJ, Metzger PP, Seelig MH, Pettit PD, Knudesen
difficile colitis. Br J Surg 1997; 84:150–159 JM, Alvarez SA. Clostridium difficile infection: risk fac-
3. Brook I. Pseudomembranous colitis in children. Journal of tors, medical and surgical management. Dig Dis 2000; 18:
Gastroenterology and Hepatology 2005; 20(2):182–186 147–160
3 Abdomen 171

Q 82
Luis de la Torre

Fig. 2

Fig. 1

A 36-h-old term newborn delivered by cesarean section • What do Figs. 1 and 2 show?
and weighing 2.7 kg had abdominal distension, biliary • What is the diagnosis?
vomiting, and no stools. Radiographic and ultrasound • What other diagnostic examination do you recom-
examinations were performed. mend to establish a diagnosis?
An intestinal duplication was suspected and a pediat- • What is the most appropriate surgical management
ric surgeon was called to evaluate the patient. in this case?
• What pathological condition should be investigated
in this case?
172 Abdomen

A 82

Fig. 4

Fig. 3

This boy has a “congenital low intestinal obstruction Colonic atresia was suspected and laparotomy was
syndrome.” performed confirming this condition (Fig. 4).
The most common surgical causes are intestinal atre- Figure 4 shows the dilated proximal colon and the
sia, anorectal malformation and Hirschsprung’s disease. distal hypoplastic segment (in Babcock forceps). A 10:1
Figure 1 shows big dilated loops without air at the disproportion was found and a colostomy was performed.
rectum. A rectal biopsy was performed and Hirschsprung’s dis-
The sonogram in Fig. 2 shows dilated intestinal loops ease was confirmed in this patient.
filled with hyperechogenic material. To compare, you After 3 years, an ileal pull-through was performed.
can see a normal loop (arrow). An intestinal duplication Because of the severe hypoplastic distal colon, a poste-
was suspected because this image looks like a cyst. rior sagittal approach was used during the pull-through.
The colon enema is a very effective study in patients A colon enema in patients with low intestinal ob-
with congenital low intestinal obstruction syndrome for struction syndrome and rectal biopsy in patients with
differentiating functional vs. mechanical obstruction. colonic atresia are mandatory.
Figure 3 shows a defunctionalized distal colon until
the splenic flexure where the contrast stops and the big
dilated loop is not filled.
3 Abdomen 173

Q 83
Luis de la Torre

Fig. 1 Fig. 2 Fig. 3

A 43-day-old boy was admitted to hospital with acute


abdominal distension and gastric vomiting.
The mother told the doctor she had been using a
glycerin suppository daily to facilitate her baby’s bowel
movements since his birth.
Physical examination showed a normal anus. The ab-
domen is shown in Fig. 1.
In the ER an abdominal radiograph was obtained
(Fig. 2).
A rectal tube was inserted, resulting in an explosive
evacuation with abundant gas; the baby improved no-
ticeably within hours.
A pediatric surgical evaluation was considered and a
colon enema was indicated (Figs. 3, 4).
• What does Fig. 1 show?
• What does Fig. 2 show?
• What do Figs. 3 and 4 show?
• What is the diagnosis?
• What is the initial management in this case?
• What pathological condition should be investigated Fig. 4
in this case?
• How do you confirm the diagnosis?
• Which medical or surgical option is better to resolve
this problem?
174 Abdomen

A 83

Fig. 5

Fig. 6
Fig. 7

This boy had “constipation syndrome” since birth and All Hirschsprung’s disease patients have rectal agan-
Hirschsprung’s disease should be suspected. glionosis, regardless of the length of the aganglionic seg-
He was well with suppositories for 43 days, but sud- ment, therefore the disease must be sought in the rectum.
denly developed acute intestinal obstruction with visible A definitive diagnosis of Hirschsprung’s disease can be
dilated loops (Fig. 1). made by demonstrating the absence of ganglion cells
Figure 2 shows greatly dilated loops in the lower ab- and nerve hypertrophy on histopathological examina-
domen correlating with low intestinal obstruction. tion of the rectal biopsy specimen.
Figures 3 and 4 show a whole spastic colon with an A rectal suction biopsy was performed 1 week af-
inflamed spike-like mucosa indicating Hirschsprung’s ter admission and the histopathological hallmarks of
colitis. In this condition, the transitional zone cannot be Hirschsprung’s disease were confirmed (Fig. 5).
demonstrated. Once the inflammatory process was resolved, a new
Colon enema is suggested as the first study for colon enema was performed easily demonstrating the
Hirschsprung’s disease; however, we recommend per- transitional zone (Fig. 6).
forming it after histopathological confirmation. One-stage transanal colectomy with endorectal pull-
Forty-eight hours after admission, the baby was much through resolved this length of aganglionosis, avoiding
better, with rectal irrigations through the rectal tube and a colostomy. The baby underwent this procedure 9 days
IV antibiotics allowing oral feeding. after admission and 28 cm of colon was resected (Fig. 7).
The baby was discharged 3 days after surgery.

Suggested Reading
1. Dasgupta R, Langer JC. Transanal pullthrough for 3. IPEG Guidelines for surgical treatment of Hirschsprung’s
Hirschsprung disease. Sem Pediatr Sur 2005; 14:64–71 disease. J Laparoendosc Adv Surg Tech 2005; 15:89–91
2. De la Torre ML, Ortega SJ. Transanal endorectal pull- 4. Marty TL et al. Rectal irrigations for the prevention of post-
through for Hirschsprung’s disease. J Pediatr Surg 1998; operative enterocolitis in Hirschsprung’s disease. J Pediatr
33:1283–1286 Surg 1995; 30:652–654
3 Abdomen 175

Q 84
Luis de la Torre

Fig. 1 Fig. 2 Fig. 3

A 9-year-old boy, weighing 25 kg, was admitted to the Two weeks after rectoscopy, he presented with nausea
emergency department because of acute abdominal pain, and a new episode of abdominal pain with 5 days of no
nausea, abdominal distension, and visible intestinal bowel movements. The plain film radiograph is shown
loops. These symptoms had been recurrent for 8 years in Fig. 2.
with asymptomatic periods, but they had been more fre- The patient was admitted to our department and an-
quent during the last 2 years after the ingestion of laxa- other rectal biopsy was performed. There were no ulcers,
tives. A plain film radiograph was obtained (Fig. 1). no abscess, and normal ganglion cells in the submucosa.
The patient had an 8-year history of mild constipation A new radiograph was obtained, shown in Fig. 3.
and multiple visits to several pediatricians, gastroenter- • What do Figs. 1–3 show?
ologists, and pediatric surgeons. The mother recalled us- • What is the diagnosis?
ing suppositories occasionally when he was a baby, and • Do you continue with the laxatives?
since then she had given him different diets, analgesics, • Which studies would you perform in this case?
laxatives, and enemas. • What medical or surgical option is better to resolve
A rectoscopy showed lymphoid follicular hyperplasia this problem?
and the results of a rectal biopsy revealed nonspecific • Does this patient have constipation or chronic low
colitis with follicular hyperplasia; no abscess and no intestinal obstruction?
submucosa were observed. Tegaserod was administered • Does this boy have constipation or inflammatory
orally, but no clinical response was observed. bowel disease?
176 Abdomen

A 84

Fig. 4 Fig. 5 Fig. 6 Fig. 7

Fig. 8 Fig. 9

Figures 1 and 2 show distended loops of small intestine, A colonoscopy showed follicular hyperplasia and an
due to low “chronic” intestinal obstruction rather than extraordinary large sigmoid. The whole colonoscope
constipation. reached the splenic flexure.
Hirschsprung’s disease and inflammatory bowel dis- During laparotomy 110 cm of sigmoid (Fig. 8) came
ease should be considered. out through the incision. A sigmoidectomy and primary
A history of constipation and the left colon dilatation colorectal anastomosis were made (Fig. 9). After surgery
with an air-fluid level in the splenic flexure (Fig. 3) ruled the patient was well and asymptomatic.
out aganglionosis of the rectum. Dolichocolon is a rare disorder in the pediatric pop-
A colon enema was redone. ulation characterized by elongation of the colon, espe-
The initial radiograph (Fig. 4) shows similar findings cially the sigmoid. It can cause constipation and recur-
to those in Figs. 1 and 2. Figures 5 and 6 show a very rent chronic volvulus expressed as abdominal pain and
large and tortuous sigmoid. Figure 7 shows a transverse distension, as in this case.
and right dilated colon. These finding are characteristics Dolichocolon (long colon) should be distinguished
of dolichocolon. from megarectosigmoid (a wide colon and sigmoid) that
is far more common.
3 Abdomen 177

Q 85
Luis de la Torre

Fig. 1 Fig. 2

A 20-month-old boy presented to the emergency depart-


ment because of severe abdominal distension, gastric
vomiting, and dehydration. The initial radiograph is
shown in Fig. 1.
On physical examination, a large abdominal mass
was palpated and a well-positioned surgical anus was
observed.
The boy had undergone anorectoplasty because of a
rectoperineal fistula.
The mother reported that the boy had not had bowel
movements for the last 17 months, but his diapers al-
ways had fecal soiling. A physician had prescribed lop-
eramide, but the mother discontinued this medication.
For the last 10 months, she had used laxatives and en-
emas without any improvement.
A colon enema with hydrosoluble contrast medium
was performed, shown in Figs. 2, 3.
• What is the diagnosis?
• What do Figs. 1–3 show?
• Would you request other studies? Fig. 3
• What treatment would you propose?
178 Abdomen

A 85

This patient was born with one of the most common


anorectal malformations (ARMs), a rectoperineal fistula,
treated surgically with anorectoplasty in the newborn
period. A common outcome in this type of ARM is con-
stipation, which should be treated from birth to avoid
complications.
The patient has chronic constipation, fecal soiling,
and an abdominal mass. This triad should be suggestive
of a megarectosigmoid.
Figure 1 shows a megafecaloma. Figure 2 demon-
strates a hemivertebra. Figure 3 shows a huge megarec-
tosigmoid with a proximal normal descending colon.
Fecal soiling and megarectosigmoid (fecal pseudoin-
continence) can be managed medically; however, some
cases will need a sigmoidectomy to resolve the problem.
Moreover, in this patient with ARM and hemiverte-
bra, it is necessary to rule out a presacral mass. These
three defects are called Currarino triad.
MR imaging was performed and an anterior menin-
gocele was found (Fig. 4). The patient underwent co- Fig. 4
lostomy followed by a plasty of the meningocele via a
posterior sagittal approach and 2 months later sigmoid-
ectomy was performed. Figure 5 shows the resected
megasigmoid; note the normal caliber of the descending
proximal colon.
At the time of writing, the patient was fecally conti-
nent and clean using 10 ml of lactulose per day.
This case illustrates two entities that should be borne
in mind when treating patients with ARM: (1) a megarec-
tosigmoid as a source of fecal pseudoincontinence and (2)
Currarino’s triad, whose frequency is underestimated.

Fig. 5

Suggested Reading
1. Currarino G, Coln D, Votteler TP. Triad of anorectal, sacral 3. Swamy S, et al. Anterior sacral meningocele as part of the
and presacral anomalies. Am J Roentg 1981; 137:395–398 Currarino triad. Ind J Radiol Imag 2003; 13:207–208
2. Peña A, el Behery M. Megasigmoid: a source of pseudoin-
continence in children with repaired anorectal malforma-
tions. J Pediatr Surg 1993; 28:199–203
3 Abdomen 179

Q 86
Luis de la Torre

Fig. 1 Fig. 2

A 3-month-old boy with imperforate anus was admitted The mother showed us a contrast radiological study
to hospital because of a 2-day history of fever, lethargy, performed for the nonfunctional stoma, which dem-
intolerance to oral intake, and three episodes of gastric onstrated an anorectal malformation without fistula
vomiting. (Fig. 2).
On admission, his temperature was 38.7ºC. Down • What does Fig. 1 show?
syndrome was eliminated, and an intestinal stoma on the • What does Fig. 2 show?
right side of the abdomen was observed. The perineum • What is the diagnosis?
is shown in Fig. 1. • What surgical procedure is indicated in this case?
An initial laboratory evaluation showed 12.4 g/dl of • What radiological studies would you order for this
hemoglobin, 15,300 white blood cells, 79% neutrophils, patient?
and 2% bands. A urine examination showed abundant
bacteria, uncountable leukocytes, and positive nitrites.
180 Abdomen

A 86

Fig. 3

Fig. 4

The patient has an anorectal malformation (ARM) with After treating the urinary tract infection, closure of
urinary tract infection; consequently, a urinary fistula or the right colostomy and a new (left) descendent colos-
urinary malformation should be ruled out. tomy were performed. Subsequently, renal ultrasonogra-
Figure 1 shows a flat perineum. This is characteristic phy was performed, which was normal.
of a perineum in a child without Down syndrome and A new high-pressure distal colostogram with hydro-
suggests clinically an ARM with a urinary fistula. soluble contrast medium was obtained 1 month after the
To demonstrate the anatomy of an ARM a high-pres- second operation. Figure 3 is an anteroposterior view
sure distal colostogram with hydrosoluble contrast is man- demonstrating the distal segment with an adequate
datory. This study allows one to define: (a) the length of length for the pull-through; in the lateral view a recto-
the distal (defunctionalized) segment, (b) the distance urethral bulbar fistula is identified.
between the distal pouch and the perineum, and (c) When performing colostography, it is not rare for the
whether or not there is an associated fistula. contrast medium to pass from the urethra to the bladder.
Figure 2 shows the distal cologram of this patient. We This situation should be kept in mind, because if barium
can observe a right colostomy that defunctionalized most is used for the colostogram, it can be introduced into the
of the colon, therefore the boy has a distal colon that is too urinary tract with grave consequences.
long; the distal pouch seems to be far from the perineum In this patient, during the colostography the contrast
and without an associated fistula. However, because it is medium passed to the bladder and refluxed to the ureter.
almost impossible to generate high pressure during the Figure 4 illustrates the vesicoureteral reflux in this case.
cologram in a right colostomy—which would allow open- The patient underwent anorectoplasty and closure of
ing of the elevator muscle and consequently demonstrat- the urethral fistula by a posterior sagittal approach, anal
ing a recto-urinary fistula and the rectal pouch—we must dilatations, closure of the colostomy, and ureter reim-
not assume that this patient does not have a fistula. plantation.
3 Abdomen 181

Q 87
Bruno Cigliano

Fig. 1

A full-term neonate with prenatal diagnosis of dextro- • What does Fig. 1 show?
cardia was admitted to hospital on the first day of life. • Which is the possible diagnosis?
The general condition of the infant was good and only a • Which other diagnostic procedures are needed to
mild polypnea was present. An ECG was performed and confirm the diagnosis?
a radiograph (Fig. 1) was obtained. The ECG was com- • What is the treatment?
patible with the prenatal diagnosis of dextrocardia.
182 Abdomen

A 87

Fig. 2

Fig. 3 Fig. 4

Figure 1 shows a mass in the right hemithorax that with a parenchymal aspect caused by anomalous liver
moves down and left to the heart or a right displacement development, probably due to a left eventration of the
of the heart caused by a solid mass in the left lung. diaphragm (Fig. 2).
A cardiologic consultation and an echocardiography MR imaging (Figs. 3, 4) confirmed the diagnosis,
are mandatory. showing anomalous development of the left liver lobe
The cardiologic consultation and ultrasonography caused by a congenital elevated left hemidiaphragm.
showed a normal heart but positioned to the right with The treatment of diaphragmatic eventration can be
a normal atrioventricular ratio. The examinations also conservative or surgical. The choice is based on clinical
revealed a structure in the middle left pulmonary field and radiological assessment.
3 Abdomen 183

Q 88
Bruno Cigliano

Fig. 1

A 26-day-old newborn, who had been breastfed, pre- A barium meal (Fig. 1) was performed, and the baby
sented with vomiting that had started since the tenth day was referred to our unit.
of life and had progressively worsened. The vomit was • Do you believe that the diagnostic study was correct?
never bile-stained, but sometimes contained fresh blood. • What is the most appropriate management in this
Constipation was also present. Abdominal examination case?
revealed an abdominal fullness in the epigastric region. • What is the differential diagnosis?
Laboratory studies showed some degree of hypochlore- • What are the complications of the surgical treatment?
mic alkalosis.
184 Abdomen

A 88

Fig. 2

Fig. 3 Fig. 4

Infantile hypertrophic pyloric stenosis (IHPS) has inter- The differential diagnosis includes milk intolerance,
ested investigators for several decades; the cause is now gastroesophageal reflux, pyloric atresia, duodenal steno-
thought to be a mechanism other than a developmental sis, infections, and neurological diseases.
defect, but no definitive cause has been found. There is The therapy of IHPS is surgical correction. It is im-
evidence of a genetic predisposition. The pylorus appears portant to prepare the infant appropriately for anesthe-
enlarged, measuring 2–2.5 cm in length and 1–1.5 cm in sia and operation.
diameter. The mucosa is normal, while marked muscle The operative procedure is extramucosal pyloromy-
hypertrophy primarily involving the circular layer pro- otomy that can be performed either during laparotomy
duces partial or complete luminal occlusion. (Fig. 3) or laparoscopy (Fig. 4).
The diagnosis is based on careful physical examina- The complications are minimal if the procedure is
tion, laboratory examinations, and ultrasonography performed by an experienced surgeon but include: per-
(Fig. 2). Rarely is a barium upper GI examination or en- sistent vomiting (incomplete myotomy), wound infec-
doscopy necessary. tion, laparocele, and damage of the duodenal mucosa.
3 Abdomen 185

Q 89
Bruno Cigliano

Fig. 1 Fig. 2

A neonate with vomiting of gastric content without bile • Which anomalies can be observed on these radio-
was hospitalized on his second day of life. graphs?
An upper abdominal distension with visible peristal- • What is the possible diagnosis?
sis was found. • Which other diagnostic procedures can be useful for
A plain thoracoabdominal radiograph was obtained the diagnosis?
with the patient in the clinostatic position (Figs. 1, 2). • What should be done in this case?
186 Abdomen

A 89

The anteroposterior radiograph (Fig. 1) shows an en- In this case, the gastroscopy showed the presence of
larged stomach distended by swallowed air with absence congenital pyloric atresia (CPA). Laparotomy confirmed
of gas beyond the stomach. this diagnosis.
This radiological findings are confirmed by the ra- CPA is a very rare malformation occurring in 1 of
diograph obtained in the latero-lateral position (Fig. 2), 100,000 live births. It can occur as an isolated lesion or
which shows a stomach distended by gas that does not in association with other genetically determined condi-
pass into the duodenum. tions such as epidermolysis bullosa or congenital aplasia
In this case, the possible diagnosis can be a high-de- cutis, or form part of the hereditary multiple intestinal
gree gastric outlet obstruction due to either pyloric apla- atresias syndrome.
sia or atresia or even caused by the presence of a com- Excision or incision of the diaphragm with pyloro-
plete antral or pyloric diaphragm. plasty and gastroduodenostomy are the procedures of
To differentiate these forms, gastroscopy can be very choice. Gastrojejunostomy should be avoided because
useful, whereas barium swallow can be contraindicated of its high mortality rate. In our case the surgical pro-
because of possible vomiting and consequent inhalation cedure consisted in excising the atresic pylorus and in
of contrast material. performing an end-to-end gastroduodenostomy.

Suggested Reading
1. Dessanti A, Di Benedetto V, Iannuccelli M, Balata A, Cossu 3. Okoye BO, Parikh DH, Buick RG, Lander AD. Pyloric
Rocca P, Di Benedetto A. Pyloric atresia: a new operation to atresia: five new cases, a new association, and a review of
reconstruct the pyloric sphincter. J Pediatr Surg 2004 Mar; the literature with guidelines. J Pediatr Surg 2000 Aug;
39(3):297–301 35(8):1242–5
2. Nawaz A, Matta H, Jacobsz A, Al-Salem A. Congenital py- 4. Toma P, Mengozzi E, Dell’Acqua A, Mattioli G, Pieroni G,
loric atresia and junctional epidermolysis bullosa: a report Fabrizzi G. Pyloric atresia: report of two cases (one asso-
of two cases. Pediatr Surg Int 2000; 16(3):206–8 ciated with epidermolysis bullosa and one associated with
multiple intestinal atresias). Pediatr Radiol 2002 Aug; 32(8):
552–5
3 Abdomen 187

Q 90
Giovanni Esposito and Ciro Esposito

Fig. 1

The patient was a 6-week-old infant, born at full term fraction of 6.8 and an indirect one of 2.2; the serum al-
after a spontaneous delivery and normal pregnancy. He kaline phosphatase levels were also elevated.
was breastfed and his initial development was normal. An abdominal US was performed (Fig. 1). On the ba-
When he was 35 days old, his mother noticed darken- sis of the US data, other examinations were performed,
ing of the urine and lightened feces color, which became and subsequently a surgical intervention was scheduled.
completely acholic after a few days. At the same time the • What does the US show?
baby began showing signs of jaundice, initially limited • What was the diagnostic suspicion?
to the sclera and afterward to the entire body. • Which other examinations were performed to define
On hospitalization the jaundiced baby was in good the diagnosis and what were their results?
condition. On palpation of the abdomen the liver, mod- • On the basis of illness evaluation, laboratory tests,
erately enlarged, had a firm consistency. There was no and all other examinations, what was the diagnosis?
splenomegaly. • What is the treatment for this disease?
Laboratory test results were normal except for the • What was the follow-up?
total bilirubin level that was over 9 mg/dl with a direct • What is the prognosis?
188 Abdomen

A 90

The abdominal US shows an enlarged liver with a mod-


erate increase in its echogenicity and the absence of any
visible biliary structures.
On the basis of the clinical features and laboratory
data, biliary atresia was suspected.
To confirm this diagnosis and to exclude other jaun-
dice conditions, especially neonatal hepatitis, 99mTc-di-
methyl-iminodiacetic acid scintigraphy (HIDA), hepatic
biopsy, and laparoscopy were performed. The HIDA
(Figs. 2, 3) showed lack of excretion of the contrast ma-
terial thought the biliary tree. The hepatic biopsy high-
lighted the characteristics of biliary atresia, with prolif-
erating bile ducts and without giant cells and focal areas
of necrosis which, on the contrary, are characteristic of
neonatal hepatitis. Lastly, the laparoscopy confirmed the
diagnosis of biliary atresia, with the characteristic aspect
of biliary cirrhosis and absence of the gallbladder that
was replaced by fibrous cords.
The treatment consists in an anastomosis between
the porta hepatis, conveniently prepared to expose the Fig. 2
intrahepatic biliary ductules, and a segment of intestinal
tract. In this case a portohepatic appendicoduodenos-
tomy was performed (Fig. 4).
The follow-up was normal and the infant was dis-
charged 15 days after surgery when the jaundice re-
gressed. At 1 year the jaundice disappeared and the total
bilirubin level was 1.6.
The prognosis of biliary atresia depends on the suc-
cessful outcome of the operation which, when per-
formed before the age of 2 months, achieves good results
only in about 30% of cases. In the rest of the cases, liver
transplantation is necessary. Fig. 3

Suggested Reading
1. Casaccia G, Catalano OA, Marcellini M, Bagolan P. Biliary
atresia associated with multiple unrelated anomalies: what
about it? Pediatr Surg Int 2006; 20:345–348
2. Dehghani SM, Haghighat M, Imanieh MH, Geramizadeh
B. Comparison of different diagnostic methods in infants
with Cholestasis. World J Gastroenterol 2006 Sep 28;
12(36):5893–6

Fig. 4
3 Abdomen 189

Q 91
Giovanni Esposito and Ciro Esposito

Fig. 1

A 3-year-old boy, in good general condition, presented mal, while auscultation of the left hemithorax revealed
with sudden abdominal pain localized in the epigastric humid rales.
and left upper regions, which appeared a few hours af- • What do the thoracic radiograph and barium meal
ter a meal. Sometimes, the child had stipsis. At the same (Fig. 1) show?
time, the patient presented respiratory symptoms such • Which other examinations were performed to define
as a cough. the diagnosis and what were their results?
A gastric disease was suspected, and therefore a sim- • What was the diagnosis?
ple chest radiograph series and a barium meal were per- • What was the surgical treatment?
formed. On physical examination the abdomen was nor- • What was the follow-up?
190 Abdomen

A 91

The barium meal showed a normal stomach, duodenum,


and first jejunal segment, while in the inferior left hemi-
thorax there were some gaseous irregular images. This
finding was confirmed by the chest radiograph.
Based on the clinical and radiological features, a
gastrografin enema was performed, which revealed the
presence of the colon in the left hemithorax (Fig. 2). The
diagnosis was Bochdalek diaphragmatic hernia. This is
a congenital malformation due to an agenesis of the dia-
phragm’s dorsal part; through this hole, one or more ab-
dominal organs may move into the thorax. This hernia is
more frequent on the left site, and it may have a real her-
nial sac. Because of the presence of abdominal viscera in
the thorax, the lung becomes more or less hypoplastic.
Clinically, congenital diaphragmatic hernia (CDH) may
occur, outside the more frequent neonatal period, in the
first years of life. Because of the different clinicoradio-
logical features of these conditions from the classic find-
ings of congenital hernia (tachypnea, cyanosis, respira-
tory failure, and presence of the polycyclic hyperlucent
endothoracic blebs with a shift of the mediastinum on
the right seen on chest radiographs), the diagnosis is Fig. 2
often missed and it may be discovered incidentally on
chest radiographs.
At surgery, a large defect of the left posterior dia-
phragm was found. The herniated colon was replaced in Suggested Reading
the abdomen and the diaphragm defect was closed. To- 1. Baeza-Herrera C, Velasco-Soria L, Garcia-Cabello LM,
day it is possible to perform this operation with laparos- Osorio-Aguero CD. Bochdalek hernia with late manifesta-
copy, and if there is a prenatal diagnosis of a very large tion. Relevant clinico-surgical features Gac Med Mex 2000
defect, fetal surgery may be considered. Jul–Aug; 136(4):311
The postoperative course was uneventful with good 2. Elhalaby EA, Abo Sikena. Delayed presentation of con-
lung expansion. The child was discharged 7 days after genital diaphragmatic hernia. Pediatr Surg Int 2002 Sep;
surgery and at follow-up was growing well. 18(5–6):480–5
3. Mei-Zahav M, Solomon M, Trachsel D, Langer JC. Boch-
dalek diaphragmatic hernia: not only a neonatal disease.
Arch Dis Child 2003 Jun; 88(6):532–5
4. Newman BM, Afshani E, Karp MP, Jewett TC Jr, Cooney
DR. Presentation of congenital diaphragmatic hernia past
the neonatal period. Arch Surg 1986 Jul; 121(7):813–6
5. Ozturk H, Karnak I, Sakarya MT, Cetinkursun S. Late pre-
sentation of Bochdalek hernia: clinical and radiological as-
pects. Pediatr Pulmonol 2001 Apr; 31(4):306–10
3 Abdomen 191

Q 92
Giovanni Esposito and Ciro Esposito

Fig. 1 Fig. 2

A full-term newborn, with normal breast-feeding since • What do the radiographs show (Figs. 1, 2)?
the third week of life, suddenly presented with post- • Which other examinations were performed?
feeding vomiting. The vomiting became more frequent • What was the diagnosis?
and severe. On physical examination, the child had se- • What was the treatment?
vere signs of dehydration without any abdominal prob- • What was the follow-up?
lems. A radiological examination, shown in Figs. 1 and
2, was carried out. After more examinations the infant
was operated on.
192 Abdomen

A 92

The barium swallow reveals some episodes of gastro-


esophageal reflux (GER) and significant gastric dilatation
and a large air–fluid level. There is a hypertrophic stenosis
of the pylorus, with the typical sign of a double railroad.
Laboratory tests showed a hydroelectrolitic imbal-
ance with hypocloremia, hyperpotassemia, and meta-
bolic alkalosis.
The diagnosis was hypertrophic stenosis of the pylorus
associated with GER. This disease is known as phreno-
pyloric syndrome or Roviralta’s syndrome. Roviralta’s
syndrome is rare, affecting 3% of pyloric stenosis cases.
The treatment consists in correcting the dehydration
and then surgery in order to perform a Fredet–Ramst-
edt-type pyloromyotomy. In our case the operation was
open, but today some surgeons prefer to do it laparo-
scopically. There was no indication for reflux correction
because the GER was due to hyperpressure inside the
stomach.
The follow-up was normal and the patient’s prognosis
good.

Suggested Reading
1. Rode H, Cywes S, Davies MR. The phreno-pyloric syn-
drome in symptomatic gastroesophageal reflux. J Pediatr
Surg 1982 Apr; 17(2):152–7
3 Abdomen 193

Q 93
Alessandro Settimi and Ciro Esposito

Fig. 1

A 4-year-old boy, without any respiratory problems, un- • Which examinations are necessary to confirm the di-
derwent chest radiography (Fig. 1) before an operation agnosis?
for hypospadias. Physical examination was normal, ex- • What is this pathological condition and what is the
cept for the finding of an elevation of hepatic dullness treatment?
found on percussion of the right hemithorax. • What did the surgeon perform in this case?
• What does the radiograph show? • What is the prognosis?
• What was the suspected diagnosis and what should it
be differentiated from?
194 Abdomen

A 93

The chest radiograph shows a remarkable elevation of


the right diaphragm with one reduced respiratory area.
The suspected diagnosis was diaphragmatic eventra-
tion, which must be differentiated from other thoracic dis-
ease (collapse of inferior pulmonary lobe or pulmonary
hypoplasia) or abdominal hepatic pathologies (inflam-
matory, cancer, Echinococcus cyst) and extrahepatic pa-
thologies (subphrenic abscess, retroperitoneal tumors).
In the past, to confirm the diagnosis, pneumoperito-
neum was created and peritoneography was performed.
Today it is useful to perform ultrasound and fluoros-
copy; for left eventration, a barium swallow with upper
gastrointestinal series followed by pulmonary scanning Fig. 2
are recommended. In this case, fluoroscopy revealed a
paradoxical motion of the right diaphragm during res-
piration that confirmed the diagnosis of diaphragmatic
eventration. compression of the lung caused by displacement of the
Diaphragmatic eventration consists in diaphragm diaphragm. The treatment in symptomatic patients is
elevation of different degrees, and is congenital or ac- surgery, and it can be performed via open surgery (lapa-
quired. The congenital form is characterized by hypo- rotomy or thoracotomy) or videosurgery (laparoscopy
plasia or atrophy of the diaphragmatic muscle, formed or thoracoscopy). The operation consists in reducing the
only by pleural and peritoneal layers (Fig. 2). The ac- diaphragm with partial resection or plication.
quired type is a result of phrenic nerve paralysis due to Conservative treatment is indicated in asymptomatic
different etiologies (traumatic, surgery, extrinsic com- patients, as in our case, and periodical follow-ups are neces-
pression). Clinically, the patients may present with re- sary. The prognosis is generally very good in both cases.
spiratory problems of varied degree in connection with

Suggested Reading
1. Becmeur F, Talon I, Schaarschmidt K, Philippe P, Moog R, 4. Tsugawa C, Kimura K, Nishijima E, Muraji T, Yamaguchi
Kauffmann I, Schultz A, Grandadam S, Toledano D. Tho- M. Diaphragmatic eventration in infants and children: is
racoscopic diaphragmatic eventration repair in children: conservative treatment justified? J Pediatr Surg 1997 Nov;
about 10 cases. J Pediatr Surg 2005 Nov; 40(11):1712–5 32(11):1643–4
2. Gierada DS, Slone RM, Fleishman MJ. Imaging evalua- 5. Yazici M, Karaca I, Arikan A, Erikci V, Etensel B, Temir
tion of the diaphragm. Chest Surg Clin N Am 1998 May; G, Sencan A, Ural Z, Mutaf O. Congenital eventration of
8(2):237–80 the diaphragm in children: 25 years’ experience in three
3. Tiryaki T, Livanelioglu Z, Atayurt H. Eventration of the pediatric surgery centers. Eur J Pediatr Surg 2003 Oct;
diaphragm. Asian J Surg 2006 Jan; 29(1):8–10 13(5):298–301
3 Abdomen 195

Q 94
Giovanni Esposito, Alessandro Settimi, and Ciro Esposito

Fig. 1

A 2-year-old boy had undergone pyeloplasty for ure- bile-stained liquid came out. Based on the findings of
teropelvic junction obstruction via a retroperitoneal ap- an abdominal radiograph, explorative laparotomy was
proach. The initial postoperative course was uneventful performed (Fig. 1).
with removal of the drainage device on postoperative • What does the abdominal radiograph show?
day 2. • What was found at surgery?
Six days after surgery the patient began vomiting. • What type of surgery was undertaken?
Feeding was stopped and a nasogastric tube was in- • What was the follow-up?
serted in the stomach from which a large quantity of
196 Abdomen

A 94

This boy had jejunojejunal intussusception. The abdomi-


nal radiograph shows some air–fluid levels, indicative of
an intestinal occlusion.
Ultrasonography was performed (Fig. 2), confirming
the diagnosis.
We performed a laparotomy and at operation a small
amount of peritoneal liquid was found with a significant
degree of high intestinal occlusion caused by a jejunoje-
junal intussusception.
Because of the good appearance of the intestinal loops
involved, the surgery consisted in an easy reduction of
the intussusception. Palpation of the invaginated loops
did not reveal any anomaly causing the intussusception. Fig. 2
The postoperative course was uneventful with rapid
normalization of the intestinal canalization. The boy was
discharged 14 days after the pyeloplasty. Six months later
the boy’s growth was normal.
In general, intussusceptions are located at the ileoce-
cal segment and their treatment consists in a barium or
air enema, which is able to solve the problem in more
than 75% of cases. When the intussusception involves
the upper intestinal loops, as in our case, surgery is the
best option.

Suggested Reading
1. Erichsen D, Sellstrom H, Andersson H. Small bowel intus- 3. Munden MM, Bruzzi JF, Coley BD, Munden RF. Sonogra-
susception after blunt abdominal trauma in a 6-year-old phy of pediatric small-bowel intussusception: differentiat-
boy: case report and review of 6 cases reported in the litera- ing surgical from nonsurgical cases. AJR Am J Roentgenol
ture. J Pediatr Surg 2006; 11:1930–1932 2007; 1:275–279
2. Grimprel E, de La Rocque F, Romain O, Minodier P, Dom- 4. Simanovsky N, Hiller N, Koplewitz BZ, Eliahou R, Udassin
mergues MA, Laporte-Turpin E, Lorrot M, Parez N, Caulin R. Is non-operative intussusception reduction effective in
E, Robert M, Lehors H, Cheron G, Levy C, Haas H. Man- older children? Ten-year experience in a university affili-
agement of intussusception in France in 2004: investigation ated medical center. Pediatr Surg Int 2006; 19:345–347
of the Paediatric Infectious Diseases Group, the French 5. Wiersma F, Allema JH, Holscher HC Ileoileal intussuscep-
Group of Paediatric Emergency and Reanimation, and the tion in children: ultrasonographic differentiation from ileo-
French Society of Paediatric Surgery Arch Pediatr 2006; colic intussusception. Pediatr Radiol 2006; 11:1177–1181
12:1581–1588
3 Abdomen 197

Q 95
Giovanni Esposito and Ciro Esposito

Fig. 1

The patient was a 7-year-old child, born at term after an child complained of abdominal pain. His abdomen was
uncomplicated pregnancy. At 3 months of age he was af- enlarged and at percussion a dull sound was revealed on
fected by meningitis, due to Escherichia coli and Pseu- the right site of the abdomen. Abdominal CT was per-
domonas, and complicated by hydrocephalus that was formed (Figs. 1, 2), followed by other procedures before
treated with a ventriculoperitoneal shunt (VPS). After planning the treatment.
1 year, a simple revision of the shunt was carried out • What does the CT scan show?
because of malfunctioning, and 6 years later, because • Which other procedures were performed?
of skin ulceration along its pressure points, the shunt • What was the diagnosis?
was replaced in the abdomen. Three days before admis- • What was the treatment?
sion to our hospital, the child had an abdominal colic • What was the follow-up?
episode complicated by convulsions. On admission, the
198 Abdomen

A 95

The CT shows a large intra-abdominal cyst with a diam-


eter of 10.7×9.7 that displaces the intestinal loops con-
necting the right kidney.
Laparoscopy was performed and a large pseudocyst
was found where the catheter shunt entered.
Laparoscopy treatment was decided consisting in
opening the pseudocyst, voiding its content, pulling the
catheter out, and replacing it in the abdomen (Fig. 3).
The follow-up was uncomplicated and a CT scan
showed the disappearance of the cyst.
At 2 years the child was doing well.
VPSs are frequently used to treat hydrocephalic chil-
dren. The frequency of complications in children with
VPS varies from 5% to 47%, and in the majority of the
cases involves the abdominal part of the catheter. Any Fig. 2
obstruction of the ventriculoperitoneal derivation sys-
tem will lead to endocranial hypertension; this should
be considered an emergency requiring immediate sur-
gery. Recently, laparoscopy using three ports has been
reported for the treatment of VPS complications.

Suggested Reading
1. Adeloye A. Spontaneous extrusion of the abdominal tube
through the umbilicus complicating peritoneal shunt for
hydrocephalus. Case report. J Neurosurg 1973; 38:758–760
2. Brunori A, Massari A, Macarone-Palmieri R, Benini B, Chi-
appetta F. Minimally invasive treatment of giant CSF pseu-
docyst complicating ventriculoperitoneal shunt. Minim
Invasive Neurosurg 1998; 41(1):38–39
3. Esposito C, Porreca A, Gangemi M, Garipoli V, De Pasquale Fig. 3
M. The use of laparoscopy in the diagnosis and treatment of
abdominal complications of ventriculo-peritoneal shunts
in children. Pediatr Surg Int 1998; 13(5–6):352–354
4. Grosfeld JL, Cooney DR, Smith J, Campbell RI. Intra-ab-
dominal complications following ventriculo-peritoneal
shunt procedures. Pediatrics 1974; 54:791–796
5. Kim HB, Raghavendran K, Kleinhaus S. Management of
an abdominal cerebrospinal fluid pseudocyst using laparo-
scopic techniques. Surg Laparosc Endosc 1995; 5(2):151–
154
3 Abdomen 199

Q 96
Christophe Chardot and Sylviane Hanquinet

Fig. 1 Fig. 2

Fig. 3 Fig. 4

A 4-week-old boy was taken to a general practitioner for • Why is prothrombin time prolonged?
routine examination. He was the third child in a family • What severe complication could occur?
without any history of congenital disease or malforma- • What should be done to avoid this?
tion. The mother’s pregnancy and delivery were unevent-
ful. The boy’s birth weight was 3,200 g. He was breastfed, Abdominal US was performed to confirm the diagnostic
with a normal transit. The mother told the doctor her hypothesis.
son’s skin and eyes had been slightly yellow since birth, • In which condition should the child be before the US
but this had recently become more obvious. He was in examination so that adequate information can be ob-
good general condition, weighing 3,800 g. His eyes and tained?
skin were moderately icteric. On examination, hepato-
megaly was found, palpated 4 cm below the costal mar- The US Doppler findings are shown in Figs. 1–4.
gin, without splenomegaly. During the examination, the • What other US signs may be seen in the syndromic
baby passed white stools. form of the disease?
Blood tests confirmed cholestasis. The Quick value • Is cholangiography needed in this case? Why?
was 25%. • What operation is necessary for this disease?
• What are the chances of success of this operation?
200 Abdomen

A 96

This patient is affected by biliary atresia (BA). The as-


sociation of neonatal jaundice (lasting after 2 weeks of
life), white stools, and hepatomegaly is highly suggestive
of this disease.
Prothrombin time is prolonged (Quick value de-
creased) due to vitamin K deficiency: severe cholestasis
induces a lack of bile salts in the intestine, and conse-
quently malabsorption of liposoluble vitamins ADEK.
Lack of vitamin K induces deficiency of coagulation fac-
tor synthesis. Severe hemorrhage can occur, especially
intracranial hemorrhage. Parenteral administration of
vitamin K should be carried out urgently. Fig. 5
Before the US examination, the child should be fast-
ing for at least 6 h, and consequently should have an
IV line with fluid, glucose, and electrolytes. The aim of hepatis. If some intrahepatic bile ducts are still patent,
fasting is to have a full gallbladder during the US. It is the biliary flow can be restored to the intestine. Yellow
important to check that the child is really nil by mouth liquid is often seen at the porta hepatis after resection of
when the US is performed, in order to avoid any misin- the extrahepatic biliary remnant, and can correspond to
terpretation if a shrunk gallbladder is found. either bile or icteric lymph. It is therefore impossible to
A shrunk and hyperechogenic gallbladder despite predict the success of the Kasai operation at this stage.
fasting, without intrahepatic bile ducts dilatation, In Europe and North America, results of the Kasai
strongly suggests biliary atresia. In the syndromic form operation are: clearance of jaundice (normal bilirubin
of BA (polysplenia syndrome or BA splenic malforma- levels) in about one-half of patients; one-third of pa-
tion syndrome), the following signs can be seen on US tients alive with their own liver at the age of 10 years;
(or CT): multiple spleens; median liver (hypertrophy of one-fourth to one-fifth of patients alive with their own
the left liver and more or less symmetrical right liver); liver at the age of 20 years.
preduodenal portal vein; absence of retrohepatic vena When the Kasai operation fails to clear jaundice, bili-
cava; and situs inversus. Figure 5 shows the operative ary cirrhosis progresses and liver transplantation is re-
picture: a cholestatic, fibrotic and enlarged liver and a quired, in most cases in the first few years of life. Today,
completely atretic gallbladder, which is characteristic of with the sequential treatment of Kasai operation and
BA. Complete atresia of the extrahepatic biliary remnant liver transplantation, if needed, overall survival of BA
is the most common form of BA. patients is about 90%.
Since the gallbladder is completely atretic, the diag-
nosis of BA is certain and cholangiography is not needed
(and would be technically unsuccessful).
In other cases of complete neonatal cholestasis (white- Suggested Reading
gray stools) with a nonatretic gallbladder, cholangiogra- 1. Chardot C. Biliary atresia. Orphanet J Rare Dis 2006;
phy is mandatory to assess the permeability of the bile 1(1):28
ducts and rule out a partial form of BA. This can be done 2. Chardot C, Serinet MO. Prognosis of biliary atresia: what
either preoperatively (by interventional radiology or by can be further improved? J Pediatr 2006; 148(4):432–5
ERCP) or intraoperatively. It is important to check the 3. Kasai M, Kimura S, Asakura Y, Suzuki Y, Taira Y, Obashi
color of bile (green or uncolored) before injecting the E. Surgical treatment of biliary atresia. J Pediatr Surg 1968;
contrast medium. 3(6):665–675
The first treatment in patients with BA is to perform 4. Lykavieris P, Chardot C, Sokhn M, Gauthier F, Valayer J,
Bernard O. Outcome in adulthood of biliary atresia: a study
hepatoportoenterostomy (Kasai operation), which con-
of 63 patients who survived for over 20 years with their na-
sists in resection of all the extrahepatic biliary remnant,
tive liver. Hepatology 2005; 41(2):366–71
and anastomosis of a Roux-en-Y jejunal loop at the porta
3 Abdomen 201

Q 97
Christophe Chardot and Sylviane Hanquinet

Fig. 1 Fig. 2 Fig. 4

A 24-year-old woman, without a family history of in-


herited disease or malformation, was in her second
pregnancy. A prenatal ultrasound at 26 weeks’ gestation
showed a female fetus with normal growth and vitality.
A cyst in the liver hilum was discovered (Fig. 1), the liver
looked normal, and a small gallbladder was identified.
The US was otherwise normal, without any other mor-
phological anomaly.
• What is the suspected diagnosis?
• What can be proposed to the mother?
Prenatal work-up showed no other fetal anomaly, and
the pregnancy continued uneventfully. The baby was
born after 38 weeks’ gestation and after a normal deliv-
ery, weighing 3,200 g. The infant’s neonatal adaptation
was good and clinical examination was normal. Neo-
natal US confirmed a cystic dilation in the liver hilum
(Fig. 2). The gallbladder and the liver looked normal.
The rest of the abdominal US was normal.
• What are the possible diagnoses, and how do you
manage the infant?

The infant tolerated breast-feeding very well, she passed


five to six gold-yellow stools every day, had no jaundice,
and her weight progressed normally. Liver test results
were normal. An MR image is shown in Fig. 3.
• How do you interpret Fig. 3? Fig. 3
• What is your final diagnosis?
• What anomaly of the common bile duct is usually
seen in this condition?
• What are the medium- and long-term risks of this
anomaly?
• What surgical treatment is therefore recommended?
• What is the prognosis for this child?
202 Abdomen

A 97

A cyst in the liver hilum may correspond to: form of biliary atresia, and Fig. 5 is the intraoperative
– A cystic form of biliary atresia, which requires urgent cholangiogram.
diagnosis and treatment after birth
– A choledochal cyst In the case of choledochal cyst, the child is likely to have
– A duodenal duplication a normal clinical neonatal course, without cholestasis.
– An ovarian cyst The surgical treatment can be postponed, and is usually
– A cystic lymphangioma performed after 3 months of life.
Clinical evolution allows one to exclude biliary atre-
The mother needs to be referred to a prenatal diagnosis sia. MR imaging findings confirm the diagnosis of cho-
and counseling center. ledochal cyst, with a major dilation of the common bile
Repeated US will follow the evolution of the cyst (size duct and hepatic duct, and a mild dilation of the gall-
and echogenicity), the liver, and the gallbladder, and will bladder and cystic duct. The intrahepatic bile ducts are
look for other morphological anomalies, especially car- regular and slightly enlarged.
diopathy, situs inversus, and interruption of the inferior Choledochal cyst is usually associated with a bilio-
vena cava with azygos continuation. pancreatic common channel: the common bile duct and
Amniocentesis is not routinely indicated in cases of the Wirsung duct end in a common biliopancreatic duct,
isolated cyst of the liver hilum. Depending on the associ- which originates above the sphincter of Oddi. This con-
ated prenatal findings, it may be advised for: dition causes a reflux of pancreatic juice in the bile ducts,
• Karyotype, although none of the suspected diagnoses with subsequent biliary epithelium abrasion, inflamma-
is commonly associated with chromosomal anoma- tion, dysplasia, and finally a risk of cholangiocarcinoma
lies. in adulthood. The cystic dilation of bile ducts may also
• Genetic screening for cystic fibrosis, if indicated. increase in childhood, with chronic cholestasis, liver fi-
• Biological screening of digestive enzymes in the am- brosis, gallstone formation, and the risk of acute biliary
niotic fluid cannot be done in this case. Their level in obstruction and/or cholangitis.
the amniotic fluid declines between 18 and 24 weeks • Surgery must:
of gestation, due to closure of the anal sphincter: it is – Disconnect the pancreatic and biliary ducts, in order
therefore too late to detect a biochemical profile of to suppress the pancreaticobiliary reflux and subse-
biliary obstruction (low GGTs and normal intestinal quent chronic biliary inflammation.
enzymes). – Resect the diseased extrahepatic bile ducts.
Consultation with a pediatric surgeon is recommended, The recommended operation is therefore resection of the
in order to inform the parents about the postnatal man- extrahepatic biliary tree from the hepatic duct (or extra-
agement of the child. hepatic biliary bifurcation) to the intrapancreatic termi-
This clinical picture and the US findings (Fig. 2) are nation of the common bile duct, and biliary reconstruc-
highly suggestive of choledochal cyst, although a cystic tion with a hepaticojejunostomy on a Roux-en-Y loop.
form of biliary atresia cannot be completely excluded. During the operation, an assay of pancreatic enzymes
The child can be fed normally, and must be closely fol- (amylase, lipase) in bile usually reveals very elevated lev-
lowed up. els, confirming the reflux of pancreatic juice in the bile
– A normal gallbladder does not exclude biliary atresia. ducts. A cholangiogram is recommended (if it has not
Clinical evolution in the first weeks of life will show been done preoperatively) in order to precisely evalu-
whether complete cholestasis appears: in the case of ate the intrahepatic extension of the biliary dilations. A
biliary atresia, the stools would rapidly become white liver biopsy is recommended to evaluate potential liver
and jaundice would appear. Blood tests would show fibrosis.
cholestasis, and the work-up would confirm obstruc- In this form with limited intrahepatic biliary dila-
tion of the bile ducts. Kasai operation should be per- tions, the prognosis is good, and the child is likely to
formed urgently, in the first weeks of life. Figure 4 have a completely normal life. In cases of large intrahe-
shows an operative view of another child with a cystic patic biliary dilations, low-grade cholestasis may persist,
3 Abdomen 203

with a risk of gallstone formation and cholangitis. Such


patients need close follow-up and ursodeoxycholic acid
may be necessary to increase the biliary flow, reduce the
risk of gallstone formation, and protect the liver against
the consequences of chronic cholestasis. Biliopancreatic
disconnection and resection of the extrahepatic biliary
tree including the intrapancreatic common bile duct are
expected to lower the risk of biliary chronic inflamma-
tion, dysplasia, and cancer. As with any abdominal sur-
gery, the parents must be informed of the risk of postop-
erative adhesions and intestinal obstruction. In all cases,
a prolonged follow-up is recommended.

Fig. 5

Fig. 6

Suggested Reading
1. Brunero M, De Dreuzy O, Herrera JM, Gauthier F, Valayer 3. Todani T, Watanabe Y, Fujii T, Toki A, Uemura S, Koike
L. [Prenatal detection of a cyst in the liver hilum. Interpre- Y. Congenital choledochal cyst with intrahepatic involve-
tation for an adequate treatment]. Minerva Pediatr 1996; ment. Arch Surg 1984; 119(9):1038–43
48(11):485–94 4. Todani T, Watanabe Y, Fujii T, Toki A, Uemura S, Koike Y.
2. Muller F, Oury JF, Dumez Y, Boue J, Boue A. Microvillar Cylindrical dilatation of the choledochus: a special type of
enzyme assays in amniotic fluid and fetal tissues at different congenital bile duct dilatation. Surgery 1985; 98(5):964–9
stages of development. Prenat Diagn 1988; 8(3):189–98
3 Abdomen 205

Q 98
Christophe Chardot and Sylviane Hanquinet

Fig. 1 Fig. 2

A 24-month-old girl was referred to the emergency de- • What simple US criterion allows you to define sple-
partment for episodes of pain and vomiting, which had nomegaly, whatever the age of the child?
appeared 3 h previously. She had been in good health • How do you interpret the following blood test re-
until then. During examination, she seemed tired and sults?
experienced several episodes of pain. Her temperature Hb 75 g/l; leukocytes 5,600/mm3; platelets 153,000/
was 37°C. She was pale and slightly subicteric. Her abdo- mm3; CRP 15 mg/l; ASAT 245 UI/l; ALAT 234 UI/l;
men was soft, but palpation of the right upper quadrant GGT 158 UI/l; total bilirubin 65 µmol/l; direct biliru-
was painful. There was no hepatomegaly, no abdominal bin 25 µmol/l; amylase 484 UI/l; lipase 198 UI/l.
mass. Splenomegaly was palpated 3 cm below the cos- • Which relevant clinical information is missing in this
tal margin. Her nappy contained normal stools, without observation?
blood. The rest of the clinical examination was normal.
• What is your first diagnostic hypothesis regarding the Under painkillers and spasmolytic therapy, the patient’s
current episode? abdominal pain resolved after a few hours. Liver and
pancreatic blood values normalized in the following
The abdominal US and CT scan show: days. Hematological investigations confirmed sphero-
– The absence of intussusception cytosis. Abdominal US confirmed the complete migra-
– A normal liver parenchyma tion of the stone, with regression of the biliary dilation,
– A gallbladder containing multiple small stones and normalization of the pancreas. Multiple small stones
(Fig. 1) persisted in the gallbladder.
– A stone in the lower common bile duct • What surgical treatment do you propose and why?
– Dilated bile ducts (Fig. 2).
– An edematous head of the pancreas (Fig. 2)
– A splenomegaly
206 Abdomen

A 98

Due to the patient’s age and the paroxystic abdominal cystectomy alone (drepanocytosis) or associated with
pain episodes, intussusception has to be ruled out. After subtotal splenectomy (spherocytosis).
this, the suspected diagnosis is biliary stone migration – Biliary lithiasis in early infancy: it may be related to
(right upper quadrant abdominal pain, vomiting, and transient immaturity of bilirubin conjugation in the
absence of fever), secondary to chronic hemolysis (pale perinatal period, especially in premature babies. If
and subicteric, splenomegaly). treatment is needed (due to stone migration) and
If the length of the spleen is more than the length of a the bile ducts are presumably normal, a conservative
normal left kidney, there is a splenomegaly. treatment by interventional radiology (percutaneous
These blood tests show an anemia which is probably transhepatic flushing of the stones) without chole-
hemolytic (elevated unconjugated bilirubin), the absence cystectomy may be attempted; if successful, biliary
of inflammatory syndrome, mild cholestasis (elevation lithiasis has a low risk of recurrence.
of liver enzymes with mild elevation of GGTs and of In the reported case, cholecystectomy is required due
conjugated bilirubin), and mild pancreatic reaction. to previous stone migration, in order to avoid recur-
These biological findings are consistent with biliary rence of such episodes and potential biliary or pan-
stone migration and chronic hemolysis. creatic complications.
The familial history reveals that the father underwent Regarding spherocytosis, severe anemia pleads in
splenectomy in childhood for hereditary spherocytosis, favor of reducing hemolysis. At this age, the recom-
and the grandfather underwent cholecystectomy in early mended treatment is subtotal (7/8) splenectomy,
adulthood for cholelithiasis. Most likely this child also which allows one to significantly reduce the level of
has hereditary spherocytosis. hemolysis and to preserve immunological functions
Biliary lithiasis in childhood can be divided into the of the splenic remnant.
following subgroups: These two procedures can be combined in the same
– Asymptomatic biliary lithiasis, without underlying operation, which can be partly or totally performed
hemolysis: no indication for surgery. by laparoscopy. After subtotal splenectomy, hemato-
– Symptomatic or complicated biliary lithiasis: indi- logical and US Doppler follow-up is needed:
cation for surgery (laparoscopic cholecystectomy). – In the early postoperative period to measure the size
It is recommended to search for hemolysis, which and check the blood supply of the splenic remnant
may require its own surgical treatment in the same (Figs. 3, 4).
operation. – In the follow-up to measure the size of the splenic
– Asymptomatic microlithiasis in hemolytic anemias: remnant, since some regrowth is usual and may
due to the high risk of severe complications, preven- (rarely) require secondary splenectomy totalization,
tive surgical treatment may be advised, either chole- if significant hemolysis reappears.

Fig. 3 Fig. 4
3 Abdomen 207

Q 99
Yves Aigrain and Pascale Philippe-Chomette

Fig. 1 Fig. 2

A 4-year-old girl with Turner syndrome was admitted to A “wait and see” approach was decided on. Two years
hospital for abdominal hemorrhage. later, this girl was readmitted for acute gastrointestinal
Rectal bleeding and melena were observed. hemorrhage and needed three blood transfusions within
• What is your strategy and which two examinations 10 days.
do you ask for first? A video capsule total intestinal endoscopy was per-
formed and was not contributive, showing only intesti-
After these examinations, a US study and CT scan were per- nal bleeding without any identifiable lesion.
formed; however, these two examinations were not con- • What is your hypothesis?
tributive and laparoscopy was performed. No Meckel di-
verticulum was observed but intestinal telangiectasias on Figures 1 and 2 show the internal and external investiga-
the small bowel (about 70-cm length) were identified. tions (endoscopy coupled with laparotomy).
• How do you interpret these images?
208 Abdomen

A 99

After laboratory tests (patient’s blood, platelet count, he- Considering that this patient presented with recur-
moglobin and hematocrit values), an esophagogastro- rent bleeding, laparotomy was performed with com-
duodenoscopy and a rectal examination with complete bined upper endoscopy, the bowel was carefully pal-
colonoscopy were performed. pated and transilluminated looking for the symptomatic
Turner’s syndrome associated with telangiectasias telangiectasias.
has been described previously. Telangiectasias are diffi- Figures 1 and 2 show intra- and extraluminal aspects
cult to see at endoscopy and on CT scans. Laparoscopy of telangiectasias. It is rather difficult to localize them
could be helpful in eliminating Meckel’s diverticulum with endoscopy alone, and therefore endoscopy should
in the case of rapid deglobulization and in establishing be coupled with laparotomy. After collegial discussion
the diagnosis of telangiectasia. Hormonal therapy has we decided to resect 70 cm of consecutive small bowel.
shown its efficacy in this syndrome but it could not be After 5 years of follow-up, there is no recurrent bleeding
proposed for this girl before puberty. and no need for hormone therapy.

Suggested Reading
1. Eroglu Y, Emerick KM, Chou PM, Reynolds M. Gastroin- 3. Vuillemin E, Rifflet H, Oberti F, Cales P, Wion-Barbot N,
testinal bleeding in Turner’s syndrome: a case report and Ben Bouali A. Turner syndrome and digestive telangiecta-
literature review. J Pediatr Gastroenterol Nutr 2002 Jul; sis: an additional value of oestrogen-progestational treat-
35(1):84–7 ment. Gastroenterol Clin Biol 1996; 20(5):510–1
2. Nudell J, Brady P. A case of GI hemorrhage in a patient with
Turner’s syndrome: diagnosis by capsule endoscopy. Gas-
trointest Endosc 2006 Mar; 63(3):514–6
3 Abdomen 209

Q 100
Yves Aigrain and Pascale Philippe-Chomette

A healthy 4-year-old boy presented with abdominal ob-


struction with fever (38.5°C) and pain.
For 2 days the boy had been vomiting and had ab-
dominal distension. The clinical examination revealed a
painful medial abdominal mass.
Figure 1 is standard abdominal radiograph and Fig. 2
is a CT scan.
• Can you comment on the case?
• What is your diagnosis?
• As a surgeon, what is your initial approach?
• After a complete recovery, what is your next ap-
proach?
• What is the main risk?

Fig. 1

Fig. 2
210 Abdomen

A 100

The repeat plain abdominal radiographs show a mass ef-


fect on the left quadrant of the abdomen with displace-
ment of the viscera.
The CT scan shows a large cystic mass with a hemor-
rhagic component. The diagnosis is an abdominal cystic
lymphangioma with inflammatory and infectious com-
plications.
The initial approach is to treat infection with paren-
teral antibiotherapy and gastric aspiration.
This benign lesion needs complete excision because
of the symptomatic effects and the risk of recurrent in-
testinal obstruction. This particular lymphangioma de-
veloped in the mesenteric area with infiltration of the
small bowel and needed small-bowel resection (Fig. 3).
Although cystic lymphangioma is a rare benign le- Fig. 3
sion, recurrence is the main risk. The resection must be
complete and for mesenteric localization bowel resec-
tion is often necessary.

Suggested Reading
1. Konen O, Rathaus V, Dlugy E, Freud E, Kessler A, Shapiro
M, Horev G. Childhood abdominal cystic lymphangioma.
Pediatr Radiol 2002 Feb; 32(2):88–94
2. Singh S, Maghrabi M. Small bowel obstruction caused
by recurrent cystic lymphangioma. Br J Surg 1993 Aug;
80(8):1012
3 Abdomen 211

Q 101
Yves Aigrain and Pascale Philippe-Chomette

A premature twin (30 weeks), weighing 900 g, with a


dilated jejunum detected at 20 weeks’ gestation was re-
ferred to our department.
At birth, the boy was clinically healthy and needed no
respiratory assistance.
A standard radiograph was obtained (Fig. 1).
• Can you comment on this examination?
• Before birth, what sort of examinations would you
request if intestinal obstruction is suspected?
• Considering the prematurity, what is your initial ap-
proach as a surgeon?

Fig. 1
212 Abdomen

A 101

A gastric and duodenal distension with “double bubble


sign” and a gasless abdomen were identified.
These findings could fit the description of a proximal
intestinal atresia or the description of a midgut volvulus.
In view of the enlarged jejunum described at 20 weeks,
we diagnosed proximal jejunal atresia, with uncertainty
about the rest of the small bowel (multiple atresias, ante-
natal volvulus, apple peel syndrome).
Prenatal ultrasound examinations detect some anom-
alies evoking proximal intestinal obstruction:
– Isolated enlarged stomach and duodenum (“double
bubble sign”) associated with polyhydramnios are
suggestive of duodenal junction obstruction, while
calcifications or hyperechogenicity of the loops may Fig. 2
indicate meconium peritonitis.

As we detected these anomalies, MR imaging was per- with a digestive enzyme assay which suggested proximal
formed within 2 weeks of the US examination. Obstruc- obstruction with no detection of cystic fibrosis.
tion is considered proximal when the dilated loops are This premature twin had jejunal atresia, and an in-
hypointense on T1-weighted images and distal when testinal permeability test revealed an associated jejunal
they are hyperintense on T1-weighted images with nor- diaphragm 3 cm below. We performed a “onetime” repa-
mal loops visible below the stomach. ration without stoma (Fig. 2) and with duodenal caliber
Amniocentesis is proposed for amniotic fluid assays reduction; at the same time a CVC was inserted. Three
for karyotyping, cystic fibrosis screening, and digestive weeks after reparation, intestinal transit started and en-
enzyme assays. teral feeding could be initiated. We recommend “one-
This boy did not undergo MR imaging considering time” reparation when there is proximal atresia without
his prematurity; however, amniocentesis was carried out involvement of the distal bowel.

Suggested Reading
1. Besner GE, Bates GD, Boesel CP, Singh V, Welty SE, Cor- 3. Garel C, Dreux S, Philippe-Chomette P, Vuillard E, Oury JF,
pron CA. Total absence of the small bowel in a premature Muller F. Contribution of fetal magnetic resonance imaging
neonate. Pediatr Surg Int 2005 May; 21(5):396–9 and amniotic fluid digestive enzyme assays to the evalua-
2. Escobar MA, Ladd AP, Grosfeld JL, West KW, Rescorla FJ, tion of gastrointestinal tract abnormalities. Ultrasound Ob-
Sherer LR, Engum SA, Rouse TM, Billmire DF. Duodenal stet Gynecol 2006; 28:282–291
atresia and stenosis. J Pediatr Surg 2004 Jun; 39(6):867–71:
long term follow up over 30 years.
3 Abdomen 213

Q 102
Felix Schier

Fig. 1 Fig. 2

A 6-year-old girl presented with acute, persistent up- Puncture and aspiration yielded hemorrhagic ascites.
per abdominal tenderness and distension. There was no • What does Fig. 1 show?
trauma or sepsis, and she was not taking any medication. • What does Fig. 2 show?
The girl had nausea and vomiting. • What is the most likely diagnosis?
Serum and urinary amylase and lipase levels were • What may be the cause?
elevated. • What is the most adequate therapy?
An ultrasound examination was performed, shown
in Figs. 1 and 2.
214 Abdomen

A 102

Figure 1 shows a transverse ultrasonographic section of Clinically, more than 90% of the children will have
the upper abdomen at the level of the pancreas. pain, more than 60% will vomit, and more than 30% will
The pancreas is markedly enlarged in all segments have fever.
and is of reduced echogenicity. Bowel rest, nasogastric tube, intravenous support,
Figure 2 shows a longitudinal cross-section at the and the correction of electrolytes, glucose, and calcium
level of the left anterior axillary line. The black area is a are the main aspects of therapy for acute pancreatitis.
broad fluid collection with fine internal echoes caudally The basic idea is to reduce pancreatic activity by stop-
to the inferior spleen pole, also cranially and ventrally to ping enteral feedings. The efficacy of the nasogastric
the left kidney. tube is controversial. In parallel, a search for a possible
The most likely diagnosis is hemorrhagic pancreatitis. anatomical, chemical, or metabolic cause should be con-
The pathophysiology of acute pancreatitis is unclear ducted, leading to a specific therapy when identified.
in most cases. The most frequent causes in children are More than 75% of patients will recover without need-
idiopathic, cystic fibrosis or diabetes, biliary tract dis- ing surgery.
ease, and trauma with ductal lesions. Also, systematic Despite the figures shown above, the sensitivity of ul-
diseases such as lupus erythematosus, sepsis, or shock trasonography in detecting acute pancreatitis is inferior
have been described as possible causes. to the sensitivity of CT.

Suggested Reading
1. Akel S, Khalifeh M, Makhlouf Akel M. Gallstone pancre- 3. Nydegger A, Couper RT, Oliver MR. Childhood pancreati-
atitis in children: atypical presentation and review. Eur J tis. J Gastroenterol Hepatol 2006; 21:499–509, Review
Pediatr 2005; 164:482–485, Review 4. Stringer MD. Pancreatitis and pancreatic trauma. Semin
2. Benifla M, Weizman Z. Acute pancreatitis in childhood: Pediatr Surg 2005; 14:239–246, Review
analysis of literature data. J Clin Gastroenterol 2003;
37:169–172, Review
3 Abdomen 215

Q 103
Felix Schier

Fig. 1 Fig. 2

A 4-year-old boy presented with right-sided lower ab- avoided moving the right psoas muscle. However, the
dominal pain that had started 4 days earlier. The pain child had a good appetite and wished to eat and drink.
increased over the last 2 days, reached a peak on the Laboratory tests demonstrated an elevated leukocyte
2nd day, and decreased slightly over the next 2 days. Ini- count and C-reactive protein level.
tially there was vomiting; his fever persisted. Ultrasound studies yielded the images shown in
There was marked tenderness on palpation of the right Figs. 1 and 2.
lower abdomen. The remaining abdomen was tender. • What is the most likely diagnosis?
Both Rovsing and Blumberg signs were positive. The • What is the therapy?
child avoided walking, had to be carried, and especially
216 Abdomen

A 103

Figure 1 is a longitudinal section of the right lower ab- Suggested Reading


domen. There is a fluid collection of 7-cm diameter cau- 1. Bixby SD, Lucey BC, Soto JA, Theyson JM, Ozonoff A, Var-
dally to the right kidney (K) with marked internal echoes. ghese JC. Perforated versus nonperforated acute appendi-
Most conspicuous is a semilunar hyperechoic structure citis: accuracy of multidetector CT detection. Radiology
with a dorsal acoustic shadow (arrow). This is a fecalith 2006; 241:780–786
within an abscess (RE, patient’s right side). 2. Brown CV, Abrishami M, Muller M, Velmahos GC. Ap-
Figure 2 shows a transversal section of the right mid- pendiceal abscess: immediate operation or percutaneous
dle abdomen. Just ventrally to the kidney (K), again, the drainage? Am Surg 2003; 69:829–832
fluid-containing structure is seen with marked internal 3. Lasson A, Lundagards J, Loren I, Nilsson PE. Appendiceal
abscesses: primary percutaneous drainage and selective in-
echoes (arrow). The abscess is almost the size of the kid-
terval appendicectomy. Eur J Surg 2002; 168:264–269
ney (K). “S” marks the vertebral column.
4. Samuel M, Hosie G, Holmes K. Prospective evaluation of
The diagnosis is perityphlitic abscess with a fecalith
nonsurgical versus surgical management of appendiceal
inside the abscess cavity.
mass. J Pediatr Surg 2002; 37:882–886
CT is not very good in detecting perforation in ap-
5. Wootton-Gorges SL, Thomas KB, Harned RK, Wu SR,
pendicitis. Stein-Wexler R, Strain JD. Giant cystic abdominal masses
The best treatment of appendiceal masses is contro- in children. Pediatr Radiol 2005; 35:1277–1288
versial at present. It would appear that evacuation of the
abscess, for example, by ultrasonographic guidance, is
indicated as soon as the diagnosis is established. Several
studies support this approach. An appendectomy will
follow later.
3 Abdomen 217

Q 104
Felix Schier

Fig. 1
Fig. 2

In a newborn boy, all attempts at feeding resulted in im- On examination, the child appeared otherwise nor-
mediate vomiting. Prenatal ultrasound examinations mal. No malformations were known and all laboratory
were normal. The abdomen was not distended and the values were normal. Radiological studies yielded the im-
vomit did not smell sour. ages shown in Figs. 1 and 2.
• What is the most likely diagnosis?
218 Abdomen

A 104

Figure 1 shows a large, oval epiphrenic mass, shifting the referred to the surgeon for fundoplication or placement
heart to the left. The abdomen is gasless. Figure 2 is a of a gastrostomy with the referring physician not be-
contrast study of the esophagus and the stomach. The ing aware of the presence of microgastria. For the same
stomach is small and located mostly intrathoracically. reasons, surgical correction may be delayed. The young-
The esophagus is unusually short. The contrast medium est child to have undergone gastric augmentation was
is promptly regurgitated and even aspirated, addition- 3 months old. As a consequence, in children with severe
ally contrasting the trachea and bronchi. reflux symptoms since birth, radiographic contrast stud-
The diagnosis is brachyesophagus and microgastria. ies should be evaluated carefully for reduced gastric res-
Congenital microgastria has been postulated to be ervoir capacity. Prolonged medical management is not
the result of impaired normal foregut development. Ap- beneficial in congenital microgastria, because the stom-
parently, only 39 cases have been described in the litera- ach size does not increase significantly with the passage
ture. Clinically, reflux signs, failure to thrive, and growth of time.
retardation are the consequences. After gastric augmentation the gastroesophageal re-
Preliminary jejunostomy feeding has been suggested flux will cease.
as a primary treatment modality. Subsequently, gastric Microgastria is seldom an isolated symptom. In most
augmentation (Hunt-Lawrence pouch) is performed. children there are associated congenital malformations.
From the few cases described so far, it seems that even They include asplenia, intestinal malrotation, cardiopul-
the gastroesophageal sphincter may regain its compe- monary anomalies, central nervous system and renal
tence once the reservoir capacity of the small stomach anomalies, laryngotracheobronchial clefts, and limb-re-
is restored. duction defects including total amelia. Prenatal ultra-
An initial diagnosis of gastroesophageal reflux is not sonographic demonstration of these anomalies should
unusual in children with microgastria. The child may be alert the surgeon.

Suggested Reading
1. Hoehner JC, Kimura K, Soper RT. Congenital microgastria. 4. Murray KF, Lillehei CW, Duggan C. Congenital microgas-
J Pediatr Surg 1994; 29:1591–1593 tria: treatment with transient jejunal feedings. J Pediatr
2. Kroes EJ, Festen C. Congenital microgastria: a case report Gastroenterol Nutr 1999; 28:343–345
and review of literature. Pediatr Surg Int 1998; 13:416–418 5. Ramos, CT, Moss RL, Musemeche CA. Microgastria as an
3. Menon P, Rao KL, Cutinha HP, Thapa BR, Nagi B. Gastric isolated anomaly. J Pediatr Surg 1996; 31:1445–1447
augmentation in isolated congenital microgastria. J Pediatr
Surg 2003; 38:E4–6
3 Abdomen 219

Q 105
Felix Schier

Fig. 1 Fig. 2 Fig. 3

Fig. 4 Fig. 5 Fig. 6

A 12-year-old boy presented with right-sided lower ab- Laboratory tests showed a slightly elevated leukocyte
dominal pain that had started 4 days earlier. The pain count and a markedly elevated C-reactive protein level.
decreased over the last 2 days. His aunt suffered from The erythrocyte sedimentation rate was markedly el-
ulcerative colitis. evated and the hemoglobin value decreased.
Initially there was vomiting, and bloody diarrhea had Ultrasound examinations yielded the images shown
set in during the last 2 days. He had a moderate fever. in Figs. 1–6.
There was tenderness and possibly a mass on palpa- • What is the most likely diagnosis?
tion of the right lower abdomen.
220 Abdomen

A 105

Figure 1 shows the terminal ileum, which is stenotic Crohn’s disease patients often have a positive family
and displays an increased distance to the adjacent small history of the disease.
bowel loops. The cecum is compressed. Multiple spicu- It is not unusual for primary manifestations of
lae are identified (a source of fistulae). Crohn’s disease to be misinterpreted as acute appendici-
In Fig. 2, in addition to the stenosis of the terminal tis or gastrointestinal infection.
ileum, a cobblestone mucosa is seen. The bowel wall is Ultrasonographic diagnosis of Crohn’s disease in
thickened. such a small child is unusual. Usually contrast studies
Figure 3 is a longitudinal section of the terminal il- are undertaken in order to search for stenosis, muco-
eum. Again, there is marked thickening of the bowel wall, sal disease, and skip lesions. Bowel wall thickening is
especially at the level of the submucosa. The muscularis better seen with CT. Recently, 18F-FDG-PET has been
propria is also thickened and demonstrates irregular in- attributed with a high diagnostic value in pediatric pa-
terphases with the broadened mesenterial fat. tients with chronic inflammatory bowel disease. In ad-
In Fig. 4, as before, there is wall thickening of the ter- dition, CE-mannitol-MR imaging has been described to
minal ileum, especially of the submucosa. The muscula- contribute significantly to the identification of disease
ris propria shows infiltrations of the massively thickened extension, severity, and intestinal complications with
mesenterial fatty tissue. adequate diagnostic accuracy. This technique has been
Figure 5 is a cross-section of the terminal ileum. The suggested for diagnostic exploration in young patients
bowel wall is thickened and contains an enlarged hyper- with suspected Crohn’s disease.
echoic submucosa. There is massive enlargement of the Wireless video capsule endoscopy (VCE) is increas-
hyperechoic mesenterial fatty tissue, almost completely ingly described, possibly also used, in children with a di-
encircling the bowel loops (“creeping fat”). A few spicu- agnosis of functional abdominal pain. Even children can
lae are cross sectioned from the muscularis propria into swallow the capsules without major problems. VCE has
the mesenterial fat. also been used to detect Crohn’s disease. VCE is more
On color-coded Doppler images (Fig. 6) a hypervas- sensitive than endoscopy for detection of macroscopic
cularity of the mesenterial fat and especially of the thick- gastric and small-bowel pathologies.
ened ileum is noticeable, a sign of acute inflammation. Laparoscopy has been used for early detection of
The diagnosis is Crohn’s disease. Crohn’s disease, also in combination with endoscopy.
There are only few data on inflammatory bowel disease It provides an early macroscopic impression of Crohn’s
in young children. It seems that these children often pres- disease in children, a picture not obtained previously.
ent with rectal bleeding and primarily colonic involve- However, since primary treatment of Crohn’s disease is
ment. Furthermore, it has been observed that half of the usually nonsurgical, and is directed by the “activity” of
children had a positive history of neonatal or early-onset the disease process, the significance of the macroscopic
bacterial infection with the use of antibiotics before the picture is relatively unimportant.
onset of inflammatory bowel disease. Some patients were The most efficient screening strategy for Crohn’s dis-
even still being breastfed, others were just weaned when ease in children seems to be measurement of the eryth-
GI symptoms started. Most children eventually needed rocyte sedimentation rate and hemoglobin. This combi-
bowel rest, parenteral nutrition, and steroid, azathio- nation appears to have a higher positive predictive value
prine, or cyclosporine medication. One-third of patients and is more sensitive, more specific, and less costly than
required surgery. Neonatal inflammatory bowel disease commercial serologic testing.
seems to be more severe in presentation and evolution.
3 Abdomen 221

Q 106
Felix Schier

Fig. 1 Fig. 2 Fig. 3

A 9-month-old child presented with intermittent vom-


iting since birth. There had been several previous epi-
sodes of pneumonia.
On auscultation there were normal breath sounds.
Laboratory values were normal except for signs of ane-
mia. Imaging studies yielded the pictures shown in
Figs. 1–3.
• What is the most likely diagnosis?
222 Abdomen

A 106

Figure 1 is an esophagogram demonstrating parts of the vulus requiring emergency surgery does occur, however.
fundus within the thorax. Some children will also have malrotation.
On the ultrasound image in Fig. 2, a layered, longitu- Radiologically, a cystic or opaque mass is seen in the
dinal, tubular structure is seen at the hiatus (arrow). posterior mediastinum in the right lower chest, occa-
In Fig. 3 the structure can be followed cranially to an sionally an air–fluid level in the cystic mass or a dilated
epiphrenic parenchymal mass. esophagus is also noted. In most cases, the gastroesoph-
The diagnosis is paraesophageal hiatal hernia. ageal junction is displaced into the stomach. Some have
Paraesophageal hernias are uncommon in children the gastroesophageal junction within the abdomen.
and are different from the more common sliding hiatus Ultrasonographically obvious pictures, as demon-
hernias. They may be combined with a sliding hernia. strated here, are usually not obtained.
Most often they are found on the right side. They also The principles of surgery are the reduction of the
occur as a complication after a previous antireflux op- contents, partial excision of the sac, crural approxima-
eration, especially if there was no crural plication. Chil- tion, and a fundoplication. Omission of a fundoplication
dren with preoperative gagging or slow corrected gastric is likely to result in reflux.
emptying seem to be at higher risk of developing a para- The laparoscopic treatment of large paraesophageal
esophageal hernia postoperatively. and mixed hiatal hernias is feasible and safe. The long-
The most common symptoms in children with para- term results, however, are less favorable. The anatomical
esophageal hernias are chest infections, vomiting, ane- and functional recurrence rate is around 40%. It is un-
mia, failure to thrive, and dysphagia. In some children, known at present which patients are at risk of failure and
the diagnosis is established incidentally. Strangulation, which technical modifications need to be made in order
in contrast to what one would expect, virtually does not to prevent recurrences.
exist. Mechanical obstruction owing to organoaxial vol-

Suggested Reading
1. Avansino JR, Lornez ML, Hendrickson M, Jolley SG. Char- 4. Schier F. Indications for laparoscopic antireflux procedures
acterization and management of paraesophageal hernias in children. Indications for laparoscopic antireflux proce-
in children after antireflux operation. J Pediatr Surg 1999; dures in children. Semin Laparosc Surg 2002; 9:139–45
34:1610–1614 5. Targarona EM, Novell J, Vela S, Cerdan G, Bendahan G,
2. Imamoglu M, Cay A, Kosucu P, Ozdemir O, Orhan F, Sa- Torrubia S, Kobus C, Rebasa P, Balague C, Garriga J, Trias
pan L, Sarihan H. Congenital paraesophageal hiatal hernia: M. Mid term analysis of safety and quality of life after the
pitfalls in the diagnosis and treatment. J Pediatr Surg 2005; laparoscopic repair of paraesophageal hiatal hernia. Surg
40:1128–1133 Endosc 2004; 18:1045–1050
3. Karpelowsky JS, Wieselthaler N, Rode H. Primary parae-
sophageal hernia in children. J Pediatr Surg 2006; 41:1588–
1593
3 Abdomen 223

Q 107
Felix Schier

Fig. 1 Fig. 2 Fig. 4

A 7-year-old boy presented with right lower quadrant


mass and pain, fever, vomiting, and leukocytosis. The
leukocyte and C-reactive protein levels were moderately
elevated.
Imaging studies yielded the pictures shown in
Figs. 1–4.
• What is the most likely diagnosis?

Fig. 3
224 Abdomen

A 107

Figure 1 is a cross-section of the terminal ileum. The There is no consistently reliable nonoperative way to
wall is thickened. Hyperechoic structures are identified separate a sporadic case of appendicitis from one whose
within the submucosa (arrowheads), representing lym- appendicitis-like symptoms are due to Yersinia. In addi-
phofollicular hypertrophy. There is slight thickening of tion, a small percentage of Yersinia patients will present
the submucosa. The muscularis propria is unchanged with true appendicitis as a complication of their disease.
(arrow). In a study of 352 patients who were hospitalized with
In Fig. 2, there is normal haustration near the cecum. symptoms of an acute appendicitis, Yersinia infections
Several enlarged lymph follicles make the shape of the were determined in almost 20% of patients by cultural
lumen irregular. and serological methods. Infections due to Y. entero-
Figure 3 is an ultrasonogram of the right lower abdo- colitica serovar 0:3 were approximately six times more
men. There are enlarged lymph nodes within the mes- frequent than those due to Y. enterocolitica serovar 0:9.
entery (L). Yersinia pseudotuberculosis could only be isolated in one
Figure 4 is a color-coded Doppler cross-sectional so- patient from a mesenterial lymph node. The majority of
nogram of the terminal ileum. The bowel wall is enlarged the infections were found in the age group of 9–12 years.
and displays increased vascularity. The mesenterial fat is The incidence was highest in the summer months, June–
not involved. The lymph follicles are hyperplastic. August.
The diagnosis is ileitis terminalis (yersiniosis). The dif- Other studies have found lower rates of yersiniosis in
ferential diagnosis to Crohn’s disease includes no spiculae “appendicitis,” especially in the United States.
and no or only little broadening of the mesenterial fat. Numerous publications discuss the advantages and
The clinical symptoms are very similar to classic ap- disadvantages of preoperative ultrasonography and CT
pendicitis: lower abdominal pain, fever, vomiting, and a in the diagnosis of appendicitis. This case demonstrates
right lower quadrant mass associated with leukocytosis. that ultrasonography, when performed by an expert,
Upon surgery, terminal ileitis is found, and eventually may be helpful in establishing the correct diagnosis.
cultures of peritoneal fluid and of mesenteric lymph
nodes will grow Yersinia enterocolitica.

Suggested Reading
1. Baier R, Puppel H, Zelder O, Heiming E, Bauer E, Syring 3. Shorter NA, Thompson MD, Mooney DP, Modlin JF. Surgi-
J. Frequency and significance of infections due to Yersinia cal aspects of an outbreak of Yersinia enterocolitis. Pediatr
enterocolitica in “acute appendicitis”. Z Gastroenterol 1982; Surg Int 1998; 13:2–5
20:78–83 4. York D, Smith A, Phillips JD, von Allmen D. The influence
2. Bennion RS, Thompson JE Jr, Gil J, Schmit PJ. The role of of advanced radiographic imaging on the treatment of pe-
Yersinia enterocolitica in appendicitis in the southwestern diatric appendicitis. J Pediatr Surg 2005; 40:1908–1911
United States. Am Surg 1991; 57:766–768
3 Abdomen 225

Q 108
Felix Schier

Fig. 1 Fig. 2

A newborn presented with vomiting immediately after


any feeding attempt. No mass or olive was palpated. The
shape of the abdomen looked normal and peristalsis was
not seen.
Laboratory values were also normal. Imaging studies
yielded the pictures shown in Figs. 1 and 2.
• What is the most likely diagnosis?
226 Abdomen

A 108

Figure 1 is a plain view radiograph. There is a mass at the This patient was a newborn. The diagnosis, however,
greater curvature (arrows). The gastric air is dislocated was not known prenatally, despite several sonographic
cranially and the transverse colon is pushed caudally. evaluations of the pregnancy. Antenatal ultrasound di-
Figure 2 is a sonogram of the antrum. The arrow- agnosis has become the rule for larger gastric duplica-
head points to the compressed antrum. Caudally to the tions.
compressed antrum, a large fluid-containing structure is Symptoms of children with gastric duplication cysts
identified with the characteristic layers of the gastroin- vary widely. There are asymptomatic duplication cysts.
testinal wall, typical for these kinds of lesions. To the left, Pain, bleeding, obstruction, palpable mass, failure to
the compressed bulbus duodeni is seen, also containing thrive, perforation, and recurrent pancreatitis have been
some fluid. described in children with gastric duplication, and some
The diagnosis is gastric duplication. have even eroded up into the chest.
Gastric duplications represent only 5% of all ali- Diagnostically, ultrasound and CT will establish the
mentary tract duplications. Most become symptomatic diagnosis, perhaps aided by an upper GI study.
before the age of 2 years. A duplication of the size pre- Surgical excision is the therapy of choice. Complete
sented here will become symptomatic from shear mass excision is usually feasible.
effect, other duplications may bleed or cause peptic ul- Laparoscopic resection of gastric duplication cysts
ceration, depending on location, size, presence of gastric has been described. Some cysts, however, are difficult to
mucosa, and whether there is a communication with the localize laparoscopically, even when aided by intraop-
lumen of the stomach (in the majority there is none). erative gastroscopy.

Suggested Reading
1. Carachi R, Azmy A. Foregut duplications. Pediatr Surg Int 4. Nakazawa N, Okazaki T, Miyano T. Prenatal detection of
2002; 18:371–374 isolated gastric duplication cyst. Pediatr Surg Int 2005;
2. Ford WD, Guelfand M, Lopez PJ, Furness ME. Laparo- 21:831–834
scopic excision of a gastric duplication cyst detected on an- 5. Rodriguez CR, Eire PF, Lopez GA, Alvarez EM, Sanchez
tenatal ultrasound scan. J Pediatr Surg 2004; 39:e8–e10 FM. Asymptomatic gastric duplication in a child: report
3. Master V, Woods RH, Morris LL, Freeman J. Gastric dupli- of a new case and review of the literature. Pediatr Surg Int
cation cyst causing gastric outlet obstruction. Pediatr Ra- 2005; 21:421–422
diol 2004; 34:574–576
3 Abdomen 227

Q 109
Felix Schier

A mature newborn had a distended abdomen, bilious


vomiting, and failed to pass meconium for 3 days. There
were no associated anomalies and all laboratory values
were normal.
An imaging study yielded the picture in Fig. 1.
• What is the most likely diagnosis?

Fig. 1
228 Abdomen

A 109

Figure 1 is a contrast study of the colon. A microcolon There is a risk to confuse meconium ileus radiologi-
extends over the whole colon length, containing numer- cally with Hirschsprung’s disease. It has been stated that
ous small meconium globules. The small bowel in the the presence of right lower quadrant intraluminal calcifi-
left upper abdomen is distended secondary to the distal cations should raise the suspicion of long-segment intesti-
obstruction. nal aganglionosis even if the operative findings are typical
The diagnosis is meconium ileus from cystic fibrosis of meconium ileus and a biopsy should be performed.
(as diagnosed subsequently). Later in life, a neonate with meconium ileus will not
As newborns, children with cystic fibrosis usually pres- have a poorer nutritional status or poorer lung function
ent with gastrointestinal or pancreaticobiliary symptoms. tests than a cystic fibrosis patient without meconium
Meconium ileus is the typical first manifestation. These ileus.
symptoms persist throughout childhood. Only later do Up to one-third of children with meconium ileus
pulmonary complications become more prominent. may be treated nonsurgically with hypertonic enemas.
Radiologically, the colon is of normal length but has Children with small-bowel atresias may constitute up to
a decreased caliber. If there is reflux into the terminal il- 20% of cases with meconium ileus.
eum, the site of obstruction will be identified. A barium Approximately 60% of children with meconium il-
enema also serves to differentiate an uncomplicated me- eus have uncomplicated disease. Uncomplicated cases
conium ileus from an intestinal atresia or volvulus. are those with simple obstruction of the terminal ileum.
Later in life, the combination of fat deposition and One-third has complications such as perforation, volvu-
pancreatic fibrosis leads to varying CT and MR appear- lus, atresia, or meconium peritonitis.
ances. A higher than normal incidence of pancreatic In this case, there is meconium obstruction without
cysts and calcification is also seen. Decreased transport cystic fibrosis.
of water and chloride increases the viscosity of bile, with If surgery is required, up to 60% of the children with
subsequent obstruction of the biliary ductules. If exten- uncomplicated disease will need bowel resection and en-
sive, this can progress to obstructive cirrhosis, portal hy- terostomy. Of the complicated cases, about half may be
pertension, and esophageal varices. Diffuse fatty infiltra- managed by bowel resection and anastomosis, the other
tion, hypersplenism, and gallstones are also commonly half of patients will require an enterostomy.
seen in these patients.

Suggested Reading
1. Chaudry G, Navarro OM, Levine DS, Oudjhane K. Abdom- 3. Escobar MA, Grosfeld JL, Burdick JJ, Powell RL, Jay CL,
inal manifestations of cystic fibrosis in children. Pediatr Wait AD, West KW, Billmire DF, Scherer LR 3rd, Engum
Radiol 2006; 36:233–240 SA, Rouse TM, Ladd AP, Rescorla FJ. Surgical consider-
2. Cowles RA, Berdon WE, Holt PD, Buonomo C, Stolar CJ. ations in cystic fibrosis: a 32-year evaluation of outcomes.
Neonatal intestinal obstruction simulating meconium ileus Surgery 2005; 138:560–571
in infants with long-segment intestinal aganglionosis: ra- 4. Munck A, Gerardin M, Alberti C, Eizenman C, Lebourgeois
diographic findings that prompt the need for rectal biopsy. M, Aigrain Y, Navarro J. Clinical outcome of cystic fibrosis
Pediatr Radiol 2006; 36:133–137 presenting with or without meconium ileus: a matched co-
hort study. J Pediatr Surg 2006; 41:1556–1560
5. Rescorla FJ, Grosfeld JL. Contemporary management of
meconium ileus. World J Surg 1993; 17:318–325
3 Abdomen 229

Q 110
Felix Schier

The radiograph is of a 6-month-old boy with coughing


after feedings. Asthma was suspected and treated earlier.
Quite often, the child choked and coughed during feed-
ings.
A physical examination was normal, and all labora-
tory values were normal. An imaging study yielded the
picture in Fig. 1.
• What is the most likely diagnosis?

Fig. 1
230 Abdomen

A 110

Figure 1 is an esophagogram, obtained by advancing a the fistula is high up in the neck, (b) the thoracoscopic
balloon catheter into the distal esophagus, followed by a approach if the fistula is within the thorax, and (c) the
second catheter ending more proximally. This avoids as- endoscopic approach using electrocautery and histoac-
piration during swallowing. Contrast medium is injected ryl glue or fibrin glue. Success with the third technique
into the proximal catheter, resulting in contrasting the is variable. It has been stated that a fistula that has not
esophagus and subsequently the fistula in a retrograde closed after two endoscopic attempts is not suitable for
manner from distally to cranially. further endoscopic treatment and therefore an external
The diagnosis is tracheoesophageal H-fistula approach should be recommended.
H-type tracheoesophageal fistulae are rare. Although Although cine-esophagography is described to be
most cases are diagnosed in the neonatal period because highly effective in demonstrating H-type tracheoesoph-
of choking and cyanosis during feeding, there are rather ageal fistulae, bronchoscopy should be used in every
frequent reports of older children and even adults with patient suspected of having a fistula, especially when
H-type fistulae. The reason is that the diagnosis is diffi- radiological methods fail. Bronchoscopy is helpful in
cult and elusive, with a high percentage of false-negative diagnosis, in evaluation of associated respiratory tract
findings. anomalies, and in treatment.
Several radiologic techniques have been described on Preoperative placement of a catheter in order to fa-
how to identify an H-type fistula. The technique demon- cilitate identification of the cyst has repeatedly been sug-
strated in Fig. 1 is simple and efficient. gested. Identification of the fistula, however, is not that
Closure of H-fistulae is attempted in three different difficult in typical cases.
approaches: (a) the conventional “open” approach if

Suggested Reading
1. Aziz GA, Schier F. Thoracoscopic ligation of a tracheo- 4. Garcia NM, Thompson JW, Shaul DB. Definitive localiza-
esophageal H-type fistula in a newborn. J Pediatr Surg tion of isolated tracheoesophageal fistula using bronchos-
2005; 40:35–36 copy and esophagoscopy for guide wire placement. J Pedi-
2. Blanco-Rodriguez G, Penchyna-Grub J, Trujillo-Ponce A, atr Surg 1998; 33:1645–1647
Nava-Ocampo AA. Preoperative catheterization of H-type 5. Karnak I, Senocak ME, Hicsonmez A, Buyukpamukcu N.
tracheoesophageal fistula to facilitate its localization and The diagnosis and treatment of H-type tracheoesophageal
surgical correction. Eur J Pediatr Surg 2006; 16:14–17 fistula. J Pediatr Surg 1997; 32:1670–1674
3. De Schutter I, Vermeulen F, De Wachter E, Ernst C, Malf- 6. Tzifa KT, Maxwell EL, Chait P, James AL, Forte V, Ein SH,
root A. Isolated tracheoesophageal fistula in a 10-year-old Friedburg J. Endoscopic treatment of congenital H-Type
girl. Eur J Pediatr 2006; 21:2006 and recurrent tracheoesophageal fistula with electrocautery
and histoacryl glue.Int J Pediatr Otorhinolaryngol 2006;
70:925–930
3 Abdomen 231

Q 111
Felix Schier

Fig. 1 Fig. 2 Fig. 3

A 7-year-old boy passed a moderate amount of fresh


blood perianally. The child was otherwise healthy and
had no abdominal pain. All laboratory values were
normal.
Imaging studies yield the pictures shown in Figs. 1–3.
• What is the most likely diagnosis?
232 Abdomen

A 111

Figure 1 is a sonogram of the lower abdomen. A circu- There seems to be a potential risk of developing carci-
lar mass of 2.3-cm diameter and of mean echogenicity is noma because of the presence of adenomatous changes
identified, interspersed with hypoechoic areas. in some juvenile polyps, warranting colonoscopic re-
As seen in the color-coded Doppler sonogram (Fig. 2), moval.
the structure has a central vessel, branching into several The patient presented here is a typical case. The mean
smaller vessels. presenting age of juvenile polyps is around 6 years, and
The double contrast study of the colon (Fig. 3) dem- boys are mostly affected.
onstrates a round mass in the descending part of the sig- It has been found that roughly 50% of the polyps are
moid (arrow). in the rectosigmoid, 15% in the descending colon, and
The diagnosis is juvenile polyp of the sigmoid. another 30% near the splenic flexure. Overall, proximal
Juvenile polyps are the most common cause of rectal polyps were seen in 37% of pancolonoscopies. The con-
bleeding in children above the age of 1 year. In this age, clusion of these observations was that although most ju-
they are the most common polypoid lesion of the colon. venile polyps are located in the left colon, a pancolonos-
They are usually relatively large. copy should be the initial procedure because: (a) 37% of
Clinically, the children present with mild, asymptom- pancolonoscopies revealed proximal polyps, (b) 32% of
atic perianal bleeding. Profuse bleeding, however, has polyps were located proximal to the splenic flexure, (c)
repeatedly been reported. Some children also have diar- persistence of symptoms from missed proximal polyp(s)
rhea, in others the polyp prolapses. necessitates a repeat study with attendant risks, and (d)
The origin and the natural history are unknown. Ear- there is a possibility of malignant transformation in an
lier it was believed that they were solitary. Colonoscopy unidentified juvenile polyp.
has demonstrated, however, that more than half of the
patients have additional polyps further up.

Suggested Reading
1. Gupta SK, Fitzgerald JF, Croffie JM, Chong SK, Pfefferkorn 3. Pratap A, Tiwari A, Sinha AK, Kumar A, Khaniya S, Agar-
MC, Davis MM, Faught PR. Experience with juvenile pol- wal RK, Shakya VC. Nonfamilial juvenile polyposis coli
yps in North American children: the need for pancolonos- manifesting as massive lower gastrointestinal hemorrhage:
copy. Am J Gastroenterol 2001; 96:1696–1697 report of two cases. Surg Today 2007; 37:46–49
2. Katz AL. Juvenile polyps and poyposis syndromes. In: 4. Ukarapol N, Singhavejakul J, Lertprasertsuk N, Wongsa-
Surgery of Infants and Children: Scientific principles and wasdi L. Juvenile polyp in Thai children-clinical and colo-
practice. Keith T. Oldham, Paul M. Colombani, and Robert noscopic presentation. World J Surg 31 2007; 395–398
P. Foglia (eds). Lippincott-Raven Publishers, Philadelphia,
p 1313–1314, 1997
3 Abdomen 233

Q 112
Felix Schier

Fig. 2 Fig. 3

A 5-week-old boy presented with projectile vomiting Imaging studies yielded the findings shown in Figs. 2
that had set in from one day to another. An “olive” could and 3.
not be palpated. • What is the most likely diagnosis?
Laboratory values showed hypochloremia and alka-
losis.
234 Abdomen

A 112

Figure 1 serves only for comparison. It is a normal, lon-


gitudinal sonogram of the pylorus and the bulbus duo-
deni.
To the extreme right the antrum is seen, entering into
the pyloric channel. All five layers of the intestinal wall
are easily distinguishable: mucosa, muscularis mucosae,
submucosa, and muscularis propria including the serosa.
This is a normal picture.
Figure 2 shows an abnormal sonogram. The plane is
almost identical to Fig. 1. The pyloric channel is mark-
edly elongated. The sphincter musculature is thickened,
hypoechoic, and less structured. The pyloric region is
enlarged to 14 mm. On the right, a transsection of the
stenotic and “multiluminal” channel is seen.
Figure 3 is a gastric contrast study and shows the
characteristic “string sign.” The pyloric channel is highly
stenosed and markedly elongated. The gastric antrum is
impressed by the hypertrophic pyloric musculature. The
diagnosis is pyloric hypertrophy. Fig. 1
Years ago, the diagnosis of pyloric hypertrophy was
established by palpating the enlarged muscle as an “olive.”
The advent of ultrasonography has replaced palpation.
However, surgeons have never trusted palpation of an As in a number of pediatric surgical diseases, the eti-
olive more than ultrasonography. It has been shown that ology of pyloric hypertrophy is still unknown. Numer-
introducing a guideline to require palpation by a surgeon ous theories are still being proposed. If some day the
first and ultrasonographic imaging second did not reduce etiology is identified, the role of surgery, namely, a de-
the total amount of ultrasonography. The final diagnosis structive intervention, will consequently be revisited.
is eventually based on ultrasonography exclusively. Today, the role of laparoscopic surgery in hypertro-
Radiologists have established criteria for the ultraso- phic pyloric stenosis is not yet well established. The con-
nographic diagnosis of hypertrophic pyloric stenosis: A ventional, open approach has passed the test of time and
pyloric canal length of 18–20 mm (normal 11 mm), a di- naturally has fewer complications. It has been stated that
ameter of 13–14 mm, and a muscle thickness of 4–5 mm it also has a higher efficacy. On the other hand, recovery
(normal <2 mm) have been stated to be diagnostic for time appears to be shorter following the laparoscopic
hypertrophic pyloric stenosis; 4.8 ± 0.6 mm, muscle approach.
length 2.1 ± 0.3 cm, and channel length 1.8 ± 0.3 cm were An indication that surgeons no longer feel comfort-
measured in this case. Thickness of the pyloric muscle is able with the conventional transverse upper abdomen
the most discriminating and accurate criterion for hy- incision is the fact that, in addition, a transumbilical ap-
pertrophic pyloric stenosis. proach has been introduced, for cosmetic reasons.
3 Abdomen 235

Q 113
Giuseppe Ascione

Fig. 1

A 7-month-old infant presented with vomiting, abdomi- dominal mass was palpable. An examining finger passed
nal pain, and lethargy. The parents reported an attack of into the anus revealed blood in the stools.
intermittent abdominal pain on the previous day. • What does the examination in Fig. 1 show?
On physical examination, the child was chubby and • What is the most likely diagnosis?
in good health. Because of abdominal distension, no ab- • What is the best treatment?
236 Abdomen

A 113

Figure 1 is an abdominal ultrasonography scan and it re-


veals the classic target sign.
The diagnosis is idiopathic intussusception (in older pa-
tients intussusception can be associated with others med-
ical conditions such as Henoch–Schönlein purpura, he-
mophilia, Peutz–Jeghers syndrome, cystic fibrosis etc.).
Ultrasonography and barium enema are the imag-
ing studies to be conducted for confirmation of the diag-
nosis.
The barium enema (Fig. 2) shows intussusception in
the ascending colon.
The treatment of intestinal intussusception is hydro-
static reduction or surgical reduction that can be per-
formed via an open or laparoscopic approach.
Hydrostatic reduction (air insufflation or barium or
water-soluble contrast) is conditioned by the patient’s
age, duration of symptoms, and presence of lead points.
The recommended pressure of air insufflation for
therapeutic enema should not exceed 120 cm of H2O.
Operative reduction (Fig. 3) is indicated if hydrostatic
reduction is unsuccessful or if perforation exists.

Suggested Reading
1. Di Fiore JW. Intussusception: Semin Pediatr Surg 1999 Fig. 2
Nov; 8 (4):214–220
2. Eklof OA, Johanson L, Lohr G. Childhood intussusception:
hydrostatic reducibility and incidence of leading points in
different age groups. Pediatr Radiol 1980 Nov; 10(2):83–86
3. Kirks DR. Air intussusception reduction: “the wind of
change” Pediatr Radiol 1995; 25(2)89–91

Fig. 3
3 Abdomen 237

Q 114
Giuseppe Ascione

Fig. 1

A 9-year-old boy had a history of fatigue, anorexia, ab- Laboratory values revealed an iron-deficiency ane-
dominal pain, diarrhea, and weight loss. mia, an erythrocyte sedimentation rate of 50 mm in 1 h,
Several types of medical therapies did not improve and a low concentration of serum albumin.
the symptoms. The boy presented to the emergency de- An upper gastrointestinal series is shown in Fig. 1.
partment after a severe episode of rectal bleeding and • What does Fig. 1 show?
abdominal distension. • What pathological condition is affecting this child?
The patient had a recent appendectomy scar; in the • How do you confirm the diagnosis in this case?
right lower quadrant there was a palpable abdominal • What medical or surgical option is better for resolv-
mass. ing this problem?
There was bilateral joint swelling at the knees. Exami-
nation of the anus was normal.
238 Abdomen

A 114

The barium meal shown in Fig. 1 demonstrates a “string


sign” leading to obstruction.
This child has a Crohn’s disease with terminal ileum
involvement.
Endoscopy (Fig. 2) with biopsies of the mucosa
(Fig. 3) confirm the diagnosis.
Figure 3 is a histologic specimen showing a granu-
loma.
Fibroendoscopy is quite accurate when ileocolic or
colonic Crohn’s disease is present.
Other noninvasive tests are sucralfate scan to detect
the presence of active disease and radio-labeled leuko-
cyte scan to determine progressive disease.
Remission can be achieved by dietary manipulations
(elemental or semielemental diet) or intestinal rest. Fig. 2
If symptoms are severe, the mainstay therapy is ste-
roids (5 mg. m2). In intractable disease with steroids, 5-
ASA preparations and immunosuppressants such as aza-
thioprine, methotrexate, and cyclosporin A have been
used.
Indications for surgery are:
a) In emergency, massive hemorrhage, perforation or
intestinal obstruction
b) Growth and sexual development failure
c) Fistulas or persistent inflammatory mass
d) Perianal lesions that may precede intestinal prob-
lems

Complete eradication is unnecessary because of the mul-


tifocality of the disease (risk of short bowel syndrome).
In all, 85% of children with Crohn’s disease will have had
surgery within 15 years after presentation of the disease. Fig. 3

Suggested Reading
1. Ruemmele FM, Roy CC. Nutrition as primary therapy in 3. Zholudef A, Zurakoswsky D. Serologic testing with ANCA,
pediatric Crohn disease: fact or fantasy? J Pediatr 2000 ASCA and anti-Ompc In children. Am J Gastroenterol
Mar; 136(3):285–91 2004 Nov 99(11):2235–41
2. Sawczenco A, Sandu BK. Presenting features of inflamma-
tory bowel disease. Arch Dis Child Nov 88(11):995–1000
3 Abdomen 239

Q 115
Craig T. Albanese

Fig. 1 Fig. 2 Fig. 3

Fig. 4 Fig. 5

A routine screening ultrasound at 22 weeks’ gestation distal microcolon (Fig. 3, arrow) can be seen. For com-
was obtained. An ultrasound of the fetal abdomen re- parison, Fig. 4 shows the spine (arrowhead), liver (aster-
vealed a dilated portion of bowel, most likely of the isk), normal fetal colon (arrows), bladder (asterisk), and
proximal colon (Fig. 1, calipers). fetal head (arrowhead).
• What is the differential diagnosis? The baby was uneventfully delivered at term. Polyhy-
dramnios was never present.
Prenatal MR imaging (Fig. 2) was performed, which also A postnatal contrast enema was performed (Fig. 5).
suggested an obstruction of the proximal colon (arrows). • What are the findings and what is your diagnosis?
The liver (asterisk), fetal head (arrowhead), and the more
240 Abdomen

A 115

There is a dilated, meconium-filled segment of bowel. It


appears to be the proximal colon, although the distal il-
eum cannot be ruled out.
The differential diagnosis is colon atresia, ileal atresia,
and meconium ileus.
The MR image shows meconium (bright signal) in
the dilated right colon with virtually no fluid/meconium
entering a small distal colon.
The postnatal contrast enema demonstrates a micro-
colon (arrows) and obstruction of the dilated proximal
colon (arrowheads; stomach, asterisk). This is most con-
sistent with a right colon atresia. A type IIIb atresia was
noted at operation. This accounts for less than 1% of all
intestinal atresias. Intestinal continuity was established
and the baby recovered uneventfully.

Suggested Reading
1. Blair GK, Jamieson DH. Colon atresia—type III. J Pediatr
Surg 2001 Mar; 36(3):530–1
2. Etensel B, Temir G, Karkiner A, Melek M, Edirne Y, Ka-
raca I, Mir E. Atresia of the colon. J Pediatr Surg 2005 Aug;
40(8):1258–68
3. Karnak I, Ciftci AO, Senocak ME, Tanyel FC, Buyuk-
pamukcu N. Colonic atresia: surgical management and
outcome. Pediatr Surg Int 2001 Nov; 17(8):631–5
3 Abdomen 241

Q 116
Francois Becmeur

Fig. 1 Fig. 2 Fig. 4

The patient in this case was born at full term weighing


3,550 g. The mother consulted the pediatric emergency
department when the infant was 13 days old because of
tears, weight loss, and frequent regurgitations. On ad-
mission, the infant weighed 3,200 g, he had a tempera-
ture of 37°C, and seemed well. The clinical examination
did not show any possible anomalies.
Plain radiographs of the abdomen and thorax were
acquired in two planes: anteroposterior and lateral
(Figs. 1, 2).
• Can you comment on the radiographs?
An esophagogastroduodenal transit study was requested,
which is shown in Figs. 3 and 4.
• What do you make of this study?
• What is your diagnosis and what are the possible
treatments?

Fig. 3
242 Abdomen

A 116

An air bubble in the base of the thorax is seen on the ra-


diographs, evoking the presence of part of the stomach
in the posterior mediastinum.
It is a large right hernia which does not allow enteral
feeding (Figs. 3, 4).
Postural treatment may be attempted, with the child
in vertical position and placement of a nasogastric
probe. The hernia can thus be reduced gradually while
the child grows in weight. In this case, as often occurs in
this situation, this medical treatment failed. We decided
to perform a laparoscopy to reduce the hernia, to close
the diaphragmatic opening, and to create an antireflux
gastroesophageal valve (Nissen–Rossetti type of fundo-
plication). The postoperative course was uneventful and
at 3 years the child was perfectly normal.

Suggested Reading
1. Fasching G, Huber A, Uray E, Sorantin E, Lindbichler F,
Mayr J. Gastroesophageal reflux and diaphragmatic motil-
ity after repair of congenital diaphragmatic hernia. Eur J
Pediatr Surg 2000 Dec; 10(6):360–4
2. Gorenstein A, Cohen AJ, Cordova Z, Witzling M, Krutman
B, Serour F. Hiatal hernia in pediatric gastroesophageal re-
flux. J Pediatr Gastroenterol Nutr 2001 Nov; 33(5):554–7
3. Mayr J, Sauer H, Huber A, Pilhatsch A, Ratschek M. Modi-
fied Toupet wrap for gastroesophageal reflux in childhood.
Eur J Pediatr Surg 1998 Apr; 8(2):75–80
4. Samujh R, Kumar D, Rao KL. Paraesophageal hernia in the
neonatal period: suspicion on chest X-ray. Indian Pediatr
2004 Feb; 41(2):189–91
3 Abdomen 243

Q 117
Ciro Esposito and Giovanni Esposito

An 8-year-old boy had undergone surgery at the age


of 2 years for subocclusive abdominal pains that were
thought to be due to a defect of fixation and rotation of
the midgut; the surgery was complicated by a postop-
erative occlusion due to adherence that required another
operation.
Since that time the child had not fed correctly with
scarce growth and he had recurrent episodes of upper
abdominal distension. At 7 years of age, vomiting ap-
peared. Therefore, the child was taken to hospital where
a plain abdominal radiograph was acquired, followed by
a contrast meal. After these studies, he was transferred
to our department for surgery. On physical examination
the child had a distended and tympanic abdomen, over-
all in the epigastrium and left hypochondrium. Before
surgery, other examinations were performed.
• What does the contrast meal show (Fig. 1)?
• Which other examinations should be performed?
• What is the diagnosis?
• What was the treatment?
• What was the follow-up?

Fig. 1
244 Abdomen

A 117

The abdominal radiograph shows the presence of a dou-


ble bubble, due to a dilated, gas-filled stomach and duo-
denum. The contrast meal (Fig. 1) reveals a large dilata-
tion of the first part of the duodenum.
Gastroduodenoscopy was performed revealing a ste-
nosis in the second part of the duodenum due to the dia-
phragm with a very small hole.
The delayed diagnosis is a diaphragmatic duodenal
stenosis.
The operation consisted in a duodenotomy and a
complete web excision, coupled with a tapering duode-
noplasty. Postoperative nasogastric decompression with
a tube is necessary. Postoperative imaging through the
nasogastric tube showed a regular gastroduodenal tran-
sit (Fig. 2).
The postoperative course was uneventful and hos-
pitalization lasted 10 days. At the 5-year follow-up, the
child was growing well without any intestinal problems.

Fig. 2

Suggested Reading
1. Diamond IR, Hayes-Jordan A, Chait P, Temple M, Kim PC. 3. McCollum MO, Jamieson DH, Webber EM. Annular
A novel treatment of congenital duodenal stenosis: image- pancreas and duodenal stenosis. J Pediatr Surg 2002 Dec;
guided treatment of congenital and acquired bowel stric- 37(12):1776–7
tures in children. J Laparoendosc Adv Surg Tech A 2006 4. van Rijn RR, van Lienden KP, Fortuna TL, D’Alessandro
Jun; 16(3):317–20 LC, Connolly B, Chait PG. Membranous duodenal stenosis:
2. Escobar MA, Ladd AP, Grosfeld JL, West KW, Rescorla FJ, initial experience with balloon dilatation in four children.
Scherer LR 3rd, Engum SA, Rouse TM, Billmire DF. Duo- Eur J Radiol 2006 Jul; 59(1):29–32
denal atresia and stenosis: long-term follow-up over 30
years. J Pediatr Surg 2004 Jun; 39(6):867–71
3 Abdomen 245

Q 118
Alessandro Settimi and Ciro Esposito

Fig. 1

Fig. 2

A 5-year-old girl, born at term after an uncomplicated In order to define the disease and to establish a thera-
pregnancy and delivery, had developed normally with- peutic plan, the child was sent to our surgical depart-
out any disease until the age of 5 months, when, during ment. On admission, the physical examination revealed
hospitalization for gastroenteritis with episodes of bili- no pathology except for a mild increase in the size of
ary vomiting, high levels of transaminases were found. the liver, which was palpated at 1 cm to the costal mar-
After discharge, the hypertransaminasemia persisted gin with a normal surface. Laboratory test results were
and the child was hospitalized again. A hepatic biopsy normal except for glutamyl oxaloacetic transaminase
was performed that showed mild inflammatory fibrosis (GOT) and glutamyl pyruvic transaminase (GPT) levels
of the liver (Fig. 1). The child was transferred to a gas- with values of 134 U/l and 394 U/l, respectively. Endo-
troenterology department where US, photoscintigraphy scopic retrograde cholangiopancreatography (ERCP)
with 99mTc-dimethyl-iminodiacetic acid scintigraphy was scheduled to define the exact diagnosis and to pro-
(HIDA), and intravenous cholangiography were per- vide a therapeutic indication (Fig. 2).
formed, which revealed nonvisualization of the gall- • What does Fig. 1 show?
bladder with a dilatation of the choledochus and a slow • What was the diagnosis?
elimination of the HIDA. • What was the therapeutic indication?
• What was the follow-up?
246 Abdomen

A 118

On the ERCP scan, the papilla of Vater was found in a


normal localization but it was difficult to cannulate.
Injection of radiopaque contrast medium into the
papilla demonstrated a duplication of the hepatic left
duct, absence of the gallbladder, and hypoplasia of the
distal choledochus with dilatation of its proximal por-
tion (Fig. 2).
The diagnosis is a complex congenital malformation
of the biliary tree, consisting in anomaly of the biliary
tract, agenesis of the gallbladder, and hypoplasia of the
distal choledochus.
The therapeutic indication is a biliodigestive diver-
sion, which the child’s parents refused.
Follow-up until the age of 7 years did not show any
aggravation of the disease. Subsequently, the child was
lost to follow-up.

Suggested Reading
1. Kabiri H, Domingo OH, Tzarnas CD. Agenesis of the gall-
bladder. Curr Surg 2006 Mar–Apr; 63(2):104–6
2. Peloponissios N, Gillet M, Cavin R, Halkic N. Agenesis of
the gallbladder: a dangerously misdiagnosed malformation.
World J Gastroenterol 2005 Oct 21; 11(39):6228–31
3. Wazz G, Branicki F, Chishti I, Taji H. Role of intraoperative
cholangiography in detecting rare bile duct anomalies. JSLS
2002 Oct–Dec; 6(4):393–5s
4. Okada T, Sasaki F, Honda S, Naitou S, Onodera Y, Todo S.
sefulness of axial planes of helical computed tomography
for diagnosis of pancreaticobiliary maljunction in early in-
fants with negative findings on magnetic resonance cholan-
giopancreatography. Pediatr Surg 2008 Mar;43(3):579-82.
5. Fitoz S, Erden A, Boruban S. Magnetic resonance cholan-
giopancreatography of biliary system abnormalities in chil-
dren. Clin Imaging. 2007 Mar-Apr;31(2):93-101.
6. Ishida M, Egawa S, Takahashi Y, Kohari M, Ohwada Y,
Unno M. Gallbladder agenesis with a stone in the cystic
duct bud. J Hepatobiliary Pancreat Surg. 2008;15(2):220-3.
3 Abdomen 247

Q 119
Giovanni Esposito and Ciro Esposito

Fig. 1

A 3-year-old child without any previous serious disease • What does Fig. 1 show?
came to our attention because of the incidental detection • Which other examinations were performed and what
of a thoracic opacity. On admission, physical examina- were their results?
tion of the child did not reveal any anomaly. After other • What was the diagnostic suspicion?
examinations, surgery was scheduled. • What was found during the intervention and what
was the treatment?
• What was the definitive diagnosis?
• What was the follow-up?
248 Abdomen

A 119

The radiograph of the thorax (Fig. 1) shows the presence


of an opacity the size of a small mandarin in the right
phrenopericardial angle.
CT was performed (Fig. 2) that demonstrated the
presence of a round mass occupying the left phrenoperi-
cardial angle. The diagnostic suspicion was a pericardial
cyst.
At operation, a Morgagni–Larrey hernia with hepatic
contents (Fig. 3) was found.
After incision of the hernial sac, the liver was replaced
in the abdomen, the sac was excised, and the margins of
the Morgagni–Larrey hole were sutured.
The postoperative course was uneventful and at
2 years’ follow-up the child is doing well. Fig. 2

Fig. 3

Suggested Reading
1. Laparoscopic treatment of Morgagni-Larrey hernia: techni- 4. Lima M, Lauro V, Domini M, Libri M, Bertozzi M, Pigna A,
cal details and report of a series. J Laparoendosc Adv Surg Domini R. Laparoscopic surgery of diaphragmatic diseases
Tech A 2005 Jun; 15(3):303–7 in children: our experience with five cases. Eur J Pediatr
2. Huttl TP, Meyer G, Geiger TK, Schildberg FW. Indications, Surg 2001 Dec; 11(6):377–81
techniques and results of laparoscopic surgery for dia- 5. Ridai M, Boubia S, Kafih M, Zerouali ON. Morgagni-Lar-
phragmatic diseases. Surg Tod 2002 Jul; 127(7):598–603 rey hernias treated by laparoscopy. Presse Med 2002 Sep 14;
3. Ipek T, Altinli E, Yuceyar S, Erturk S, Eyuboglu E, Akcal T. 31(29):1364–5
Laparoscopic repair of a Morgagni-Larrey hernia: report of
three cases. Surg Today 2002; 32(10):902–5
3 Abdomen 249

Q 120
Ciro Esposito and Giovanni Esposito

Fig. 1

A 10-year-old girl without any significant disease pre- • What did the US show?
sented with abdominal colic accompanied by bile- • Which other procedures were performed and what
stained vomiting. was their result?
On physical examination, a mobile, aching mass was • What was the diagnosis?
revealed in the upper region of her abdomen. • What was the treatment?
US was performed (Fig. 1), which confirmed the pres- • What was the follow-up?
ence of the mass. Other procedures were subsequently
required to make a precise diagnosis and establish the
treatment.
250 Abdomen

A 120

The US (Fig. 1) image shows a large cystic mass that is


surrounded by some dilated intestinal loops.
To define its nature, laparoscopy was performed that
revealed an intestinal lymphangioma.
Laparotomy was subsequently performed.
The diagnosis was confirmed.
The treatment consisted in the simple excision of the
lymphangioma (Fig. 3).
The postoperative course was uneventful and the
girl was discharged 7 days after the intervention. At the
1-year follow-up, she was doing well.

Suggested Reading
1. Menon P, Rao KL, Vaiphei K. Isolated enteric duplication
cysts. J Pediatr Surg 2004 Aug; 39(8):67–69
2. Morgan K, Ricketts RR. Lymphangioma of the falci- Fig. 2
form ligament—a case report. J Pediatr Surg 2004 Aug;
39(8):1276–9
3. Ng WT. Mesenteric lymphangioma infected with non-ty-
phoidal Salmonella: the first case. Pediatr Surg Int 2005
Jun; 21(6):504–5. Epub 2005 May 19
4. Ratan SK, Ratan KN, Kapoor S, Sehgal T. Giant chylolym-
phatic cyst of the jejunal mesentry in a child: report of a
case. Surg Today 2003; 33(2):120–2
5. Takeuchi K, Takaya Y, Maeda K, Maruo T. Peritonitis
caused by a ruptured, infected mesenteric cyst initially in-
terpreted as an ovarian cyst. A case report. J Reprod Med
2004 Jan; 49(1):65–7

Fig. 3
3 Abdomen 251

Q 121
Vincenzo Di Benedetto and Ciro Esposito

A neonate, born at 34° weeks’ gestation because of ma-


ternal gestosis, and delivered by cesarean section, at
birth presented with respiratory distress with intense
dyspnea and ingravescent cyanosis that required im-
mediate respiratory assistance with intubation and a
ventilator at high frequencies and low pressures. After
acquiring a standard thoraco-abdominal radiograph, an
intervention was scheduled. This was performed after
stabilization of the infant’s respiratory function and nor-
malization of laboratory values, which were consider-
ably altered, showing hypoxia and acidosis.
• What does Fig. 1 show?
• What was the diagnosis?
• When was the intervention performed?
• What was found during the intervention?
• What was the follow-up?

Fig. 1
252 Abdomen

A 121

The radiograph shows the presence of a large gaseous


area inferiorly and medially in the left hemithorax and
the presence of other translucent areas laterally, with a
complete shift of the mediastinum and dextrocardia.
The diagnosis was of congenital diaphragmatic her-
nia (Bochdalek hernia) with transposition of the stom-
ach and of some intestinal loops in the thorax.
Two days after birth, once stabilization of the respi-
ratory function was obtained and the oxygenation and
acid-base balance were normalized, the neonate was op-
erated on.
The intervention consisted in a subcostal left lapa-
rotomy that showed a large posterior diaphragmatic
defect through which the stomach and left colon were Fig. 2
migrated in the thorax (Fig. 2). After repositioning these
organs in the abdomen, the diaphragmatic defect was
closed with interrupted sutures, without leaving a drain
in the thorax according to the suggestions of Cloutier.
After a radiographic check-up of the chest 8 days after
operation, which was normal with a good expansion of
the lung, the neonate was discharged on the 10th postop-
erative day. After 1 year, the child was in good condition.

Suggested Reading
1. Crankson SJ, Al Jadaan SA, Namshan MA, Al-Rabeeah AA, 4. Luis AL, Avila LF, Encinas JL, Andres AM, Suarez O, Elorza
Oda O. The immediate and long-term outcomes of new- D, Rodriguez I, Martinez L, Murcia J, Lassaletta L, Tovar
borns with congenital diaphragmatic hernia. Pediatr Surg JA. Results of the treatment of congenital diaphragmatic
Int 2006 Apr; 22(4):335–40 hernia with conventional therapeutics modalities. Cir Pe-
2. Kamata S, Usui N, Kamiyama M, Tazuke Y, Nose K, Sawai diatr 2006 Jul; 19(3):167–72
T, Fukuzawa M. Long-term follow-up of patients with 5. Rygl M, Pycha K, Stranak Z, Melichar J, Krofta L, Toma-
high-risk congenital diaphragmatic hernia. J Pediatr Surg sek L, Snajdauf J. Congenital diaphragmatic hernia: onset
2005 Dec; 40(12):1833–8 of respiratory distress and size of the defect: Analysis of
3. Koumbourlis AC, Wung JT, Stolar CJ. Lung function in in- the outcome in 104 neonates. Pediatr Surg Int 2007 Jan;
fants after repair of congenital diaphragmatic hernia. J Pe- 23(1):27–31
diatr Surg 2006 Oct; 41(10):1716–21
3 Abdomen 253

Q 122
Francois Becmeur

Fig. 1 Fig. 2 Fig. 3 Fig. 4

A 6-month-old infant, born at 37 weeks and weighing However, the symptoms continued, and the child
3,090 g, presented with greenish vomiting, diarrhea, re- vomited all the time, although the amount of procinetic
fusal of bottle-feeding, no tears, and no painful crises, was increased. Moreover, abdominal pain appeared,
which had started when he was 2 months old. These epi- with increasingly long crises until the pain became con-
sodes appeared regularly. tinuous.
• What are the plausible diagnoses? The baby was taken to hospital again. On clinical ex-
amination, a flexible, distended, and swollen abdomen
The pediatrician diagnosed an allergy to cow’s milk pro- was found with no signs of defense.
teins, and thus performed the Diallertest® (patch test), • Which diagnoses do you eliminate and which do you
which was positive. A dietetic treatment was started, consider further?
substituting the usual milk by Peptijunior®. The greenish • How did the doctor in the emergency department pro-
vomiting continued and the doctor suggested comple- ceed?
mentary examinations. An abdominal ultrasound was • Which examinations were requested?
performed to search for an obstruction. The radiologist
was satisfied with the sonogram of the cardia and did A plain anteroposterior abdominal radiograph and an
not perform supplementary examinations or radiogra- ultrasound scan were acquired (Figs. 1, 2). Subsequently
phy of the abdomen, although the symptoms had been an esophagogastric-duodenal transit was carried out
persisting for 1 month. (Figs. 3, 4)
A diagnosis of gastroesophageal reflux (GER) was made. • What do you observe?
• Which treatment would you have proposed for the • Which treatment do you propose?
GER?
A few hours after treatment, abdominal meteorism and
The medical therapy was with anti-acid drugs and a pain appeared.
stimulant of intestinal motility. • What went wrong?
254 Abdomen

A 122

The GER is due to bad gastroduodenal voiding. Any bile-


stained vomiting should suggest an occlusion caused by
a vaterian obstruction (bulb of Vater: bile duct).
Volvulus around the base of the narrow midgut com-
mon mesentery can provoke recurrent episodes of sub-
acute intestinal obstruction.
Laparoscopic surgery is the right treatment. The last
radiograph showed a gastric probe not in place: it is at
the bottom of the esophagus. It is enough to descend it
in the stomach.

Suggested Reading
1. Boyle JT. Gastroesophageal reflux disease in 2006: The im-
perfect diagnosis. Pediatr Radiol 2006 Sep; 36(Supplement
14):192–195
2. Gold BD. Review article: epidemiology and management of
gastro-oesophageal reflux in children. Aliment Pharmacol
Ther 2004 Feb; 19 Suppl 1:22–7
3. Ida S. Evaluation and treatment of gastroesophageal re-
flux in infants and children. Nippon Rinsho 2004 Aug;
62(8):1553–8
4. Ostlie DJ, Holcomb GW 3rd. Laparoscopic fundoplication
and gastrostomy. Semin Pediatr Surg 2002 Nov; 11(4):196–
204
5. Suwandhi E, Ton MN, Schwarz SM. Gastroesophageal
reflux in infancy and childhood. Pediatr Ann 2006 Apr;
35(4):259–66
4
Genitourinary
Disorders

Case 123–151
256 Genitourinary Disorders

Introduction

Clinical history and physical examination represent an Urography on its own is rarely performed on chil-
important aspect in the management of infants and chil- dren, even if it is indicated in some cases of obstructive
dren with urological diseases. uropathies to obtain more anatomo-functional data.
However, the role of instrumental and in particular MR imaging and MR urography require anesthesia in
of radiological examinations is fundamental for making the pediatric age and they are mainly adopted to study
a certain and correct diagnosis in all clinical cases. renal masses or in cases of ureteral duplications with a
In the last 20 years, there has been a considerable nonfunctioning upper or lower renal pole.
improvement in the diagnosis of children with urologi- Urodynamic studies always complete the diagnos-
cal problems. The most important reason for this is the tic work-up in patients with neurological or functional
advent of ultrasonography as a prenatal and postnatal vesical dysfunctions.
screening tool. Obviously, the diagnostic work-up can change case
Nowadays, this examination allows a complete and per case and it is related to the clinical experience of the
accurate study of the upper urinary tract. pediatric urologist and is generally focused on obtaining
Obviously, a good knowledge of the most common a correct diagnosis, avoiding certain invasive diagnostic
congenital urinary malformations is essential for the examinations without benefit to the child.
correct interpretation of the ultrasonographic findings. The therapeutic procedures described in the clinical
Two other examinations that are frequently adopted cases presented in this section are: endoscopic, laparo-
to confirm the diagnosis are voiding cystography and scopic, video-assisted, and open surgical techniques.
isotopic renography. Frequently, these procedures are complementary, allow-
Ultrasound, voiding cystography, and renal scanning ing resolution of the majority of cases with very good re-
together allow one to study the majority of obstructive sults and guarantying young patients not only resolution
congenital uropathies in pediatric patients. of their disease but a good quality of life too.
4 Genitourinary Disorders 257

Q 123
Jean Stephane Valla

Fig. 1 Fig. 2

A 14-year-old boy presented with a 1-year history of in- • What do Figs. 1 and 2 show?
termittent right flank pain radiating from the loin to the • What pathological condition is affecting this adoles-
groin. He had also had two episodes of hematuria. The cent?
physical examination was normal. • Which other diagnostic examinations are necessary
Ultrasonography showed mild dilatation of the upper in this case?
urinary tract on the right side. Excretory urography was • How do you manage this condition?
performed.
258 Genitourinary Disorders

A 123

The IV urogram shows a right-sided hydronephrosis


and dilatation of the proximal ureter up to the level of
the transverse process of L3 (Fig. 1, frontal view; Fig. 2,
lateral view). In addition there is a medial deviation of
the ureter at this level, usually described as an “S” or
“fish hook” deformity at the point of obstruction.
This adolescent has a retrocaval ureter. Retrocaval
(circumcaval) ureter is an uncommon anomaly. It origi-
nates form a developmental error in the formation of the
vena cava and not of the ureter; this can cause varying
degrees of ureteral obstruction. Retrocaval ureter should
be suspected in any case of pyelectasis and ureterectasis
of the upper third ureter on the right side.
In order to reduce irradiation, the MAG-3 scan Fig. 3
(technetium-99m mercaptoacetyltriglycine) is likely to
replace IV urography, CT, and diuretic renography. This
examination allows one to confirm the diagnosis, to un-
derline other associated malformations, and to assess
the function of the right kidney.
Surgical repair is indicated only when symptoms de-
velop or functionally significant obstruction exists. In
our opinion, minimally invasive repair, by a trans- or
retroperitoneal approach, should be considered before
open surgery, provided the surgeon is comfortable with
advanced laparoscopic techniques. During the proce-
dure the patency of the ureter must be verified to deter-
mine whether it is necessary or not to excise the retroca-
val segment and to avoid any residual ureteral stenosis.
Figure 3 is an intraoperative uretero-pyelograph. In
Fig. 4, the intraoperative view is shown (retroperitoneo- Fig. 4
scopic approach) after dissection. Figure 5 shows the fi-
nal result after division of the pelvis, uncrossing of the
ureter, and ureteropelvic reanastomosis.

Suggested Reading
1. Simforoosh N et al. Laparoscopic pyelostomy for retrocaval
ureter without excision of the retrocaval segment: first re-
port of 6 cases. J Urol Vol 175 2006; p 2166–2169
2. Uthappa MC et al. Retrocaval ureter: MR appearances. Br J
Radiol 2002; Vol 75 p 177–179

Fig. 5
4 Genitourinary Disorders 259

Q 124
Jean Stephane Valla

Fig. 1 Fig. 2 Fig. 3

Fig. 4 Fig. 5

A 4-year-old girl presented with a recurrent upper uri- • How would you interpret the radiographic studies in
nary tract infection with fever up to 40°C, irritability, Figs. 1–5?
and left flank pain. The antenatal ultrasound examina- • Which other examinations are needed in this case?
tion of her kidney was normal. She had been hospitalized • On the basis of which arguments could you distin-
6 months earlier for 1 week because of an initial episode guish a primary vesicoureteral reflux (VUR), referred
of pyelonephritis due to Escherichia coli. At that time, to as a congenitally deficient ureterovesical junction,
ultrasonography (Figs. 1–3) and retrograde cystography from a secondary VUR due to bladder or sphincter
(Figs. 4, 5) were performed; the girl was discharged with disease?
continuous antibiotic prophylaxis. • How do you manage this case?
260 Genitourinary Disorders

A 124

Fig. 6 Fig. 7 Fig. 8

The first ultrasonography was judged as normal: normal A new retrograde cystogram revealed persistence of
echostructure and renal growth (Fig. 1, right kidney: the high-grade left VUR. Urodynamics testing yielded
71 mm; Fig. 2, left kidney: 75 mm) no visible ureter, nor- normal results. A radioisotope renal scan (dimercapto-
mal bladder (Fig. 3, capacity, thickness). The retrograde succinic acid, DMSA) confirmed the loss of left renal
cystogram confirmed a normal bladder and unilateral function (Fig. 7).
left high-grade reflux (Figs. 4, 5). Follow-up with antibi- All these arguments point to renal scarring due to
otic prophylaxis was appropriate at that time. primary VUR: unilateral VUR, normal bladder and
The present recurrent pyelonephritis needs rapid in- sphincter, and recent unilateral renal loss of growth and
travenous antibiotherapy for at least 10 days after collec- function.
tion of urine samples for bacteria identification (E. coli) Because of the patient’s age, the nature of the VUR,
and blood samples for infection and renal function eval- the recurrent urinary tract infection in spite of antibiotic
uation. A more complete investigation is then necessary. prophylaxis, medical management should be given up.
A careful history and physical examination revealed: Correction of the VUR by classical surgery or submu-
no voiding symptom, no incontinence, no constipation, cosal endoscopic injection was suggested to the parents,
normal neuro-urological test results. some weeks after the latest episode. In this case, ureteral
A new renal ultrasound found asymmetric renal reimplantation was performed using a minimally inva-
growth (left side 66 mm, right side 75 mm; Fig. 6) which sive technique (Fig. 8).
was not present 6 months earlier.

Suggested Reading
1. Demede D et al. Evidence-based medicine and vesico ure-
teral reflux. Ann Urol (Paris) Vol 43 2006; p 161–74
2. Greenbaum LA, Mesrobian HG. Vesico ureteral reflux. Pe-
diatr Chir North Ann Vol 53 2006; p 413–427
3. Steyaert H, Valla JS. Minimally invasive urologic surgery
in children: an overview of what can be done. Eur J Pediatr
Surg Vol 15 2005; p 307–13
4 Genitourinary Disorders 261

Q 125
Jean Stephane Valla

Fig. 1 Fig. 2

A 15-year-old boy presented with recurrent left flank


pain with hematuria, but without dysuria and no fever.
Urinary culture did not demonstrate urinary tract infec-
tion. After questioning, the mother told us that during
pregnancy a left renal pelvis dilatation was noted; this
dilatation was controlled after birth, and had regressed
at 1 year of age. No ultrasonographic follow-up was
made after 2 years of age.
• What are the first diagnostic hypotheses to explain
pain with hematuria in an adolescent?
• How do you interpret the renal sonography study
(Figs. 1–3)?
• What is the suspected pathological condition?
• How do you complete the diagnostic work-up?
• How do you manage this case?

Fig. 3
262 Genitourinary Disorders

A 125

Fig. 4 Fig. 5 Fig. 6

The two most frequent etiologies to explain flank pain each kidney (in this case, left kidney 17%, right kidney
with hematuria are lithiasis and obstructive uropathy, in 83%) and the degree of obstruction (in this case major
particular ureteropelvic junction obstruction (UPJO). obstruction on the left side). Angiography (Figs. 4, 5)
These two causes could be associated. could be useful in detecting crossing vessels (extrinsic
The ultrasonography shows: cause of UPJO).
a. Sagittal US (Fig. 1), pelvis and calices are largely di- Surgery is needed in this case; because the left renal
lated, renal parenchyma is thin. function was down to 15%, pyeloplasty must be preferred
b. Transversal US (Fig. 2), anteroposterior diameter of to nephrectomy and if possible by using a minimally in-
the pelvis is up to 40 mm. vasive approach (Fig. 6). Nevertheless, the patient and
c. Doppler US (Fig. 3), no lower pole crossing vessel is parents must be informed of a possible failure of this re-
clearly visible. constructive surgery (10% of cases) in such a distended
and poorly functioning kidney. This case demonstrates
The suspected pathological condition is pyeloureteral that it would have been better to make an earlier diagno-
junction obstruction because no lithiasis is visible and sis by performing annual ultrasound examinations and
the ureter is not distended. not to stop the surveillance after 1 year.
A radionuclide renal study using Tec-99 MAG 3 is
mandatory to calculate the relative renal function of

Suggested Reading
1. Balster S, Schiborr M, Brinkmann OA, Hertle L. Ob- 4. McMann LP, Kirsch AJ, Scherz HC, Smith EA, Jones RA,
structive uropathy in childhood. Aktuelle Urol 2005 Aug; Shehata BM, Kozielski R, Grattan-Smith JD. Magnetic res-
36(4):317–28 onance urography in the evaluation of prenatally diagnosed
2. Kaselas C, Papouis G, Grigoriadis G, Klokkaris A, Kaselas hydronephrosis and renal dysgenesis. J Urol 2006 Oct;
V. Pattern of renal Function Deterioration as a predictive 176(4 Pt 2):1786–92
factor of unilateral ureteropelvic junction obstruction 5. Sheu JC, Koh CC, Chang PY, Wang NL, Tsai JD, Tsai TC.
treatment. Eur Urol 2006 Jun 15 Ureteropelvic junction obstruction in children: 10 years’
3. McDaniel BB, Jones RA, Scherz H, Kirsch AJ, Little SB, experience in one institution. Pediatr Surg Int 2006 Jun;
Grattan-Smith JD. Dynamic contrast enhanced MR urog- 226:519–23
raphy in the evaluation of pediatric hydronephrosis: Part
2, anatomic and functional assessment of ureteropelvic
junction obstruction. AJR Am J Roentgenol 2005 Dec;
185(6):1608–14
4 Genitourinary Disorders 263

Q 126
Jean Stephane Valla

Fig. 1 Fig. 2

A full-term normal-weight baby was supervised before • How do you interpret Fig. 2?
birth because an ultrasound examination at 22 weeks • What is the suspected diagnosis?
of pregnancy had revealed a dilatation of the left pelvis. • How do you complete the diagnostic work-up?
This dilatation increased at 32 weeks, reaching 22 mm • How do you manage this case?
in diameter (Figs. 1). At 4 days of life, ultrasonography
confirmed the dilatation of the left upper urinary tract
(Fig. 2). A urine sample was sterile.
264 Genitourinary Disorders

A 126

Fig. 4

Fig. 3

Figure 2 shows a typical aspect of ureteropelvic junction


obstruction (UPJO): a central cystic structure with con-
necting peripheral cysts representing the dilated calyces.
The renal parenchyma is thin but its echogenicity looks
normal.
Hydronephrosis on a prenatal examination is today
the most common mode of presentation of UPJO.
Two examinations are mandatory before making
a therapeutic decision: first a retrograde cystogram
(Fig. 3), which shows a normal urethra, normal bladder,
and bilateral stage III vesicoureteral reflux with diluted
contrast liquid in the left pelvis. Second, a MAG-3 renal
scan (Fig. 4) allows one to measure the value of the left
kidney (34%), to confirm the obstruction.
Management can be debatable; in this case antibiotic Fig. 5
prophylaxis was given. A clinical, bacteriological, and
radiological follow-up was performed each month. At
3 months, ultrasonography and MR imaging demon- pelvis) was performed at 6 months (Fig. 5, on the right,
strated no improvement of the left pelvic dilatation; the dilated pelvis, on the left, normal ureter, in the middle,
left ureter was not visible. At 4 months, this infant was stenotic segment). Preoperative cystoscopy allowed as-
sent to hospital because of febrile urinary tract infection. sessment of the two ureteral orifices to be made (mild
An Anderson–Hynes dismembered pyeloplasty (exci- malformation) and catheterization of the left side to be
sion of the abnormal segment as well as redundant renal carried out.
4 Genitourinary Disorders 265

Q 127
Sabine Sarnacki

Fig. 1 Fig. 2

Fig. 3 Fig. 4

A 12.5-year-old girl presented with a 3-week history of Questioning of the patient and the clinical examination
abdominal pain. Clinical examination showed a healthy finally allowed the correct diagnosis to be made.
girl with patent signs of puberty (Tanner stage IV) and • How do you interpret the US images (Figs. 1–4)?
an abdominopelvic mass, which was not mobile. A pel- • How do you interpret the results of the laboratory
vic ultrasound examination was performed before the tests?
consultation (Figs. 1–4). • What are the main questions and the principal fea-
Laboratory tests were also performed on the basis of tures of the clinical examination that make the US
the first US examination, yielding the following results: examination useless, and in this case confusing?
– ACE: 8 ng/ml (normal <5 ng/ml) • What is the main diagnosis that leads to a very simple
– CA 19-9: 2,252 U/ml (normal <39 U/ml) operation and a complete relief of the symptoms?
– CA 125: 1,129 U/ml (normal <35 U/ml)
– β-HCG: <1 UI/l (normal <5 UI/L)
– Alpha-fetoprotein (AFP): 2 ng/ml (normal <15 ng/
ml)
266 Genitourinary Disorders

A 127

The discovery of an abdominopelvic mass in a girl of Questioning the patient and a complete clinical ex-
12–13 years with patent signs of puberty raises several amination in this case would have avoided a misinter-
diagnoses that could be explored through clinical, radio- pretation of the US and laboratory tests.
logical, and biological examinations. The presence of the Figures 1 and 2 show a hypoechogenic mass that was
symptoms for several weeks is not common in adnexal interpreted as an ovarian cyst. There is, however, a clear
torsion, but this diagnosis must be considered. The mo- line of sedimentation that could not be present in an
bility of the mass may clearly point to an ovarian nature, ovarian cyst. Figure 3 shows an adult-type uterus with
but the immobility cannot eliminate this hypothesis. In signs of hormonal impregnation, and Fig. 4 a normal
contrast, the fact that the mass is painful does not favor right ovary with multiple follicles.
the diagnosis of an ovarian tumor but rather the diagno- The markers for malignant germinal tumors are nega-
sis of an obstruction of the genital tract with retention of tive (alpha-fetoprotein and HCG). The elevation of ACE,
menstruation. CA 19-9, and CA 125 levels is nonspecific, due to the
In the presence of an abdominopelvic mass, labora- inflammatory context.
tory tests are required only when there is a suspicion The main question to ask is whether this girl with
of ovarian tumor to detect the presence of a malignant patent signs of puberty has already started having her
component (alpha-fetoprotein for the yolk sac tumor periods, and the sign to search for during the clinical
component and HCG for the choriocarcinoma compo- examination is the presence of a hymenal swelling.
nent), which may require preoperative chemotherapy. The diagnosis was hematocolpos due to imperforate
Measurements of CA 19-9, CA 125, and ACE levels are hymen.
useless in a pubertal girl, since adult-type ovarian carci-
noma is exceptional in this age group.

Suggested Reading
1. Nazir Z, Rizvi RM, Qureshi RN, Khan ZS, Khan Z. Con- 3. Stone SM, Alexander JL. Images in clinical medicine. Im-
genital vaginal obstructions: varied presentation and out- perforate hymen with hematocolpometra. N Engl J Med
come. Pediatr Surg Int 2006; 22:749–753 2004; 351:e6
2. Posner JC, Spandorfer PR. Early detection of imperforate 4. Wall EM, Stone B, Klein BL. Imperforate hymen: a not-so-
hymen prevents morbidity from delays in diagnosis. Pedi- hidden diagnosis. Am J Emerg Med 2003; 21:249–250
atrics 2005; 115:1008–1012
4 Genitourinary Disorders 267

Q 128
Sabine Sarnacki

Fig. 1 Fig. 2 Fig. 3

Fig. 4 Fig. 5

A 12-year-old girl presented with a 2–3-month history of An MR imaging study was requested by the surgeon.
coccygeal pain. Clinical examination showed a healthy • How do you interpret Figs. 2–5?
girl with patent signs of puberty (Tanner stage IV) and a • What are the important points to assess on these im-
normal aspect of the sacrococcygeal region. ages?
• Which important question should be asked? • How do you manage this situation?
A US examination was performed and showed normal
ovaries with peripheral follicles and a mass in the pelvis
(Fig. 1).
• What is the other region that is important to analyze
on this examination?
• What is the first diagnosis to be proposed?
268 Genitourinary Disorders

A 128

The most important question to ask this girl who had section where both ovaries with multiple follicles can be
reached puberty is about the date of onset of her pe- seen from one part of the fluid collection to another in
riod. The pain related to a hematocolpos typically has a the blind hemivagina. The ipsilateral hemiuterus is an-
coccygeal projection, and this diagnosis should be con- teverted and is not seen on this section. Figures 4 and 5
sidered in this pubertal girl. If periods had already oc- show sagittal sections of the pelvis, where the mucosal
curred, the notion of a two-step menstruation with two line of the open hemivagina reaching the perineum can
different colors (one red and one brown) is important be followed.
to consider and gives information about the anatomical The diagnosis of müllerian duplication could be
type of the potential malformation (incomplete septum, made on the basis of a simple US. This examination al-
see below). lows one to determine the uterine didelphys, the blind
The US examination shows a hypoechogenic mass hemivagina, and the location of the cervix. MR imaging
with a well-defined wall and a line of sedimentation is, however, interesting for the precise study of the level
within. The presence of two normal ovaries (Fig. 3) with of the vaginal partition. Retention of blood often gives an
signs of hormonal activity eliminates an ovarian origin. ovoid or round shape to the filled hemivagina, such that
The retroperitoneal region has to be analyzed for the the vaginal partition is not sagittal, but transversal, as in
presence of both kidneys. Müllerian duplication is usu- the present case. Axial and sagittal MR imaging repre-
ally associated with a renal agenesis on the side of the sentations give a more precise evaluation of the distance
interrupted genital tract. This malformation proceeds of the septum from the perineal level, which is helpful in
from mesonephric anomalies with absence of the wolff- anticipating the difficulties of the intervention.
ian duct opening to the urogenital sinus and of the ure- The degree of emergency for curing the malforma-
teral bud sprouting (and, therefore, renal agenesis). The tion depends on the symptoms. The intervaginal septum
‘inductor’ function of the wolffian duct on the müllerian may be seen as a ‘buttonhole’ on the anterolateral wall
duct is also failing and there is usually a uterovaginal of the normal vagina when the septum is incomplete
duplicity plus ipsilateral blind hemivagina with the re- but is difficult to locate when the septum is complete. A
nal agenesis. This malformation is thus associated with true collection swelling in the vagina is mandatory in
a large unilateral hematocolpos with a partial resorp- this case to determine precisely the site of incision of the
tion of the intervaginal septum. These types can be as- septum. If the girl is symptom-free, the distension of the
sociated with a vaginal ectopic ureter and interseptal or hemivagina can be followed with a US examination and
interuterine communication. Vaginal or complete cervi- the intervention planned when it is filled. In the present
covaginal unilateral agenesis, ipsilateral with the renal case, the intervention can be planned at once. Some au-
agenesis, with or without communication between both thors recommend preoperative hormonal treatment to
hemiuteri, is less common. block ovary functions, with the aim of preventing later
Figures 2–5 are fat-saturated T2-weigthed sequences onset of endometriosis. Resection of the vaginal septum
which demonstrate the fluid nature of the collection. is performed via the normal vagina. Preoperative cystos-
Figure 2 shows the hematocolpos in the blind hemiva- copy could attest to the expected trigonal agenesis, and
gina on a transversal section of the pelvis. A hematome- laparoscopy may provide additional information on the
tra is visible in the right hemiuterus. Figure 3 is an axial pelvic and urological anatomy beyond radiologic tests.

Suggested Reading
1. Acien P, Acien M, Sanchez-Ferrer M. Complex malforma- 4. Zurawin RK, Dietrich JE, Heard MJ, Edwards CL. Didel-
tions of the female genital tract. New types and revision of phic uterus and obstructed hemivagina with renal agenesis:
classification. Hum Reprod 2004; 19:2377–2384 case report and review of the literature. J Pediatr Adolesc
2. Acién P. Incidence of Müllerian defects in fertile and infer- Gynecol 2004; 17:137–141
tile women. Hum Reprod 1997; 12:1372–1376
3. The American Fertility Society. Classification of adnexal
adhesions, distal tubal occlusion, tubal occlusion secondary
to tubal ligation, tubal preganancies, müllerian anomalies
and intrauterine adhesions. Fertil Steril 1988; 49:944–955
4 Genitourinary Disorders 269

Q 129
Antonio Savanelli, Marianna De Marco, and Hana Dolezalova

Fig. 2

Fig. 1

A child who was affected by urinary tract infections


(UTIs) since birth had been diagnosed with right ure-
thral duplication and bilateral vesicourethral reflux,
which was treated in another hospital first with endo-
scopic injection and then with a right ureteral implanta-
tion using the Cohen technique.
Recurrent UTIs persisted after surgery and a serious
urinary incontinence appeared.
Because of the complaint of urinary incontinence, the
child was admitted to our hospital at the age of 5 years.
Firstly, ultrasound and voiding cystography studies were
performed, which are shown in Figs. 1 and 2.
• What does the renal ultrasound show (Fig. 1)?
• What does the second examination (Fig. 2) show?
Fig. 3
Following these studies, another examination was neces-
sary so as to examine the urinary continence.
• Which study was performed?
• What does the urodynamic study show (Fig. 3)?
• Which malformation is causing persistence of the
symptoms in this girl?
• Why did she develop urinary incontinence?
• What is the optimal treatment for this condition?
270 Genitourinary Disorders

A 129

The girl had a duplicated right kidney, bilateral vesico-


urethral reflux, and ipsilateral blind ectopic ureterocele,
misdiagnosed during the first operation. Cystoscopic
detection of ureteroceles can be quite variable and fre-
quently confused. A compressible ureterocele may come
to resemble only a minor mucosal fold with bladder fill-
ing; even when the ureterocele has poor detrusor support
and prolapses, at cystoscopy it may be misdiagnosed as
a bladder diverticulum. In the other hospital the child
also underwent intravenous pyelography that was nega-
tive for ureterocele. Most ectopic ureteroceles are associ-
ated with the upper pole of a duplex kidney that shows
minimal or no function, as in our case, and in these
cases the radiographic signs of ureterocele are primar- Fig. 4
ily negative. Ureteroceles are described as blind when no
kidney or upper pole associated with ureterocele can be
demonstrated. Despite thorough radiological investiga-
tion in all patients, a correct assessment of the anatomic (Fig. 3) shows a good detrusor contraction with urinary
defect can be achieved only by surgical exploration. If leakage at the end of filling.
preoperative radiological evaluation is equivocal, a high Persistent urinary incontinence was due to the flow
index of suspicion after cystography and intraoperative of urine from the bladder directly into the postsphin-
recognition of an unusual anatomic presentation of the teric urethra. It was attributed to failure to resect the
ectopic ureterocele are essential for appropriate manage- misdiagnosed ureterocele.
ment and a successful outcome. Finally, we performed cystoscopy, which revealed the
The first examination (Fig. 1) in this case is an ultra- real malformation. The urethral diverticulum is actually
sound and shows no pelvic dilatation and a normal renal the distal part of the ureterocele that comes out in the
structure. urethra. Ureterocelectomy and neck bladder reconstruc-
The second study (Fig. 2) is a cystogram showing a tion were performed (Fig. 4).
large and irregular bladder neck and a dilated urethra, In Fig. 4 it is possible to see the intravesical hole re-
with an image resembling a urethral diverticulum. sponsible for the urinary incontinence.
In order to evaluate urinary incontinence it is useful The child remains continent 1 year after the surgical
to perform a urodynamic study. In our case the study treatment.

Suggested Reading
1. Byun E, Merguerian PA. A meta-analysis of surgical prac- 4. Hoebeke P, De Kuyper P, Goeminne H, Van Laecke E, Ever-
tice patterns in the endoscopic management of uretero- aert K. Bladder neck closure for treating pediatric inconti-
celes. J Urol 2006; 176(4 Pt2):1871–7 nence. Eur Urol 2000 Oct; 38(4):453–65
2. Castagnetti M, Cimador M, Sergio M de Grazia E. Trans- 5. Shekarriz B, Upadhyay J, Fleming P, Gonzales R, Barthold
ureteral incision of duplex system ureteroceles in neonates: JS. Long-term outcome based on the initial surgical ap-
does increase the need for secondary surgery in intravesical proach to ureterocele. J Urol 1999; 162(3Pt2):1072–6
and ectopic cases? BJU Int 2004; 93(9):1313–7
3. Chertin B, Rabinowitz R, Pollack A, Koulikov D, Fridmans
A, Hadas-Halpern I, Farkas. Does prenatal diagnosis influ-
ence the morbidity associated with left in situ non-func-
tioning or poorly functioning renal moiety after endoscopic
puncture of ureterocele? J Urol 2005; 173(4):1349–52
4 Genitourinary Disorders 271

Q 130
Antonio Savanelli, Flavio Perricone, Gianfranco Vallone,
and Pier Francesco Rambaldi

Fig. 2

Fig. 1

A 10-year-girl with abdominal pain and pelvic and cali-


ceal dilatation diagnosed on ultrasound was admitted to
our unit.
During a previous hospitalization 4 years earlier, the
patient, after an episode of abdominal pain, underwent
ultrasonography. A caliceal and pelvic dilatation was
detected with an anteroposterior (AP) diameter of the
renal pelvis of 34 × 27 mm.
At the same time, the girl also underwent cystogra-
phy, which was normal, and Tc-99 MAG-3 dynamic re-
nal scintigraphy (Fig. 1).
• What does Fig. 1 show?
The girl underwent annual ultrasonographic follow-ups Fig. 3
that showed an unchanging condition.
The patient returned to hospital at the age of 10 years,
after another ultrasound study for a renal colic episode Another investigation was then performed.
(Fig. 2). • What is the diagnosis?
After analgesic treatment and when the symptoms • What is the preferred therapeutic approach?
disappeared, the girl underwent an ultrasound examina- • Which preoperative investigation would have al-
tion after 12 h (Fig. 3). lowed an etiologic diagnosis to be made?
• What does Fig. 3 show? • What is the follow-up?
272 Genitourinary Disorders

A 130

Figure 1 shows a renal scintigraphy study with normal improve the dilatation over time, and poor relative renal
relative renal function and adequate elimination after function.
furosemide (80%). It shows a urinary dilatation without The girl underwent surgery with Anderson–Hynes
obstruction. dismembered pyeloplasty.
The ultrasound image (Fig. 2) shows a caliceal and A dynamic MR image should permit a more spe-
pelvic dilatation with AP diameter of 66 mm and a thin- cific preoperative diagnosis to be made, by depicting
ning cortical parenchyma. the anomalous vessel; however, the etiologic diagnosis
The ultrasound after 12 h (Fig. 3) shows an improve- should not change the therapeutic planning.
ment with AP diameter of 20 mm. Follow-up entails 6-month ultrasound and scintigra-
The intravenous urogram (IVU), necessary to im- phy evaluations.
prove the diagnosis and to provide other anatomo-func- The MAG-3 scintigraphy (Fig. 4) provides evidence
tional details, shows a reduced and delayed elimination of worsening renal function despite a good excretion
of contrast media and hydronephrosis signs. Since the phase, and the ultrasound shows a small left kidney
marked improvements in the quality of ultrasound ex- without dilatation.
aminations, the indications to perform an IVU have The delay in the diagnosis and the treatment of hy-
been more restricted, but there is still a role for this mo- dronephrosis as an anomalous vessel probably caused
dality in some patients before surgical intervention. the renal damage. The presence of pelvic–caliceal sys-
The improved dilatation and pain lead to the diagno- tem dilatation associated with renal colic in childhood
sis of intermittent hydronephrosis, which in patients of should always suggest an anomalous vessel and indicate
this age is typically produced by a crossing renal vessel. the surgical treatment.
Indications for surgery in children with unilateral
hydronephrosis are the presence of symptoms, failure to

Fig. 4
4 Genitourinary Disorders 273

Q 131
Antonio Savanelli, Francesca Alicchio, and Luigi Mansi

Fig. 1

Fig. 2

Fig. 3

A newborn boy had an antenatal ultrasound diagnosis


of dilatation of the upper urinary tracts. Urinary tract
infections (UTIs) were not reported.
• Which examinations do you perform?
The examinations shown in Figs. 1 and 2 were per-
formed. Fig. 4
• What does Fig. 1 show? What does Fig. 2 show?
• Why are other examinations necessary?
• What does Fig. 3 show? • What is the follow-up?
• What is the diagnosis? • What does Fig. 4 show?
• What is the treatment of this malformation in the • What is the final treatment?
newborn?
274 Genitourinary Disorders

A 131

The newborn could have a congenital bilateral vesico- Children with high-grade reflux have a spontaneous
ureteral reflux (VUR) and he needs to undergo an ultra- resolution rate of about 30%.
sound examination and voiding cystourethrography. Indications for surgical treatment are recurrent symp-
The ultrasound (Fig. 1) allows the diagnosis of a left tomatic urinary infections, despite prophylactic antimi-
pyeloureteral duplication to be made. Figure 2 shows a crobials, failure to improve the grade of reflux over time,
cystography. You can see a huge left refluxing megaure- new scars and compliance problems. A nonfunctioning
ter corresponding to the inferior renal segment and a lower renal segment can be removed. At follow-up, ra-
moderate right VUR. The urethral profile is normal. dioisotope cystography (Fig. 4) shows persistent VUR
To complete the diagnostic approach, a Tc-99m after 1 year of prophylactic treatment.
DMSA static renal scan is necessary, so as to estimate The principles of management of VUR associated
the renal function and the presence of reflux nephropa- with complete ureteral duplication do not differ substan-
thy, or dynamic renography. The Tc-99m DMSA isoto- tially from those for management of reflux into a single
pic scan is the gold standard for evaluating the function ureteral system. The most common surgical treatment is
of the involved renal tissue, but the differential renal intravesical repair described by Cohen, where the ure-
function is more reliably assessed with Tc-99m MAG-3 ters are placed in a common transtrigonal submucosal
dynamic renography. tunnel in their common sheath without changing the
In Fig. 3 you can see a MAG-3 dynamic renogram ureteral hiatus. In order to obtain such a ratio, when the
showing a normal renal function and a good washout. ureter caliber is wider than 10 mm, the last 10 cm of the
The diagnosis is VUR of severe grade at the left side ureter needs to be tapered.
of the lower renal segment and of moderate grade at the Endoscopic subureteric injection of bulking agents is
right. another modern approach. Proposals for a randomized
Preventing urinary infections by using prophylactic clinical trial of immediate endoscopic treatment versus
antimicrobials is generally the first treatment step. conservative management are pending at this time.

Suggested Reading
1. Badwan KH, Diamond DA. Vesicoureteral reflux: diagnosis 4. Dillon MJ, Goonasekera CD. Reflux nephropathy. J Am Soc
and management. J Med Liban 2005 Apr–Jun; 53(2):61–5 Nephrol 1998 Dec; 9(12):2377–83
2. Bhide A, Sairam S, Farrugia MK, Boddy SA, Thilaganathan 5. Smellie JM, Barratt TM, Chantler C, Gordon I, Prescod
B. The sensitivity of antenatal ultrasound for predicting NP, Ransley PG, Woolf AS. Medical versus surgical treat-
renal tract surgery in early childhood. Ultrasound Obstet ment in children with severe bilateral vesicoureteric reflux
Gynecol 2005 May; 25(5):489–92 and bilateral nephropathy: a randomised trial. Lancet 2001
3. Cohen AL, Rivara FP, Davis R, Christakis DA. Compliance Apr 28; 357(9265):1329–33
with guidelines for the medical care of first urinary tract
infections in infants: a population-based study. Pediatrics
2005 Jun; 115(6):1474–8
4 Genitourinary Disorders 275

Q 132
Antonio Savanelli, Pier Francesco Rambaldi, Gianfranco Vallone,
and Barbara Greco

Fig. 1
Fig. 2

Fig. 3 Fig. 4

A newborn was affected by monolateral left hydrone- • What is the diagnosis?


phrosis identified on antenatal ultrasound (Fig. 1). • What is the therapeutic planning?
• What does the antenatal ultrasound show? • What do the examinations in Figs. 3 and 4 show (per-
• What other examinations do you perform in this formed when the boy was 3 and 6 months old, re-
case? spectively)?
• What does Fig. 2 show? • What is the follow-up?
276 Genitourinary Disorders

A 132

The antenatal ultrasound (Fig. 1) allows one to see a


thinner renal parenchyma and a monolateral severe dila-
tation of the calyx and pelvis without ureteral dilatation.
We performed a new ultrasound at 5 days of life con-
firming a severe dilatation with an AP diameter of the
pelvis of 34 mm. A MAG-3 renogram was planned after
the first month of life, maintaining prophylactic therapy
until the diagnosis of obstruction is excluded.
Figure 2 shows a renogram with a good renal relative
function and late washout.
The diagnosis is monolateral dilatation of the upper
urinary tract without obstruction.
In this case, there are no indications to surgical treat-
ment. An ultrasound and another MAG-3 renogram Fig. 5
were repeated after 3–6 months.
The ultrasound (Fig. 3) shows an improved dilatation
of the upper urinary tract with an AP diameter of the
pelvis of 19 mm and the MAG-3 (Fig. 4) shows an im-
proved washout too.
Follow-up studies using a MAG-3 renal scan and an
ultrasound should be performed at 12 and 24 months.
In this case, at follow-up the AP diameter of the pelvis
improved reaching 14 mm (Fig. 5), and the MAG-3 re-
nogram showed a good washout after furosemide ad-
ministration.

Suggested Reading
1. Bhide A, Sairam S, Farrugia MK, Boddy SA, Thilaganathan 2. Chandrasekharam VV, Shah MA. Outcome of patients with
B. The sensitivity of antenatal ultrasound for predicting antenatally detected pelviureteric junction obstruction. Pe-
renal tract surgery in early childhood. Ultrasound Obstet diatr Nephrol 2005; 20:547
Gynecol 2005 May; 25(5):489–92 3. Riccabona M. Assessment and management of newborn
hydronephrosis. World J Urol 2004; 22:73
4 Genitourinary Disorders 277

Q 133
Antonio Savanelli, Gianfranco Vallone, Barbara Greco, and Luigi Mansi

Fig. 1a Fig. 1b

A 9-month-old child with an antenatal diagnosis of left


hydronephrosis was admitted to our unit. The patient
had a history of a urinary tract infection (UTI) occur-
ring postnatally. In order to exclude vesicoureteral reflux
(VUR), voiding cystourethrography was performed. The
cystography was normal. Antibiotic prophylaxis for the
UTI was begun soon after cystography. Two months
later renal ultrasonography (Fig. 1a, b) was performed.
• What does Fig. 1 show?
After admission in our unit, the diagnostic examinations
were completed with excretory urography and a MAG-3
renal scan (Fig. 2).
• What do you see in Fig. 2? Fig. 2
• What is the diagnosis?
• What is the therapeutic approach?
• What is the follow-up for this malformation?
278 Genitourinary Disorders

A 133

Fig. 3
Fig. 4

UTIs are the most common cause of fever due to bacte- UTIs. Surgical treatment is indicated in the event of de-
rial infection in the neonatal age. The causes of UTI can teriorating renal function with permanent obstruction.
be VUR, obstructive uropathies like posterior urethral These patients are followed up with ultrasonography
valves (PUV), megaureter etc. and nuclear renography until stable improvement or
Figure 1 (a, b) shows severe hydronephrosis of the complete resolution of hydroureteronephrosis is noted.
left kidney with renal parenchymal thickness. An excretory urogram should be limited to cases in
An intravenous urography study showed delayed con- which anatomical details are required before surgical
trast excretion with dilution of material contrast within treatment. In our case the last echogram repeated after
the calices and pelvis, with a late image of hydrouretero- 2 years showed remarkable improvement (Fig. 3). The
nephrosis. A diuretic radionuclide urography with MAG- MAG-3 scan that showed a conserved parenchymal
3 (Fig. 2) confirmed the delayed elimination of the tracer function remained unmodified in spite of the persis-
without obstruction and a normal renal function. tence of incomplete washout (Fig. 4).
The diagnosis was of a partially obstructive megaure-
ter. Comment: The diagnosis and the treatment of a not re-
Spontaneous resolution or improvement of many fluxing obstructive megaureter still represent one of the
cases of urinary tract dilatation is becoming more com- most challenging dilemmas in pediatric urology today.
mon. Many asymptomatic primary megaureters in in- The indication to treatment is the presence of clinical
fancy improve spontaneously and do not require surgi- symptoms, decrease of renal function, and dilatation.
cal treatment. When the upper urinary tract is dilated, The aims of diagnosis, treatment, and long-term follow-
patients may need antibiotic prophylaxis, because the up are the preservation of renal function and the pre-
dilatation of the upper urinary tract increases the risk of vention of UTIs.
4 Genitourinary Disorders 279

Q 134
Alfonso Papparella, Mercedes Romano, and Pio Parmeggiani

Fig. 2

Fig. 1

Fig. 4

Fig. 3

An 11-year-old patient presented with a soft mass in the An echo color Doppler of the spermatic vessels was re-
upper part of the left scrotal region. The lesion was pain- quested.
less and was associated with ipsilateral testicular hypot- • What does the test show (Figs. 2, 3)?
rophy. • What does Fig. 4 show?
• What is the most likely diagnosis? (See Fig. 1) • What are the most likely grade and classification of
• What should the clinical examination evaluate? this pathological condition?
• Would you suggest further diagnostic investigations?
• What is the surgical treatment for this condition?
280 Genitourinary Disorders

A 134

The diagnosis is varicocele. Palpation should evaluate


the size of the testis, the condition of the pampiniform
plexus and the spermatic cord, in addition to the pa-
tency of the vaginal peritoneal duct. Palpation highlights
a dilation of the pampiniform plexus that is evident even
with the patient in the standing position; it resembles
the typical “bag of worms” and is associated with ipsi-
lateral testicular hypotrophy. In the clinostatic position,
the varicose swelling decreases, and it increases after the
Valsalva maneuver. Examination of the contralateral re-
gion is normal.
The power color Doppler shows a 2.8-s reflux on
the left side after the Valsalva maneuver (normal range:
0.8 s). Fig. 5
The comparative testicular ultrasound shows a re-
duced size in the left testis compared with the right testis
(Fig. 4)
The classification is grade II varicocele with type I re-
flux (renospermatic) and testicular hypotrophy.
Instrumental tests to be performed include: abdomi-
nal–scrotal ultrasound (symptomatic, bilateral varico-
cele), scrotal echo color Doppler, retrograde phlebogra-
phy of the internal spermatic vein that could be useful
in recurrences, and CT scan in the case of obstructive
varicocele.
The therapeutic approach in this patient can be liga-
tion of the spermatic vessels, either using the retroperi-
toneal approach according to Palomo with ligation of
the spermatic artery or the inguinal approach according
to Ivanissevich (ligation of the spermatic, cremasteric, Fig. 6
and deferential veins); the subinguinal microsurgical ap-
proach (ligation of the spermatic and cremasteric veins,
external spermatic vein and gubernaculums); derivative clipped (Figs. 5, 6), and then sectioned (Palomo proce-
microsurgical anastomosis;or sclerotizing treatment. dure). A retroperitoneoscopic technique could also be
More recent approaches use transperitoneal videosur- applied for varicocele.
gery, where the spermatic vein and artery are isolated Complications include hydrocele, testicular atrophy,
several centimeters away from the internal inguinal ring, recurrence, and persistence.

Suggested Reading
1. Esposito C, Monguzzi GL, Gonzalez-Sabin MA, Rubino R, 2. Esposito C, Monguzzi G, Gonzalez-Sabin MA, Rubino R,
Montinaro L, Papparella A, Amici G. Laparoscopic treat- Montinaro L, Papparella A et al. Results and complications
ment of pediatric varicocele: a multicenter study of the of laparoscopic surgery for pediatric varicocele. J Pediatr
Italian society of video surgery in infancy J Urol 2000; Surg 2001; 36(5):767–9
163(6):1944–1946 3. Koyle MA et al. Laparoscopic Palomo varicocele ligation in
children and adolescents: results of 103 cases. J Urol 2004;
172:1749–52
4 Genitourinary Disorders 281

Q 135
Alfonso Papparella, Mercedes Romano, and Pio Parmeggiani

Fig. 1 Fig. 2 Fig. 3

A 1-month-old girl was hospitalized following the pre- • What does Fig. 1 (micturitional cystourethrography)
natal diagnosis of an abdominal mass. show?
The general objective examination was normal; • What do Figs. 2 and 3 show?
laboratory test results (alpha-fetoprotein and CA 125) • What additional diagnostic tests would you suggest?
were within normal ranges. Results from the urinaly- • What is the most likely diagnosis?
sis showed small amounts of blood with leukocyturia, • Which is the most appropriate surgical management
whereas the urine culture was positive for Escherichia in this case?
coli (105). A micturitional cystourethrography and a • What other conditions can we suspect?
complete abdominal ultrasound were performed.
282 Genitourinary Disorders

A 135

The diagnosis in this girl is self-amputated ovarian cysts.


Ovarian cysts are generally rare in the pediatric age.
They represent a heterogeneous group of conditions that
range from functional (nonneoplastic) ovarian cysts, to
ovarian torsion, benign tumors, or even malignant and
extremely aggressive neoplasm.
The micturitional cystourethrography shows a nor-
mal-sized and normally localized bladder, with a lateral
deflection of vesical imaging probably due to mass com-
pression (Fig. 1). The course and caliber of the urethral
duct are regular and there is no vesicoureteral reflux.
The abdominal ultrasound shows a roundish mass
with linear and exogenous margins, located in an an- Fig. 4
teromedian position compared to both the lower pole of
the right kidney and the subhepatic region; the mass is
about 4 × 3 × 3 cm, and has a mixed solid–liquid echo-
structure (Figs. 2, 3).
CT and MR imaging should be performed to better
understand the origin and the localization of the cyst;
as a matter of fact, the former confirmed the presence
of the lesion in the subhepatic regions, whereas the lat-
ter located it in the right iliac fossa, and ruled out the
involvement of other organs or apparatuses.
In such cases, the value of a diagnostic laparoscopy is
beyond doubt, as it is able to evaluate the anatomical re-
lationships of the mass, verifying its nature and the organ
of origin through video-guided biopsies. Moreover, it
can also be surgical or can help choose the most suitable Fig. 5
surgical strategy. Laparoscopy is essential for the man-
agement of pediatric pathologies involving the annexes.
In this specific case, laparoscopy was able to detect
a 4-cm cystic formation in the subhepatic region. The
mass was mobile, well delimited by the surrounding or- Histological examination indicated a self-amputated
gans, filled with blood, and attached by a thin long vessel ovarian cyst, morphologically devoid of malignant fea-
(Figs. 4, 5). tures. A self-amputated ovarian cyst results from a pre-
Although the diagnostic test results were normal, to natal or neonatal torsion, followed by necrosis and cal-
completely rule out a neoplastic formation the mass was cification.
totally removed through a right pararectal laparotomic The differential diagnosis of ovarian cysts includes
incision, to avoid disseminating the fluid within the ab- lymphangiomas and intestinal duplications, hepatic tu-
dominal cavity in case of cyst rupture. mors, hypersplenism, neuroblastoma, and renal masses.
4 Genitourinary Disorders 283

Q 136
Antonio Marte, Maria Domenica Sabatino, and Pio Parmeggiani

Fig. 1 Fig. 2

A 14-year-old boy presented with recurrent lower ab- • What does Fig. 1 show?
dominal pain. His mother told the doctor that her child • What does Fig. 2 show?
had been diagnosed with a kidney anomaly at birth. • What pathological condition is affecting this child?
When he was 1 year old he underwent the examinations • What is the way to manage this condition?
shown in Figs. 1 and 2.
284 Genitourinary Disorders

A 136

This patient has an ectopic right pelvic kidney, an abnor-


mality of renal migration with an incidence of 1 in 5,000.
The ectopic kidney can be asymptomatic or associated
with vesicoureteral reflux (VUR) 26%, ureteropelvic
junction (UPJ) obstruction 37%, and functional uretero-
vesical junction (UVJ) 15%.
Figure 1 shows the ectopic pelvic kidney (note the prox-
imity of the bladder) with a mild dilation of the pelvis.
The DTPA nuclear scintigram (Fig. 2) shows a bilat-
eral good function of both kidneys with a mild delay
of the tracer washout on the right side—split function:
right kidney, glomerular filtration rate (GFR) 32.7 ml/
m’, relative function 41.7%; left kidney, GFR 37.7 ml/m2,
relative function 58.3%. Fig. 3
On admission we planned MAG-3 nuclear scintig-
raphy with indirect voiding cystourethrography (VCG)
and MR urography.
Another important aspect the surgeon must consider
is that the ectopic kidney is quite often associated with
other malformations such as skeletal, cardiovascular,
pulmonary, and genital system malformations. In girls,
Rokitansky-Mayer-Kuster-Hauser syndrome is frequent.
In this case, echocardiography showed a mild mitral
insufficiency.
The URO-MR study (Figs. 3, 4) confirms the position
of the kidney, the iliac pelvic blood supply, the abnormal
pelvicaliceal rotation, and the dilation.
The anterior view of the MAG-3 nuclear scintig-
raphy shows a reduction of right renal function (split
right function (%), 39.1%) and confirms the obstruction Fig. 4
(Fig. 5).
The nuclear VCG shows no VUR.
Because of the reduction of the right renal function
(<40%) and the abdominal symptoms, the patient un-
derwent laparoscopic-assisted pyeloplasty according to
Anderson-Hynes.
After the 1-year follow-up, the child was well and free
of symptoms.

Suggested Reading
1. Allen D, Bultitude MF, Nunan T, Glass JM. Misinterpreta-
tion of radioisotope imaging in pelvic kidneys. Int J Clin
Fig. 5
Pract Suppl 2005; 147:111–2
4 Genitourinary Disorders 285

Q 137
Antonio Marte, Maria Domenica Sabatino, and Pio Parmeggiani

Fig. 1

Fig. 2

A 5-year-old boy presented with an 8-month history of a • What do Figs. 1 and 2 show?
febrile Escherichia coli urinary tract infection (UTI) with • Is there a need for other examinations?
abdominal pain. The patient was referred to us by his • What pathological condition is affecting this child?
pediatrician for other diagnostic investigations. • What is the best way to manage this condition?
The patient was on antibiotic prophylaxis with co-tri-
moxazole and intermittently took spasmolytic drugs.
The mother showed us the examinations performed
during a previous hospitalization (Figs. 1, 2).
286 Genitourinary Disorders

A 137

This child has a horseshoe kidney. Figure 1 shows an


ultrasound scan demonstrating the abnormality of posi-
tion as well as the connecting isthmus.
Figure 2 shows a typical appearance of a horseshoe
kidney on an excretory urogram with a mild hydrone-
phrosis of the left segment. The renal pelvis remains
anterior with the ureter crossing the isthmus. The renal
axis appears to be vertical with the lower poles lying
closer than the upper pole. A frequent urographic find-
ing is a low-lying kidney and the lower outer border of
the kidney appears to continue across the midline (ar-
row), in this case the right kidney.
Associated urological anomalies (52%) are frequently
identified in patients with horseshoe kidney, includ- Fig. 3
ing primary vesicoureteral reflux (VUR), ureteropelvic
junction obstruction, and ectopic ureter.
Figure 3 is a MAG-3 renal scan that shows the two
kidneys fused through their lower poles. The left kidney Suggested Reading
and the isthmus show minimal function, elimination of 1. Cascio S, Sweeney B, Granata C, Piaggio G, Jasonni V,
tracer after IV furosemide; and after micturition it does Puri P. Vesicoureteral reflux and ureteropelvic junction in
not show elimination of tracer material from the pelvis. children with horseshoe kidney. Treatment and outcome. J
Split function percentage is: effective renal plasma flow Urol 2002; 167:2566–8
(ERPF) left kidney 8.3%; ERPF right kidney 91.7%. The 2. Pitts WR jr, Muecke EC. Horseshoe kidneys: a 40 years ex-
right kidney has a normal function. perience. J Urol 1975; 113:743–6
Most horseshoe kidneys are asymptomatic through- 3. Yoannes P, Smith AD. the endourological management
out the patient’s life. However, segmental hydronephro- of complications associated with horseshoe kidney.
J Urol 2002; 168:5–8
sis, reflux, lithiasis, or malignancy may lead to surgical
intervention because of UTI, abdominal pain, or hema-
turia. Because of the recurrent UTI, the abdominal pain,
and the poor function of the left kidney, the patient un-
derwent left segment nephrectomy.
The intraoperative view shows a large parenchyma-
tous renal isthmus that made the dissection difficult.
After a 10-month follow-up, the patient was well and
free of symptoms.
4 Genitourinary Disorders 287

Q 138
Antonio Marte, Maria Domenica Sabatino, and Pio Parmeggiani

Fig. 2

Fig. 1

A 3-month-old boy with a history of a febrile urinary


tract Escherichia coli infection underwent urologic eval-
uation. The boy was on antibiotic prophylaxis.
The boy had been operated on at birth for myelome-
ningocele (MMC) prenatally detected with US. He did
not need a ventricular shunt. The physical examination
and blood test results were all normal. The child seemed
to be leaking urine constantly. The motility of the lower
limbs was intact. We performed the examinations shown
in Figs. 1–3.
• What does Fig. 1 show?
• What does Fig. 2 show?
• What does Fig. 3 show?
• How should we manage this case?
Fig. 3
288 Genitourinary Disorders

A 138

The patient has a neuropathic bladder without urinary


retention.
Figure 1 is a US study of the bladder showing a thick-
ened bladder. The renal US is normal.
Figure 2 is a voiding cystourethrography (VCG) that
shows a typical neuropathic bladder with trabecula-
tion and anomalies of the bladder profile. There is a left
grade-1 (according to international classification: grade
1–5) vesicoureteral reflux (VUR).
The urodynamics shows a hyperactive bladder with
high pressures and detrusor-sphincter dyssynergia. The
leak point pressure (LPP) is 50 cm of water and the leak
point volume (LPV) 30 cc. Pressures over 40 cm H2O
pose a risk for the upper urinary tract.
The patient was treated with oral oxybutynin, CIC,
and antibiotic prophylaxis.
Despite the treatment, he developed a bilateral VUR
at the age of 3 years (Fig. 4). The MAG-3 nuclear scin-
tigraphy showed a relative function of 55.5% on the left
and of 44.5% on the right side. There was no evidence of
obstruction or scars.
The patient continued with antibiotic prophylaxis Fig. 4
and CIC and when he was 6 years old underwent endo-
scopic correction of the VUR with a subureteral Deflux
injection.
In order to increase the patient’s bladder capacity, at The stool incontinence is treated with a continent en-
the age of 11 years he underwent intradetrusor botulin ema device two times a week and occasionally the boy
A toxin injection of 200 UI. The injection was repeated takes loperamide.
8 months later. At the time of writing, the patient is In some cases, neuropathic bladder secondary to
14 years old, infection-free, and dry between the CIC MMC has an unfavorable evolution which requires blad-
without oxybutynin; moreover, his bladder capacity der augmentation and/or an increase of the bladder out-
has increased from 180 cc to about 400 cc. A follow-up let resistance in order to prevent urinary tract infections,
VCUG was normal, and the urodynamics study yielded intractable VUR, and continuous incontinence.
the following results: LPP 54 cm H2O, LPV 411 c.

Suggested Reading
1. Bauer SB. Evaluation and management of the newborn with 2. Marte, A Vessella, P Cautiero, M Romano, M Borrelli, C
myelomeningocele. In Gonzales ET, Roth D (eds) Common Noviello, R Del Gado, P Parmeggiani. Efficacy of toxin-A
Problems in Pediatric Urology. Mosby-Year Book Inc 1991; Botulinum for treating intractable bladder hyperactivity
pp 169–80 in children affected by neuropathic bladder secondary to
myelomeningocele: an alternative to enterocystoplasty. Mi-
nerva Pediatrica 2005; 57(1):35–40
4 Genitourinary Disorders 289

Q 139
Antonio Marte, Maria Domenica Sabatino, and Pio Parmeggiani

Fig. 1

Fig. 2

We were asked to see a full-term newborn female. Dur- A US study was performed (Fig. 2).
ing the mother’s pregnancy, a prenatal ultrasound per- • What does Fig. 1 show?
formed at 32 weeks showed a cystic mass in the retro- • What does Fig. 2 show?
peritoneum on the left side (Fig. 1). • What pathological condition is affecting this baby?
The baby appeared to be healthy. Physical examina- • Does she need to undergo other examinations?
tion confirmed a left upper quadrant mass which was • What is the best way to manage this condition?
hard and with an irregular surface. Otherwise, the child
was normal.
290 Genitourinary Disorders

A 139

Fig. 3

Fig. 4

This girl has a left multicystic dysplastic kidney (MCDK) tracer in the left retroperitoneum. Only the right well-
presenting with an abdominal mass on the upper left functioning kidney is seen (Fig. 3).
quadrant (Fig. 1). VCUG can detect associated vesicoureteral reflux.
The lesion feels hard and has a knobbly surface, while The management of this case was conservative with
the hydronephrotic kidney has a smooth surface (other periodic ultrasonographic checkups every 6 months un-
postnatal presentations are: flank pain, urinary tract in- til the patient was 3 years of age, and yearly until she was
fection, or hypertension). 6 years old.
The postnatal US examination (Fig. 2) shows: ran- MCDK might persist without any change, increase in
domly arranged and varied-sized cysts, the presence of size, or undergo spontaneous involution. Most cases of
interfaces between the cysts, a nonmedial location of the unilateral MCDK undergo spontaneous involution.
largest cyst, a lack of an identifiable renal sinus, and an Complications of multicystic kidney disease include
absence of renal parenchyma. The right kidney shows hypertension and infection. The danger of malignancy
compensatory hypertrophy. MCDK is found commonly in this lesion is considered remote.
in children with other major anomalies such as those of The surgical treatment of MCDK should be consid-
the respiratory, cardiac, gastrointestinal, and musculo- ered in only few selected conditions: with a very large
skeletal system. MCK (>6 cm), when the retained mass appears to be
A nuclear renogram is the best imaging study with growing or not involuting, when adequate follow-up
which to differentiate MCDK from hydronephrotic cannot be assured, when the diagnosis is in question,
kidney and to detect a contralateral ureteropelvic junc- and if hypertension or symptoms develop.
tion (UPJ) obstruction. A CT scan would require se- At the age of 6 years the patient presented with hy-
dation, it exposes the child to more radiation, and it is pertension and left flank pain. The mass appeared nearly
more expensive. Multicystic kidneys do not function. unchanged on US. Therefore, the girl underwent retro-
The MAG-3 nuclear scintigraphy shows that there is no peritoneoscopic nephrectomy (Fig. 4).
4 Genitourinary Disorders 291

Q 140
Antonio Marte

Fig. 1
Fig. 2

A 6-year-old girl presented with a history of secondary


nocturnal enuresis, urinary urgency associated with re-
current flank pain and febrile urinary tract infections
(UTIs). The patient was on antibiotic prophylaxis with
cotrimoxazole. The mother showed us previous exami-
nations (Figs. 1–3).
• What do Figs. 1 and 2 show?
• What does Fig. 3 show?
• Which pathological condition is affecting this girl?
• Are other examinations necessary?
• What is the best way to manage this condition?

Fig. 3
292 Genitourinary Disorders

A 140

This girl has a bilateral vesicoureteral reflux (VUR).


The renal sonograms (left–right) in Figs. 1, 2 show a
dilatation of both pelvises.
Figure 3 is a voiding cystourethrography (VCUG)
scan during the voiding stage and shows a grade-III bi-
lateral VUR, dilation of both pelvises, and trabeculation
of the bladder wall.
Because the patient has a history of recurrent UTI
and urinary urgency, a DMSA nuclear scan and urody-
namics were requested.
The DMSA scan (Fig. 4) shows both the kidneys
scarred with central and peripheral scars.
The urodynamics demonstrates a hyperactive blad-
der with high waves of detrusor contraction during the
bladder filling.
The girl was kept on antibiotic prophylaxis with amox-
icillin-clavulanic acid and oxybutynin 0.3 mg/kg/die.
After 1 year of therapy the urinary symptoms resolved
but a nuclear cystogram (Fig. 5) revealed the persistence Fig. 4
of bilateral VUR.
The parents were offered three options: (a) to go on
with continuous antibiotic therapy, (b) open bilateral re-
implantation, (c) endoscopic correction of the VUR.
The parents chose the third option and the girl un-
derwent endoscopic correction of the VUR with a bulk-
ing agent (Deflux).
Eight months later the nuclear voiding cystogram
showed no reflux. The girl’s nocturnal enuresis resolved
but she is still taking oxybutynin for sporadic urgency.

Fig. 5

Suggested Reading
1. Elder JS, Diaz M, Caldamone A, Cendron M, Greenfield S, 2. Stenberg A, Hensle TW and Lackgren G. Vesicoureteral
Hurwitz R, Kirsch A, Koyle MA, Pope J, Shapiro E. Endo- reflux: a new treatment algorithm. Curr Urol Rep 2002;
scopic therapy for vesicoureteral reflux: a meta-analysis. I. 3:107–14
Reflux resolution and urinary tract infection. J Urol 2006; 3. Unver T, Alpay H, Bivikli NK, Ones T. Comparison of di-
175:716–22 rect radionuclide cystography and voiding cystourethrog-
raphy in detecting vesicoureteral reflux. Pediatr Int 2006;
48:287–91
4 Genitourinary Disorders 293

Q 141
Marcelo Martinez-Ferro

Fig. 1 Fig. 2 Fig. 3

Fig. 4

On a routine prenatal ultrasound scan (Fig. 1), per- On an ultrasound scan performed at 20 weeks, the cystic
formed at 32 weeks, a 21-mm cystic mass lying over the mass was not seen.
urinary bladder was observed in the fetal abdomen. The • Is this possible or could it be a misdiagnosis?
rest of the fetal anatomy was normal for a female fetus • How can you explain the fact that after presenting a
and the amniotic fluid volume was also normal. considerable growth, by the end of pregnancy this
At 34 weeks (Fig. 2) the mass had doubled in volume mass diminished in size?
and now measured 43 mm. Its content was liquid and • What could be the reason for the ultrasonographic
completely anechogenic with thin walls. It occupied a (from anechogenic to echogenic with septa) changes
big portion of the abdominal cavity. that this mass showed?
At 36 weeks (Fig. 3) the mass reduced in size measur- • Would you expect to palpate this mass after birth? If
ing 33 mm, but its content had turned echogenic with so, what clinical characteristics would you expect to
a fluid/debris pattern. The walls of the mass were thick find during palpation?
and crumpled. • What is the most probable diagnostic condition?
An immediate postnatal ultrasound revealed a 5-cm • Would you ask for further postnatal imaging stud-
echogenic mass with multiple internal septa (Fig. 4). ies?
• What are the possible differential diagnoses for a fetal • What is the best way to manage this condition?
abdominal cystic mass?
294 Genitourinary Disorders

A 141

This patient has a prenatally diagnosed ovarian cyst.


The differential diagnoses for fetal cystic abdominal
masses are: choledochal cyst, intestinal duplications,
mesenteric cysts, hydronephrosis, urachal cyst, and om-
phalomesenteric cysts among others.
It has been proposed that placental hormone stimu-
lation is the trigger factor that provokes follicular stimu-
lation and growth.
There are no reports of fetal ovarian cysts before
27 gestational weeks, probably because the ovarian tis-
sue requires some degree of maturity in order to respond
to hormonal stimulation. These data together with the Fig. 5
determination of fetal sex are important diagnostic tools
that help in the differential diagnosis.
The diagnostic criteria are: (1) female fetus (2) older
than 37 weeks (3) absence of urogenital and gastrointes-
tinal anomalies, (4) presence of a cystic mass in the fetal
abdominal cavity.
Depending on their ultrasound pattern, the cysts are
called simple (completely anechogenic with thin walls)
and complex or complicated (presence of septa, fluid/
debris, calcification).
The morphological changes shown by the mass are
due to its complication with torsion of the vascular ped-
icle and internal hemorrhage followed by necrosis. Tor-
sion may occur in up to 50% of cases in cysts larger than
5 cm. Fig. 6
Figure 5 shows the size of the mass as palpated after
birth. The mass could be moved all around the abdomen
without resistance.
Laparoscopy is the treatment of choice for compli-
cated cysts, as it provides diagnosis and treatment. No
further imaging studies are required in this case.
Figure 6 shows the necrotic aspect of the left ovary
(Ov) during laparoscopy. The vascular pedicle (Ped) was
completely torted and the left tube was amputated up
to the left uterine horn. In addition, the torted vascular
pedicle partially occluded the sigmoid colon. The cyst
was resected and excised through the umbilicus.
One-week postoperative cosmetic results were opti-
mal after laparoscopy (Fig. 7).

Suggested Reading
1. Brandt ML, Helmrath MA. Ovarian cysts in infants and
children. Semin Pediatr Surg 2005; 14:78–85
Fig. 7
4 Genitourinary Disorders 295

Q 142
Antonio Savanelli, Salvatore Iacobelli, and Hana Dolezalova

Fig. 1 Fig. 2

A 26-month-old child presented with a 5-month history


of urinary tract infections (UTIs). A US scan and a urog-
raphy study were performed, shown in Figs. 1 and 2.
• What do you see in Figs. 1 and 2?
• Which procedures do you need so as to complete the
diagnosis?
• What does Fig. 3 show?
• What is the differential diagnosis?
• What is your diagnosis?
• Which treatment do you prefer in this case?
• What is the follow-up?
• What does Fig. 4 show?

Fig. 3

Fig.4
296 Genitourinary Disorders

A 142

The diuretic urography shows, on the right, regular elim- The UPJO can be treated medically or surgically. Sur-
ination of the contrast medium with a normal-looking gical indications are: symptomatic patient, UTI with fe-
pyelocaliceal system and ureter. On the left side we can ver and failure of growth, and poor renal function. Open
see a delayed washout with hydronephrosis and no visu- pyeloplasty using the Anderson–Hynes dismembered
alized ureter. After 12 h, there is persistence of contrast pyeloplasty is still the most widely used method.
medium on the left side. The US scan shows a high de- At the 6-month follow-up, we performed a US scan
gree of hydronephrosis on the left. The anteroposterior and dynamic scintigraphy. The last US scan shows a
diameter of the pelvis measures 38 mm. small hydronephrosis on the left with a normal ureter.
In order to complete the diagnosis, MAG-3 scintig- The renal scintigram (Fig. 4) shows a renogram with a
raphy was indicated. We can see a dishomogeneous left good washout after the diuretic test.
renal perfusion and capitation with poor washout after
the diuretic test. Effective renal plasma flow (ERPF) in Comment: Stenosis of the pyeloureteral junction rep-
the left kidney was 54.1%; ERPF in the right kidney was resents the most common malformation of the upper
45.9% (Fig. 3). urinary tract. US scans and renal scintigraphy concur in
Unilateral hydronephrosis can be found in the fol- most cases in providing a definitive diagnosis. Diuretic
lowing pathologies: vesicoureteral reflux, obstructive urography has been supplanted by scintigraphy, but is
megaureter, ureterocele and ectopic ureter, and uretero- still occasionally used in the preoperative phase for a
pelvic junction obstruction (UPJO). better diagnostic definition.
As there is no dilated ureter, the most probable diag-
nosis is UPJO.

Suggested Reading
1. Ismail A, Elkholy A, Zaghmout O, Alkadhi A, Elnaggar 2. Peters CA. Urinary tract obstruction in children. J Urol
O, Khairat A, Elhassanat H, Mosleh A, Hamad B, Elzomer 1995; 154:1874
J, Elkaabi A. Postnatal management of antenatally diag- 3. Rodriguez LV, Lock J, Kennedy WA, Shortliffe, LD. Evalua-
nosed ureteropelvic junction obstruction. J Ped Urol 2006; tion of sonographic renal parenchymal area in the manage-
2:163–168 ment of hydronephrosis. J Urol 2001; 165:548
4 Genitourinary Disorders 297

Q 143
Antonio Savanelli, Emanuela Giordano, and Barbara Greco

Fig. 1

Fig. 2

Fig. 3 Fig. 4

A 6-day-old baby was prenatally diagnosed with hydro- • What does Fig. 2 show?
nephrosis and bilateral megaureter. The postnatal ultra- • What is the therapeutic approach?
sound investigation showed the presence of a bilateral py- • What does Fig. 3 show?
elocalyceal dilatation, megaureters with tortuous course, • What will the therapeutic approach be in the case of
and distended bladder with thickened bladder wall persistent acute kidney failure or eventual complica-
(Fig. 1). Blood test results showed acute kidney failure. tions?
• Based on Fig. 1, what could the pathological condi- • What is the follow-up for this condition?
tion be? • What does Fig. 4 show?
• What is the second instrumental investigation to be
performed?
298 Genitourinary Disorders

A 143
Hydronephrosis and bilateral megaureter suggest the num in the posterior urethra (type III valves according
presence of several pathologies: vesicoureteral reflux to Young’s classification; believed to originate from in-
(VUR), bilateral obstructive megaureter, and posterior complete canalization between the anterior and poste-
urethral valves (PUVs). The finding of a distended blad- rior urethra) (Fig. 3).
der with thickened bladder wall on the ultrasound scan In the case of complications or no improvement, it
suggests the presence of cervical–ureteral obstructive will be necessary to perform vesicostomy.
pathology as well as PUVs (Fig. 1). PUV is a congenital The patient is monitored with:
condition that occurs only in boys and consists in a situ- (a) Laboratory tests to check the renal function.
ation where the urethral valves, which are small leaflets (b) Ultrasound of the loins and urinary tracts after
of tissue, have a narrow, slit-like opening that partially 6 months to check the dilatation of the upper urinary
impedes urine outflow. In fact, this pathological condi- tracts.
tion represents the most common cause of severe con- (c) MCUG after 6 months to check the urethral gauge or
genital obstruction of the lower urinary tract. When the persistence of PUVs after ablation.
there are PUVs the ultrasound scan, occasionally, shows (d) Renal scintigraphy (diuresis renography), which is a
a dilated posterior urethra or bright kidneys with loss of valuable tool in the evaluation of renal function and
corticomedullary differentiation. washout, once the patient is 1 month older.
The second instrumental investigation is the micturat- (e) Urodynamic testing, which is a functional test of
ing cystourethrogram (MCUG). This diagnostic test may the bladder and bladder outlet function. It is very
be performed via urethral or suprapubic catheter and re- important to perform this test in order to check for
quires the introduction of iodate intravenous urography the eventual presence of urinary incontinence, which
agents into the bladder. Radiographs are taken when the occurs in patients treated by valve ablation with an
bladder is full and while the patient passes urine. The incidence of 13%–38%.
position and integrity of the bladder and the urethra
are clearly shown during this examination. MCUG is Figure 4 shows the renal scintigraphy after the treatment
very important in analyzing the evacuation of the blad- of the PUVs.
der and the filling of the urethra and in searching for a Diuresis renography should be performed only
possible VUR, bladder outline, or a dilated obstructed when the patient is at least 1 month old. 99mTc MAG-3
posterior urethra. is the recommended radiopharmaceutical, at a dose
Figure 2 shows the patient’s MCUG with PUVs: the of 1.85 MBq/kg (50 µCi/kg) and a minimum dosage of
bladder dilatation and anomalies, such as bladder diver- 37 MBq (1 mCi). 99mTc MAG-3 yields higher extraction
ticula, and a very large ectasia of the posterior urethra from the kidneys than 99mTc DTPA, which is another less
are seen. There is no presence of VUR. preferred agent used in diuresis renography. Renal up-
When planning the therapy, first of all, it is impor- take by 99mTc MAG-3 is 55% compared with 20% uptake
tant to pay attention to the young patient’s fluid and the by 99mTc DTPA. This higher extraction results in better
electrolyte and acid-base balance. It is also important images for qualitative and quantitative analysis, and this
to achieve an adequate provisional urinary drainage by is of particular benefit in the pediatric period, when re-
inserting a fine urethral catheter and, subsequently, to nal function is immature. In this case the MAG-3 scan
ablate the valve through a diagnostic and therapeutic shows normal renal function of the left kidney and a de-
cystoscopy. crease in the function of the right side (Fig. 4).
Figure 3 shows the results of the cystoscopic investi- In urodynamic testing the bladder has a good capac-
gation. A small cystoscope is introduced in the urethral ity with a good compliance: max. vol. 186 ml; leak point
external meatus up to the urethra and, when possible, in pressure (LPP) 30 cm H2O; max. pressure of emptying
the bladder. In the present case, there is the presence of 50 cm H2O.
a perforated urethral membrane below the verumonta-

Suggested Reading
1. Gatti JM, Kirsch AJ. Posterior urethral valve. Pre and post- 3. Lopez Pereira P, Martinez Urrutia Mj, Espinosa L et al.
natal management. Current Urology Reports 2001; 2:138 Bladder dysfunction as a prognostic factor in patients with
2. Glassberg KI. The valve bladder syndrome: 20 years later. posterior urethral valves. BJU International 2002; 90:308
J Urol 2001; 166:1406
4 Genitourinary Disorders 299

Q 144
Bruno Cigliano

Fig. 2

Fig. 1

A full-term female newborn had a large intra-abdomi-


nal cystic lesion, which was detected on ultrasound per-
formed during the third trimester of pregnancy (Fig. 1).
At birth, her abdomen was distended (Fig. 2) and a large
soft mass was palpable. Ultrasound was performed show-
ing a large cystic mass containing a smaller cystic lesion,
with no relationship with the kidneys, liver, or spleen
(Fig. 3). An intestinal duplication was suspected and a
pediatric surgeon was called to evaluate her condition.
• What does Fig. 3 show?
• What is the possible diagnosis in this baby?
• What other diagnostic examinations do you recom-
mend to establish a correct diagnosis? Fig. 3
• What is the most appropriate management in this
case?
300 Genitourinary Disorders

A 144

Fig. 5

Fig. 4

Figure 3 shows a large cystic lesion in the abdomen that


has no relationship with the kidneys, liver, or spleen.
There are various diagnostic possibilities (mesenteric
cyst, urachal cyst, intestinal duplication, ovarian cyst
etc.), but the presence of the “daughter sign” is strongly
suggestive of an ovarian cyst.
Ovarian cysts are the most common intraperitoneal
masses found in female newborns.
The management of these lesions depends on their
dimension, evolution, and ultrasonographic appearance.
Ultrasonography is generally sufficient for diagnosis;
it is also accurate in predicting complicated cases and
is recommended for monitoring spontaneous resolution.
Usually, cysts that are 4–5 cm in diameter or larger, not
decreasing in size, or complicated (Fig. 4) are treated.
Treatment may consist of US-guided needle aspiration,
surgery (Fig. 5), or laparoscopy (Fig. 6). Laparoscopic
treatment is a safe and quick technique in neonates and Fig. 6
in our opinion is the best treatment.
4 Genitourinary Disorders 301

Q 145
Craig T. Albanese

Fig. 1 Fig. 2

A routine prenatal ultrasound at 18 weeks’ gestation was


performed. The left kidney (arrows) is pictured (Fig. 1)
as is the fetal heart (arrowheads).
• What are the findings?
Figure 2 is another view of the kidney (between arrows).
MR imaging was performed (Fig. 3), showing the fetal
kidney (arrow) and lung (asterisk).
• What is the diagnosis?

Fig. 3
302 Genitourinary Disorders

A 145

This child has a unilateral ureteropelvic junction (UPJ)


obstruction. There is marked hydronephrosis. Since it
was unilateral, there was no impact on amniotic fluid
production (i.e., there was no oligohydramnios).
The child underwent a dismembered pyeloplasty at
4 months of life.

Suggested Reading
1. Boubaker A, Prior JO, Meyrat B, Bischof Delaloye A,
McAleer IM, Frey P. Unilateral ureteropelvic junction ob-
struction in children: long-term follow-up after unilateral
pyeloplasty. J Urol 2003 Aug; 170(2 Pt 1):575–9
2. Perez-Brayfield MR, Kirsch AJ, Jones RA, Grattan-Smith
JD. A prospective study comparing ultrasound, nuclear
scintigraphy and dynamic contrast enhanced magnetic
resonance imaging in the evaluation of hydronephrosis. J
Urol 2003 Oct; 170(4 Pt 1):1330–4
3. Rodriguez LV, Spielman D, Herfkens RJ, Shortliffe LD.
Magnetic resonance imaging for the evaluation of hydro-
nephrosis, reflux and renal scarring in children. J Urol 2001
Sep; 166(3):1023–7
4 Genitourinary Disorders 303

Q 146
François Becmeur

Fig. 2

Fig. 1

A 5-year-old boy had several episodes of pyelonephritis.


A double kidney was suspected based on the findings of
a renal ultrasound study.
Cystography (Fig. 1) was performed. The pediatri-
cian asked for intravenous urography (Figs. 2–5) in or-
der to complete the data acquired from the renal ultra-
sound (Fig. 6). Finally, a DMSA renal scan was necessary
to specify the functional characteristics of each kidney Fig. 3
(Fig. 7).
• Describe the figures.
• Which therapeutic approach do you propose?
304 Genitourinary Disorders

Fig. 4

Fig. 5
4 Genitourinary Disorders 305

Fig. 6

Fig. 7
306 Genitourinary Disorders

A 146

The micturating cystography shows the existence of a Intravenous urography makes it possible to assess,
major vesicoureteral reflux on the left side of the lower during the first minutes of the examination, a functional
pole. right kidney and a functional upper left pole. The late
It was important to know whether the lower left re- opacification (45 min and 1 h) of the lower pole was due
nal pole was functional. The answer would determine to retrograde flow because of vesicorenal reflux. The re-
the choice of treatment. Indeed, if the lower pole were nal DMSA scan confirms the absence of functionality of
functional, a preserving treatment should be proposed: the left lower pole.
medical, endoscopic, or surgical. To specify the func- Left lower polar nephrectomy is the only treatment
tional anatomy of the kidneys, the pediatrician decided indicated for this child. Partial nephrectomy can be car-
to perform intravenous urography and renal scanning ried out by retroperitoneoscopy.
with DMSA.

Suggested Reading
1. Blumenthal I. Vesicoureteric reflux and urinary tract infec- 4. Heidenreich A, Ozgur E, Becker T, Haupt G. Surgical
tion in children. Postgrad Med J 2006 Jan; 82(963):31–5 management of vesicoureteral reflux in pediatric patients.
2. Darge K, Riedmiller H. Current status of vesicoureteral re- World J Urol 2004 Jun; 22(2):96–106
flux diagnosis.World J Urol 2004 Jun; 22(2):88–95 5. Riccabona M, Fotter R. Modern imaging technology for
3. Garin EH, Olavarria F, Garcia Nieto V, Valenciano B, Cam- childhood urinary tract infection Radiologe 2005 Dec;
pos A, Young L. Clinical significance of primary vesicoure- 45(12):1078–8
teral reflux and urinary antibiotic prophylaxis after acute
pyelonephritis: a multicenter, randomized, controlled study.
Pediatrics 2006 Mar; 117(3):626–32
4 Genitourinary Disorders 307

Q 147
François Becmeur

Fig. 2

Fig. 1

A 7-year-old girl had a fever of 39.5°C, right lumbar • What is the probable cause?
pains, and burning micturition. • What do you see on the retrograde cystogram
• Which examinations do you suggest in emergency? (Fig. 1)?
• Which treatment do you propose? • How can you evaluate the repercussions of this path-
ological condition on renal function (Fig. 2)?
During the last 3 years, the child had several episodes • Which treatment do you propose for this condition?
of fever. • What is its effect on renal function?
308 Genitourinary Disorders

A 147

This girl probably has acute pyelonephritis. Renal function can be evaluated by a renal DMSA
In emergency, urine analysis and renal ultrasound scan, which in this case shows a relative function of 36%
must be performed. The diagnosis of pyelonephritis will on the right kidney and of 64% on the left one. We ob-
be confirmed by urine culture and the bacteria will be served the late effects of pyelonephritis of the inferior
identified; ultrasonography will eliminate renal abscess, third of the right kidney.
lithiasis, or hydronephrosis. Surgical treatment of the reflux either by endoscopic
For antibiotic treatment, intravenous antibiotic ther- or by a ureterovesical reimplantation is indicated. For
apy is indicated consisting in beta-lactamine+aminosides this treatment, we recommend certain hygienic and di-
for 48 h, followed by oral administration according to etetic measures with abundant drink and regular mictu-
the antibiogram for 10 days. rition in the course of the day.
Vesicoureteral reflux must be investigated with retro- The function of the right kidney will not recover, but
grade cystography. should remain stable if the patient does not have any
The cystography shows a grade-3 vesicoureteral re- episodes of UTI.
flux in the inferior pole with a complete ureteric duplica-
tion on the right.

Suggested Reading
1. Capdevila Cogul E, Martin Ibanez I, Mainou Cid C, Toral 4. de La Vaissiere B, Castello B, Quinet B, Cohen R, Grim-
Rodriguez E, Cols Roig Mf, Agut Quijano T, Caritg Bosch prel E. Management of acute pyelonephritis in patients
J, Camarasa Pique F. First urinary tract infection in healthy older than 3 months: survey conducted in 39 paediatric
infants: epidemiology, diagnosis and treatment An Esp Pe- emergency departments of the Ile de France Region in 2004
diatr 2001 Oct; 55(4):310–4 Arch Pediatr 2006 Mar; 13(3):245–50
2. Halevy R, Smolkin V, Bykov S, Chervinsky L, Sakran W, 5. Lin KY, Chiu NT, Chen MJ, Lai CH, Huang JJ, Wang YT,
Koren A. Power Doppler ultrasonography in the diagnosis Chiou YY. Acute pyelonephritis and sequelae of renal scar
of acute childhood pyelonephritis. Pediatr Nephrol 2004 in pediatric first febrile urinary tract infection. Pediatr
Sep; 19(9):987–91 Nephrol 2003 Apr; 18(4):362–5
3. Kirsch AJ, Grattan-Smith JD, Molitierno JA Jr. The role
of magnetic resonance imaging in pediatric urology. Curr
Opin Urol 2006 Jul; 16(4):283–90
4 Genitourinary Disorders 309

Q 148
Antonio Savanelli, Francesca Alicchio, and Pier Francesco Rambaldi

Fig. 2

A 15-day-old baby presented with loss of weight and


urinary tract infection (UTI). On the ultrasound scan,
only a reduction in the volume of the right kidney was
evident without dilatation of the urinary tract.
• Which diagnosis is suspected?
• Which instrumental examination could be per-
formed?
• What does Fig. 1 show?
• What is the treatment for vesicoureteral reflux (VUR)
in the newborn and what does Fig. 2 show?
• What do you think the next step in follow-up should
be?

The patient, at 16 months of age, continued to have in-


sufficient weight gain and recurrent UTI.
• Which treatment should be indicated?
Fig. 1
310 Genitourinary Disorders

A 148

The patient could be affected by a congenital bilateral


VUR.
This condition represents the most frequent uropathy
in the pediatric age. It can be either primitive due to a
congenital malformation or a delay of development of
the vesicoureteral junction or secondary due to anatom-
ical or functional vesical or ureteral factors.
In 26% of cases, the antenatal ultrasound does not
show dilatation of the upper urinary tract.
The examination in Fig. 1 is a cystourethrogram
(CUGM) that shows bilateral VUR of a severe degree on
the right and moderated degree on the left.
The treatment of VUR in a baby comprises antibiotic Fig. 3
prophylaxis of UTI. The examination in Fig. 2 shows a
renal scintigram with MAG-3 that demonstrates a re-
markable reduction in right renal function.
The next diagnostic step in the follow-up is nuclear
cystography, which in this case demonstrates the same
situation, and renal scintigraphy with MAG-3 confirms
the finding of a nonfunctioning right kidney (Fig. 3).
Surgical treatment is indicated. We performed right
retroperitoneoscopic nephrectomy (Fig. 4) and endo-
scopic treatment of the VUR by Deflux (Fig. 5).

Fig. 4
Suggested Reading
1. Badwan KH, Diamond DA. Vesicoureteral reflux: diagnosis
and management. J Med Liban 2005 Apr–Jun; 53(2):61–5
2. Bhide A, Sairam S, Farrugia MK, Boddy SA, Thilaganathan
B. The sensitivity of antenatal ultrasound for predicting
renal tract surgery in early childhood. Ultrasound Obstet
Gynecol 2005 May; 25(5):489–92
3. Cohen AL, Rivara FP, Davis R, Christakis DA. Compliance
with guidelines for the medical care of first urinary tract
infections in infants: a population-based study. Pediatrics
2005 Jun; 115(6):1474–8
4. Dillon MJ, Goonasekera CD. Reflux nephropathy J Am Soc
Nephrol 1998 Dec; 9(12):2377–83
5. Smellie JM, Barratt TM, Chantler C, Gordon I, Prescod NP,
Ransley PG, Woolf AS. Medical versus surgical treatment
in children with severe bilateral vesicoureteric reflux and
bilateral nephropathy: a randomised rial. Lancet 2001 Apr
28; 357(9265):1329–33

Fig.5
4 Genitourinary Disorders 311

Q 149
Antonio Savanelli, Barbara Greco, and Concetta De Luca

Fig. 1
Fig. 3

Fig. 2

A newborn was antenatally diagnosed with a renal cyst • What other examinations are necessary?
of the upper renal pole. The perinatal ultrasound showed • What does Fig. 2 show?
a left ureteral duplication with dilatation of the ureteral • What does Fig. 3 show?
and upper segment system. • What is the final diagnosis?
• What does the ultrasound (Fig. 1) show? • What is the treatment?
• What is the possible diagnosis? • What is the follow-up?
312 Genitourinary Disorders

A 149

The ultrasound (Fig. 1) shows a duplex kidney with dila-


tation of the upper left segment. Dilatation of the upper
tract in ureteral duplication and a normal lower urinary
tract are compatible with the diagnosis of ectopic ure-
ter and ureterocele. The absence of a ureterocele in the
bladder on the ultrasound scan confirms the diagnostic
suspicion of ectopic ureter. Today, ultrasound may be
able to trace the ureter into the pelvis and into an abnor-
mally low position beyond the bladder.
In this case, we needed to perform voiding cystoure-
thrography, excretory urography with renography, and
cystoscopy.
The cystography shows reflux in the left ectopic ure-
ter. The cystoscopy demonstrates the ectopic ureter.
Figure 3 shows a urogram. A normal right kidney
and left duplex kidney can be seen. The ureter and the
upper pelvis are dilated.
The diagnosis is refluxing ectopic ureter in the left
duplex kidney.
Management of the upper segment of the ectopic re-
noureteral unit most often involves surgical treatment. Fig. 4
The procedure is dependent on the function of the ec-
topic segment. In this case, a pyeloureterostomy (upper
segment ureter to lower segment pelvis) was preferred
over partial nephrectomy, and partial left ureterectomy Suggested Reading
was performed. The distal ureteral stump should be left 1. Bieri M, Smith CK, Smith AY, Borden TA. Ipsilateral ure-
as short as possible. tero-ureterostomy for vesicoureteral reflux in duplicated
Follow-up includes ultrasound, cystography, and a ureters. J Urol 1998; 159:3,1016
MAG-3 renogram at 6 months. 2. De Caluwe D, Chertin B, Puri P. Long-term outcome of the
At follow-up the patient showed an improved dilatation retained ureteral stump after lower pole heminephrectomy
of the upper pelvis and persistent refluxing ureteral stump, in duplex kidneys. Eur Urol 2002; 42(1):63
which had determined a UTI. The reflux into the reflux- 3. el Ghoneimi A, Miranda J, Truong T, Monfort G. Ectopic
ing ectopic ureteral stump (Fig. 4) was treated success- ureter with complete ureteric duplication: conservative
surgical treatment. J Pediatr Surg 1996, 31:467
fully by endoscopic injection of macroplastic material.
4 Genitourinary Disorders 313

Q 150
Brice Antao and Azad Najmaldin

Fig. 1 Fig. 2

A 15-year-old girl presented to the Accident and Emer-


gency department with sudden onset of right-sided ab-
dominal pain. She was known to have an antenatal diag-
nosis of solitary right kidney. On examination she was
pyrexial at 38˚C and had a tender palpable mass on the
right side of her abdomen.
• What is the differential diagnosis?
• What initial investigations would you request?
• What are the findings in Fig. 1 that support the diag-
nosis?
• What is the investigation shown in Figs. 2 and 3, and
why was it performed?
• What is the diagnosis and how do you manage this
condition?
• What are the potential complications of this condi-
tion and how can these be managed?

Fig. 3
314 Genitourinary Disorders

A 150

The differential diagnoses include appendix mass, mass


associated with Crohn’s disease, intestinal duplication
cyst, hepatobiliary mass, torted ovarian cyst, pyelone-
phritis, hydronephrosis, obstructing urinary calculi, in-
fected lymphatic cysts, and tumors.
A baseline blood and urine analysis may prove help-
ful. The patient had a high white blood cell count and
C-reactive protein and her urine analysis confirmed a
coliform infection. Ultrasonography is easily available,
noninvasive, and often informative. Figure 4 confirmed
an absent left kidney and an abnormally long and en-
larged right renal mass and an unusual but nondilated
collecting system. No other abnormality was identified
in the abdomen and pelvis.
A CT scan of the abdomen (Fig. 1) clearly demon- Fig. 4
strated features of a cross-fused renal ectopia in the right
side of abdomen and no other abnormalities.
Figures 2 and 3 are MR urography images. This inves-
tigation provides anatomical and functional evaluation The potential complications include recurrent infec-
of the urinary tract in a single examination without the tion and scaring. Malignancy has also been reported,
use of ionizing radiations. T2 sequences in coronal and albeit sporadically. Regular ultrasonography to monitor
axial section with contrast enhancement demonstrated the size of the collecting system and ureter may prove
a cross-fused ectopia. The right moiety lies superior–lat- helpful in the long term. A DMSA scan may be neces-
erally, while the left moiety which lies inferior–medially sary to assess scaring and function. An isotope mictur-
has a bifid collecting system. There is no evidence of hy- ating cystourethrogram or a formal contrast cystogram
droureter or hydronephrosis. with or without cystoscopy will add more information
The diagnosis is an infected right crossed renal ecto- and exclude vesicoureteric reflux as an associated prob-
pia. The management is conservative and includes: intra- lem. An acutely infected kidney associated with signifi-
venous antibiotics, possible intravenous fluids, analgesia, cant hydronephrosis may require percutaneous drainage.
and close monitoring (pulse, respiration, urine output, Subsequent surgery will depend on whether or not the
degree of pain, tenderness and size of the mass). Repeat patient remains symptomatic or has associated prob-
ultrasonography is also used to monitor progress. The lems such as obstruction, reflux, or a nonfunctioning
patient responded well to the above regimen and was al- symptomatic moiety.
lowed home after 4 days. She was reviewed in the clinic
with repeat ultrasonography after a few days, weeks, and
months.

Suggested Reading
1. Grattan-Smith JD, Perez-Bayfield MR, Jones RA, et al. MR 2. Stimac G, Dimanovski J, Ruzic B, Spajic B, Kraus O. Tumors
imaging of kidneys: functional evaluation using F-15 per- in kidney fusion anomalies-report of five cases and review
fusion imaging. Pediatr Radiol 2003; 33:293–304 of the literature. Scand J Urol Nephrol 2004; 38(6):485–9
3. Taweel W, Sripathi V, Ahmed S. Crossed fused renal ectopia
with hydronephrosis. Aust NZ J Surg 1998; 68(11):808–9
4 Genitourinary Disorders 315

Q 151
Gianluca Terrin, Annalisa Passariello, and Hana Dolezalova

Fig. 1a Fig. 1b

A routine prenatal ultrasound scan performed at day 1 of life and suspended on day 7 for feeding intol-
20 weeks’ gestational age on a consanguineous 31-year- erance. A urinary and gastrointestinal tract radiological
old woman (gravida 1) revealed marked bladder dis- examination was performed.
tension, hydroureteronephrosis, polyhydramnios, and • What do Figs. 2–4 show?
ascites. A dilated posterior urethra was not identified
on subsequent ultrasound studies. Two in-utero bladder This clinical condition suggests a diagnosis of MMIH
evacuative punctures were performed during pregnancy. syndrome such as prune belly syndrome (PBS).
A normal 46 XX karyotype was detected at amniocente- • Describes the main features used to distinguish be-
sis. The maternal Coombs test was negative. tween these two entities.
• In this case, is it possible to distinguish obstructive
from nonobstructive in-utero bladder distension? Bladder distension persisted, associated with recurrent
• What is megacystis microcolon intestinal hypoperi- urinary tract infections. Enteral feeding intolerance
stalsis syndrome (MMIHS)? persisted and total parenteral nutrition was adopted.
On day 28 the presence of intestinal malrotation was
The neonate was born at 35 weeks (birth weight 4,350 g). diagnosed by a laparoscopic procedure. Subsequently,
Physical examination showed laxity of the abdominal vesicostomy and correction of an intestinal malrotation
musculature and a renal bladder ultrasound revealed were performed. Histological examination showed a
megacystis with bilateral hydroureteronephrosis. Shortly normal number of ganglia in the autonomic intestinal
after delivery the child required intubation and mechan- plexus and normal acetylcholinesterase staining. Inad-
ical ventilation (Fig. 1). After placing a catheter in the equate bladder specimens were obtained.
bladder, more than 800 ml of urine was drained and re- • Is it possible to differentiate MMIHS from PBS on
inspection of the abdominal wall revealed a prune aspect the basis of the intestinal histological examination?
(Fig. 2). The cyst rapidly refilled after aspiration. Acute • Describe the prognosis of MMIHS.
renal failure was observed. Oral feeding was started on
316 Genitourinary Disorders

Fig. 2 Fig. 3

Fig. 4
4 Genitourinary Disorders 317

A 151 structive enlarged bladder may coexist. Intestinal malro-


tation and Hirschsprung’s disease have been described
in association with PBS. The current hypothesis regard-
ing the etiopathogenesis of PBS proposes a mesodermal
arrest between the 6th and 10th weeks of gestation or an
in utero urethral obstruction. Consequently, fetal mega-
Congenital anomalies of the genitourinary system are cystis development determines the increase of intra-ab-
detected in about 1:500 fetuses during routine prena- dominal pressure that induces laxity of the abdominal
tal ultrasound screening. The bladder forms one of the musculature and a defect of intestinal fixation and rota-
most readily identified structures in the fetus and is tion.
seen at 12 weeks of gestation. Prenatal differentiation Pathological findings in MMIHS vary considerably
between obstructive and nonobstructive megacystis and include changes of the neural tissue and muscle of
is mainly based on the amount of amniotic fluid, renal the bowel wall. This variability may be due to the focal
echogenicity, and bladder wall thickness. The presence nature of some of the pathological findings reported or
of oligohydramnios, progressive bladder wall thickening, may reflect pathological heterogeneity within MMIHS.
and dilated posterior urethra is suggestive of obstructive More recently, a marked reduction of contractile and
uropathy (e.g., urethra valves). A normal amount of am- cytoskeleton proteins in smooth muscle cells combined
niotic fluid suggests a nonobstructive bladder distention. with reduced expression of intramuscular interstitial
Nonobstructive forms of megacystis are seen in isolated cells of Cajal, pacemaker cells coordinating intestinal
congenital megacystis, in nonrefluxing–nonobstructive motility, in the gut were reported. Excessive smooth
megaureter–megacystis, and in MMIHS. The former, in muscle glycogen storage and vacuolar degeneration on
this clinical setting, is the most probable diagnostic hy- histological bladder examination of patients affected by
pothesis, because the presence of polyhydramnios sug- MMIHS were also described. Interestingly, in both syn-
gests an intestinal implication. dromes abundant fibrocytes and collagen are present in
MMIHS is an inherited disorder transmitted in an the gastrointestinal tract; moreover, in PBS excessive fi-
autosomal recessive fashion, causing a functional ob- brous tissue associated with both increased musculature
struction of both the gastrointestinal and genitourinary and defective or dysplastic muscles of the urinary tract
tracts. Polyhydramnios is the major prenatal feature in- have been identified. Immunohistochemical and ultra-
dicating gut involvement. Recent evidence suggests that structural studies on urinary tract smooth muscle cells
MMIHS is a result of the functional deficiency of the α-3 in patients with PBS have not been reported.
or β2/β4 subunits of the nicotinic-acetylcholine recep- The prognosis in MMIHS is generally poor. To date,
tor (nAChR) in peripheral autonomic and enteric gan- up to about 10% of children have survived and the ma-
glia. A defective cholinergic signal determines the fate of jority required total parenteral nutrition. Postoperative
a functional bladder and intestinal hypoperistalsis and, complications, sepsis, and liver failure are the most
additionally, the α-3 subunits seem to be involved in di- common causes of death. Intestinal transplantation was
rect cell-to-cell communication during fetal maturation adopted in a limited number of cases of MMIHS. The
and differentiation of the involved organs. prenatal diagnosis of this condition remains a challenge,
Figure 2 shows an enlarged bladder without reflux, and the contribution of prenatal ultrasound is limited.
while Fig. 3 demonstrates a dilated stomach associated Recently, digestive enzyme measurement of the amni-
with delayed gastric emptying time. Figure 4 shows a di- otic fluid has been proposed to differentiate MMIHS
lated stomach and narrow microcolon. from other forms of megacystis during the prenatal pe-
Findings in PBS and MMIHS may overlap. PBS, riod. Molecular genetics may contribute significantly to
which usually presents in male infants, is characterized the prenatal diagnosis and to familial counseling. High-
by the triad of laxity of the abdominal wall musculature, frequency polymorphisms in neuronal nicotinic acetyl-
hydroureteronephrosis, and cryptorchidism. In females, choline receptor genes have been reported. Although no
as in males in whom not all of the triad is present, the loss-of-function mutations have been identified, these
condition is referred to as pseudo-PBS. In PBS the blad- genes remain strong candidates for involvement in
der is capacious and urethral abnormalities with an ob- MMIHS.
318 Genitourinary Disorders

Suggested Reading
1. Lev-Lehman E, Bercovich D, Xu W, Stokton DW, Beaudet 4. Richardson CE, Morgan JM, Jasani B, Green JT, Rhodes
A. Characterization of the human β-4 nAChR gene and J, Williams GT, Lindstrom J, Wonnacott S, Thomas GAO,
polymorphisms in CHRNA3 and CHRNB4. J Hum Genet Smith V. Megacystis-microcolon intestinal hypoperistalsis
2001; 46:362–66 syndrome and the absence of the α3 nicotinic acetylcholine
2. Levin TL, Soghier L, Blitman NM, Vega-Rich C, Nafday receptor subunit. Gastroenterology 2001; 121:350–57
S. Megacystis-microcolon intestinal hypoperistalsis and 5. White SM, Chamberlain P, Hitchcock R, Sullivan PB, Boyd
prune belly: overlapping syndromes. Pediatr Radiol 2004; PA. Megacystis-microcolon intestinal hypoperistalsis syn-
34:995–98 drome: the difficulties with antenatal diagnosis. Case report
3. McHugo J, Whittle M. Enlarged fetal bladders: aetiology, and review of the literature. Prenatal Diagn 2000; 20:697–
management and outcome Prenatal Diagn 2001; 21:958–63 700
5
Cardiovascular
Disorders

Case 152–167
320 Cardiovascular Disorders

Introduction

A congenital heart defect (CHD) is an abnormality that nary venous return, ventricular septal defect, patent
is present at birth. Congenital heart surgery was prac- ductus arteriosus, atrioventricular septal defects, aor-
ticed before the heart–lung machine was developed as topulmonary window, truncus arteriosus).
surgeons started to work on abnormalities of the arteries 2. CHDs with reduced pulmonary blood flow: these are
of the heart. Dr. John Streider at the Massachusetts Gen- characterized by a reduced pulmonary flow, second-
eral Hospital in Boston tied off a patent ductus arterio- ary to a right ventricular obstruction, with arterial
sus in a child on 6 March 1937. hypo-oxygenation directly related to the degree of
The human heart begins to develop at the end of the pulmonary flow reduction (e.g., tetralogy of Fallot,
first month of fetal life and takes about another 8 weeks Ebstein disease).
before it resembles an adult heart. During this period, 3. CHD with parallel circulations: in these pathologi-
about 8 out of every 1,000 newborns develop some form cal conditions, unoxygenated systemic venous return
of congenital heart defect ranging from very mild to directly enters into the systemic arterial circulation,
quite severe. The incidence of CHD ranges between 2.5% whereas oxygenated pulmonary return re-enters into
and 12%, being one of the most common congenital the pulmonary bed. A typical example is the transpo-
anomalies in human beings. Of these, ventricular septal sition of the great arteries, in which the aorta arises
defects and atrial septal defects are the most frequently from the right ventricle, and the pulmonary trunk
diagnosed, with about 15.6/10,000 birth survivors in the from the left ventricle. In these cases, survival is war-
former, and around 4/10,000 in the latter. However, the ranted by the presence of an anomalous communica-
real incidence of CHD seems to be higher, according to tion between the two circulations, aimed at ensuring
data that 10%–25% of fetal deaths are due to CHD. an adequate mixing between oxygenated and un-
The exact cause of CHD is unknown, but recent in- oxygenated blood. The greater the communication,
formation suggests there may be genetic influences in- the higher the mixing and the capability to prolong
volved. In some cases, they are associated with other survival. These CHD cases are commonly associated
medical conditions, such as the mother contracting Ger- with other types of CHD, which involve the sites of
man measles (rubella) while pregnant. It has been esti- the mixing, such as a patent ductus arteriosus, an
mated that 25% of CHD cases are encountered within atrial septal defect, and sometimes a ventricular sep-
complex congenital syndromes, and the cause of CHD tal defect.
can be attributed to the pathogenetic factor responsible 4. Ductus-dependent CHDs: all CHDs with severe ob-
for the syndrome; almost 6% of CHDs are due to envi- structions to the right or left ventricular flow, there-
ronmental teratogenic factors; finally, metabolic diseases fore requiring a patent ductus to assure an adequate
and collagenopathies of the mother are frequently en- pulmonary or systemic flow (e.g., critical aortic valve
countered in CHD. However, the vast majority of CHD stenosis, critical pulmonary valve stenosis, severe
cases still do not have a clarified complex etiopathogen- aortic coarctation, pulmonary atresia, aortic arch in-
esis, in which genetic, environmental, and maternal fac- terruption).
tors are implicated. 5. CHDs with ventricular outflow obstructions: all the
CHDs are traditionally classified into five general CHDs in which the symptoms depend on the pres-
groups: ence of an obstruction, at different levels, to the right
1. CHDs with excessive pulmonary blood flow: these or left ventricular flow. However, in these cases the
diseases are characterized by a left-to-right shunt of degree of obstruction is not so dependent on a pat-
a variable quantity of circulating blood, due to an ent ductus to have a sufficient pulmonary or systemic
anomalous intra- and/or extracardiac connection be- flow. These obstructions cause a pressure overload to
tween the systemic and pulmonary circulation. This the involved ventricle. (e.g., noncritical pulmonary
results in an augmented pulmonary flow with right and aortic valve stenosis, supravalvular and subval-
and then left chamber overload, depending on the vular aortic stenoses, aortic coarctation).
site of the anomalous connection, the age of the di-
agnosis, and the presence of hypertensive pulmonary Some CHDs are diagnosed shortly after birth or even
disease (e.g., atrial septal defects, anomalous pulmo- while the fetus is in the uterus by using ultrasound or
5 Cardiovascular Disorders 321

echocardiography. They may be diagnosed later when nary trunk shadow, hypoperfusion of the pulmonary
the child is of school age, or in rare circumstances, the fields, or sometimes, due to the development with
congenital cardiac defect remains hidden until adult- age of high-flow systemic-to-pulmonary collaterals,
hood. If one omits all patients born before 1990 and an altered pulmonary blood flow pattern in one or
those not diagnosed in the first year of life, assuming sta- both lungs (i.e., plethora of one lung or part of it and
ble mortality in early adulthood, nearly 760,000 adults oligemia of the other). Typical radiological signs such
will have CHD by 2020. as calcified patent ductus arteriosus or boot-shaped
When signs or symptoms of CHD are considered, heart shadow in tetralogy of Fallot are diagnostic.
one indicator of some types of CHD in a newborn is a 3. Echocardiography is certainly the most important
faint bluish color of the skin. Some children with heart examination with which to correctly define the CHD
defects may not thrive, and many suffer from congested affecting the patient. Two-dimensional Doppler echo-
lungs, which may be related to heart failure. However, cardiography has the advantage of being reproducible,
the two main signs raising suspicion for CHD are cyano- easy, safe, and with its transthoracic, or in more com-
sis and congestive heart failure. Congestive heart failure plex cases, transesophageal approach it not only helps
is responsible for delayed growth, fatigue, effort breath- define the diagnosis, but also indicates the therapeutic
lessness, dyspnea, cough, re-entrant intercostal spaces, approach (medical, interventional, surgical, pallia-
recurrent respiratory infections, palpitations from par- tion/correction, etc.). Its use in pediatric cardiology
oxysmal atrial tachycardia, and atrial fibrillation. Right requires a strict methodology: first, the cardiologist
heart failure furthermore causes fluid retention, hepato- must define the position of the heart within the chest.
megaly, sometimes distal edemas, and finally severe car- Then, the sequential definition of the “situs” (solitus/
diac cachexia. Physical examination shows the delay of inversus), of the veno-atrial concordance, of the atrio-
somatic and often mental growth, heart murmurs, split- ventricular concordance, of the spatial relation of the
ting of the second heart sound, hyposphygmic femoral two ventricles, of the ventricular-arterial concordance,
pulses, hepatomegaly, etc. and of the spatial relationship of the great arteries rep-
If a defect in a newborn is suspected, the pediatrician resents the next step of the echocardiographic evalu-
will recommend an electrocardiogram and probably an ation. The addition of Doppler color-flow imaging
echocardiogram. allows a reasonable estimate of the Qp/Qs ratio to be
1. Electrocardiography may suggest CHD or, on the made (pulmonary to systemic output ratio, useful to
contrary, be almost normal. Generally, signs of CHD discriminate between CHD with excessive and CHD
are left or right ventricular hypertrophy, left or right with reduced pulmonary blood flow). The recent in-
deviation of the QRS axis, intraventricular delay, or troduction of three-dimensional echocardiography
bundle branch blocks (often right bundle block). has the potential of providing a three-dimensional re-
Atrial enlargement may be responsible for supraven- construction of the cardiac chambers and of the CHD,
tricular tachycardias, atrial flutter, or fibrillation. which is of great interest for the surgical approach.
2. Chest radiography is still important when there is a 4. Cardiac catheterization, besides echocardiography, is
significant suspicion of CHD. First of all, the local- a gold standard diagnostic tool for all CHDs. In par-
ization of the left ventricular apex together with the ticular, it plays a key role in determining the stage of
gastric bulla is important so as to define the “situs”: the CHD, by measuring intra-atrial, intraventricular,
a right-sided apex and bulla are defined as a “situs intrapulmonary, and intra-aortic pressures. More-
inversus,” often associated with CHD. Cardiomegaly over, contrast-enhanced imaging strictly defines the
(cardio/thoracic ratio >0.50) is suggestive of ventric- morphology of the CHD. Moreover, it allows the cal-
ular overload and/or congestive heart failure. CHD culation of pulmonary and systemic flow (Qp/Qs)
with excessive pulmonary blood flow shows typical and of pulmonary resistances, and it identifies the
enlargement of the pulmonary trunk shadow on the sites of shunts by blood gaseous analysis at different
left margin of the cardiac silhouette, as well as en- levels of sampling. Moreover, direct administration
largement of the right and left pulmonary arteries to of vasodilators into the pulmonary bed also defines
the periphery of the lung field. In general, pulmonary the reversibility of a pre-existing pulmonary hyper-
vascular markings are increased or plethoric. Often tension. Finally, this technique is also crucial in the
the shadow of the aortic arch is abnormally small, therapeutic management, since it is possible to carry
due to the left-to-right shunt. On the other hand, out:
CHD with reduced pulmonary blood flow demon- a. Septostomy, to improve arteriovenous blood mix-
strates an atypical reduction or absence of the pulmo- ing in the transposition of the great arteries
322 Cardiovascular Disorders

b. Balloon valvuloplasty in pulmonary or aortic Other tests increasingly used to diagnose CHDs today
valve stenosis include CT and MR imaging. CT scan is generally em-
c. Balloon angioplasty with stent release in vascu- ployed in left ventricular obstruction, mainly in supra-
lar stenoses (first of all in the coarctation of the aortic valve stenosis and, more frequently, in coarctation
aorta) of the aorta. It precisely defines the anatomy and the
d. Atrial and ventricular septal defect closure with degree of aortic narrowing. Vascular rings are another
umbrella-shaped devices possible field of application.
e. Coil embolization to occlude patent ductus arte- Finally, the use of MR imaging is growing in daily
riosus, arteriovenous fistula, and systemic-to-pul- practice because of its precise anatomic and functional
monary collaterals cardiovascular definition, its noninvasiveness, its high
f. Myocardial biopsies to diagnose cardiomyopathy sensitivity, and its objectiveness. It can be stated that
or to evaluate acute/subacute/chronic rejection in there is no type of CHD which cannot be diagnosed with
heart transplant recipients MR imaging, and in which MR imaging is not useful in
the functional, preoperative, postoperative, and prog-
nostic evaluation.
5 Cardiovascular Disorders 323

Q 152
Juan A. Tovar

Fig. 1 Fig. 2

A 2-month-old baby presented with a pulsating mass in A frame of the right humeral angiography is depicted
his right elbow. in Fig. 2.
He was born after an uneventful delivery weighing He was treated via a transarterial route. The pulsating
1,500 g. He had respiratory distress requiring ventila- mass disappeared and the radial pulse was preserved.
tory assistance for 1 week. During this period and in the • Describe what you see in Fig. 1.
ensuing days he had several venous punctures in both • What is the difference between aneurysm and pseu-
arms for blood sampling. doaneurysm?
His respiratory condition improved and he was dis- • What sort of procedures may be indicated in these
charged without major problems. Two weeks after dis- cases?
charge, the parents noticed the mass in his right arm. • Are there any sequelae to be expected?
324 Cardiovascular Disorders

A 152

This baby had a humeral artery pseudoaneurysm. Arteriography or angio-MR imaging depicts the
Arterial injuries in children, and particularly in young anatomy of the lesion and orient the treatment.
babies, are most often iatrogenic. The progressively more In this case the pseudoaneurysm has a neck and the
aggressive approach to the treatment of various neonatal main artery is patent.
conditions requiring frequent venous sampling, arterial Treatment consisted in percutaneous transarterial
monitoring, or central lines has increased the risk of ar- embolization.
terial, venous, or arteriovenous lesions. Several substances are indicated for this purpose, in-
A false aneurysm or pseudoaneurysm is the result of cluding thrombin. Open surgery with anatomical repair
extravasation of blood through a partially ruptured arte- is also a valid option.
rial wall. The space created by the spillage heals forming Most patients can be cured without permanent se-
a false vascular wall that is devoid of the normal arterial quelae.
wall layers and that becomes more or less dilated and
pulsatile.

Suggested Reading
1. Gow KW, Mykytenko J, Patrick EL, Dodson TF. Brachial 3. Rey C, Marache P, Watel A, Francart C. Iatrogenic false
artery pseudoaneurysm in a 6-week-old infant. Am Surg aneurysm of the brachial artery in an infant. Eur J Pediatr
2004; 70:518–21 1987; 146:438–9
2. Pezzullo JA, Wallach MT. Successful percutaneous throm-
bin injection of a brachial artery pseudoaneurysm in a neo-
nate. AJR Am J Roentgenol 2002; 178:244–5
5 Cardiovascular Disorders 325

Q 153
Francesco Onorati, Giacomo Sica, and Attilio Renzulli

Fig. 1

Fig. 3

Fig. 4 Fig. 2

A 17-year-old boy complaining of headache and dys- A mono-2D Doppler echocardiography image is shown
pnea during submaximal efforts was referred to our de- in Fig. 2.
partment. He had no medical problems until the age of • What is the anomaly of the aortic image?
15 years, after which he progressively developed recur- • What are the main findings on echocardiography?
rent headache, fatigue, and ongoing shortness of breath • Which other investigations should be performed in
for maximal efforts. He also reported a few episodes of this case?
nose bleeding.
Physical examination showed systemic hypertension Angiography was also performed as shown in Fig. 3.
in the upper body (180/110 mmHg in the arms), with • Why was preoperative angiography carried out and
femoral hypotension (90/55 mmHg in the limbs). what is indicated by the arrow and the triangle?
• Which congenital heart disease could be suspected
according to the blood pressure pattern? A CT scan is shown in Fig. 4.
• Which anatomic structures are pathological?
The patient underwent chest radiography (PA projec- • What are the therapeutic options?
tion), as shown in Fig. 1.
• What are the typical rib findings (arrows) of this dis-
ease?
326 Cardiovascular Disorders

A 153

The patient was affected by coarctation of the aorta on the extension of the narrowing and the age of the pa-
(AoCo), a congenital narrowing of the aortic isthmus tient, surgery varies from resection of the AoCo and di-
(Fig. 5, arrow). Depending on the grade of the aortic rect reconstruction (end-to-end anastomosis) with wide
narrowing and the patency of the ductus arteriosus, mobilization of the aortic arch and ascending aorta in
signs and symptoms develop at different ages: severe newborns, infants, and young children, to an enlarge-
narrowing characterizes neonatal and infant AoCo, the ment or replacement of the descending aorta with a dia-
less severe forms present in childhood or adulthood. mond-shaped or tubular Dacron graft. Figure 5 shows
AoCo causes high blood pressure in the ascending an intraoperative view of the CoA (arrow) and the com-
aorta and aortic arch branches, leading to systemic hy- pletion of the operation. Several operations have been
pertension with headache and nasal bleeding. Hyper- reported for bypassing the lesion with a Dacron tube:
tension also causes progressive dilation of the ascend- subclavian-to-descending aorta (Clagett operation) or
ing aorta and left ventricular hypertrophy, leading to aortic–aortic bypass. In complex cases of multiple ob-
cardiomegaly and congestive heart failure. Similarly, the structions to the left ventricular outflow, an ascending-
isthmic narrowing explains why physical examination to-supradiaphragmatic aorta bypass and apico-aortic
demonstrated hypertension of the upper body and hy- conduit can be performed.
potension with poor femoral pulses in the lower body.
Chest radiography shows the typical “rib notching,”
caused by the hypertrophy of the intercostal arteries,
due to a collateral circulation through the intercostal
and mammary arteries between the upper and lower
thoracic aorta. Ectatic ascending aorta and aortic arch
can be noted in long-lasting AoCo.
2D echocardiography can depict AoCo, particularly
in neonates and small infants. Associated intracardiac
defects or other congenital anomalies can be easily
detected. The severity of AoCo can often be assessed
through color Doppler signaling (star, Fig. 2). Figure 2
shows a parasternal view of the AoCo (arrow) and a
moderate aortic arch dilation (triangle), with the origins
of the innominate and left carotid arteries.
With the advent of CT or MR imaging, cardiac cath-
eterization and aortography are nowadays rarely per-
formed. However, cardiac catheterization with aortogra-
phy still remains a good diagnostic tool with which to
better define the anatomy of the lesion and collateral cir-
culation as well as any associated congenital cardiac le-
sions; it also provides hemodynamic data. Moreover, in
selected cases AoCo can be treated with interventional
cardiology, stenting the narrowed area.
The CT scan demonstrated progressive narrowing of
the descending thoracic aorta (arrow). Other findings
suggestive of AoCo are dilated intercostal and mam-
mary circulation (triangle). Collaterals are better appre-
ciated on 3D-rendered images.
Besides interventional techniques, the gold standard
is still surgical therapy. The aim of surgery is to restore a
normal aortic flow, resecting the narrowed aorta and all
the surrounding apoptotic tissue. Therefore, depending Fig. 5
5 Cardiovascular Disorders 327

Q 154
Francesco Onorati, Giacomo Sica, and Attilio Renzulli

Fig. 1

Fig. 2

A 17-year-old girl was admitted to our institution


complaining of a persistent barking cough, with a few
episodes of apnea causing cyanosis and one episode of
unconsciousness. The patient reported progressive dys-
phagia in the last 5 years. She recently fed poorly and
therefore felt quite sick; in particular, the patient re-
ported great difficulties in swallowing liquids and solids,
with episodes of choking and regurgitation.
Physical examination did not show any relevant signs,
apart from expiratory wheezing.
• Does the chest radiograph (PA projection, Fig. 1) Fig. 3
show any abnormality?

Immediately thereafter, the patient underwent CT scan-


ning as demonstrated in Fig. 2.
• What is the main finding of the CT imaging depicted
by the arrow? • Why did the patient undergo aortography?
• Which vascular structure has an abnormal course?
Cardiac catheterization and aortography were per- • What pathological condition is affecting the patient?
formed (Fig. 3). • What is the treatment of choice?
328 Cardiovascular Disorders

A 154

The patient had so-called right lusoria subclavian ar- The clinical suspicion of right lusoria subclavian
tery—a congenital malformation belonging to a par- artery disease was confirmed. The CT scan clearly de-
ticular group of vascular anomalies defined as “vascular fined the anatomy of the origin and course of the su-
rings.” Vascular rings are congenital anomalies in which pra-aortic branches. An isolated right common carotid
the aortic arch and its branches completely or incom- course from the arch to the neck is shown, as well as a
pletely compress either the trachea, or the esophagus, or normal position of the left common carotid and of the
both by encircling them. Vascular rings are caused by left subclavian artery; in particular, the aberrant right
inappropriate persistence or resorption of the six pairs subclavian artery arises as the last branch of the aortic
of aortic arches connecting the ventral and dorsal aortae arch from the posterior wall, running behind and thus
during embryological development. These arches vari- compressing the esophagus. The retroesophageal artery
ously recede, fuse, and remodel to form the typical left- in dysphagia lusoria is frequently visible on unenhanced
sided aortic arch and its major branches. scans even if intravenous contrast medium administra-
In this case the right subclavian artery arises from tion clearly detects the anomaly.
the posterior wall of the aortic arch, next to the origin Alternatively, high-field (1.5 T) MR angiography
of a normal-positioned left subclavian artery. The aber- should be preferred in children and adolescents, since
rant artery compresses the esophagus and partially the it has a comparable clear definition of the anatomy, but
trachea to reach the right arm. Furthermore, esopha- avoids patient exposure to ionizing radiation and the
geal compression causes tracheobronchial hyper-reac- use of nephrotoxic contrast medium.
tivity. Because of this, the patient reported some signs In order to exclude associated cardiac and/or vascu-
and symptoms of respiratory impairment (cough, apnea, lar malformations, the patient underwent cardiac cathe-
cyanosis, unconsciousness, and expiratory wheezing). terization and aortography (Fig. 3), which excluded con-
Moreover, the right-sided course of the artery posterior genital cardiac diseases, confirming the posterior origin
to the esophagus causes its compression that has been of the right subclavian artery from the aortic arch.
traditionally blamed for the digestive symptoms (dys- Surgery is the only therapy for symptomatic cases.
phagia, choke, and regurgitation). Surgical exploration did not show a constraining liga-
The plain chest radiograph in the frontal view ap- mentum arteriosus, but demonstrated the vascular na-
peared normal. No relevant signs of cardiac or vascular ture of the esophageal compression: surgery therefore
pathologies could be detected. This is a common finding consisted of the repositioning of the aberrant right sub-
in right lusoria subclavian artery disease. On the con- clavian artery to the aortic arch through a prosthetic
trary, other vascular rings, such as double aortic arch or graft interposition. Postoperatively, the patient experi-
right-sided aortic arch, can be easily suspected on chest enced immediate resolution of her dysphagia.
radiographs (upper mediastinal shadow enlargement).

Suggested Reading
1. Cameron D (2004). Congenital anomalies of the aortic 3. Woods RK et al. Vascular anomalies and tracheoesophageal
arch. In: Gardner TJ, Spray TL (eds) Operative Cardiac Sur- compression: a single institution’s 25-year experience. Ann
gery 5th Edition. Oxford University Press, Oxford, 851–861 Thorac Surg 2001; 72:434–439
2. Kamiya H et al. Surgical treatment of aberrant right subcla-
vian artery (arteria lusoria) aneurysm using three different
methods. Ann Thorac Surg 2006; 82:187–190
5 Cardiovascular Disorders 329

Q 155
Francesco Onorati, Giacomo Sica, and Attilio Renzulli

Fig. 1 Fig. 2

A 6-year-old boy was admitted to hospital complaining


of recurrent pneumonia, progressive dyspnea for mild
to moderate efforts, and fatigue. Case-history findings
revealed the patient was small for gestational age and
his weight at birth was below the 5th percentile. Physical
examination demonstrated a loud continuous murmur,
which was maximal over the pulmonary artery and radi-
ated upward beneath the mid-third of the clavicle. An
electrocardiogram (ECG) demonstrated left ventricular
hypertrophy.
A chest radiograph is shown in Fig. 1.
• What relevant findings on the left edge of the medias-
tinal shadow can be appreciated?
• How do the pulmonary fields appear?
Fig. 3
Thereafter, the patient was scheduled for echocardiogra-
phy, as shown in Fig. 2.
• What is the main finding of the study? The patient then underwent angiography, as shown in
• Which vessel is indicated by the arrow? Fig. 3.
• Which anatomic structures are indicated by the tri- • What was the main indication for aortography?
angle and the star? • What pathological condition is affecting this child?
• Which technique is employed to treat this disease?
330 Cardiovascular Disorders

A 155

Fig. 4 Fig. 5

The patient had a patent ductus arteriosus (PDA). PDA scending thoracic aorta is seen (arrow), together with
is an abnormal persistence of a patent lumen in the fe- the enlarged left pulmonary artery (star). An ascending
tal ductus arteriosus, which usually connects the upper aorta is also detected (triangle). Doppler signaling can
descending aorta with the proximal portion of the left be helpful in demonstrating the PDA and the direc-
pulmonary artery. tion of the shunt (arrow). Pulmonary pressure can be
Symptoms and signs of PDA are due to left-to-right assessed with a noninvasive method through Doppler
shunting, with increased pulmonary flow eventually echocardiography.
leading to pulmonary hypertension. This hemodynamic In newborns, attempts should be made to close the
abnormality is responsible for the recurrent pneumo- PDA with medical treatment (indometacine).
nia and progressive dyspnea reported by the patient. Cardiac catheterization and aortography are no lon-
Chronic pulmonary hypertension leads to biventricular ger indicated in the diagnosis of PDA. However, the pos-
hypertrophy and finally to congestive heart failure. sibility to treat patients with an interventional cardiology
A chest radiograph in PA view (Fig. 1) shows mod- technique through coil-embolization of the PDA makes
erate cardiomegaly, with enlargement of the first (aortic, angiography a useful technique for diagnosing and
arrow) and second (pulmonary artery, star) left arches, treating a PDA in the same session. Aortography (Fig. 3)
and increased pulmonary vascularization. showed the large PDA, contrasting both the descending
In adults, a calcified PDA can sometimes be appreci- thoracic aorta and, in a retrograde fashion, the common
ated. (PA) and the two main pulmonary arteries; furthermore,
Echocardiography was performed to confirm the following percutaneous coil embolization (Figs. 4, 5) the
suspicion of a PDA, as well as to rule out associated car- PDA was successfully treated, with no residual shunting
diac abnormalities. The patient did not have any associ- from the aorta to the pulmonary artery. Surgery can be
ated lesion, but a large PDA was clearly demonstrated performed with very low operative risks on patients not
on the sagittal view of the high parasternal window; in responding to less invasive techniques or on patients
particular, a large PDA continuing into the upper de- with associated cardiac abnormalities.
5 Cardiovascular Disorders 331

Q 156
Francesco Onorati, Giacomo Sica, and Attilio Renzulli

Fig. 1

Fig. 2
Fig. 3
sis and clubbing of the fingers and toes. A systolic mur-
mur in the second intercostal space was found.
An electrocardiogram (ECG) showed moderate right
ventricular hypertrophy.
Blood analysis demonstrated polycythemia, with a
hemoglobin (Hb) value above 20 g/dl, and hyperbiliru-
binemia (3.2 mg/dl).
A chest radiograph is shown in Fig. 1.
• Are the pulmonary fields hypoperfused?
Following chest radiography, the patient underwent
transthoracic echocardiography, as shown in Fig. 2.
Fig. 4 • What does Fig. 2 shows?
Cardiac catheterization and angiography (Figs. 3, 4)
A 13-year-old boy weighing 34 kg and complaining of were then performed.
cyanosis, shortness of breath, and hypoxic spells was re- • Why is cardiac catheterization mandatory before sur-
ferred to our institution. The child frequently resorted to gical therapy?
the “squatting” position (knee–chest position). • What kind of congenital heart disease is affecting this
On physical examination, he showed intense cyano- patient?
• What is the best way to manage this condition?
332 Cardiovascular Disorders

A 156

The patient had tetralogy of Fallot (TOF). TOF is a mal- ization and angiography are useful for planning the sur-
formation characterized by displacement of the infun- gical strategy and ruling out stenoses of the pulmonary
dibular septum and its malalignment, leading to right branches and coronary anomalies (anterior descending
ventricle outflow tract stenosis or atresia, and a large artery running transversely on the right ventricular in-
ventricular septal defect (VSD). Malalignment generates fundibulum), often associated with TOF. Finally, the
a subaortic VSD with an overriding aorta onto the right “stop-flow” technique shows any systemic-to-pulmonary
ventricle. The pulmonary valve is congenitally stenosed. collaterals.
Distal stenoses in the pulmonary arteries can be found. In this case, coronary angiography also demonstrated
Severe obstruction of the right outflow tract leads a circumflex-to-left bronchial artery coronary fistula
to right ventricular hypertrophy. Cyanosis depends on (Fig. 3, arrow). The main pulmonary artery (Fig. 4, cir-
the severity of the pulmonary stenosis and is caused not cle), overriding aorta (double circle), infundibular (tri-
only by the VSD but mainly by the overriding aorta. Cy- angle) and pulmonary valve (arrow) stenosis, as well as
anosis, furthermore, leads to polycythemia and clubbing the misaligned septum (asterisk) are shown.
of fingers and toes. Surgery still remains the only treatment for TOF. The
The chest radiograph showed hypoperfused pulmo- main steps of surgical treatment are: closure of VSD and
nary fields with a normal heart size due to the lack of resection of the parietal band through a right atrial ap-
pulmonary blood flow, congestive heart failure, and a proach; relief of the pulmonary valve stenosis; and as-
decreased pulmonary vascularization. sessment of the pulmonary annulus, main pulmonary
Transthoracic echocardiography clearly showed the artery, and infundibulum by Hegar dilators. In the case
VSD (star) and aortic overriding (arrow) and narrowing of hypoplastic pulmonary valve annulus, a transannular
of the right ventricle infundibulum (triangle) secondary patch should be implanted.
to infundibular hypertrophy in parasternal long axis; In very sick infants with other associated abnormali-
the parasternal short axis depicted a comparable right ties and pulmonary valve atresia, an initial palliative ap-
and left ventricular hypertrophy, due to equalized intra- proach through a modified Blalock–Taussig shunt can
ventricular pressures secondary to the large VSD. be performed.
Although surgery can be performed on the basis of
the echocardiographic findings alone, cardiac catheter-

Suggested Reading
1. Bernardes RJ et al. A comparison of magnetic resonance 3. Karl TR and Brizard CPR (2004). Tetralogy of Fallot. In:
angiography with conventional angiography in the diagno- Gardner TJ, Spray TL (eds) Operative Cardiac Surgery 5th
sis of tetralogy of Fallot. Cardiol Young 2006; 16:281–288 Edition. Oxford University Press, Oxford, 689–705
2. Dorfman AL and Geva T. Magnetic resonance imaging
evaluation of congenital heart disease: conotruncal anoma-
lies. J Cardiovasc Magn Reson 2006; 8:645–659
5 Cardiovascular Disorders 333

Q 157
Francesco Onorati, Giacomo Sica, and Attilio Renzulli

Fig. 1 Fig. 2

A 3-year-old boy was admitted to hospital because of cardiac catheterization and angiography were carried
recurrent episodes of cyanosis leading to unconscious- out.
ness. The patient had a history of asthma treated with • What abnormality of the mediastinal shadow can be
bronchodilators. His parents reported that he had de- detected on the chest radiograph?
veloped stridor during the last year, exacerbated when
the child lay on his back, and noisy breathing during Cardiac catheterization and aortic angiography (Fig. 2)
the night. They also reported recurrent respiratory in- led to the final diagnosis.
fections. Physical examination demonstrated tachypnea • What is the anatomy of the aortic arch on aortogra-
and expiratory wheeze. phy?
Chest radiography (AP, LL projections) was per- • What pathological condition is affecting this child?
formed. Thereafter, according to the radiology findings, • Are CT scanning and MR imaging useful to correctly
diagnose this congenital anomaly?
334 Cardiovascular Disorders

A 157

Fig. 3

The patient had a double aortic arch, a rare congenital


malformation consisting of a normal ascending aorta,
with a normal aortic valve, which divides into two arches
as it leaves the pericardial sac; a left and right aortic arch
can be identified, joining together again posteriorly,
forming a normal descending aorta. This malformation
belongs to the so-called vascular rings, where an arterial
ring surrounds the esophagus, aorta, or both. In this case, Fig. 4
an intraoperative (Fig. 3) examination showed a left aor-
tic arch passing anteriorly and to the left of the trachea
and being joined by the ductus arteriosus (ligated, white tivity and specificity. However, the variety of the lesion
arrow), where it becomes the descending aorta, and a and the complexity of the anatomy generally lead to a
right arch, which passes posteriorly and to the right of full angiographic examination in order to better define
the esophagus to reach the left-sided descending aorta, the anatomy and to rule out any other associated car-
thus forming a complete vascular ring. diac pathology. The study in Fig. 2 clearly defined the
Signs and symptoms are related to the tracheoesoph- anatomy of the malformation with a right (arrow) and
ageal compression. Our patient demonstrated mainly left (triangle) arch, and it confirmed the normal aortic
respiratory symptoms, due to a predominant tracheal valve function.
compression, secondary to a well-developed left arch,
but no esophageal symptoms, due to a less-developed
right arch.
Enlargement of the upper mediastinal shadow on a Suggested Reading
chest radiograph of a child with respiratory difficulties 1. Cameron D (2004). Congenital anomalies of the aortic
or dysphagia should alert one to the likelihood of a vas- arch. In: Gardner TJ, Spray TL (eds) Operative Cardiac Sur-
cular ring. Figure 1 shows an enlarged upper mediastinal gery 5th Edition. Oxford University Press, Oxford, 851–861
shadow (arrow), suggesting the presence of a right-sided 2. Sivanandam S et al. Prenatal diagnosis of conotruncal mal-
aortic arch; the lateral view of this chest radiograph formations: diagnostic accuracy, outcome, chromosomal
demonstrates a narrow tracheal air column (star), sug- abnormalities, and extracardiac anomalies. Am J Perinatol
gesting extrinsic compression of the trachea. 2006; 23:241–245
Cardiac catheterization is no longer mandatory be- 3. Weinberg PM. Aortic arch anomalies. J Cardiovasc Magn
Reson 2006; 8:633–643
cause CT techniques (Fig. 4) have shown good sensi-
5 Cardiovascular Disorders 335

Q 158
Masayuki Fujioka, Carl Muroi, Nadia Khan and Yasuhiro Yonekawa

Fig. 2 Fig. 3

Fig. 1

A 15-year-old boy had a bitemporal headache of sudden • What cranial nerve disturbance should be suspected
onset with subsequent vomiting and consciousness dis- and what mechanism should be considered as a pos-
turbance when he was watching TV at home. sible cause at first?
He was transferred to the emergency room of the
hospital. CT scanning of the head was performed (Fig. 1).
On his arrival at the hospital he was unconscious • What does the CT scan show?
(Glasgow Coma Scale 6) with weak spontaneous respira-
tion and with his right pupil dilated. Cerebral angiography was performed immediately after
the CT scanning.
• What do Figs. 2 and 3 show?
• What is the best way to manage this condition?
336 Cardiovascular Disorders

A 158

Fig. 4 Fig. 5

The right pupil dilatation on arrival at the hospital rep- mia therapy for brain protection were initiated in the in-
resents right oculomotor nerve palsy. If the cause of the tensive care unit after the surgery.
anisocoria is brainstem (midbrain) compression, emer- The CT scans obtained 7 days after the clipping
gency surgery would be required for decompression. showed large low-dense areas in the right cerebellar
The presence of brain herniation, therefore, should be hemisphere and the right cerebral occipital lobe. The de-
investigated first. layed brain infarcts after SAH suggested the presence of
The CT scan (Fig. 1) shows a subarachnoid hemor- cerebral vasospasm.
rhage (SAH) resulting from the rupture of the brain vas- The postoperative cerebral angiography (Figs. 4, 5)
cular structure. The distribution of the SAH seems sym- showed severe vasospasm of the right internal carotid
metrical with a localization in the basal cistern and in artery, the right vertebral artery, the basilar artery, and
the fourth ventricle. The CT scan shows no intracerebral the bilateral posterior cerebral arteries. Intra-arterial in-
hemorrhage leading to brain herniation. Several condi- fusion of papaverine was performed at the same time.
tions (vascular anomaly including cerebral aneurysm, After a 6-month follow-up, this boy is independent
arteriovenous malformation, and venous angioma) in his daily life, although the mild ataxia in the right ex-
should be investigated as the possible cause of the SAH. tremities and the left homonymous hemianopsia remain.
Cerebral angiography (Figs. 2, 3) shows the cerebral The pediatric case of cerebral aneurysm is considered
aneurysms at the top of the basilar artery and at the rare compared to adult cases. In general, multiple aneu-
proximal portion of the right posterior cerebral artery rysms are present in around 20%–30% of adult cases of
(P1 segment). The ruptured aneurysm(s) seemingly SAH. In the pediatric cases, around 10% of patients have
damages the peripheral portion of the oculomotor nerve multiple aneurysms. Hypertension is one of the most
that runs between the posterior cerebral artery and the important factors associated with the multiplicity. In
superior cerebellar artery. our case, the radiological studies (chest radiograph, CT
Aneurysmal clipping surgery (pterional craniotomy) angiography, MR angiography, and aortic angiography)
with ventricular drainage was performed to prevent an- suggested and showed the coarctation of the aorta that is
eurysmal re-rupture. Barbiturate therapy and hypother- known to be associated with cerebral aneurysm.
5 Cardiovascular Disorders 337

Q 159
Masayuki Fujioka, Carl Muroi, Nadia Khan and Yasuhiro Yonekawa

A 9-year-old boy presented with a severe throbbing head-


ache of sudden onset with nausea and projectile vomit-
ing. This was followed by deterioration in consciousness,
reaching a score of 7 on the Glasgow Coma Scale (GCS),
and by seizures. On arrival to our emergency room, he
had a GCS score of 4. A CT scan of his brain showed
an intracerebral hemorrhage in the left frontal lobe with
its breakthrough into the ventricular system. Ventricle
drainage was performed immediately.
Four-vessel cerebral angiography was performed as
shown in Fig. 1.
• What is the cause of hemorrhage?
• What is the therapeutic possibility for this 9-year-old
child?

Fig. 1
338 Cardiovascular Disorders

A 159

The left internal carotid artery angiography (Fig. 1)


shows an aneurysm of 3-mm diameter arising from
the artery of Huebner that originates from the distal
segment of the left A1 portion. This cerebral aneurysm
caused the intracranial hemorrhage.
The cerebral aneurysm was treated with a polyvinyl
alcohol embolization by selective catheterization of the
origin of the Huebner artery (Fig. 2).
Figure 3 shows the follow-up angiography after the
intravascular treatment of the aneurysm.

Suggested Reading
1. Huang J, McGirt MJ, Gailloud P, Tamargo RJ. Intracranial
aneurysms in the pediatric population: case series and lit-
erature review. Surg Neurol 2005; 63:424–323
2. Jain P and Mehta V. Anteriorcommunicating artery aneu- Fig. 2
rysm in a 3-year-old girl. Child’s Nervous System 2002;
18:71–73

Fig. 3
5 Cardiovascular Disorders 339

Q 160
Masayuki Fujioka, Carl Muroi, Nadia Khan and Yasuhiro Yonekawa

Fig. 1 Fig. 2 Fig. 3

A 5-year-old boy presented with a cortical infarct in the • What is the angiographic diagnosis of this patient?
right middle cerebral artery territory. Cerebral angiogra-
phy showed stenotic changes of the bilateral internal ca- The child had no other systemic disease and his general
rotid arteries (ICA) and the right middle cerebral artery examination was normal.
(MCA) (Fig. 1). A vertebral angiogram showed stenosis • What is the clinical diagnosis?
of the posterior cerebral artery (PCA; Fig. 2). • What is the common clinical presentation of this dis-
• What are these typical collateral formations called ease in children?
that appear in the regions of the lenticulostriate ar- • What other single examination is required to plan a
teries (Fig. 1) and of the thalamoperforators (Fig. 2)? cerebral revascularization procedure?

After demonstration of the external carotid circulation


on lateral view, a typical ethmoidal collateral circulation
and a transdural anastomosis are observed in Fig. 3.
340 Cardiovascular Disorders

A 160

Fig. 4 Fig. 5 Fig. 6

Fig. 7

This 5-year-old boy presents the typical collateral vessel Signs and symptoms of repeated cerebral ischemia or
formations usually observed in moyamoya angiopathy. infarcts as seen in Fig. 4 are the most common presenta-
These collateral vascular networks usually develop (a) at tion of this disease in children.
the site of stenosis or occlusion of the ICA (Fig. 1) with A cerebral perfusion scan using H215O PET to dem-
formation of the lenticulostriatal moyamoya or “puff of onstrate the cerebral perfusion reserves (Fig. 5 baseline
smoke” vessels or (b) at the site of PCA stenosis (Fig. 2) and Fig. 6 after a Diamox challenge) is mandatory to tai-
with moyamoya collateral formation in the thalamoper- lor the number, the side (right vs. left), and the location
forators. Transdural anastomoses are also characteristi- (MCA, ACA, or PCA arterial distribution) of the cere-
cally seen when the disease is well advanced (Fig. 3). bral revascularization procedure (Fig. 7).
The angiographic diagnosis is moyamoya angiopathy. This boy underwent left STA–MCA bypass surgery
The clinical diagnosis is moyamoya disease but not (superficial temporal artery–middle cerebral artery) be-
moyamoya syndrome. cause the PET scan showed cerebral hypoperfusion in
the left cerebral hemisphere. He is doing fine at 6 months
after the surgery.
5 Cardiovascular Disorders 341

Q 161
Masayuki Fujioka, Carl Muroi, Nadia Khan and Yasuhiro Yonekawa

Fig. 1 Fig. 2

Fig. 3 Fig. 4

A 10-year-old girl presented with deteriorating neuro- • What is the possible diagnosis?
logical symptoms of increasing frequency of bifrontal • What do the repeat T2-weighted MR images (at
headaches along with periodic temporary weakness of 10 years of age) show (Figs. 1, 2)?
both upper extremities, especially the left side. She had • What is the angiopathy demonstrated on the right
her first headache attack at the age of 5 years. She was and left internal carotid angiograms (AP views,
diagnosed as having hemolytic anemia at the age of Figs. 3, 4)?
4 months due to a rare unstable hemoglobinopathy with • What is the clinical diagnosis of this case?
abnormal oxygen affinity called “Hb Alesha.” MR imag-
ing of her brain at the age of 5 years was normal.
342 Cardiovascular Disorders

A 161

Fig. 5 Fig. 6

This 10-year-old girl was considered to have repeated Figures 5 and 6 also demonstrate the moyamoya col-
transient ischemic attacks in the form of headaches and laterals on the ICA angiograms (lateral views).
transient weakness of the bilateral sides. This case is an example of “moyamoya syndrome.”
The MR images show collateral vessel formations at Moyamoya syndrome represents a typical moyamoya
the level of the basal ganglia bilaterally (Figs. 1, 2). angiopathy accompanying systemic diseases.
The internal carotid angiography (Figs. 3, 4) confirms The patient is doing fine and returned to school
the diagnosis of “moyamoya angiopathy” characterized 6 months after the bilateral STA–MCA anastomosis
by the typical internal carotid artery (ICA) stenosis in surgery.
the supraclinoid segment and by the typical “moyam-
oya collaterals” at the level of the basal ganglia (“puff of
smoke”).

Suggested Reading
1. Khan N, Schuknecht B, Boltshauser E, Capone A, Buck A, 3. Yonekawa Y, Goto Y, Ogata (1992). Moyamoya disease: Di-
Imhof HG, Yonekawa Y. Moyamoya disease and Moyamoya agnosis, Treatment, Recent Achievement: In: Barnett HJM
syndrome: experience in Europe; choice of revascularisation et al. (eds): Storke Pathophysiology, Diagnosis and Man-
procedures. Acta Neurochir (Wien) 2003; 145(12):1061–71 agement Churchill Livingstone, New York, 721–47
2. Yonekawa Y, Taub E. Moyamoya disease: Status 1998. The
Neurologist 1999; 5:13–23
5 Cardiovascular Disorders 343

Q 162
Frédéric Gauthier, Sophie Branchereau, and Chiara Grimaldi

Fig. 1 Fig. 2

A 1.5-year-old girl had a clinical history of prematurity We performed an abdominal color Doppler US that is
(28 gestational weeks) and neonatal bacterial meningitis shown in Fig. 1. The child required a surgical interven-
that needed an umbilical venous catheter. She was re- tion.
ferred to our department for the first time at 8 months of • What do you need to know in order to plan the op-
age for hematemesis and melena. eration?
• How do you manage this patient? • Figure 2 shows the suprahepatic phlebography: how
do you interpret it?
• Which is your surgical management?
344 Cardiovascular Disorders

A 162

In managing this patient, you have to assess the degree of


anemia and the presence of esophageal varices with gas-
troesophageal endoscopy. In this case, we found grade-4
esophageal varices and portal hypertension gastropathy.
In Fig. 1, we can see a portal anechogenic biliary tree
dilation contrasting with the colored blood vessels of the
cavernoma.
When planning surgery, you need to know whether
the intrahepatic portal system and the mesenteric sys-
tem are patent.
Figure 2 shows the right intrahepatic portal system
without communication with the left intrahepatic portal
system.
According to the phlebography findings, it is not pos-
sible to perform a mesorex shunt. Since there was a pat-
ent superior mesenteric vein, we decided to perform a
mesocaval shunt with jugular graft interposition.

Suggested Reading
1. Gauthier-Villars M, Franchi S, Gauthier F, Fabre M, Pari-
ente D, Bernard O. Cholestasis in children with portal vein
obstruction. J Pediatr 2005; 146(4):568–73
2. Valayer J, Hay JM, Gauthier F, Broto J. Shunt surgery for
treatment of portal hypertension in children. World J Surg
1985; 9(2):258–68
5 Cardiovascular Disorders 345

Q 163
Christophe Chardot and Sylviane Hanquinet

Fig. 1 Fig. 2 Fig. 3

An 18-month-old boy, who was born after a normal preg- • The platelet count is 80,000/mm3. How do you in-
nancy and normal delivery, presented with a 2-month terpret this finding? In addition to blood group and
history of repeated episodes of melena. Clinical exami- standard coagulation tests, which other hematologi-
nation showed a baby in good general condition, with cal investigation is needed before considering sur-
pale lips but without jaundice. Significant splenomegaly gery?
(5 cm below the costal margin) was found, without hep- • Apart from the risk of GI bleeding, what main com-
atomegaly. Blood count showed microcytic anemia, with plications may arise in this condition?
a hemoglobin level of 83 g/l. Routine liver test results • How can these potential complications be detected?
(ASAT, ALAT, GGT, bilirubin, INR) were normal. • What is the ideal treatment in such a case? Why?
• What is the most likely diagnosis? • What are the anatomical prerequisites and which in-
• What is abdominal US likely to show? vestigations are needed to assess them?
• Which investigation is most likely to identify the ori- • Figure 1 represents the venous sequence of a superior
gin of the gastrointestinal (GI) bleeding? mesenteric arteriography. How do you interpret it?
• What findings are expected, according to the clinical • Figure 2 represents a retrograde transparenchymal
presentation? portography through the right hepatic vein. How do
• Which additional questions should be asked to the you interpret it?
parents regarding familial history and the neonatal • Where is the recessus of Rex located on this retro-
period, in order to find a cause of the current prob- grade portography (Fig. 3)?
lem? • How do you interpret the postoperative US Doppler
study of the shunt (Fig. 4)?
346 Cardiovascular Disorders

Fig. 4
5 Cardiovascular Disorders 347

A 163

The association of upper GI bleeding, normal liver, and – Bile duct compression by varices in the liver hilum,
significant splenomegaly suggests prehepatic portal hy- causing cholestasis, intrahepatic bile duct dilatation,
pertension whose main cause is portal vein obstruction and liver fibrosis. This complication can be detected
with subsequent portal cavernoma. by liver tests and abdominal US.
An abdominal US is likely to show: – Liver nodules (adenomas or nodular regenerative
– A normal liver (Fig. 5) hyperplasia) due to the modifications of intrahepatic
– Varices of the liver hilum and nonvisualization of the portal flow. This can be detected by abdominal US.
portal vein (Figs. 5, 6) – Pulmonary arteriovenous shunts. This complication
– Increased thickness of the hepatogastric omen- is supposedly due to excessive opening of the pulmo-
tum (more than the aortic diameter) due to varices nary capillaries secondary to the action of intestinal
(Fig. 7) vasoactive substances that are no longer eliminated
– Splenomegaly by the liver and directly arrive to the lungs via por-
– Retroperitoneal varices and possibly spontaneous tosystemic shunts. Pulmonary shunts can be de-
splenorenal shunts tected by oxymetry (hypoxia) and be confirmed by
pulmonary perfusion scintigraphy. Figure 10 shows a
Upper GI endoscopy is required after upper GI bleed- normal scintigram on the left and severe pulmonary
ing in order to identify the origin of the bleeding. En- shunts on the right.
doscopy is likely to show signs of portal hypertension: – Pulmonary hypertension: Intestinal vasoactive sub-
esophageal (Fig. 8) and/or gastric varices (Fig. 9), and/or stances may cause spasm of the small pulmonary
hypertensive gastropathy. arteries. This complication can be detected by echo-
Portal vein obstruction in infancy is due to portal cardiography (signs of pulmonary hypertension) and
vein thrombosis, whose main causes are: infection or confirmed by heart catheterism.
thrombosis of an umbilical catheter; umphalitis; and in- The ideal treatment of portal vein obstruction with
herited prothrombotic coagulopathy. portal cavernoma is to restore a physiological portal
Therefore, the neonatal history of the child should be flow towards the liver. This relieves portal hyperten-
precisely known, and familial cases of thrombotic dis- sion and avoids the long-term complications of por-
eases should be sought. tosystemic shunts. Restoration of a physiologic por-
Thrombocytopenia is due to hypersplenism. In order tal flow may be achieved by shunting the obstructed
to exclude prothrombotic coagulation disorders, a de- portal vein, usually between the superior mesenteric
tailed investigation of coagulation is advisable, includ- vein and the intrahepatic left portal vein (Rex reces-
ing protein C, protein S, antithrombin 3 levels, and the sus) with an internal jugular vein of the patient. The
search for mutations of factors II or V. anatomical prerequisites are:
The main risk of portal vein obstruction is GI bleed- – Normal liver parenchyma without severe fibrosis or
ing. Other complications may arise: cirrhosis (liver biopsy).

Fig. 5 Fig. 6 Fig. 7


348 Cardiovascular Disorders

Fig. 8 Fig. 9 Fig. 10

– Patent intrahepatic portal system: this may be seen renchymography in the right liver, and the retrograde
on US. In cases where US does not show the intra- opacification of a complete intrahepatic portal system
hepatic portal system, its patency has to be assessed (right and left branches opacified). The portal system is
by either hepatic retrograde portography (retrograde slightly hypoplastic, but will re-expand after restoration
opacification of the portal system via hepatic veins of a physiologic portal flow.
and liver parenchyma), percutaneous transhepatic The recessus of Rex is the area where the left portal
portography, or intraoperative portography. vein divides into portal branches to segments 2, 3, and
– Patent intra-abdominal portal system, ideally with 4, and to the umbilical vein. In Fig. 3, the catheter comes
patent splenic vein, superior mesenteric vein, and from the left hepatic vein, with the area of parenchymog-
splenomesenteric confluence. This can be seen on raphy in the left lobe, and opacification of a large reces-
abdominal US Doppler, and if needed on angio-CT sus of Rex connected to the portal branches of segments
scan, MR imaging, or arteriography. 2, 3, and 4 as well as to the left portal vein. The umbilical
– Patent internal jugular veins, allowing retrieval of one vein is not opacified. Figure 10 shows an intraoperative
of them (US). view of the anastomosis between the autologous jugular
vein and the recessus of Rex.
Figure 1 shows a patent mesenteric vein (on the right The US Doppler (Fig. 4) shows a patent shunt, with a
side of the aortic catheter), as well as a splenic vein and satisfactory blood flow: blood velocity is between 15 and
splenomesenteric confluence. The portal vein is ob- 30 cm/s with variations according to respiratory move-
structed. Numerous varices are seen in the liver hilum ments. The liver is normal. Note that after a Rex shunt,
and in front of the stomach. the flow in the left portal vein is reversed.
Figure 2 shows the catheter in the right hepatic vein
(introduced from the right jugular vein), the area of pa-

Suggested Reading
1. de Ville de Goyet J, Alberti D, Clapuyt P, Falchetti D, 2. de Ville de Goyet J, Alberti D, Falchetti D, Rigamonti W,
Rigamonti V, Bax NM et al. Direct bypassing of extrahe- Matricardi L, Clapuyt P et al. Treatment of extrahepatic
patic portal venous obstruction in children: a new tech- portal hypertension in children by mesenteric-to-left por-
nique for combined hepatic portal revascularization and tal vein bypass: a new physiological procedure. Eur J Surg
treatment of extrahepatic portal hypertension. J Pediatr 1999; 165(8):777–81
Surg 1998; 33(4):597–601
5 Cardiovascular Disorders 349

Q 164
Giovanni Esposito, Ciro Esposito, and Giuseppe Amici

Fig. 1

A 10-year-old child, without any previous medical prob- After some more examinations, the diagnosis was
lems, pricked the anterior side of his left thorax with a made and treatment was planned.
nail while playing. After a few hours he began experi- • What does the chest radiograph demonstrate
encing respiratory difficulties with dyspnea and later (Fig. 1)?
retrosternal pain appeared. Chest radiography was per- • What was the suspected diagnosis?
formed, after which the child was admitted to our hos- • Which other examinations were performed and what
pital. On physical examination, he presented dyspnea, a were their results?
fast pulse rate, augmentation of the heart flatness, distant • What was the evolution of the disease?
heart sounds, and subcyanosis of the lips and fingers. • What was the treatment?
Laboratory tests showed a slight anemia (RC 3,800,000 • What was the follow-up?
and Hb 11.3).
350 Cardiovascular Disorders

A 164

The chest radiograph shows an augmentation of cardiac


volume with a classic “fiasco” image.
The suspected diagnosis was pericardial effusion.
An ECG and a pericardiocentesis were carried out.
The ECG showed the presence of a low-voltage complex
like a pericardial effusion. The pericardiocentesis, per-
formed under local anesthesia via the subxiphoid route,
led to drainage of 150 ml of blood.
Based on these results and clinical outcome, the di-
agnosis of traumatic hemopericardium was made. The
condition improved after the first drainage, but there was
a recurrence of symptoms which required new blood
drainage (200 ml). Therefore, surgery was proposed.
The treatment consisted in a left thoracotomy at the
fifth intercostal space through which the pericardium Fig. 2
was opened and 700 ml of blood came out. On heart
examination, no wounds were found and the etiological
hypothesis was of a laceration of the pericardial vessels.
A pericardial flap was resected and a partial suture was
performed leaving a small hole inferiorly.
The postoperative course was regular and the ECG
normalized. Following a radiographic checkup (Fig. 2),
the patient was discharged 7 days after surgery.

Suggested Reading
1. Bowers P, Harris P, Truesdell S, Stewart S. Delayed hemo- 6. Tsang TS, Barnes ME, Hayes SN, Freeman WK, Dearani
pericardium and cardiac tamponade after unrecognized JA, Butler SL, Seward JB. Clinical and echocardiographic
chest trauma. Pediatr Emerg Care 1994 Aug; 10(4):222–4 characteristics of significant pericardial effusions following
2. Cil E, Senkaya I, Tarim O. Delayed hemopericardium due cardiothoracic surgery and outcomes of echo-guided peri-
to trivial chest trauma. Cardiol Young 1998 Jul; 8(3):390–2 cardiocentesis for management: Mayo Clinic experience,
3. Tulzer G, Lechner E, Gitter R. Emergencies in pediatric 1979-1998. Chest 1999 Aug;116(2):322-31
cardiology. Ther Umsch 2001 Feb; 58(2):76–9 7. Palatianos GM, Thurer RJ, Pompeo MQ, Kaiser GA. Clini-
4. Mok GC, Menahem S. Large pericardial effusions of in- cal experience with subxiphoid drainage of pericardial effu-
flammatory origin in childhood. Cardiol Young 2003 sions. Ann Thorac Surg 1989 Sep;48(3):381-5
Apr;13(2):131-6
5. Atar S, Chiu J, Forrester JS, Siegel RJ. Bloody pericardial
effusion in patients with cardiac tamponade: is the cause
cancerous, tuberculous, or iatrogenic in the 1990s? Chest
1999 Dec;116(6):1564-9
5 Cardiovascular Disorders 351

Q 165
Giovanni Esposito, Ciro Esposito and Alessandro Settimi

A full-term 3-month-old infant, born after a normal


pregnancy, presented with splenomegaly. On physi-
cal examination the spleen was palpable in the left iliac
fossa exceeding the xiphoumbilical line. The child had
a pained look, his skin was pale, and the subcutaneous
tissue was scanty. Laboratory tests yielded the following
results: RBC 2,740,000; Hb 7.5;WBC 8,500; PLT 70,000;
glutamyl oxaloacetic transaminase (GOT) 195 U/l;
glutamyl pyruvic transaminase (GPT) 99 U/l; immu-
noglobulin G (IgG) 1,130; IgM 196; IgA 142. TORCH
(Toxoplasma, Rubella, Cytomegalovirus, Herpes), vene-
real disease, and hepatitis markers were negative. Sple-
noportography was performed (Fig. 1), after which the
patient underwent surgical treatment.
• What does the splenoportography show?
• What was the diagnosis and the possible cause?
• Which operation was performed?
• Which examinations were carried out during the fol-
low-up?

Fig. 1
352 Cardiovascular Disorders

A 165

The splenoportography shows the obstruction of the


splenic vein, the patency of the portal vein, and the pres-
ence of a collateral circle that drains the splenic blood in
the portal vein through the gastric veins and the coro-
nary vein of the stomach. Splenomanometry revealed an
endosplenic pressure of 25 cm H2O.
Based on these findings, the suspected diagnosis was
of a thrombosis of the splenic vein probably due to sep-
sis developed during intrauterine or perinatal life caus-
ing splenic vein thrombosis and impairment of hepatic
function (as demonstrated by the anemia, platelet count
decrease, and increase of IgM and GOT).
Splenectomy was performed followed by the implan-
tation of splenic tissue on the right rectum muscle in or-
der to avoid postsplenectomy sepsis.
The postoperative course was uneventful. The follow-
up, which included a clinical examination and labora-
tory tests, demonstrated normal growth of the child. Af-
ter 1 year, a splenic scintiscan with 99mTc-marked red
blood cells (Fig. 2) was carried out and revealed a good
function of the splenic tissue graft.

Fig. 2

Suggested Reading
1. Berk DR, Ahmed A. Portal, splenic, and superior mes- 4. Ikeda M, Sekimoto M, Takiguchi S, Yasui M, Danno K, Fu-
enteric vein thrombosis in a patient with latent essential jie Y, Kitani K, Seki Y, Hata T, Shingai T, Takemasa I, Ike-
thrombocythemia and hyperhomocysteinemia. J Clin Gas- naga M, Yamamoto H, Ohue M, Monden M. Total splenic
troenterol 2006 Mar; 40(3):227–8 vein thrombosis after laparoscopic splenectomy: a possible
2. Gupta N, Sahni V, Singh P, Das A, Kar P. Spontaneous bac- candidate for treatment. Am J Surg 2007 Jan; 193(1):21–5
terial peritonitis in isolated splenic vein thrombosis with 5. Reddy B, Gonzalez-Angulo AM, Kemmerly SA. Splenic vein
portal hypertension. Indian J Gastroenterol 2006 Sep–Oct; thrombosis presenting as Fusobacterium nucleatum bacte-
25(5):263–4 remia. J La State Med Soc 2006 Jul–Aug; 158(4):183–5
3. Hiraiwa K, Morozumi K, Miyazaki H, Sotome K, Furu-
kawa A, Nakamaru M, Tanaka Y, Iri H. Isolated splenic vein
thrombosis secondary to splenic metastasis: a case report.
World J Gastroenterol 2006 Oct 28; 12(40):6561–3
5 Cardiovascular Disorders 353

Q 166
Vincenzo Farina, Claudia Tiano, Selvaggia Lenta, and Luigi Scippa

Fig. 1 Fig. 2

Fig. 4

Fig. 3

A systolic murmur of grade-2 (of 6) intensity was found


in a newborn girl, without any symptoms. Fig. 5
The cardiologist performed the examinations shown
in Figs. 1–3.
• What do Figs. 1–3 show? When the girl was 7 years old the mother reported an
• What pathological condition is affecting this child? episode of severe dyspnea during physical exercise.
• Why did the cardiologist repeat the examinations ap- • Which examination did the doctors decide to per-
proximately every 6 months? form?
• What does Fig. 4 show? • What did they find (Fig. 5)?
• Did the condition change during the follow-up? • What is the best way to manage this condition?
354 Cardiovascular Disorders

A 166

During the first cardiac examination, this child under- sure of the ventricular septal defect. The child contin-
went an ECG and an echocardiography. ued with the follow-up, undergoing periodically ECG,
As you can see in Fig. 1, the ECG shows a left axial Holter-ECG, and echocardiography. At 6 years of age it
deviation and prominent Q waves in the inferior and was decided to start therapy with an ACE inhibitor be-
right precordial leads, with absent left precordial Q cause of dilatation and thickening of the right ventricle
waves. The echocardiography shows corrected transpo- and because of the increase in episodes of Mobitz II A-V
sition of the great arteries (c-TGA): double discordance block.
between the atria and ventricles (Fig. 2) and between the At 7 years of age, on the basis of the mother’s report
ventricles and great arteries with aortic left positioning of an episode of dyspnea occurring during physical exer-
(Fig. 3); a moderate ventricular septal defect; stenosis of cise, the cardiologist decided to repeat the routine exam-
the pulmonary valves; right ventricle dilatation without inations (ECG, Holter-ECG, and echocardiography) and
contractility abnormalities; moderate homologous A-V to also perform an ergometric test. This test, as shown
valve insufficiency. in Fig. 5, revealed a complete A-V block as early as the
c-TGA is a rare congenital cardiac pathology with an third minute.
incidence of less than 1%. In 95% of cases, an associa- Consequently, the doctors decided to evaluate the
tion with other cardiac abnormalities such as ventricu- child with an electrophysiologic study and to consider
lar septal defects, pulmonary valves stenoses, A-V valve her for a pacemaker implantation.
insufficiency and, with longer follow-up, dysrhythmias Many patients with c-TGA have survived to adult-
is reported in the literature. All patients should be seen hood even without surgical correction; however, the dys-
periodically in order to evaluate their clinical, imaging, rhythmias frequently occur with age and are associated
and hemodynamic conditions and to decide the right with deterioration of RV function. Surgical treatment
treatment. should be considered at the earliest sign of RV dysfunc-
Six months after the diagnosis, the child underwent tion. All patients should be seen periodically to evalu-
another cardiac evaluation and, as shown in Fig. 3, ate their condition and decide on the right therapeutic
the ECG revealed a Mobitz I A-V block with Luciani- approach. Patients diagnosed as having c-TGA with or
Wenckebach periodism alternated with AVB 2:1; the without cardiac defects should be followed up carefully
echocardiographic findings did not change. When she during their clinical course to identify and treat different
was 1 year old, we witnessed a complete regression of type of dysrhythmias that can appear anytime and that
the pulmonary valve stenosis and the spontaneous clo- can represent a high risk of death.

Suggested Reading
1. Harska V, Duncan BW, Maver JE Jr, Freed M, del Nido PJ, factors for late mortality after the atrial switch operation:
Jonas RA. Long term outcome of surgical treated patients thirty years of follow-up in 417 patients at a single center.
with corrected transposition of the great arteries. Circula- Circulation 2006 Oct 31; 114(18):1905–13
tion 2005; 10:1987–91 3. Warnes CA. Transposition of the great arteries. Circulation
2. Lange R, Horer J, Kostolny M, Cleuziou J, Vogt M, Busch 2006 Dec 12; 114(24):2699–709
R, Holper K, Meisner H, Hess J, Schreiber C. Presence of a
ventricular septal defect and the Mustard operation are risk
5 Cardiovascular Disorders 355

Q 167
Vincenzo Farina, Claudia Tiano, Selvaggia Lenta, and Luigi Scippa

Fig. 1 Fig. 2

Fig. 3 Fig. 4

A 3-month-old girl was admitted to our department be-


cause of the presence of a holosystolic murmur, of grade-
3 intensity (of 6), at the apex. Physical examination re-
sults and her growth were normal.
The cardiologist performed the examinations shown
in Figs. 1–3.
• What do Figs. 1 and 2 show?
• What does Fig. 3, during follow-up, show?
• Which therapy was started?
• Which other diagnostic examinations need to be per-
formed in this case (Figs. 4, 5)?
• What pathological condition is affecting this child?
• What is the best way to manage this condition? Fig. 5
356 Cardiovascular Disorders

A 167

This child is affected by cardiac rhabdomyoma. In accordance with the oncologist, to better define
Figures 1 and 2 show a highly echogenic well-cir- the mass characteristics, the patient underwent cardiac
cumscribed mass (4 × 2.5 cm) with a posterior intraven- MR imaging (Figs. 4, 5), showing an increase in the
tricular location that leads to distortion of the posterior size of the cardiac mass (5.5 × 4.1cm) with involvement
leaflet of the mitral valve (secondary moderate mitral in- of the cardiac apex and disruption of the right cardiac
sufficiency) with a moderate increase of the blood flow chamber.
across the valve, but not interfering with cardiac output. The patient was affected by isolated cardiac rhabdo-
During follow-up, the ECG (Fig. 3) shows signs of myoma, which, without life-threatening hemodynamic
left ventricular hypertrophy and the echocardiographic instability or arrhythmias, is not an absolute indication
study shows atrial enlargement (20 mm) with mild tri- for surgery.
cuspid insufficiency. There are reports in the literature that large rhabdo-
Despite the absence of clinical signs related to the myomas may significantly regress in size or disappear
pathology of origin, in order to increase the flow across completely. In our patient, the mass increased during
the mitral valve, the patient started therapy with an ACE follow-up and the only possible therapy was excision
inhibitor. of the mass or cardiac transplantation. At the time of
Because of the close association between cardiac writing the girl is 2 years old and surgical intervention
rhabdomyomas and tuberous sclerosis (TS) syndrome is not indicated; however, the child is being monitored
reported in 86% of cases, the patient underwent screen- for early detection of arrhythmias or flow hemodynamic
ing for TS including ophthalmologic and neurological abnormalities.
tests, abdominal ultrasonography, laboratory tests, and
brain CT, which all yielded normal results.

Suggested Reading
1. De Wilde H, Benatar A. Cardiac rhabdomyoma with long- 2. Durairaj M, Mangotra K, Makhale CN, Shinde R, Mehta
term conduction abnormality: Progression from pre-exci- AC, Sathe AS. Cardiac rhabdomyoma in a neonate: appli-
tation to bundle branch block and finally complete heart cation of serial echocardiography. Echocardiography 2006
block. Med Sci Monit 2007 Jan 18; 13(2):CS21–23 Jul; 23(6):510–2
3. Uzun O, Wilson DG, Vujanic GM, Parsons JM, De Giovanni
JV. Cardiac tumours in children. Orphanet J Rare Dis 2007
Mar 1; 2(1):11 [Epub ahead of print]
6
Musculoskeletal
Disorder

Case 168–179
358 Musculoskeletal Disorder

Introduction

During the last 30 years the focus of pediatric ortho- similar to the use of radiographs as a standard radiologi-
pedics has changed considerably as a result of many cal tool following Roentgen’s discovery of x-rays.
important studies. From a situation where diagnosis Arthroscopy has become a high-performance tech-
was founded on clinical expertise, now there is an in- nique that improves the diagnosis and the treatment of
creased use of sophisticated imaging and surgical tech- articular pathologic problems such as meniscus or liga-
niques that require a precise knowledge of the anatomy ment tears and cartilage damage that is idiopathic or
of skeletally immature bone. Important issues faced in secondary to trauma and to changes in blood supply. In
this field include an understanding of the behavior of these applications, the performance of arthroscopy con-
growth cartilage together with how it interacts with the tinues to improve with time, according to the increasing
developing blood supply and the factors that can lead skill of the operator and the technological improvements
to necrosis. of the instruments and surgical tools.
One of the most important steps forward in the treat- In the field of tumoral and tumor-like lesions of bone
ment of pediatric orthopedic diseases was the develop- and soft tissue, important progress has been made in the
ment of noninvasive diagnosis and prophylaxis of one of establishment of the limb-sparing technique for malig-
the most frequent congenital diseases of the musculosk- nant bone tumors to avoid amputation and the associ-
eletal apparatus: developmental displacement of the hip. ated damaging psychological problems. Also, the mini-
As the result of a worldwide ultrasound screening pro- mally invasive radiological embolization technique has
gram, the incidence of serious dislocation and the need been a vital step forward in the treatment of, for exam-
for invasive surgical intervention have been reduced ple, large, benign, but apparently aggressive aneurysmal
considerably. bone cysts.
Another key development in this field has been the Last but not least, it is important to stress that a com-
application of more advanced imaging techniques. Thus, bination of progress made in anesthesia and the wide-
in addition to the use of ultrasonography, MR imaging spread application of minimally invasive surgical tech-
has considerably improved the evaluation of soft tissue niques has lad to a huge improvement of this pediatric
diseases by enhancing the visibility of cartilage. This surgical specialization. It has opened up the real possi-
tissue is clearly of great musculoskeletal interest, but bility to cure any disease of the musculoskeletal system,
previously was not detectable by any other noninvasive be it congenital, acquired, genetic, malformative, infec-
investigative techniques. MR imaging and CT have rev- tious, or posttraumatic, as well as benign and even ma-
olutionized bone and soft tissue investigation in a way lignant tumors of uncertain prognosis.
6 Musculoskeletal Disorder 359

Q 168
Francesco Sadile and Fabrizio Cigala

Fig. 1 Fig. 2 Fig. 3 Fig. 4

A 1-month-old girl was born at term with no clinical at- • What do Figs. 1 (right hip) and 2 (left hip) show at
risk features of orthopedic disease; the Ortolani-Barlow 45 days of age?
test result was normal at birth and 2 weeks later. • Which pathological condition is affecting this child?
Her mother was referred to an orthopedic specialist • Why did the specialist perform the second ultra-
because of a retrieved familial history of developmental sound examination?
displacement of the hip (DDH) not previously reported. • What do Figs. 3 (right hip) and 4 (left hip) show at
An ultrasound examination of both hips was per- 3 months of age?
formed at 45 days of life. • What is the best way to manage this condition?
• What are the results at mid-term follow-up?
360 Musculoskeletal Disorder

A 168

This child is affected by developmental dysplasia of the


right hip (DDH).
Frequently at birth, clinical tests such as the Ortolani-
Barlow click sign and limitation of hip abduction yield
normal results, whereas 1 month later clinical examina-
tion is more effective.
In particular, the clinical evaluation made at birth
should be repeated every 2 weeks to check and to avoid
false-negative or false-positive ultrasonographic find-
ings.
Furthermore, together with a clinical examination, Fig. 5
an anamnestic history of DDH is particularly important
in cross-linked cases difficult to detect in a postpartum
interview: it is recommended that a form is filled in ev-
ery time.
According to Graf ’s ultrasound hip examination
method, Fig. 1 shows a type-IIC hip and Fig. 2 a type-
IIB hip.
Even if the natural history of these cases is benign,
i.e., complete maturation within 3 or 4 months of age, a
different evolution may be seen both clinically and so-
nographically.
In particular, type IIC is classified as at-risk hip with
no abduction device.
Figures 3 and 4 at 3 months of age show an appar-
ently good restoration of the alpha and beta angles ac-
cording to Graf ’s method, but the right hip was clinically
unstable and Ortolani’s maneuver was positive. Fig. 6
In instances of clinical and ultrasonographic dissoci-
ation, it is mandatory to perform a standard or frog-leg
radiography of the pelvis.
Normally, a frog-leg position after birth is advisable
for all newborns so as to facilitate a complete and stable
acetabular maturation; in cases of immature or worse
hips, as in this example, an abduction device is indicated
such as the Milgram type in Fig. 5, which shows, on the
right, a worsening of the right acetabulum, hypopla-
sia of the ossification center, but a full hip centering at
5 months after detection of right hip instability.
The mid-term results of hips treated with abduction
devices are similar to what is shown in Fig. 6, a progres-
sive restoration and maturation of hip parameters, con-
firmed by long-term results shown in Fig. 7.

Fig. 7
6 Musculoskeletal Disorder 361

Q 169
Francesco Sadile and Fabrizio Cigala

Fig. 1 Fig. 2

Fig. 4

Fig. 3

A boy was born at full term, weighing 3,050 kg and At the 5-month follow-up, the right knee was clini-
measuring 48 cm, with no at-risk signs of orthopedic cally normal, whereas the left hip showed a limitation of
disease. abduction and internal rotation.
At 50 days of life, culture of synovial fluid led to the • What do Figs. 1 and 2 show at 5 months?
diagnosis of septic arthritis of the right knee caused by • Is a harness indicated in this case to prevent hip sub-
Candida albicans. Parenteral therapy with Ambisone luxation?
was started for a period of 30 days. • What do Figs. 3 and 4 show at 3 years?
Just before starting this therapy, the pediatric ortho- • What are the results at long-term follow-up?
pedic consultant noticed a painful stiff contralateral hip,
but hip radiographs did not show anything of impor-
tance.
362 Musculoskeletal Disorder

A 169

Fig. 6

mal femoral metaphysis, and partial extrusion from the


cavity with no sign of acetabular dysplasia.
The Milgram abduction harness prevents any other
degree of subluxation within 4 months.
Figures 3 and 4 show an initial ossification of the
proximal femoral head, an enlargement of the proximal
femoral metaphysis, and no sign of acetabular dysplasia.
Figures 5 (at 6 years of age) and 6 (at 11 years of age)
show the positive evolution of skeletal growth of the
lower limbs; in particular, Fig. 5 shows axial normality
of the right knee and Fig. 6 demonstrates the very good
remodeling of the left hip, particularly when compared
with Fig .3.

Suggested Reading
Fig. 5
1. Cigala F, Lotito FM, Sadile F. La terapia ortopedica negli es-
iti di artrite settica. In: Nuove frontiere in pediatria medica
e chirurgica CIC Eds 1986; 149–153
This child is affected by mycotic septic arthritis of the left 2. Hechard P, Carlioz H. Méthode pratique de prévision des
hip and right knee. inégalités de longuer des membres inférieurs. Rev Chir Or-
Frequently at birth, the knee is flexed and somewhat thop 1979; 7:373–375
stiff, whereas the hip should be stable with a range of 3. Nade S. Acute arthritis in infancy and childhood J Bone
motion that tends to improve with skeletal growth. Joint Surg 1983; 65B:234–241
In particular, passive articular motion should not 4. Paterson D. Septic arthritis of hip joint. Orthop Clin North
cause any pain or crying in a newborn with apyretic sub- Am 1978; 9:135–142
cutaneous swelling. 5. Zamzam MM. The role of ultrasound in differentiating sep-
Figures 1 and 2 show a hypoplasia of the proximal tic arthritis from transient synovitis of the hip in children.
femoral center of ossification, an osteolysis of the proxi- J Pediatr Orthop 2006; 15B:418–422
6 Musculoskeletal Disorder 363

Q 170
Francesco Sadile and Fabrizio Cigala

Fig. 1

A 13-year-old girl was referred to an orthopedic institu- • Generally, what is the clinical presentation at the on-
tion suffering from left hip pain 1 year after undergoing set of any hip disease?
a right hip operation not clearly reported to the special- • Which diagnostic examinations are necessary in
ist at the time of the visit. these cases?
At the time of the first episode of right hip pain, the • What does Fig. 1 show?
contralateral side was clinically painless with a complete • Which radiological examinations should be per-
range of motion (ROM) and was radiographically nor- formed?
mal on both views. • Which pathological condition is affecting this child
on the left side?
• What are the results at long-term follow-up?
364 Musculoskeletal Disorder

A 170
This child was affected by slipped capital femoral epiphy-
sis (SCFE) bilaterally with sequential onset of the disease
from right to left.
At the onset of any developmental juvenile hip dis-
ease the clinical signs are (1) stiff hip in flexion (positive
Thomas’s maneuver), (2) limitation of internal rotation,
and (3) limitation of abduction.
Frequently, at the onset of a SCFE, these clinical signs
are not found at the contralateral side and there are no
objective radiographic predictive signs, which in the lit-
erature is known as “so-called contralateral sound hip.”
In particular, a clinical evaluation should be made
more frequently and any variations from normal are to Fig. 2
be considered at-risk signs of subsequent bilateral oc-
currence of SCFE.
At a very early phase of femoral head slipping, as
shown in Fig. 1, an anterior ultrasonographic hip view
can show an initial sign of the posterior slipping of the
femoral proximal epiphysis (note how the arrow—femo-
ral head—slips from up to down).
The first orthopedic surgeon in agreement with part
of the scientific surgical community, preferred to cure
only the hip affected by slipping because there was no
true diagnosis of this condition on the other side yet.
Figures 2 and 3 are radiographs showing the right hip Fig. 3
fixation and the slipping of the contralateral left hip.
Figures 4 and 5 show the contralateral left Knowles
pinning.
Long-term results depend on the severity of the slip-
ping; in mild and moderate cases (<50°), especially in
young patients with a great potential of bone remodeling,
good results occur in the majority; in severe cases (>50°)
the results depend on the type of corrective osteotomy,
such as cervical osteotomy according to Dunn and in-
tertrochanteric according to Southwick’s procedure, and
on the complications, such as avascular head necrosis
and chondrolysis, respectively. Fig. 4

Suggested Reading
1. Bidwell TA, Susan Stott N. Sequential slipped capital femo-
ral epiphysis: who is at risk for a second slip? ANZ J Surg
2006; 76:973–976
2. Cigala F, Sadile F. Southwick’s osteotomy in the treatment of
chronic slipped upper femoral epiphysis. Fixation with AO
blade plate. Italian J Orthop and Traum 1984; 10:461–468
3. Cigala F, Sadile F, Lotito FM, Coppola C. Southwick’s oste-
otomy in the treatment of chronic slipped capital femoral
epiphysis fixed by AO blade-plate at 130°. Long-term re-
sults. Mapfre Medicina 10 Suppl 1999; 1:138–145
Fig. 5
6 Musculoskeletal Disorder 365

Q 171
Francesco Sadile and Fabrizio Cigala

Fig. 1

Fig. 2

An 11-year-old fast-growing boy suffered from occa- • What do Figs. 1 and 2 show?
sional left knee pain for 4 months. • What pathological condition is affecting this child?
In the last month, he complained of limping espe- • Why did the child complain of knee pain initially?
cially after a walk or sports activity. A radiograph of the • What is the best way to manage this condition?
left knee was normal. • What is the natural history of this condition if un-
On hospital admission, a pelvic radiograph was ac- treated?
quired in the AP and “frog leg” view.
366 Musculoskeletal Disorder

A 171

Fig. 4

Fig. 3

Fig. 5 Fig. 6

This child is affected by chronic slipped capital femoral branch of the ischiatic nerve and the medial saphenous
epiphysis of the left hip (SCFE). sensory branch of the femoral nerve.
SCFE can occur in acute or chronic form. In the slight and mild types, the choice of therapy is
Chronic SCFE is classified into slight, mild, and se- cartilage pinning with Knowles rods or similar devices
vere types according to the gravity of the slipping: <30°, such as Kirchner wires, depending on the patient’s age
from 30° to 50°, and >50°, respectively, according to (see Figs. 3, 4).
Southwick’s radiological method. If untreated the head tends to fall step by step medi-
Figures 1 and 2 show an asymmetry of the Klein line ally and posteriorly causing severe deformity of the hip
on the left side in relation to the right one as seen in the and consequently early osteoarthritis. Thus, it is manda-
AP and frog leg views. tory to treat these patients early so as to achieve the best
Knee pain as an expression of early hip pain is ex- results, like those shown in Figs. 5 and 6 at the end of
plained on an anatomical basis as a short circuit medi- treatment.
ated by a full canal by-pass from the obturator sensory
6 Musculoskeletal Disorder 367

Q 172
Francesco Sadile and Fabrizio Cigala

Fig. 2

Fig. 1

A 10-year-old boy suffered from left hip pain for


4 months. He underwent radiography of the left femur
in AP and lateral views.
• What do Figs. 1 and 2 show?
On the same day, the boy fell and was taken to hospital.
• What does Fig. 3 show?
• What should the correct diagnosis have been?
• Which diagnostic procedures should have been initi-
ated? Fig. 3
368 Musculoskeletal Disorder

A 172

Fig. 4 Fig. 5

Fig. 6

Figures 1 and 2 show osteoporosis and cortical erosion have been made 2.5 months earlier. Total-body scintig-
of the proximal femoral metaphysis. raphy, thoracic CT, and MR imaging of the hip would
The correct diagnosis, by examining the radiograph have been required.
in Fig. 3, should have been “pathological fracture at the The child was treated preoperatively according to an
femoral neck.” adjuvant chemotherapeutic protocol.
As shown in Fig. 4, in the emergency room the frac- Subsequently, as shown in Figs. 5 and 6, a proximal
ture was treated percutaneously with Kirchner wires and femoral resection (Fig. 5) was carried out with a custom-
immobilized in a plaster of Paris cast. made prosthesis (Fig. 6).
Once a risk fracture was established at this level, At the 5-year follow-up the child was free of disease.
the procedures that should have been initiated include The prosthesis was working well and showed no lines of
plaster cast immobilization and fine-needle aspiration radiolucency. The limb was about 6 cm short.
or open biopsy. The diagnosis of Ewing sarcoma would

Suggested Reading
1. Campanacci M. Tumori dell’osso e della parti molli. A.Gaggi 3. Iaccarino V, Sadile F, Vetrani A, Fulciniti F, Troncone G,
Ed. Bologna 1990 Riccio R, Misasi N. Percutaneous intralesional brushing of
2. Huvos AG. Bone tumors. Diagnosis treatment and progno- cystic lesions of bone: a technical improvement of diagnos-
sis. W Saunders Ed. Philadelphia 1990 tic cytology. Skeletal Radiology 1990; 19:187–190
4. Mirra JM. Bone tumors. Diagnosis and treatment. JP Lip-
pincott Philadelphia 1991
6 Musculoskeletal Disorder 369

Q 173
Giovanni Esposito and Ciro Esposito

A 7-year-old girl fell down and sustained an injury to


the right arm. She had pain in the upper third of the
right arm and functional impotence. After radiography
(Fig. 1) was performed, she underwent surgery.
• What does the radiograph show?
• What was the diagnosis?
• What was the treatment?
• What did the disease consist in?
• What was the follow-up?

Fig. 1
370 Musculoskeletal Disorder

A 173

The radiograph shows a fracture of the upper third of cally or with a local pain. They occur more frequently in
the humerus over a bone cyst with some displacement the metaphysis of the humerus as in the case illustrated
of the fragments. The cyst resembles an osteolysis with here or in the metaphysis of the femur, tibia, and rarely
cleanly incised margins and a thinning or ballooning in the ribs, ileum, calcaneum, and talus. They may be
cortex, traversed by bone septum. complicated by a fracture, both spontaneous or trau-
The diagnosis was bone cyst complicated by a trau- matic. When a fracture complicates a symptomatic cyst
matic fracture. the pain becomes continuous and it is accompanied by
The treatment comprised surgical curettage of the cyst, the symptoms of the fracture.
reduction of the fracture, and a homologous bone graft. When the cyst is intact, treatment includes curettage
After surgery the arm was immobilized for 4 weeks. of the cavity followed by an intracystic injection of corti-
After removing the plaster, the radiographic checkup sone and by a bone graft or radical subperiosteal excision
revealed that bone union had taken place. After 2 years, of the cyst followed by an autologous or homologous
there were no signs of recurrence. bone graft. When a fracture is present, the treatment is
Unicameral bone cysts are due to atrophic and de- like the one illustrated in our case.
generative phenomena that may present asymptomati-

Suggested Reading
1. Bhatnagar R, Nzegwu NI, Miller NH. Diagnosis and treat- 3. Hart ES, Grottkau BE, Rebello GN, Albright MB. Broken
ment of common fractures in children: femoral shaft frac- bones: common pediatric upper extremity fractures–part
tures and supracondylar humeral fractures. J Surg Orthop II. Orthop Nurs 2006 Sep–Oct; 25(5):311–23
Adv 2006 Spring; 15(1):1–15 4. Scaglietti O, Marchetti PG, Bartolozzi P. The effect of meth-
2. Carmichael KD, Joyner K. Quality of reduction versus tim- ylprednisone acetate in the treatment of bone cysts. Results
ing of surgical intervention for pediatric supracondylar hu- of three years follow-up. J Bone Joint Surg 1979; 61:200-3
merus fractures. Orthopedics 2006 Jul; 29(7):628–32 5. Schroeden B. The results of treatment of simple cysts with
curettage and cryotherapy. J Pediatr Orthop 1996; 47: 27-
30
6 Musculoskeletal Disorder 371

Q 174
Giovanni Esposito, Ciro Esposito, and Alessandro Settimi

A young girl, without any previous medical problems,


suffered from pain in the lower third of the left thigh
with functional impotence of the left leg. In the same
site, a painless, hard, consistent swelling had appeared
2 months earlier. After radiography of the femur (Fig. 1)
and other examinations, she underwent surgery.
• What does the radiograph of the femur demon-
strate?
• Which other examination were necessary?
• What was the diagnosis?
• What was the treatment?
• What was the follow-up?

Fig. 1
372 Musculoskeletal Disorder

A 174

The radiograph shows an osteocartilaginous mass with a


large base and fungus shape in the supracondyle region
of the femur.
The diagnosis was very simple: a huge exostosis of
the femur. Exostosis is a hyperplasia of cartilaginous tis-
sue projecting from the surface of a bone, generally long
ones. It is usually a benign and asymptomatic disease
that can appear with pain accompanied by a mass in the
same region.
When there is an exostosis the entire skeleton should
be examined for other localizations. In our case, there
were two exostoses of the femur (a bigger one in the
medial side, the other in the lateral side of the upper
third of femur), and in addition two other exostoses
were located in the right humerus (Fig. 2). The definitive
diagnosis was multiple exostoses disease. This entity is
characterized by a dominant inheritance and it is more
frequent in males. In general, exostosis is accompanied
by segmentary deformities and growth decrease in the
length of the affected bone.
The treatment consists in removing the exostosis,
because of its potential malignant transformation into Fig. 2
osteochondroma. In our case, we resected only the large
exostosis located in the femur. The small exostosis of the
femur and the exostosis of the humerus were excised
6 months later at another institution.
The postoperative course was uneventful and the
girl was discharged following a radiographic evaluation
10 days after surgery (Fig. 3).

Suggested Reading
1. Alvarez C, Tredwell S, De Vera M, Hayden M. The geno-
type-phenotype correlation of hereditary multiple exosto-
ses. Clin Genet 2006 Aug; 70(2):122–30
2. Darilek S, Wicklund C, Novy D, Scott A, Gambello M,
Johnston D, Hecht J. Hereditary multiple exostosis and
pain. J Pediatr Orthop 2005 May–Jun; 25(3):369–76
3. Porter DE, Lonie L, Fraser M, Dobson-Stone C, Porter
JR, Monaco AP, Simpson AH. Severity of disease and risk
of malignant change in hereditary multiple exostoses. A
genotype-phenotype study. J Bone Joint Surg Br 2004 Sep;
86(7):1041–6
Fig. 3
6 Musculoskeletal Disorder 373

Q 175
Craig T. Albanese

Fig. 1 Fig. 2 Fig. 3

A routine prenatal coronal ultrasound through the fetal


chest and abdomen at 21 weeks’ gestation is presented in
Fig. 1 (abdomen, black arrows; stomach, asterisk; heart,
arrowheads). Color flow Doppler (Fig. 2) demonstrates
a branch of the left hepatic vein (arrowhead) above the
level of the diaphragm (abdomen, black arrows; infra-
diaphragmatic inferior vena cava, arrow; heart, H).
The abnormality was imaged with MR imaging
(Figs. 3, 4).
• What is the diagnosis?

Fig. 4
374 Musculoskeletal Disorder

A 175

The ultrasound demonstrates a left congenital diaphrag-


matic hernia. The left lobe of the liver and the stomach
are herniated into the left hemithorax. Subsequent fe-
tal MR imaging (Fig. 3) with a fluid-sensitive sequence
demonstrates the stomach (asterisk) and colon (arrows)
herniated into the left hemithorax. The heart (arrow-
head) is displaced into the right chest. Fetal MR imaging
(Fig. 4) with an iron-sensitive sequence demonstrates
the left lobe of the liver (asterisk) herniated into the left
hemithorax. The herniated colon is also depicted (ar-
rowheads).
A postnatal chest radiograph (Fig. 5) shows a large
left-side diaphragmatic hernia involving the stomach
(asterisk), liver, and intestines, with secondary medi-
astinal shift to the right. At birth, this child had severe
respiratory failure, required extracorporeal membrane
oxygenation support, and succumbed from pulmonary
hypoplasia and pulmonary hypertension at 16 days of
life.

Fig. 5

Suggested Reading
1. Deprest J, Jani J, Van Schoubroeck D, Cannie M, Gallot D, 3. Jani J, Keller RL, Benachi A, Nicolaides KH, Favre R, Grata-
Dymarkowski S, Fryns JP, Naulaers G, Gratacos E, Nico- cos E, Laudy J, Eisenberg V, Eggink A, Vaast P, Deprest J.
laides K. Current consequences of prenatal diagnosis of Antenatal-CDH-Registry Group. Prenatal prediction of
congenital diaphragmatic hernia. J Pediatr Surg 2006 Feb; survival in isolated left-sided diaphragmatic hernia. Ultra-
41(2):423–30 sound Obstet Gynecol 2006 Jan; 27(1):18–22
2. Dillon E, Renwick M, Wright C. Congenital diaphragmatic
herniation: antenatal detection and outcome. Br J Radiol
2000 Apr; 73(868):360–5
6 Musculoskeletal Disorder 375

Q 176
Giovanni Esposito and Ciro Esposito

An 8-year-old child, without a significant medical his-


tory, fell from a tree. At the emergency unit, a lacerated
wound of the left parieto-occipital region, contusion of
the left shoulder, and concussion were found.
After radiography of the head (Fig. 1), the boy was
transferred to a neurosurgical unit. A few minutes later
the child had several epileptic Jacksonian crises. Conse-
quently, he underwent urgent surgical treatment.
• What did the radiograph show?
• What was the diagnosis?
• What was the treatment?
• What was the follow-up?

Fig. 1
376 Musculoskeletal Disorder

A 176

Fig. 2

This patient had a skull fracture. The radiography showed The treatment consisted in craniotomy with removal
a diastatic fracture of the left parietal bone. of the splinters.
The diagnosis was cranial fracture complicated by a The postoperative course was uneventful with a
compression of the brain due to splinters. Other diag- gradual disappearance of the epileptic crisis and a good
nostic procedures that can be performed in this case are recovery (as shows the EEG in Fig. 2); the child was dis-
a MRJ and an EEG. charged 10 days later.

Suggested Reading
1. Adamsbaum C, Rolland Y, Husson B. Pediatric neuroimag- 3. Fischer B, Wit J. Emergency ward management of traumatic
ing emergencies. J Neuroradiol 2004 Sep; 31(4):272–80 head injury in children. Unfallchirurg 2006 Nov 23
2. Da Dalt L, Marchi AG, Laudizi L, Crichiutti G, Messi G, 4. Jankowitz BT, Adelson PD. Pediatric traumatic brain in-
Pavanello L, Valent F, Barbone F. Predictors of intracranial jury: past, present and future.
injuries in children after blunt head trauma. Eur J Pediatr 5. Dev Neurosci 2006; 28(4–5):264–75
2006 Mar; 165(3):142–8 6. Kim KA, Wang MY, Griffith PM, Summers S, Levy ML.
Analysis of pediatric head injury from falls. Neurosurg Fo-
cus 2000 Jan 15
6 Musculoskeletal Disorder 377

Q 177
Juan A. Tovar

A boy was born to a 31-year-old primiparous woman af-


ter a long, cephalic delivery. He weighed 4.3 kg and did
not require resuscitation.
Immediately after birth, a bulge in his left clavicle
was seen. This area was obviously painful and the baby
cried upon mobilization. He was otherwise normal and
started feeding a few hours after birth.
A plain radiograph of the left clavicle is depicted in
Fig. 1.
• Can you describe the lesion shown in Fig. 1?
• Can you suggest the best form of treatment in these
cases? Fig. 1
• What is the best moment for operation?
• Describe the sequelae that can be expected.
378 Musculoskeletal Disorder

A 177

This baby has a fracture of the clavicle.


This is a rather common lesion after delivery of large
babies, when shoulder dystocia occurs, or in premature
deliveries. However, most cases are seen after normal
deliveries of normal-weight babies.
Figure 1 shows a mid-clavicular fracture with dis-
placement of the fragments and conservation of the cla-
vicular axis.
These lesions do not require specific treatment. Sev-
eral forms of bracing and contentions are advised but,
in fact, none is necessary since this mesenchymal bone
heals rapidly and without sequelae.
In no case is surgery justified.

Suggested Reading
1. Chez RA, Carlan S, Greenberg SL, Spellacy WN. Fractured
clavicle is an unavoidable event. Am J Obstet Gynecol 1994;
171:797–8
2. Gonen R, Spiegel D, Abend M. Is macrosomia predictable,
and are shoulder dystocia and birth trauma preventable?
Obstet Gynecol 1996; 88:526–9
3. Kaplan B, Rabinerson D, Avrech OM, Carmi N, Steinberg
DM, Merlob P. Fracture of the clavicle in the newborn fol-
lowing normal labor and delivery. Int J Gynaecol Obstet
1998; 63:15–20
6 Musculoskeletal Disorder 379

Q 178
Francesco Sadile and Fabrizio Cigala

Fig. 1 Fig. 2

A 9-year-old girl was born at term with no clinical at- • What do Figs. 1 (anteroposterior view) and 2 (lateral
risk signs of orthopedic disease. view) show?
Her parents were referred to our orthopedic institu- • How would you classify this secondary pathological
tion because she suffered indirect trauma in the right condition affecting the patient?
knee during a handball game. • What is the best way to manage this condition?
Figures 1 and 2 show two radiographic planes of the • What are the results at long-term follow-up?
knee examination performed on admission to the emer-
gency unit.
380 Musculoskeletal Disorder

A 178

Fig. 4

Fig. 3

This girl is affected by posttraumatic fracture of the ante- rotated with the posterior aspect still in place, whereas
rior tibial prominence (tibial spine avulsion). type III represents a completely displaced fracture clearly
Figures 1 and 2 show a detachment of the anterior visible on radiographs.
portion of the intercondylar prominence with a com- In our case, the trauma is classified as type III, i.e.,
plete loss of bone continuity of the fracture fragment requiring surgical reduction and fixation to restore the
from the tibial floor. normal biomechanics of the knee.
This particular fracture is categorized as fracture–dis- Normally, in types I and II the gold standard treat-
location of the anterior ligament that requires fracture ment is a conservative plaster cast of the lower limb with
and ligament fixation. The latter produces knee laxity the knee in slight flexion (20°–30°) for 3 or 4 weeks.
characterized clinically by a positive Lachman test (an- Figures 3 and 4 at 3-month follow-up show an appar-
terior tibial translation at almost 15°–20° of passive flex- ently good surgical reduction by an arthroscopically as-
ion) and by anterior subluxation of the lateral condyle at sisted wire-fixation method.
jerk or pivot shift testing. The mid- and long-term results are very satisfactory
According to Meyer’s and McKeever’s classification, from an anatomic as well as a functional point of view.
type I fractures are defined as incomplete or minimally The anterior laxity disappears very quickly, as demon-
displaced spine avulsion, type II is a complete fracture strated by knee testing at follow-up.
6 Musculoskeletal Disorder 381

Q 179
Francesco Sadile and Fabrizio Cigala

Fig. 1 Fig. 2

An 8-year-old boy was referred to our orthopedic insti- • What do Figs. 1 (anteroposterior view) and 2 (lateral
tution because of an indirect trauma on his right wrist view) show?
following a bad tumble during a running game with his • How would you classify this secondary pathological
friends. condition affecting the child?
Figures 1 and 2 show two radiographic planes of • What is the best way to manage this condition?
the elbow examination performed on admission to the • What are the results at long-term follow-up?
emergency unit.
382 Musculoskeletal Disorder

A 179

This boy is affected by fracture of the neck of the right


radius.
Figures 1 and 2 show a complete fracture–dislocation
of the head of the radius.
This particular fracture is categorized as fracture–dis-
location at the elbow requiring surgical open reduction
and fixation.
According to Judet’s classification, type I and II frac-
tures are defined as incomplete or minimally displaced
fracture of the neck of the radius up to 30°, type III is
defined as having more than 30°–60° tilt with a variable
degree of displacement, whereas type IV represents a
completely displaced fracture clearly visible on radio-
graphs.
In our case, the trauma is classified as type IV, i.e.,
requiring surgical reduction and fixation to restore the Fig. 3 Fig. 4
normal elbow anatomy and function.
Normally, in type I a plaster cast is sufficient for com-
plete healing of the trauma; for types II to III the gold
standard method is closed reduction—under general
anesthesia—and a conservative plaster cast of the elbow
for 3–4 weeks with rehabilitation.
Figures 3 and 4 show the postoperative follow-up
with complete good surgical reduction using Kirchner’s
wire fixation method.
The mid- and long-term results are very satisfactory
but generally depend on the amount of growth cartilage
damage (type IV) and blood supply interruption (type
IV), which are not easy to assess just after trauma; in fact,
no method can predict the prognosis of this type of frac-
ture. Accurate analysis of the radiographs and appropri-
ate classification along with the best choice of treatment
(closed or open) contribute to achieving good results
with no necrosis and no limitation of the articular func-
tion of the elbow.

Suggested Reading
1. Weber GB, Brunner Ch, Freuler F. Treatment of fractures
in children and adolescents. Springer-Verlag Berlin Heidel-
berg New York 1980
7
Emergency
and Trauma

Case 180–221
7 Emergency and Trauma 385

Q 180
Juan A. Tovar

Fig. 1

A 7-year-old boy fell from his bicycle on the curb and hit A section of the CT scan with contrast, performed
his lower thorax and upper abdomen. 2 h after the trauma, is depicted in Fig. 1.
He was admitted to hospital with good vital signs, Over the ensuing hours the abdomen became dis-
moderate tachycardia, and bruises in the lower thoracic tended and painful, hemoglobin levels fell to 7g/dl, urine
wall. output decreased, and tachycardia developed.
His blood hemoglobin level was 10 g/dl and blood Blood was given at 25 ml/kg until vital signs stabi-
gases and electrolytes were normal. Serum amylase and lized and urine output was normalized.
glutamic-oxaloacetic-transaminase (GOT) and glutamic- • What does Fig. 1 show?
pyruvic-transaminase (GPT) levels were slightly in- • Are the kidneys normal?
creased. • What additional tests would you order?
He was admitted to the ICU, where a central line and • What would your therapeutic approach be at this
nasogastric and urinary catheters were inserted. stage?
386 Emergency and Trauma

A 180

This child had splenic rupture, a rather common lesion, The symptoms of hemodynamic instability and the
that caused retroperitoneal hemorrhage and moderate decrease in hemoglobin show that bleeding continued
hemoperitoneum. until stabilization.
It is not unusual to have some increase of serum liver The therapeutic approach should be close observa-
and pancreatic enzymes because these organs are also tion and replacement of blood loss. These hemorrhages
traumatized, although mildly. are generally low-pressure ones and subside when intra-
In Fig. 1, the splenic hilum is disrupted and a com- peritoneal pressure increases with hemoperitoneum.
plete separation of both splenic halves is seen. The spleen heals nicely in the vast majority of these
However, parenchymal vascularization remains good cases with full preservation of function.
and the splenic artery is well depicted. Surgery is only indicated when significant blood loss
The pancreas is displaced forward and the kidneys continues (requiring more than 40 ml/kg of blood). In
look normal. such cases, repair of the organ is not always possible and
Additional tests are probably not necessary, except for splenectomy may be unavoidable.
repeated hemoglobin measurements. The splenic lesion Splenectomy is potentially dangerous particularly
can be monitored by ultrasonography. in early childhood because the patient becomes more
prone to suffering OPSI (overwhelming postsplenec-
tomy infection).

Suggested Reading
1. Brown RL, Irish MS, McCabe AJ, Glick PL, Caty MG. Ob- 4. Sjovall A, Hirsch K. Blunt abdominal trauma in children:
servation of splenic trauma: when is a little too much? J Pe- risks of nonoperative treatment. J Pediatr Surg 1997;
diatr Surg 1999; 34:1124–6 32:1169–74
2. Ein SH, Shandling B, Simpson JS, Stephens CA. Nonop- 5. Wesson DE, Filler RM, Ein SH, Shandling B, Simpson JS,
erative management of traumatized spleen in children: how Stephens CA. Ruptured spleen—when to operate? J Pediatr
and why. J Pediatr Surg 1978; 13:117–9 Surg 1981; 16:324–6
3. Schwartz MZ, Kangah R. Splenic injury in children after
blunt trauma: blood transfusion requirements and length
of hospitalization for laparotomy versus observation. J Pe-
diatr Surg 1994; 29:596–8
7 Emergency and Trauma 387

Q 181
Juan A. Tovar

Fig. 1 Fig. 2

Fig. 3 Fig. 4

A 9-year-old girl was involved in a car crash with her Several IV contrast-enhanced CT scan sections are
seat belt attached. shown in Figs. 1–4.
She was admitted to hospital with tachycardia and Over the following 2 days, the abdomen remained
several bruises in various parts of her body. distended and tender, hemoglobin levels fell to 7 g/dl,
Urine output was reduced, blood pressure was stable, urine output was maintained, and tachycardia disap-
and she had severe abdominal pain. No neurological peared.
signs or limb fractures were detected. Blood transfusion was not required.
Hematocrit was 30%, serum glutamic-oxalacetic • What do Figs. 1–4 show?
transaminase (SGOT) and serum glutamate-pyruvate • What would your therapeutic approach be at this
transaminase (SGPT) levels were increased, and total stage?
bilirubin was slightly increased. Amylases were normal.
388 Emergency and Trauma

A 181

This child had liver rupture that caused retroperitoneal Most of these patients stabilize and do not require
hemorrhage and moderate hemoperitoneum. any operative treatment. Hemorrhages are generally low-
It is usual to have increased serum liver enzymes be- pressure ones and the bleeders as well as the bile ducts
cause of the crushed parenchyma and cell damage. are sealed. If this does not happen, bile collections may
Figures 1–4 show that there was no intrapleural blood, be seen in the ensuing days. In this case, percutaneous
that the upper part of the liver was obliquely severed to drainage may suffice.
the left of the inferior vena cava, which was intact, that Surgery in only indicated when significant blood
the wound crossed to the right side along the axis of the continues, when there are associated hollow viscus le-
right portal vein, which was also undamaged, and that sions, and when massive bile leaks occur.
there was blood between the right kidney and the liver. Secondary surgery or percutaneous procedures may
Both kidneys seem intact. be indicated in a few cases for treating persistent bile or
Close monitoring is necessary, and ultrasonography pus collections or hemobilia, but most heal without ma-
is very useful for this purpose. jor operative procedures.

Suggested Reading
1. Ameh EA, Chirdan LB, Nmadu PT. Blunt abdominal 4. Kumar R, Holland AJ, Shi E, Cass DT. Isolated and multi-
trauma in children: epidemiology, management, and man- system hepatic trauma in children: the true role of non-op-
agement problems in a developing country. Pediatr Surg Int erative management. Pediatr Surg Int 2002; 18:98–103
2000; 16:505–9 5. Landau A, van As AB, Numanoglu A, Millar AJ, Rode H.
2. Avanoglu A, Ulman I, Ergun O, et al. Blood transfusion re- Liver injuries in children: the role of selective non-opera-
quirements in children with blunt spleen and liver injuries. tive management. Injury 2006; 37:66–71
Eur J Pediatr Surg 1998; 8:322–5 6. Ozturk H, Dokucu AI, Onen A, Otcu S, Gedik S, Azal OF.
3. Hendren WH, Warshaw AL, Fleischli DJ, Bartlett MK. Non-operative management of isolated solid organ injuries
Traumatic hemobilia: non-operative management with due to blunt abdominal trauma in children: a fifteen-year
healing documented by serial angiography. Ann Surg 1971; experience. Eur J Pediatr Surg 2004; 14:29–34
174:991–3
7 Emergency and Trauma 389

Q 182
Juan A. Tovar

Fig. 1

A 9-year-old boy fell while riding a bicycle. Upon physi- A CT scan was performed and one of the sections is
cal examination, the abdomen was tender and a round shown in Fig. 1.
bruise was visible above the umbilicus. • Describe Fig. 1.
His hematocrit was normal but he had leukocytosis • Describe the different therapeutic options in cases
and high serum amylase and lipase levels. like this one.
Abdominal ultrasonography revealed a thickened • What would your therapeutic approach be at this
body of the pancreas. stage?
He was kept NPO and improved over the next few • In which circumstances would you indicate an opera-
days. Amylase levels decreased but they increased again tion?
on day 8.
390 Emergency and Trauma

A 182

This child had a blunt abdominal trauma. The handlebar Nonoperative treatment is generally successful. Un-
bruise demonstrates that there was severe pressure on der pancreatic rest, the pseudocyst usually regresses and
the organs located between the anterior abdominal wall the lesions heal.
and the spine. Other therapeutic options are puncture and external
Under these circumstances the pancreas is crushed drainage as well as cystogastrostomy via a transgastric,
and the parenchyma and ducts may be disrupted. laparoscopic, or endoscopic approach.
Enzymes are released and the pancreas is inflamed. However, when major disruption of the Wirsung
If ductal lesions are relatively large, exocrine secretion duct occurs, the cyst may be very large and without ten-
accumulates leading to a pseudocyst. dency to heal. In these cases the parenchyma may be
The CT scan section in Fig. 1 depicts one of these severed between the head and the body. More extensive
pseudocysts located in front of the pancreas. The paren- operative treatment may be required in these rare cases.
chyma does not look fractured. Endoscopic retrograde Distal pancreatectomy with preservation of the spleen
cholangiopancreatography (ERCP) or magnetic RCP when possible is an option. Pancreaticojejunal anasto-
may be of interest in theses cases. mosis may help to preserve the entire pancreas in cases
of complete body disruption.

Suggested Reading
1. Bosboom D, Braam AW, Blickman JG, Wijnen RM. The 4. Patty I, Kalaoui M, Al-Shamali M, Al-Hassan F, Al-Naqeeb
role of imaging studies in pancreatic injury due to blunt ab- B. Endoscopic drainage for pancreatic pseudocyst in chil-
dominal trauma in children. Eur J Radiol 2006; 59:3–7 dren. J Pediatr Surg 2001; 36:503–5
2. Canty TG Sr, Weinman D. Management of major pancre- 5. Saad DF, Gow KW, Cabbabe S, Heiss KF, Wulkan ML. Lap-
atic duct injuries in children. J Trauma 2001; 50:1001–7 aroscopic cystogastrostomy for the treatment of pancreatic
3. Canty TG Sr, Weinman D. Treatment of pancreatic duct pseudocysts in children. J Pediatr Surg 2005; 40:e13–7
disruption in children by an endoscopically placed stent. J 6. Stringer MD. Pancreatic trauma in children. Br J Surg 2005;
Pediatr Surg 2001; 36:345–8 92:467–70
7 Emergency and Trauma 391

Q 183
Jean Stephane Valla

Fig. 1 Fig. 2

A 9-year-old boy suffered from sudden pain in the peri-


umbilical area, with subsequent localization of the pain
to the right lower quadrant. The pain was constant and
was made worse by movement. After some hours he
was vomiting and had a low-grade fever. The boy was
referred to us in emergency 24 h after the onset of symp-
toms.
• What is the suspected pathological condition affect-
ing this child?
• What is the most important component of the diag-
nostic work-up?

The surgeon performed the examinations shown in


Figs. 1–3.
• What do Figs. 1–3 show?
• Which other diagnostic examinations are necessary
in this case?
• What is the best way to manage this condition? Fig. 3
392 Emergency and Trauma

A 183

This child is suspected of suffering from acute appendici-


tis because symptoms are typical of this condition. How-
ever, the presentation may be variable, depending on the
patient’s age and other factors. The diagnosis is often dif-
ficult in the following patients: toddlers, adolescent girls,
obese patients, immunosuppressed children etc.
Physical examination is the most important compo-
nent of the diagnostic work-up: localized tenderness in a
child is due to appendicitis unless proved otherwise. De-
spite many advances in imaging and diagnostic modali-
ties, appendicitis remains largely a clinical diagnosis.
Ultrasonographic findings in acute appendicitis in-
clude:
– Appendiceal distension with a diameter greater than Fig. 4
6 mm (Fig. 1)
– “Target sign” (Fig. 2)
– Appendicolith (stercolith in appendiceal lumen) – Urines analysis is useful to rule out pyelonephritis.
(Fig. 3) – Plain thoracic radiography is useful to rule out right
– High periappendiceal echogenicity and peritoneal lung infection.
fluid in the pericecal area or pouch of Douglas – CT could be used if the appendix is not visualized by
ultrasound.
In this case no other diagnostic examinations are nec-
essary; in doubtful cases, the following examinations This boy must be managed by prompt appendectomy us-
could be useful: ing laparoscopy (Fig. 4).
– Blood samples: white blood cells count, neutrophil We favor laparoscopic appendectomy especially in the
count, and C reactive protein are poor sensitive diag- following circumstances: girls, obese patients, suspected
nostic tests in children with symptoms that have been ectopic appendix, uncertain diagnosis.
present for less than 24 h. Transaminase and amylase
levels should be measured to rule out hepatitis or
pancreatitis.

Suggested Reading
1. Arca MJ Caniano DA (2003) In: Mattei P (eds). Paediatric 3. Valla JS. Evidence Based Surgery: Appendectomy by lapa-
Surgery. Lippincott Williams and Williams Philadelphia roscopy in children: better or not? Ped EndoSurg Inn Tech
p. 395–398 2001; Vol 5 p 247–251
2. Doria AS et al. US or CT for diagnosis of appendicitis in
children and adults? A meta analysis. Radiology 2006; 10
1148/RADIO/2411050913
7 Emergency and Trauma 393

Q 184
Jean Stephane Valla

Fig. 2

Fig. 1

A 3-year-old boy was referred to our department in


emergency because of continuous periumbilical pain, fe-
ver (38.5°C), and anorexia that had started 18 h earlier.
There was no vomiting, no constipation, and no diar-
rhea. Abdominal examination was difficult because the
child felt poorly, was in pain, and was frightened. As-
sessing for true local tenderness or rebound tenderness
was inconclusive; the abdomen was slightly distended.
Rectal examination was refused by the family. The rest of
the examination was normal.
• What are findings from the abdominal radiography
(Fig. 1) and the ultrasonography (Figs. 2, 3)?
• How do you manage this young patient?

Fig. 3
394 Emergency and Trauma

A 184

In this case the diagnostic work-up is not clear. The clin-


ical and radiological examinations are not conclusive:
there is simple adynamic ileus on the abdominal radio-
graph (Fig. 1), pericecal fluid (Fig. 2), and hyperecho-
genicity of the right iliac fossa (Fig. 3), but the appendix
is not visualized—a rather common situation in infants.
A period of observation and reexamination will often
solve the problem. This boy was sedated and reexamined
1 h later. Diffuse periumbilical tenderness was evident at
that time, which justified performing a CT scan in emer-
gency, all the more so because of the blood sample re-
sults: white blood cell count (WBC), 29,400; polynuclear
neutrophil (PNF), 89; C-reactive protein (CRP), 153.
The CT scan confirmed the presence of free fluid in
the peritoneal cavity and showed a fecalith located on
the left of the medial line because of the mesoceliac ec- Fig. 4
topic position of the appendix (Figs. 4, 5). The final diag-
nosis is appendicular generalized peritonitis.
The patient must be carefully treated with intrave-
nous infusion, triple antibiotics, and analgesia. When
stable, the child is taken to the operating room. The
procedure can be conducted by laparoscopy. A perito-
neal liquid sample must be taken. The fecalith must be
removed (Fig. 6) as well as the appendix. The abdominal
cavity must be copiously irrigated and aspirated dry. An-
tibiotics are continued for at least 5 days.

Suggested Reading
1. Paya K et al. Perforating appendicitis. An indication of lap-
aroscopy? Surg. Endosc 2000; Vol 14 p 182–4 Fig. 5
2. Peng YS et al. Clinical criteria for diagnosis perforated
appendix in paediatric patients. Pediat Emerg Care 2006;
Vol 22 p 475–9
3. Yakmurlu A et al. Laparoscopic appendectomy for perfo-
rated appendicitis: a comparison with open appendectomy.
Surg Endosc 2006; Vol 20 p1051–4

Fig. 6
7 Emergency and Trauma 395

Q 185
Salam Yazbeck

Fig. 2

Fig. 1

An 8-year-old girl collided with a post while skiing. In


the emergency room she was pale, anxious, and com-
plained of diffuse abdominal pain. Physical examination
confirmed the diffuse abdominal pain with peritoneal ir-
ritation signs. Her heart rate was 120 bpm and her blood
pressure was 105/60. After initial resuscitation she un-
derwent plain abdominal radiography (Fig. 1), abdomi-
nal ultrasound (Fig. 2), and CT of the abdomen (Fig. 3).
• Describe Figs. 1–3.
• What would your management plan be?
• What would be an indication for exploratory lapa-
rotomy in this case?
• What should your diagnosis and management be if
this patient had a massive rectal bleeding episode af-
ter 4–5 days? Fig. 3
396 Emergency and Trauma

A 185

Figure 1 shows a plain abdominal radiograph that is Most patients with liver trauma will not need surgery.
normal without intra-abdominal free air or bowel dila- However, laparotomy is mandatory if the vital signs can-
tation. On ultrasound, a deep laceration in segment 5 not be stabilized or if the patient needs multiple transfu-
of the liver is noted. The CT scan confirms the isolated sions (>40 cc/kg).
liver trauma in the same area. Massive rectal bleeding is a well-described compli-
Management should start immediately with IV fluids cation of liver trauma, whereby an intrahepatic artery
(20 cc/kg) on arrival in the emergency room before any erodes in the biliary tree and the patient presents with
imaging examination. Once vital signs are stable, imag- massive GI bleeding, called hemobilia. This will neces-
ing can be undertaken. The patient should be admitted sitate urgent embolization of the involved artery, and if it
to the intensive care unit for close monitoring of vital is not possible a laparotomy with partial liver resection
signs. will be needed.

Suggested Reading
1. Landau A, Van As AB, Numanoglu A et al. Liver injuries
in children: the role of selective non-operative treatment.
Injury 2006; 37:66–71
2. Leone Re Jr, Hammond JS. Non-operative treatment of pe-
diatric blunt hepatic trauma. Ann Surg 2001; 67:138–142
7 Emergency and Trauma 397

Q 186
Salam Yazbeck

Fig. 1 Fig. 2

A 10-year-old boy was brought to the emergency room a normal platelet count as well as normal liver function
3 h after falling from his bicycle while going downhill. tests. The serum amylase level was slightly elevated.
He hit the handle bar with his abdomen during the fall. The emergency room physician ordered an abdominal ul-
On arrival, he complained of abdominal pain, which in- trasound and a CT scan of the abdomen (Figs. 1 and 2).
creased with walking. He had vomited twice. • What would your management plan be?
On physical examination, his heart rate was 130 bpm, • What is your interpretation of Figs. 1 and 2?
blood pressure was 120/80, and his oxygen saturation • What would you recommend next?
100%. On abdominal examination, there was voluntary • What is the most common complication that can be
guarding, with a slight tenderness in the epigastrium expected in this case?
where a superficial abrasion was noted. • How would you manage this complication?
Laboratory investigations showed a white blood cell
(WBC) count of 22,000, a hemoglobin count of 135, and
398 Emergency and Trauma

A 186

Initial management is to assure stabilization of the vital


signs before any imaging is done.
The ultrasound shows a fracture line in the pancreas
at the level of the superior mesenteric artery with a fluid
collection anteriorly. The CT scan shows a fracture in
the body of the pancreas.
The patient was kept NPO with parenteral nutrition.
Oral feeding was initiated on day 8 but it was not tol-
erated and the patient received nasojejunal gavage. A
follow-up ultrasound examination on day 11 showed a
heterogeneous peripancreatic collection of 6×3×4 cm.
Nasojejunal gavage was continued and the patient was
sent home on day 19.
The boy was readmitted on day 25 with nausea; there
was epigastric tenderness on physical examination. He
tolerated the gavage. Ultrasound showed a pancreatic Fig. 3
pseudocyst that was ready for drainage (Fig. 3).
Percutaneous drainage was done and the amount of
drainage decreased rapidly in a few days. The patient
was sent home on postprocedure day 4. The drain was
removed 10 days later and the patient was followed up
in the outpatient clinic. He resumed a normal oral diet
and underwent a follow-up ultrasound examination
6 weeks post-trauma that showed only a slight scarring
in the pancreas.
Other management possibilities of pancreatic injury
are described in the literature; some are more surgically
aggressive, others demand higher technology. The ap-
proach described here is neither surgically aggressive
nor technologically demanding.

Suggested Reading
1. Bass J, Di Lorenzo M, Desjardins JG et al. Blunt pancre- 3. Wales PW, Shuckett B, Kim PC. Long-term outcome after
atic injuries in children: the role of percutaneous external non-operative management of complete traumatic pancre-
drainage in the treatment of pancreatic pseudocysts. J Pedi- atic transection in children. J Pediatr Surg 2001; 36:823–
atr Surg 1988; 23:721–724 827
2. Canty TG Sr, Weinman D. Management of major pancreatic
duct injuries in children. J Trauma 2001; 50:1001–1007
7 Emergency and Trauma 399

Q 187
Nancy Rollins and Korgun Koral

Fig. 1 Fig. 2

Fig. 4

A 12-year-old girl presented to the emergency room


with a 3-day history of vomiting and a 2-day history of
abdominal pain in the left lower quadrant. The white
blood cell count was elevated (17,300/ml).
• What radiological study should be performed first?
• Which test should be performed next: ultrasonogra-
phy or CT?
• What are the pros and cons for each?
Fig. 3
CT of the abdomen was performed with coronal refor-
mations (Figs. 1–3; bl, urinary bladder; c, cecum).
• What is the diagnosis?
• What does Fig. 4 show?
• What is the thickness threshold for appendicitis on
ultrasonography?
• What are some complications of this condition and
how are they treated?
400 Emergency and Trauma

A 187

Fig. 6

Fig. 5

At many institutions the first examination is supine and responds to a fluid-filled structure and connects to the
upright radiography of the abdomen. This helps identify cecum from its medial aspect. The coronal reformation
other causes of abdominal pain such as gastrointestinal (Fig. 3) shows the same findings. Figures 4 and 5 are im-
obstruction, bowel perforation, appendicolith, and lower ages of another patient showing an appendicolith (arrow
lobe pneumonia/pleural effusion. In about two-thirds of in Fig. 4) in the fluid-filled appendix whose wall is thick-
the cases the presentation is convincing enough, and ened. There is some periappendiceal fluid. Figure 6 is an
without further testing surgery is performed. image from an ultrasonography examination of the right
At many pediatric institutions ultrasonography is the lower quadrant in another patient, showing a blind-end-
test of preference. Lack of ionizing radiation, patient ing tubular structure which is not compressible under
preparation, and portability are advantages of ultraso- manual pressure. The diameter of this tubular structure
nography. Depending on the surgeon’s preference CT is 6 mm. The most commonly used threshold is 6 mm,
may also be used, provided that the technical param- although some use 7 mm.
eters related to radiation are adjusted for the pediatric The most common complication of appendicitis is
population. CT provides the advantage of identifying perforation resulting in local inflammation and abscess
appendicitis in children who have excess body fat, retro- formation. Abscesses can be subhepatic, pelvic (most
cecal appendix, or unrelated intra-abdominal pathology commonly in cul-de-sac), or periappendiceal. In many
such as omental infarct, mesenteric adenitis, renal/ure- instances surgeons prefer percutaneous drainage of the
teral stone, and hydronephrosis. Figures 1 and 2 show abscess and intravenous antibiotic therapy before ap-
a blind-ending tubular structure with thickening and pendectomy.
enhancement of the wall. This tubular structure cor-

Suggested Reading
1. Kuhn JP, Slovis TL, Haller JO. Caffey’s Pediatric Diagnostic 2. Siegel MJ, Coley BD. Pediatric Imaging. Lippincott Wil-
Imaging. Elsevier Ed. 2004 liams & Wilkins Ed. 2006
7 Emergency and Trauma 401

Q 188
Nancy Rollins and Korgun Koral

Fig. 1 Fig. 2

A 15-year-old patient presented with a 4-day history of • What are the findings?
abdominal pain localized to the right lower quadrant. • Is there appendicitis? Can appendicitis be confidently
The white blood cell count was elevated to 12,500/ml. excluded?
Plain radiographs of the abdomen were unremarkable. • What is differential diagnosis?
Contrast-enhanced CT of the abdomen was performed • What is the most likely diagnosis?
(Figs. 1, 2). • What is the cause of this condition and how is it
treated?
402 Emergency and Trauma

A 188

There is a localized focus of fatty stranding forming the The most likely diagnosis is omental infarction.
shape of a mass in the right lower quadrant anterior to The exact cause of omental infarction is not known
the descending colon. An enlarged appendix cannot be but the mechanism involves interruption of arterial
identified. blood supply to the omentum. Definitive diagnosis and
Acute appendicitis cannot be definitively excluded, treatment are achieved by surgery. Conservative man-
because in appendicitis periappendiceal inflammatory agement is occasionally chosen; however, the similarity
changes may obscure the visualization of the appendix. of the clinical presentation and sometimes the imaging
The differential diagnosis includes acute appendicitis, findings to appendicitis makes surgery the usually pre-
epiploic appendagitis, fibrosing sclerosing mesenteritis, ferred treatment.
and pancreatitis.

Suggested Reading
1. Federle MP. Diagnostic Imaging: Abdomen. Amirsys Ed.
2005
7 Emergency and Trauma 403

Q 189
Alba Cruccetti and Luciano Mastroianni

Fig. 1

Fig. 2

A 10-year-old girl presented to our emergency depart-


ment with a 2-day history of abdominal pain and fever.
On examination the abdomen was not distended and
there was no guarding, only a mild tenderness in the in-
ferior abdominal quadrant.
The white blood cell count was normal, with absence
of left shift.
The first radiological investigation performed is
shown in Fig. 1.
• What does Fig. 1 show?
• Why did the surgeon perform the second examina-
tion?
• What do Figs. 2 and 3 show? Fig. 3
• What pathological condition is affecting this child?
• What is the best way to manage this condition?
404 Emergency and Trauma

A 189

This premenarchal girl is affected by a pelvic mass.


The abdominal ultrasound scan shows a pelvic mixed
mass with an inhomogeneous structure (size 3×1.8 cm)
located behind the uterus and attached to the right
ovary (Fig. 1). No blood flow within the mass was de-
tected with color and power Doppler sonography. The
left ovary was not identified.
MR imaging shows a heterogeneous pelvic medial
ovarian mass (size 3.5 × 4 × 2.5 cm) with a predominately
hyperintense signal containing small areas with hypoin-
tense lesions on T1- and T2-weighted images; the right
ovary and the uterus seem to be normal (Figs. 2–4).
Alpha-fetoprotein (αFP) and beta human chorionic
gonadotropin (βHCG) levels were normal. Fig. 4
The diagnosis of left ovarian torsion precipitated by a
large mass was suspected on the basis of the MR imag-
ing and ultrasound scan.
A diagnostic laparoscopy showed the left ovarian
mass to be twisted and attached to the right ovary and
to the uterus (Fig. 5). The left ovarian mass appeared
necrotic after it was untwisted: left oophorectomy and
right oophoropexy were performed.
The final histology showed a mature teratoma with
necrotic areas.
Ovarian pathology is uncommon in children, but it
must be included in the differential diagnosis of all girls
who present with abdominal pain. Ultrasound is usually
the first-line examination performed in an emergency
setting, but CT and MR imaging can be useful in cases
of ambiguous ultrasound findings, especially in patients Fig. 5
with subacute symptoms and a suspected adnexal mass.

Suggested Reading
1. Bader T, Ranner G, Haberlik A. Torsion of a normal adnexa 4. James S. Meyer, Carroll M. Harmon, M. Patricia Harty,
in a premenarcheal girl: MRI findings. Eur Radiol 1996; Richard I. Markowitz, Anne M. Hubbard and Richard D.
6:704–6 Bellah. Ovarian torsion: Clinical and imaging presentation
2. Cass DL. Ovarian torsion. Semin Pediatr Surg 2005; 14:86– in children. J Pediatr Surg 1995; 30:1433–1436
92
3. Cass DL, Hawkins E, Brandt ML, Chintagumpala M, Bloss
RS, Milewicz AL, Minifee PK, Wesson DE, Nuchtern JG.
Surgery for ovarian masses in infants, children, and adoles-
cents: 102 consecutive patients treated in a 15-year period.
J Pediatr Surg 2001; 36:693–9
7 Emergency and Trauma 405

Q 190
Alba Cruccetti and Luciano Mastroianni

Fig. 2

Fig. 1

A 3-month-old girl presented to our emergency depart- The first radiological investigation performed is
ment with a 2-day history of irritability, abdominal pain, shown in Fig. 1.
and vomiting. • What does Fig. 1 show?
On examination, her abdomen was distended with • Why did the surgeon perform the second examina-
diffuse guarding and tenderness on the right lower tion?
quadrant, but no palpable mass. • What does Fig. 2 show?
The white blood cell count was 21,500/mm3, with a • What pathological condition is affecting this child?
left shift. • What is the best way to manage this condition?

Suggested Reading
1. Bakir B, Poyanli A, Yekeler E and Acunas G. Acute torsion 3. Steinberg R, Karmazyn B, Dlugy E, Gelber E, Freud E,
of a wandering spleen: imaging findings. Abdom Imaging Horev G and Zer M. Clinical presentation of wandering
2004; 29:707–709 spleen. J Pediatr Surg 2002; 37:E30
2. Brown CV, Virgilio GR and Vazquez WD. Wandering
spleen and its complications in children: a case series and
review of the literature. J Pediatr Surg 2003; 38:1676–1679
406 Emergency and Trauma

A 190

Fig. 3

Fig. 4

This infant is affected by wandering spleen. Wandering around its pedicle (Fig. 3), and it had no ligamentous
spleen is a rare condition characterized by incomplete attachments. The spleen appeared dusky, but after detor-
fixation of the spleen by its ligaments causing hypermo- sion it returned to a more normal color. Extraperitoneal
bility with migration to the lower abdomen or pelvis. splenopexy was performed, returning the spleen to the
Wandering spleen involves a risk of torsion. Diagnosis left upper quadrant.
is difficult because of the lack of symptoms until splenic The postoperative period was uneventful. The patient
torsion occurs. Patients may present with an asymptom- recovered well, but because of a rising platelet count a
atic abdominal mass, an acute abdomen, or pain associ- liver–spleen scan was performed. This scan showed
ated with a mass. The most common clinical presenta- abnormal splenic uptake (Fig. 4), and the patient was
tion is an acute, chronic, or intermittent abdominal pain treated as an asplenic patient with vaccines and pro-
due to splenic torsion. phylactic antibiotics. A repeat liver–spleen scan several
Figure 1 shows a plain abdominal radiograph with months later showed return of splenic perfusion, and
several markedly distended loops of bowel and absence the patient’s platelet count normalized.
of gas in the rectum, in the right and left lower quad- Wandering spleen is rare, especially in the pediatric
rant. Ultrasonography revealed a medially and lower population, and diagnosis is difficult because of the lack
displaced spleen. A barium enema was normal without of symptoms until splenic torsion occurs. Laboratory
signs of extrinsic compression of the intestinal loops. findings are nonspecific, but the diagnosis can be con-
The CT scan (Fig. 2) shows the enlarged spleen lo- firmed with imaging techniques.
cated in the left lower quadrant and pelvis.
The patient was taken to the operating room for
exploration. At operation the spleen was torsed 360°
7 Emergency and Trauma 407

Q 191
François Luks

Fig. 2

Fig. 1

A 22-day-old infant, born prematurely at 27 weeks’ ges- The following morning, the infant was noted to be tachy-
tation, was breathing spontaneously on supplemental pneic (up to 70 breaths/min) and had a transient episode
oxygen via a nasal cannula, was on intravenous alimen- of hypotension. He was intubated and placed on assisted
tation, and received trophic enteral feeding at 1 ml/h. ventilation.
In the last 48 h, the infant had several episodes of ap- Physical examination revealed increased, diffuse ab-
nea followed by bradycardia, and has passed one blood- dominal tenderness, and abdominal wall changes were
streaked stool. His abdomen was noted to be distended. also noted (Fig. 2).
A nasogastric tube was inserted and an abdominal ra- A repeat radiograph was obtained (Fig. 3). It was de-
diograph was obtained (Fig. 1). cided to proceed with surgical exploration of the abdo-
• What is the differential diagnosis? men.
• What does the radiograph show? • What does the physical examination of the abdomen
• What should further work-up and treatment consist suggest?
of? • Which finding on the repeat radiograph prompted
surgical management?
• What are the therapeutic options?
408 Emergency and Trauma

Fig. 3
7 Emergency and Trauma 409

A 191

Feeding intolerance, increased and/or bloody nasogas- patient, necrosis of most of the colon was found (Fig. 6).
tric aspirates, increased frequency of apnea and brady- The small bowel was normal. A subtotal colectomy was
cardia episodes, bloody stools, and abdominal distension performed, leaving a Hartman’s pouch and a terminal
can all be signs of necrotizing enterocolitis (NEC). The ileostomy.
differential diagnosis includes sepsis, colonic immaturity Other therapeutic options include primary anasto-
(causing abdominal distension, vomiting, and a pseudo- mosis (rarely used) and exteriorization of the diseased
obstructive picture), and midgut volvulus (minimal ab- segment of bowel. The latter approach is used in cases of
dominal distension initially, but the signs of intestinal dire emergency when formal resection and the creation
ischemia seen with advanced volvulus may be mistaken of one or more stomas would dangerously prolong op-
for NEC). The initial radiograph (Fig. 1) shows evidence erative time. It usually requires a second-look operation
of intestinal pneumatosis, characterized by a stippled gas or bed-side resection of the exteriorized loop.
pattern within the intestinal wall (presumably the colon, In the extreme low-birth-weight infant (less than
although it is very difficult to differentiate the large from 750 g), bed-side drainage of the peritoneal cavity has
the small bowel in the neonate). At the hepatic flexure, been advocated as temporizing or even definitive treat-
a head-on view of the bowel shows a “target” pattern of ment. A recently published randomized study showed
gas: a central, luminal gas bubble, with concentric hy- that, for extreme low-birth-weight infants, both ap-
perdense mucosa/submucosa, a radiolucent layer of gas proaches had similar results. The role of peritoneal
bubbles, and a hyperdense muscularis and serosa. drainage in larger infants is more controversial.
NEC is typically divided into: stage I, “suspected”
NEC, without radiographic evidence of pneumatosis or
perforation; stage II, “definite” NEC, characterized by
pneumatosis intestinalis, a fixed, atonic bowel loop on
serial examinations, or portal venous gas; and stage III
(“advanced” NEC), when complications occur (perfora-
tion, abscess) or there is evidence of systemic deteriora-
tion (septic shock, leuko- and thrombocytopenia).
The appearance of the abdominal wall the next
morning suggests significant distress or ischemia of the
underlying bowel. The erythema, with its distribution
along the colonic frame, suggests underlying large bowel
necrosis.
The repeat supine radiograph shows a radiolucent
shadow over the right aspect of the liver, suggesting
free air accumulating to the right of the falciform liga-
ment (Fig. 4, arrows). Detection of free air is difficult in
newborns. An alternative is to obtain a cross-table lat-
eral view of the supine infant: free air will be visible as a
pocket of gas over the liver (Fig. 5, arrows).
Indications for surgical treatment of NEC include
hard signs of intestinal perforation (free air on a radio-
graph) or evidence of intestinal ischemia or necrosis (ab-
dominal wall erythema/abscess, persistence of an atonic
loop on successive radiographs). Clinical deterioration
without hard evidence of intestinal perforation may also
require surgical exploration but, in these cases, the out-
come is dismal regardless of the approach. In the present Fig. 4
410 Emergency and Trauma

Fig. 5 Fig. 6

Suggested Reading
1. Lessin MS, Luks FI, Wesselhoeft CW Jr et al. Peritoneal 4. Schmolzer G, Urlesberger B, Haim M et al. Multi-modal
drainage as definitive treatment for intestinal perforation approach to prophylaxis of necrotizing enterocolitis. J Peri-
in infants with extremely low birth weight (<750 g). J Pedi- natol 2006 Jun; 26(6):342–7
atr Surg 1998 Feb; 33(2):370–2 5. Sharma R, Hudak ML, Tepas JJ 3rd et al. Impact of ges-
2. Moss RL, Dimmitt RA, Barnhart DC et al. Laparotomy tational age on the clinical presentation and surgical out-
versus peritoneal drainage for necrotizing enterocolitis and come of necrotizing enterocolitis. Pediatr Surg Int 2006 Jul;
perforation. N Engl J Med 2006 May 25; 354(21):2275–6 22(7):573–80
3. Pierro A, Hall N. Surgical treatments of infants with necro- 6. Singh M, Owen A, Gull S, Morabito A et al. Surgery for
tizing enterocolitis. Semin Neonatol 2003 Jun; 8(3):223–32 intestinal perforation in preterm neonates: anastomosis vs
stoma. J Pediatr Surg 2006 Apr; 41(4):725–9
7 Emergency and Trauma 411

Q 192
Isabelle Vidal, Marc-David Leclair, and Yves Heloury

Fig. 1 Fig. 2

A 15-year-old boy fell astride parallel bars during gym- visible hematoma. The testicle was not isolated or dif-
nastics training and sustained a blunt trauma of the left ferentiated by palpation.
scrotum. Scrotal ultrasound was performed for the right testis
He had no pelvic injury, no dysuria, and no hematu- (Fig. 1) and the left testis (Fig. 2).
ria. The contralateral testis was normal. • How would you manage this young man?
On admission, he presented with extreme scrotal pain. • What should he be warned about concerning the
Physical examination revealed a boy who had achieved evolution of his testis?
puberty, and a swollen, severely tender scrotum with a
412 Emergency and Trauma

A 192

Surgical exploration found a ruptured albuginea, with be performed when there is suspicion of testicular rup-
partial necrosis of the exteriorized pulp. A minimal re- ture, suspicion of albugineal rupture, expanding hema-
section was performed, and the tunica albuginea was tomas, dislocation refractory to manual reduction, ab-
closed. Pathologic analysis of the pulp did not show any sent or abnormal vascular flow at Doppler sonography,
malignancy. and all serious trauma where clinical and radiological
This boy may have severe blunt trauma of the testicle, findings cannot confirm the absence of these injuries.
after a perineal high-energy crash. In this context, we This boy presenting with a rupture of the albuginea tu-
should not forget to check for the absence of pelvic frac- nica and protrusion of the testicular pulp needs surgical
ture (examination of pelvis mobility) and the absence of exploration as soon as possible. It has been proven that
urethral trauma that could be suspected if he had he- early surgical management can reduce testicular infarc-
maturia. tion and increases the possibilities of conservative treat-
It is difficult to differentiate between hematocele (he- ment.
matoma limited by the tunica vaginalis of the testis) and Operative management involves evacuation of the
intratesticular hematoma contained in the tunica albu- hematocele, debridement of necrotic or devitalized tis-
ginea, the first inner layer of testicle. If there is a hema- sue, copious irrigation, meticulous attention to hemosta-
tocele, on physical examination the testicle might not be sis, and closure of the tunica albuginea. Total orchidec-
palpated inside this collection, with an opaque transillu- tomy is rarely necessary, and conservative management
mination, whereas in an intra-albuginea hematoma, the can be performed for more than 50% of parenchyma
testicle and epididymis can be isolated or differentiated loss. This surgical procedure should be accompanied by
by palpation. good analgesia and antibiotic prophylaxis. For this boy,
Ultrasound provides information for testicular trau- surgical exploration confirmed the albugineal rupture.
ma staging: testis contusion, rupture of the albuginea, or About half of his testicular parenchyma was external-
testicle fracture, with the testicular pulp outside of the ized by this dysjunction, was partly necrotic, and had
albuginea, presenting a high risk of necrosis. Doppler to be removed. There was no other injury of the testis
study allows examination of the vascularization. This ex- or epididymis. Careful hemostasis was done, and the al-
amination showed a heterogeneous echo pattern at the buginea was sutured. Because there was no significant
upper pole of the left testicular parenchyma with loss of hematoma, no drainage was left.
contour definition, corresponding to a rupture of the tu- Progressive testicular atrophy may occur despite a
nica albuginea with protrusion of the testicular pulp. successful repair. A careful follow-up should include clin-
Operative indications for blunt trauma include all ical examination and testicular ultrasound for the first
situations where there is risk of necrosis of the testicular year. If there is no testicular atrophy, most often neither
parenchyma. An explorative surgical procedure should endocrine nor exocrine testicular function is altered.

Suggested Reading
1. Buckley JC, McAninch JW. Use of ultrasonography for
the diagnosis of testicular injuries in blunt scrotal trauma.
J Urol 2006; 175:175–178
2. Hendry WF. Testicular, epididymal and vasal injuries. BJU
International 2000; 86:344–348
7 Emergency and Trauma 413

Q 193
Isabelle Vidal, Marc-David Leclair, and Yves Heloury

Fig. 1 Fig. 2

Fig. 3 Fig. 4

A 7-year-old boy was admitted to hospital for left lumbar • What special slices should you ask for?
and flank trauma. Two hours earlier, he had fallen off • The CT study is shown in Figs. 1–4; how would you
his bicycle and landed flat on his back with high kinetic analyze it?
energy. He presented with pain in the lumbar region on • How would you manage this boy?
palpation, and had massive hematuria. His abdomen
was tender, without guarding, and was less painful than Twenty-four hours later, his hemoglobin rate dropped to
his back. He had stable hemodynamic parameters and 7.9 g/dl; he vomited, had abdominal distension, but no
had no vomiting. abdominal pain.
The patient had normal biological hepatic and pan- • Would you still continue with medical surveillance?
creatic values. The hemoglobin count was 10.7 g/dl 3 h
after the shock and 9.4 g 6 h later. An abdominal CT On day 5 after his accident, he had no fever, no ileus, and
scan was performed. was carefully mobilized. He was discharged on day 7.
• What would your follow-up consist of?
414 Emergency and Trauma

A 193

This boy probably had a left retroperitoneal trauma, with hemorrhage, or segmental infarctions without associ-
at least renal contusion (macroscopic hematuria). One ated lacerations, or expanding subcapsular hematomas
cannot rule out associated abdominal injuries. compressing the kidney. Grade 5 is associated with shat-
Because renal trauma is suspected, you should ask for tered or devascularized kidney, complete laceration or
a contrast-enhanced CT examination. This allows one to thrombus of the main renal artery or vein, or uretero-
view the intraperitoneal organs and renal vasculariza- pelvic avulsions. Grades 1 and 2 are classified as minor
tion, to check that the contralateral kidney is functional, traumas, and grades 3–5 as major traumas. This boy had
and to check whether there is active bleeding. However, a renal injury corresponding to grade 3.
the most important aspect for renal injury is to scan The patient should be hospitalized, with bed rest. His
the urinary tract later, after administration of contrast hemodynamic parameters should be carefully moni-
material, with excretory urography, which allows visu- tored. The only need for surgical exploration on admis-
alization of the urinary tract and shows extravasation of sion would be in the case of vascular injuries.
contrast material. The patient presented with an intestinal ileus, con-
The CT examination showed a significant retroperi- secutive to the significant hemoretroperitoneum. He
toneal perirenal hematoma (Fig. 1) and a renal fracture had no hemodynamic trouble, and did not need surgical
with cortical laceration (Fig. 2). There was no extravasa- management. He had to remain fasting; if he had vom-
tion of contrast material, meaning that there was no uri- ited, a nasogastric stent could have been helpful.
nary extravasation and that this parenchymal laceration With this conservative management, his evolution
did not extend into the collecting system (Figs. 3, 4). was good: intestinal reflex ileus disappeared on the third
Renal injuries were graded by the American Asso- day. Most kidney traumas do not require surgery, even
ciation for the Surgery of Trauma (AAST). Grade 1 cor- in cases of deep parenchymal laceration, unless there are
responds to renal contusions, or nonexpanding subcap- complications (persistent perirenal effusion of urine and
sular hematomas; grade 2 to nonexpanding perinephric blood, abscess, infection, secondary hypertension, renal
hematomas confined to the retroperitoneum, or super- atrophy).
ficial cortical lacerations less than 1 cm in depth with- The patient should undergo ultrasonography 1 month
out collecting system injury; grade 3 to renal lacerations later so as to check the evolution of the trauma and to
greater than 1 cm in depth that do not involve the col- decide whether sports can be resumed. In severe trauma,
lecting system; grade 4 to renal lacerations extending a DMSA scan can be performed 1 year later to obtain
through the kidney into the collecting system, or injuries a prognosis of renal function; blood pressure should be
involving the main renal artery or vein with contained checked once a year.

Suggested Reading
1. Broghammer JA, Langenburg SE, Smith SJ, Santucci RA. 2. Buckley JC, McAninch JW. Pediatric renal injuries: man-
Pediatric blunt renal trauma: Its conservative management agement guidelines from a 25-year experience. J Urol 2004;
and patterns of associated injuries. Urology 2006; 67:823– 172:687–690
827
7 Emergency and Trauma 415

Q 194
Isabelle Vidal, Marc-David Leclair, and Yves Heloury

Fig. 1 Fig. 2

A 9-year-old boy was brought to the emergency depart-


ment after suffering abdominal trauma. He had fallen
on his belly while walking 3 h earlier. He presented with
left lumbar pain and left flank guarding. He had good
hemodynamic constants. He had no medical history be-
fore this accident.
Before arriving at hospital, abdominal ultrasonogra-
phy was performed. This examination showed a major
perirenal effusion, with renal parenchyma contusion.
There was no intraperitoneal injury.
• Would you ask for another radiological examina-
tion?
• Comment on Figs. 1 and 2.
• What management would you advise for this young
boy?
• Figure 3 is a percutaneous contrast study of the left
renal collecting system. Does it help you to define the
exact nature of the renal trauma?

Renal DMSA scintigraphy was performed on the same


day. The left kidney had no function.
• What would you propose?

Fig. 3
416 Emergency and Trauma

A 194

It is surprising that this boy presented with blunt lum- shock. Rupture of the occult hydronephrotic pelvis is
bar trauma after such a mild-energy collision. Usually, a suspected. You should keep in mind that this left kidney
fall from one’s own height does not result in such severe may be nonfunctional, as there is no contrast. There is
pain. We should stay wary and suspect an underlying no vascular dysfunction, since the parenchyma was well
condition that could explain these clinical findings. opacified on arterial time.
Ultrasonography is not precise enough for evalua- Because of this loss of function, we cannot assess
tion of kidney trauma. We should perform contrast-en- whether the perirenal collection is only hematic or if
hanced CT (paying special attention to later views) and there is urine leakage. It was decided to perform per-
urography. The American Association for the Surgery cutaneous nephrostomy, on day 1, to drain the injured
of Trauma (AAST) grading system of kidney injury is pelvis and for opacification of the pelvis.
based on radiological features on CT: severity of paren- The contrast study confirms hydronephrosis, and on
chymal laceration, kidney vascularization, and existence a later study (Fig. 3, arrows), there is significant contrast
of collecting system or renal pedicle injury. material leakage. You can conclude that there is rupture
Figure 1 is a CT scan with injection of contrast ma- of the pelvis. Additionally, a puncture of the left peri-
terial. It is an early view after contrast medium injec- renal effusion was made during this procedure to drain
tion (arterial time): it confirms left kidney trauma, with the hematuric fluid.
major retroperitoneal effusion. Both kidneys are well The diagnosis of ruptured hydronephrosis of a non-
vascularized, and there is no pedicle injury. Figure 2 is functional kidney was proposed. Conservative treatment
a later study (urogram), to check the evacuation of con- is not logical, since there is no renal function to preserve.
trast material by the kidneys. The right kidney has nor- We performed left ureteronephrectomy, via a retroperi-
mal and well-opacified cavities, but the left one shows toneal approach. Intraoperative findings were the same
no excretion. Moreover, the limits of the left pelvis are as before: perirenal urinoma and fracture of the pelvis.
well seen (black arrow): this pelvis is larger than the con- A polar vessel was found, in front of a severe stenosis of
tralateral one. The boy had an undiagnosed left hydrone- the pyeloureteral junction. There was no postoperative
phrosis that explains this severe trauma after such a mild complication.

Suggested Reading
1. Mulligan JM, Cagiannos I, Collins JP, Millward SF. Uretero- 3. Sebastia MC, Rodriguez-Dobao M, Quiroga S, Pallisa E,
pelvic junction disruption secondary to blunt trauma: ex- Marinez-Rodriguez M, Alvarez-Castells A. Renal trauma
cretory phase imaging (delayed films) should help prevent in occult ureteropelvic junction obstruction: CT findings.
a missed diagnosis. J Urol 1998; 159:67–70 Eur Radiol 1999; 9:611–615
2. Rogers CG, Knight V, MacUra KJ, Ziegfield S, Paidas CN,
Mathews RI. High-grade renal injuries in children—is con-
servative management possible? Urology 2004; 64:574–579
7 Emergency and Trauma 417

Q 195
Isabelle Vidal, Guillaume Podevin, Etienne Suply, and Yves Heloury

Fig. 1 Fig. 2

A 9-year-old boy was brought to the emergency depart-


ment after a bicycle accident. He presented with trauma
of the right hemithorax. On clinical examination, he had
a chest wall hematoma and abdominal pain in the right
hypochondrium. There was hemodynamic stability.
• What explorations would you perform after this ex-
amination?
• What does Fig. 1 show?
• What could your approach be? In which conditions?
• What complications could occur in this type of ab-
dominal trauma?

Ten days after this accident, the boy presented with acute
abdominal pain and a fever of 39°C. Biological tests Fig. 3
showed an inflammatory syndrome and an increased
serum bilirubin rate. Ultrasonography findings showed
significant abdominal effusion.
• Which complication can be evoked?
• CT was performed. What do Figs. 2 and 3 show?
• Which diagnosis could be proposed?
• How would you manage this complication?
418 Emergency and Trauma

A 195

This setting could correspond to blunt hepatic trauma. toma) and bile duct injuries leading to bilious collection
The liver is the second most frequently injured organ in (bilioma), or hemobilia.
children with blunt abdominal trauma, after the spleen. All these findings are suggestive of bilious effusion.
The first thing to do is to evaluate trauma severity The CT scan shows a former liver fracture (Fig. 2) and
and look for associated thoracic, retroperitoneal, and massive peritoneal effusion (Fig. 3). Because the pa-
other intraperitoneal injuries. Biological investigations tient’s hemodynamic parameters remained stable, this
(full blood count, liver function test, serum amylase, major intraperitoneal leakage is not hematic, but is more
serum lipase, coagulation test, and pretransfusion test) likely to be bilious. Bile duct injury is the most likely
and thoracoabdominal CT should be performed. CT is complication.
the investigation of choice in hemodynamically stable Antibiotics should be started, and this effusion should
patients, to stage injuries and to make a complete ab- be drained: either by transcutaneous radiological target
dominal evaluation. drainage or through a surgical approach (laparoscopy
Figure 1 is a CT scan showing liver laceration and or laparotomy). Another approach could be endoscopic
contusion between the seventh and eighth segment of retrograde cholangiographic examination and biliary
the liver and a mild peritoneal effusion which is prob- drainage, but in this case, the bile duct injury is far from
ably a hemoperitoneum. There is no spleen injury. the hepatic hilus and is not accessible to endoscopy. The
For children, the most frequent approach is nonop- third approach would be partial hepatectomy, to resect
erative management when there is no hemodynamic hepatic segments where there is bilious injury.
trouble or after successful resuscitation (hemodynamics For this child, we made a US-guided puncture fol-
stability obtained by transfusion inferior to 25–40 ml/ lowed by CT drainage and, because drainage was not
kg/24 h). These patients should be monitored daily with sufficient, a peritoneal washing was performed via lapa-
clinical examinations (fever, pain, hemodynamics), bio- rotomy with drainage by a stent placed in front of the
logical tests (full blood count, liver function test), and bile duct injury. This stent was removed 15 days later,
radiological investigations if complications occur. and the boy had no other postoperative complications.
Possible complications are vascular injuries (such A CT scan performed 1 month later showed cicatriza-
as massive hemorrhage, compressive subcapsular he- tion of the liver.
matoma, arteriovenous fistula, superinfection of hema-

Suggested Reading
1. Almaramhi H, Al-Qahtani AR. Traumatic pediatric bile 3. Landau A, van As AB, Numanoglu A, Millar AJ, Rode H.
duct injury: nonoperative intervention as an alternative to Liver injuries in children: the role of selective non-opera-
surgical intervention. J Pediatr Surg 2006; 41:943–945 tive management. Injury 2006; 37:66–71
2. Hackam DJ, Potoka D, Meza M, Pollock A, Gardner M, 4. Pryor JP, Stafford PW, Nance ML. Severe blunt hepatic
Abrams P, Upperman J, Schall L, Ford H. Utility of radio- trauma in children. J Pediatr Surg 2001; 36:974–9
graphic hepatic injury grade in predicting outcome for
children after blunt abdominal trauma. J Pediatr Surg 2002;
37:386–389
7 Emergency and Trauma 419

Q 196
Isabelle Vidal, Guillaume Podevin, Françoise Schmitt, and Yves Heloury

Fig. 1

An 8-year-old boy was brought to the emergency unit The results of blood tests showed a hemoglobin level of
after a motor vehicle crash. He was secured in the car by 10.0 g/dl, normal liver function tests, and serum amy-
a “two-point” seat belt. lase and lipase rates increased 12-fold (335 UI/l and
On examination, there was abdominal pain, right 1,909 UI/l, respectively).
hypochondrium guarding, abdominal wall hematoma, • What is your diagnosis?
vomiting, and conjunctival paleness. His hemodynamic • Which treatment could you suggest?
parameters stayed stable. There was no sign of trauma of
the limbs or vertebrae.
• Which laboratory tests are needed immediately?
• Which radiological examination would you ask for
first?
• What does Fig. 1 show?
420 Emergency and Trauma

A 196

This boy had a blunt abdominal trauma. There may be


injuries of the intraperitoneal organs and intra-abdomi-
nal bleeding (suspected by the guarding associated with
the pallor).
In these cases, laboratory tests are needed, including
full blood count and the usual preoperative tests to per-
form good resuscitation, liver function tests (AST, ALT,
gamma GT, alkaline phosphatase, and bilirubin), and
pancreatic tests (serum amylase and lipase) in order to
look for trauma of these organs.
In this emergency context, it is better to ask directly
for a full-body CT scan, which is the most accurate exam-
ination for abdominal traumas. It can reveal more than
50% of pancreatic injuries. In addition, it simultaneously Fig. 2
checks the chest, the vertebral spine, and the brain.
Figure 2 is an abdominal CT scan showing a trans-
versal linear area of attenuation in the pancreatic isth- is noninvasive and accurate for the detection of duct in-
mus that reveals a transection of the pancreas (white ar- juries; it could be a good examination to use, if available
row); it is associated with a small collection under the in emergency.
liver and a traumatism of the lower part of the spleen In pancreatic trauma with injury of the major duct,
(black arrow). there are three possible approaches. First, initial medical
The boy suffered from a severe trauma of the pancreas treatment (diet, parenteral nutrition, antibiotics, analge-
with biological repercussions, although there was no cor- sic, and possible drainage of pancreatic collection) may
relation between the severity of the traumatism and the be initiated, with a secondary percutaneous or surgical
initial level of serum amylase and lipase. The CT scan drainage of pseudocysts if they develop, but this often
shows a transection of the gland, which corresponds to a implies a long hospital stay. Second, operative treat-
grade-III lesion in the pancreas injury scale of the Amer- ment includes two options: either drainage of pancre-
ican Association for the Surgery of Trauma (AAST). atic production through a Roux-en-Y loop applied on
CT is not accurate enough in the diagnosis of pancre- the pancreatic fracture, or partial distal pancreatectomy
atic injuries because it does not allow direct visualization when the distal pancreatic segment is small. Third, ERP
of the pancreatic ducts, which is essential for the choice stenting could be a good alternative if it is achievable.
of treatment. The “gold standard” technique is endo- This boy was treated surgically with a Roux-en-Y jeju-
scopic retrograde pancreatography (ERP), which accu- nostomy drainage. Medical treatment was then initiated
rately determines the type and location of duct injury comprising a combination of rest, diet, and octreotide
and may allow direct endoscopic treatment by stenting. for 12 days, after which the child recovered well, without
Magnetic resonance cholangiopancreatography (MRCP) any infection or complication.
7 Emergency and Trauma 421

Q 197
Isabelle Vidal, Guillaume Podevin, Anne Dariel, and Yves Heloury

Fig. 2

Fig. 1

Fig. 3

An 11-year-old boy was involved in a motorbike accident. Soon after his admission, his hemodynamic status be-
On admission, he had left hypochondrium pain without came unstable: tachycardia (130 beats/min), low blood
guarding and hematuria without any lumbar pain. There pressure (60 mmHg/30). Blood transfusion and macro-
was no abdominal bruise or wound. molecule filling were required.
He was hemodynamically stable. Blood test results • What is the best way to manage this event?
were normal (full blood count, liver and pancreatic • After successful management, what preventive mea-
function test). Abdominal US was performed (Fig. 1). sures should be taken?
• Would you ask for another examination?
• What do Figs. 2 and 3 show?
422 Emergency and Trauma

A 197

This boy underwent splenectomy via laparotomy and his tion or devascularization (not shown). There is no other
hemodynamic status became stable. abdominal injury. The injury is grade V according to the
In general, hypochondrium pain is probably due to American Association for Surgery of Trauma grading
splenic trauma, and hematuria suggests kidney trauma. scale: completely shattered spleen and hilar vascular in-
The hemodynamic status must be evaluated: paleness, jury, which devascularizes the spleen. This extravasation
tachycardia or bradycardia, low blood pressure, agita- of contrast material corresponds to active bleeding.
tion, confusion. The patient requires an immediate intervention for
The most important sign to look for is intraperitoneal the hemodynamic instability and for the spleen injury
effusion. The US (Fig. 1) shows intraperitoneal effusion with active bleeding. He should undergo urgent lapa-
between the liver and right kidney, which in this context rotomy for splenectomy to be performed. A total sple-
is probably hematic. nectomy is necessary because the spleen is totally shat-
An abdominal CT scan with and without contrast tered and devascularized. The renal injury is managed
material should be performed to detect any abdominal nonoperatively.
or intestinal solid injuries. CT is more sensitive than US. Close surveillance is required in an intensive care
US is not sensitive for abdominal solid trauma: it can de- unit, followed by a stay in the children’s surgical ward.
tect subcapsular or intraparenchymal hematoma but not Restricted activity after discharge is advised for the first
capsular tear or laceration. few days. Prevention of infections is recommended to
The CT scan (Fig. 2) shows a massive hemoperito- splenectomized patients: pneumococcal vaccine and
neum (A), a devascularized and completely shattered antibiotic prophylaxis based on penicillin. The patient
spleen (B), and (Fig. 3) an active extravasation of contrast should always carry a special card for splenectomized
material in the splenic hilum and parenchyma (C), as patients with him. The parents’ education is important:
well as a left renal laceration without urinary extravasa- they have to consult a doctor when the child has fever.

Suggested Reading
1. Castagnola E, Fioredda F. Prevention of life-threatening 2. Lutz N, Mahboubi S, Nance ML, Stafford PW. The signifi-
infections due to encapsulated bacteria in children with cance of contrast blush on computed tomography in chil-
hyposplenia or asplenia: a brief review of current recom- dren with splenic injuries. J Pediatr Surg 2004; 39:491–494
mendations for practical purposes. Eur J Haematol 2003; 3. Sato M, Yoshii H. Reevaluation of ultrasonography for
71:319–326 solid-organ injury in blunt abdominal trauma. J Ultra-
sound Med 2004; 23:1583–1596
7 Emergency and Trauma 423

Q 198
Isabelle Vidal, Guillaume Podevin, Anne Dariel, and Yves Heloury

Fig. 2

Fig. 1

Fig. 4
Fig. 3

A 13-year-old boy was punched in the abdomen while He was hemodynamically stable. Blood test results
playing with his brother. He immediately had abdominal were normal (full blood count, liver function test, serum
pain, but did not vomit. On admission, he had left hy- lipase). Abdominal US was performed (Figs. 1, 2).
pochondrium pain without guarding or rigidity. There • Would you ask for another examination?
was no abdominal excoriation, bruise or wound, and no • What do Figs. 3 and 4 show?
hematuria. There was no other pertinent sign on initial • The patient remained hemodynamically stable. What
examination. is the best way to manage this abdominal trauma?
• Under certain circumstances this child could un-
dergo urgent laparotomy—which ones?
424 Emergency and Trauma

A 198

The hypochondrium pain is probably due to splenic face area, or a laceration of ≤1 cm parenchymal depth.
trauma, which is the most frequent abdominal trauma Grade II is a subscapular hematoma of 10%–50% sur-
before liver or kidney injury. The hemodynamic status face area, or an intraparenchymal hematoma ≤5 cm in
should be evaluated: paleness, tachycardia or bradycar- diameter, or laceration of 1–3 cm parenchymal depth
dia, low blood pressure, agitation, confusion. that does not involve a trabecular vessel. Grade IV is lac-
The US study shows intraperitoneal effusion in the eration involving the segmental or hilar vessels produc-
pelvis (Fig. 1) and a splenic subcapsular hematoma ing major devascularization (≥25% of spleen). Grade V
without any detection of laceration (Fig. 2). In this con- is a completely shattered spleen or a hilar vascular injury
text, the intraperitoneal effusion is most probably a he- which devascularizes the spleen.
moperitoneum. This patient was managed nonoperatively. Conserva-
An abdominal CT scan with and without contrast tive management of isolated blunt splenic injuries has
material should be performed to detect abdominal solid become widely accepted for hemodynamically stable
injuries and to look for pneumoperitoneum. In addition, children. Treatment and management of children with
CT is used to stage solid organ injuries. splenic trauma can be more conservative than for adults.
The CT scan shows a splenic subcapsular hematoma The child is often monitored in an intensive care unit
(probably ≥50% of surface area), two parenchymal lac- and then in the children’s surgical ward. Bed rest is im-
erations (≥3 cm parenchymal depth) probably involving perative and the patient should remain lying down for a
the trabecular vessels (Fig. 3, A), and a hemoperitoneum specified period depending on the severity of the spleen
around the liver and spleen (Fig. 3, B) and in the pelvis injury and on the clinical evolution.
(Fig. 4, A). There is no devascularization, no active ex- This child could require immediate intervention
travasation of contrast material, and no other abdominal for hemodynamic instability. Secondary hemorrhage
injury. is found in three situations: secondary splenic rupture,
This is a grade-III injury according to the American persistent active bleeding which is originally unknown,
Association for Surgery of Trauma grading scale: sub- or hemorrhagic resurgence of hematoma. This situation
scapular hematoma of ≥50% surface area, and laceration is very rare in children, except when it concerns adoles-
of ≥3 cm parenchymal depth involving the trabecular cents and older children.
vessels. Grade I is a subcapsular hematoma of ≤10% sur-

Suggested Reading
1. Dobremez E, Lefevre Y, Harper L, Rebouissoux L, Lavrand 3. Ozturk H, Dokucu AI, Onen A, Otcu S, Gedik S, Azal OF.
F, Bondonny JM, Vergnes P. Complications occurring dur- Non-operative management of isolated solid organ injuries
ing conservative management of splenic trauma in chil- due to blunt abdominal trauma in children: a fifteen-year
dren. Eur Pediatr Surg 2006; 16:166–170 experience. Eur J Pediatr Surg 2004; 14:29–34
2. Godbole P, Stringer MD. Splenectomy after paediatric
trauma: could more spleens be saved? Ann R Coll Surg
Engl 2002; 84:106–108
7 Emergency and Trauma 425

Q 199
Deepika Nehra, Samuel Rice-Townsend, and Sanjeev Dutta

Fig. 1 Fig. 2

An 18-year-old girl presented with intermittent abdomi- • What is the most notable finding (indicated by the
nal pain, nonbilious emesis, anorexia, and malaise that arrows) on the CT scan?
had lasted several days. • Why is this a concerning finding and what pathologi-
Medical history revealed chronic difficulty with feed- cal condition does it indicate?
ing and frequent emesis as a neonate, progressing to a • Describe how the suspected abnormality typically
pattern of intermittent recurrent episodes of diffuse ab- occurs.
dominal pain associated with nonbilious emesis, charac- • Why is the suspected condition considered a surgical
teristically resolving within days to a week. emergency?
She had a laparoscopic cholecystectomy 3 years ear- • How do patients with the suspected condition typi-
lier for presumed symptomatic cholelithiasis, with no cally present?
report of intra-abdominal disease at that time. • What are the treatment options for such an abnor-
The surgeon ordered a CT scan of the abdomen and mality?
pelvis, shown in Figs. 1 and 2. • Describe the typical surgical management.
426 Emergency and Trauma

A 199

Fig. 4

Fig. 3

A “whirlpool” pattern (arrows) involving a branch of sult in episodes of abdominal pain and vomiting which
the superior mesenteric artery (SMA) is seen on the CT may temporarily resolve only to precipitate a life-threat-
scan. This pattern results from clockwise rotation of the ening crisis later. Older patients have higher complica-
bowel and the superior mesenteric vein (SMV) around tion rates due to delayed diagnosis.
the pedicle of the SMA and is a hallmark of midgut vol- Surgical intervention is the only treatment. All pa-
vulus. In a case of suspected midgut volvulus, the CT tients with volvulus or suspected volvulus should have
scan should also be assessed for signs of bowel ischemia. an operation. During laparotomy, the volvulus is un-
These include bowel wall thickening, mesenteric edema twisted in a counterclockwise fashion. If the SMA ped-
with engorgement of mesenteric vessels, and intramural icle appears narrow, the Ladd procedure is indicated.
air. Free intraperitoneal air indicates bowel perforation. This involves division of the mesenteric (Ladd) bands,
Upper gastrointestinal (UGI) series and US are other placement of the bowel in a position of nonrotation to
useful imaging modalities in the diagnosis of midgut widen the mesenteric base, and appendectomy.
volvulus. This patient was taken for emergent laparotomy for
Typically, volvulus occurs in the setting of malrota- suspected midgut volvulus. At laparotomy no volvulus
tion. The abnormal positioning of the duodenojejunal was apparent, but a highly unusual rotational abnormal-
junction and cecum with malrotation results in a short ity was found. The transverse colon appeared to encircle
bowel mesentery and narrow SMA pedicle. The midgut the mesenteric root in a spiral fashion (Fig. 3).
has a propensity to twist around this narrow base, com- This is an extremely unusual finding. Typically a
pressing the SMA and placing the entire midgut at risk “whirlpool” pattern is highly specific for midgut volvu-
for ischemia and eventual necrosis. lus. In this case, initial attempts at unspiraling the colon
Midgut volvulus classically presents with vomiting from around the root of the mesentery were unsuccess-
and high intestinal obstruction in the neonate. Vomiting ful, and a bowel resection with primary reanastomosis
(classically bilious) occurs in more than 90% of all in- was performed (Fig. 4).
fants with volvulus. In older children, the symptoms are The patient had an uneventful recovery with no fur-
vaguer including failure to thrive, recurrent abdominal ther symptoms.
pain, and chronic diarrhea. Intermittent volvulus can re-
7 Emergency and Trauma 427

Q 200
Deepika Nehra, Samuel Rice-Townsend, and Sanjeev Dutta

Fig. 1 Fig. 2

A 25-week gestational age newborn appeared critically


ill and was emergently intubated for severe respiratory
distress. A nasogastric (NG) tube was passed with dif-
ficulty.
The patient decompensated further and bilious secre-
tions were suctioned from the endotracheal (ET) tube.
The abdomen became distended with ventilation.
Initial laboratory test results were remarkable for a
white blood cell count (WBC) of 40×103 cells/µl with a
left shift (12% bands).
Postintubation chest radiography was performed
(Fig. 1).
• What are the salient findings on the radiograph
(Fig. 1)?
• What are the most likely etiologies for the findings in
Fig. 1?
• What is the most likely underlying etiology in this
particular case?

The patient was taken emergently to the operating room.


• Based on the postoperative chest radiograph (Fig. 2) Fig. 3
and the contrast study (Fig. 3), what procedure was
performed on this child and why was this procedure
deemed most appropriate? • In the setting of this intervention, what important
• What are other management options of the acute complication should be carefully ruled out at the
condition? time of definitive repair?
428 Emergency and Trauma

A 200

The left lateral decubitus film (Fig. 1) is useful in the eval- with esophageal atresia (EA) with distal tracheoesopha-
uation of pneumothorax (PTX) when upright position- geal fistula (TEF). Contrast studies are neither indicated
ing is difficult. The lucency along the right chest base and nor routinely performed to confirm the diagnosis.
right heart border represent PTX and pneumomediasti- The child required emergent occlusion of the TEF
num, respectively. Note the grossly distended stomach. to improve respiratory status, but would probably not
The NG tube is unusually positioned with the tip midline physiologically tolerate a thoracotomy for definitive re-
at T12; normally it should veer left toward the stomach. pair. For this reason, he had emergent placement of a he-
Important causes of PTX in a newborn include pa- moclip at the GE junction to prevent further aspiration
renchymal air leak and perforated viscus. Air leaks are of gastric contents through the fistula. The clip is evident
common in premature newborns on mechanical venti- on the chest radiograph (Fig. 2). A contrast study (Fig. 3)
lation for respiratory distress syndrome (RDS). Esopha- was performed to assess the integrity of the GE junction
geal perforation typically occurs in premature neonates clip and it confirmed the EA and functionality of the clip
after repeat attempts at ET intubation or passage of naso/ in preventing reflux through the fistula.
orogastric tubes. The treatment is typically medical with Other surgical options for the acute management of
broad-spectrum antibiotics, parenteral or gastrostomy a patent fistula include: ligation of the fistula; placement
feedings, and tube thoracostomy for PTX or effusion. of a gastrostomy tube to evacuate the gastric contents
Risk factors for iatrogenic esophageal perforation are: and prevent reflux; and inflation of a Fogarty catheter
prematurity, multiple attempts at intubation or passage within the fistula which is a temporizing measure to
of naso/orogastric tubes, bloody aspirate, and unusual help bridge toward definitive repair.
positioning of naso/orogastric tube. The child’s status improved greatly. He underwent de-
This boy was managed conservatively for suspected finitive repair with thoracotomy for ligation of the TEF
iatrogenic esophageal perforation with needle thora- and repair of the EA at 3 months of age. The clip was re-
cotomy to evacuate the PTX and broad-spectrum anti- moved with careful inspection for stricture at the distal
biotics. esophagus.
The ongoing bilious ET aspirate, gastric distention, At the time of writing, the boy was 4 months old and
and persistent infiltrate on chest radiographs are all had been doing well since discharge. The most likely long-
clues to the underlying etiology. This boy was diagnosed term complication is gastroesophageal reflux disease.

Suggested Reading
1. Konkin DE, O’hali WA, Webber EM, Blair GK. Outcomes 3. Spitz L (2005) Esophageal Atresia and Thracheoesophageal
in esophageal atresia and tracheoesophageal fistula. Journal Malformations. In: Ashcroft KW, Holcomb GW, Murphy
of Pediatric Surgery 2003; 38(12):1726–1729 JP (eds) Pediatric Surgery 4th edition. Elsevier Saunders,
2. Sapin E, Gumpert L, Bonnard A, Carricaburu E, Sava E, Philadelphia, pp 352–370
Contencin P, Helardot PG. Iatrogenic pharyngoesophageal
perforation in premature infants. European Journal of Pe-
diatric Surgery 2000; 10(2):83–87
7 Emergency and Trauma 429

Q 201
Deepika Nehra, Samuel Rice-Townsend, and Sanjeev Dutta

Fig. 1

Fig. 2

A 14-year-old boy underwent laparoscopic cholecystec-


tomy at an outside hospital for chronic cholecystitis. He
presented 1 week later with a 2-day history of jaundice.
He denied any associated pain, nausea, vomiting, fe-
ver, and diarrhea. He also denied prior episodes of jaun-
dice.
• What are important pathological conditions to con-
sider when evaluating a patient with postoperative
jaundice?
• What questions can you ask to narrow the differential
diagnosis?

Liver function tests (LFTs) were ordered: total biliru-


bin 16.3 mg/dl, conjugated bilirubin 9.2 mg/dl, aspar- Fig. 3
tate transaminase (AST) 127 U/l, alanine transaminase
(ALT) 387 U/l, alkaline phosphatase (AP) 367 U/l.
• What do these results indicate?
• Based on the transhepatic cholangiogram (PTC)
US was performed to evaluate the biliary system. The shown in Fig. 2, what complication does this child
probe was applied to the mid abdomen and angled to- most likely have? What do the arrows correspond to?
ward the right side. • As evidenced by the finding on the CT scan (Fig. 3),
• What does the US image (Fig. 1) demonstrate? how is this patient’s condition initially managed?
• What are the common complications following lapa- • What are treatment principles and options in a pa-
roscopic cholecystectomy? tient with iatrogenic bile duct injury?
430 Emergency and Trauma

A 201
Important etiologies to consider in a postoperative pa-
tient with jaundice include increased bilirubin produc-
tion, cholestasis, hepatocellular (HC) disease, and preex-
isting conditions. A resorbing hematoma, drug-induced
hemolysis, or blood transfusions can increase bilirubin
production. Benign postoperative cholestasis and sepsis
can lead to intrahepatic cholestasis in a postoperative
patient. In contrast, extrahepatic cholestasis is usually
secondary to common bile duct stones, cholecystitis,
pancreatitis, or bile duct stricture or injury. HC injury
may result from ischemia, sepsis, anesthetics, TPN, or
viral hepatitis. Preexisting conditions, like Gilbert’s or
Dubin–Johnson syndromes, should also be considered.
Urine color? When jaundice is due to elevated con-
jugated bilirubin, the urine turns a tea color as the wa-
ter-soluble conjugated form is excreted. Unconjugated
bilirubin is not water-soluble and does not affect urine Fig. 4
color. Stool color? When jaundice is a result of complete
biliary obstruction, the patient’s stools are clay-colored and posterior branch of the right hepatic duct, suggest-
or acholic. ing injury to the right hepatic duct (Fig. 4).
The patient had a conjugated hyperbilirubinemia Important principles in the treatment of a BDI in-
which is indicative of an obstructive pattern. The el- jury are: (a) controlling sepsis and any ongoing bile leak,
evated transaminases suggest HC damage. (b) relieving biliary obstruction, (c) delineating biliary
The ultrasound (Fig. 1) is remarkable for a dilated in- anatomy with further imaging, and (d) planning surgery.
trahepatic duct and normal-caliber extrahepatic duct. In this case, the patient was not septic and there was no
Complications from laparoscopic cholecystectomy obvious biloma to drain. Initial imaging includes US
are rare. The most important are major bleeding, wound and endoscopic retrograde cholangiopancreatography
infection, bile leak, and bile duct injury (BDI). The (ERCP). Bile leaks are usually amenable to endoscopic
most common pattern of BDI results from misidentifi- management with placement of a biliary stent. A percu-
cation of the common bile duct (or right hepatic duct) taneous transhepatic drain can relieve biliary obstruc-
for the cystic duct followed by deliberate clipping of the tion (Fig. 3). The US and PTC further delineate biliary
misidentified duct. Other complications include bowel anatomy and identify the specific location of the injury.
perforation, liver laceration, and abscess formation from Definitive surgical repair usually necessitates a Roux-
spillage. en-Y hepaticojejunostomy.
This patient has postoperative cholestasis from iatro- This boy underwent a Roux-en-Y repair with post-
genic injury to the biliary tree at the porta hepatis. The operative resolution of his jaundice and bile flow ob-
transhepatic cholangiogram shows a dilated anterior struction.

Suggested Reading
1. Connor S, Garden OJ. Bile duct injury in the era of lapa- 4. Way LW, Stewart L, Gantert W, Liu K, Lee CM, Whang K,
roscopic cholecystectomy. The British journal of surgery Hunter JG. Causes and prevention of laparoscopic bile duct
2006; 93(2):158–168 injuries: analysis of 252 cases from a human factors and
2. Faust TW, Reddy KR. Postoperative jaundice. Clinics in cognitive psychology perspective. Annals of surgery 2003;
liver disease 2004; 8(1):151–166 237(4):460–469
3. Sicklick JK, Camp MS, Lillemoe KD, Melton GB, Yeo CJ,
Campbell KA, Talamini MA, Pitt HA, Coleman J, Sauter
PA, Cameron JL. Surgical management of bile duct inju-
ries sustained during laparoscopic cholecystectomy: peri-
operative results in 200 patients. Annals of surgery 2005;
241(5):786–792
7 Emergency and Trauma 431

Q 202
Deepika Nehra, Samuel Rice-Townsend, and Sanjeev Dutta

Fig. 1

Fig. 2

Baby A was born by normal spontaneous vaginal deliv- feeding and had only a minimal green smear for stool.
ery (NSVD) after an uncomplicated pregnancy. Shortly She presented to the emergency department and an ab-
after delivery the child developed respiratory distress, re- dominal radiograph was acquired (Fig. 2).
quiring supplemental oxygen, and abdominal distention. • What are the salient findings in Figs.1 and 2?
An initial abdominal radiograph was acquired (Fig. 1). • What potential etiologies can explain these findings?
Baby B was born by NSVD after an uncomplicated • What is the main difference between Fig. 1 and 2?
pregnancy. She had an uneventful stay in the nursery • What is the most likely diagnosis in each case?
and was discharged on day-of-life 2 after she had passed • What imaging study can help confirm the diagnosis
meconium and was tolerating feeds. About 24 h later, in each case?
she had considerable dark yellow emesis associated with • How would you manage Baby A? Baby B?
432 Emergency and Trauma

A 202

There is a “double bubble” sign with a distended gas- should either have a UGI series (to rule out volvulus) or
filled stomach and a grossly dilated proximal duodenum have an exploratory laparotomy.
in both figures. The double bubble sign indicates duode- Baby A and B highlight the fact that the presence or
nal obstruction. absence of distal gas cannot be used to definitively diag-
The most common causes of duodenal obstruction nose the patient with either duodenal atresia or midgut
are duodenal atresia, duodenal stenosis, annular pan- volvulus.
creas, and rotational abnormalities. In the case of a rota- In Fig. 1, the distal gas makes malrotation with volvu-
tional abnormality, the duodenal obstruction is second- lus highly suspected. Emergent operation is performed.
ary to either congenital peritoneal bands (Ladd’s bands) At laparotomy, although there is malrotation there are no
or midgut volvulus. obvious Ladd’s bands and there is no volvulus. The cause
In Fig. 1 there is clearly gas in the more distal small of the duodenal obstruction is found to be an annular
bowel/colon. In Fig. 2 there is no visible gas distal to the pancreas. The malrotation is corrected and the annular
proximal duodenum. pancreas is bypassed with a duodenoduodenostomy.
The presence or absence of bowel gas distal to the In Fig. 2, the absence of distal gas makes duodenal
proximal duodenum helps to determine the most likely atresia highly likely and the patient is taken for elective
etiology. In the case of an isolated double bubble with no operation. At laparotomy, there is obvious malrotation
distal gas, duodenal atresia is most likely. However, con- with midgut volvulus and a normal duodenum. The vol-
trary to popular belief, an isolated double bubble does vulus is reduced and a Ladd’s procedure performed.
not entirely rule out malrotation, although the surgeon Any patient with a double bubble sign in the presence
expects some distal gas in this situation. In the presence of distal gas should be taken for emergent operation. In
of distal gas, midgut volvulus must be highly suspected. the case of an isolated double bubble, if the patient is
An upper gastrointestinal (UGI) series helps to fur- taken promptly to the operating room no further imag-
ther evaluate the case. In duodenal atresia, the duode- ing is needed but, if surgery is to be delayed, it is prudent
num has a club-shaped end. In malrotation with volvulus, to obtain a UGI series to rule out malrotation with mid-
the proximal duodenum terminates in a characteristic gut volvulus.
“corkscrew” or “birdbeak” fashion. Alternatively, ultra- Both patients did well postoperatively with no com-
sonography can be used to rule out malrotation at risk plications.
for volvulus. Children with an abnormal ultrasound

Suggested Reading
1. Hajivassiliou CA. Intestinal obstruction in neonatal/pediat- 3. Rathaus V, Grunebaum M, Ziv N, Kornreich L, Horev G.
ric surgery. Seminars in Pediatric Surgery 2003; 12(4):241– The bubble sign in the gasless abdomen of the newborn.
253 Pediatric Radiology 1992; 22(2):106–109
2. Orzech N, Navarro OM, Langer JC. Is ultrasonography a 4. Schmidt H, Abolmaali N, Vogl TJ. Double bubble sign. Eu-
good screening test for intestinal malrotation? Journal of ropean Radiology 2002; 12(7):1849–1853
Pediatric Surgery 2006; 41(5):1005–1009
7 Emergency and Trauma 433

Q 203
Yves Aigrain and Pascale Philippe-Chomette

Fig. 2

Fig. 1

Fig. 3

A 5-day-old boy with bilious vomiting was referred to • What other radiological examinations could be help-
our department. Clinical examination showed bilious ful in making the diagnosis?
vomiting, no abdominal distension but a painful abdo- • How do you interpret the US image (Fig. 2) and how
men. A standard abdominal radiograph was acquired do you interpret the examination shown in Fig. 3?
(Fig. 1). • Is a gastrointestinal tract examination needed?
• What is the main diagnosis? • What is your approach?
434 Emergency and Trauma

A 203

This boy is affected by neonatal midgut volvulus with in- The contrast image shows gastric distension with vol-
testinal malrotation. vulus and duodenal obstruction.
Clinically, the boy is 5 days old with bilious vomit- Because of the whirlpool sign on the Doppler US im-
ing, no abdominal distension, and is in pain. The stan- age and all the clinical signs, we should operate on the
dard radiograph shows gastric distension without distal child without performing any other examination.
aeration. The only thing to do before laparotomy is rehydra-
Most patients who present with acute midgut volvu- tion of the child. Intravenous fluids are administrated
lus are infants (30% present within the first week of life) rapidly to correct hypovolemia with no further delay for
and the most common symptom is vomiting (95%). intervention.
Pain and tenderness are constant. If the child is not in a life-threatening situation, a gas-
After plain film radiography revealing either a gasless trointestinal tract examination should be done to define
abdomen or upper intestinal distension (in 20% of cases the malrotation. Without active volvulus, it can be rather
a duodenal obstruction is identified by a double bubble difficult to identify the inversed position of the superior
sign), US color Doppler can help with the localization mesenteric vein and the superior mesenteric artery on
of mesenteric vessels to confirm the diagnosis of intes- US images.
tinal malrotation and the gastrointestinal tract examina- We operated on the child, who had midgut acute
tion can identify the malrotation or the total upper ob- volvulus and intestinal malrotation; the duodenum and
struction. jejunum lay to the right of the spine and the complete
The US image shows the clockwise whirlpool sign on small bowel was on the right. The small bowel had to
color Doppler US. It is the definite sign of midgut volvu- be rotated in a counterclockwise fashion to reduce the
lus. We can see the wrapping of the superior mesenteric volvulus; dissection continued until the cecum and the
vein and the mesentery around the superior mesenteric ascending colon was moved to the left away from the
artery. duodenum widening the base of the mesentery.

Suggested Reading
1. Epelman M. The whirlpool sign. Radiology 2006 Sep; 3. Orzech N, Navarro OM, Langer JC. Is ultrasonographic
240(3):910–1 a good screening test for intestinal malrotation? J Pediatr
2. Long FR, Kramer SS, Markowitz RI, Taylor GE. Radio- Surg 2006 May; 41(5):1005–9
graphics patterns of intestinal malrotation in children. Ra-
diographics 1996 May; 16(3):547–56
7 Emergency and Trauma 435

Q 204
Yves Aigrain and Pascale Philippe-Chomette

Fig. 1

Fig. 3

An 11-year-old boy presented with painful abdominal


distension and vomiting. Six years previously, he had
suffered a thoracic penetrating injury without subse-
quent surgical exploration.
The pain was localized in the upper left quadrant of
the abdomen.
• Comment on the standard chest (Figs. 1, 2).
• What is your hypothesis?
• Barium enema findings (Fig. 3) were conclusive;
why?
• What is your approach?

Fig. 2
436 Emergency and Trauma

A 204

On the standard chest radiograph, air is seen in the left Diaphragmatic repair was performed at the same
diaphragmatic area; the abdominal radiograph shows time by laparotomy.
colonic distension, and colonic splenic flexure is not vis- Blunt or penetrating injuries need complete investi-
ible. gations to confirm diaphragmatic integrity. The diagno-
The first hypothesis is colonic incarceration due to sis of colonic incarceration could be missed with a high
unrecognized traumatic diaphragmatic hernia. life-threatening risk for the patient. The diagnosis is sug-
The barium enema study shows opacification of the gested on plain film radiographs showing basilar opaci-
rectum, rectosigmoid, and a stop at the level of the fication and obliteration of a clear diaphragmatic border.
splenic flexure without transverse opacification, con- A nasogastric tube may help identify an intrathoracic
firming colonic incarceration. position of the stomach. CT helps in identifying other
The child was immediately operated on. The trans- intra-abdominal organs herniated into the chest.
verse colon and splenic flexure were incarcerated in the If the diagnosis is made shortly after injury, the repair
thoracic area with suffusion and colonic suffering. After is best made via laparotomy because of the high inci-
complete reintegration of the colon, the child recovered dence of associated intra-abdominal injuries.
fully.

Suggested Reading
1. Esme H, Solak O, Sahin DA, Sezer M. Blunt and penetrat-
ing traumatic ruptures of the diaphragm. Thorac Cardio-
vasc Surg 2006 Aug; 54(5):324–7
2. Von Hoppell, UO, Bautz P, De Groot M. Penetrating tho-
racic injuries: What we learnt. Thorac Cardiovasc Surg
2000; 48:55–61
7 Emergency and Trauma 437

Q 205
Felix Schier

A 10-year-old boy presented with right-sided lower


abdominal pain for 1 day. He experienced nausea, but
there was no vomiting and no diarrhea. There was no
fever. The pain had an increasing tendency. The child
did not want to drink or eat. He avoided any movement
of the psoas muscle. There was a marked tenderness on
palpation of the right lower abdomen. The Blumberg
and Rovsing signs were both positive.
Among the laboratory test results, there was a normal
leukocyte count and a slightly elevated C-reactive pro-
tein (CRP) level.
Ultrasound yielded the image shown in Fig. 1.
• What is the most likely diagnosis?
• Which treatment is indicated?
• Is a laparoscopic approach preferable or an open,
conventional approach?

Fig. 1
438 Emergency and Trauma

A 205

The child has gangrenous appendicitis. mesenteric fat, a “bull’s eye sign” is seen (arrow) with a
In general, laboratory values are not very useful in diameter of 1.5 cm. Also, there is marked thickening of
the differential diagnosis of acute appendicitis. It has the bowel wall.
been found that the change in total neutrophil count on CT scans for the diagnosis of appendicitis are un-
the first day after onset of symptoms and the change in usual in Europe. In the USA, they seem to be used more
CRP level on the second and third days during in-hospi- frequently, although their specificity largely depends on
tal observation may serve as useful parameters in differ- the experience of the radiologist attending and they re-
entiating acute appendicitis from other acute abdominal quire radiation.
diseases. Usually, however, there are no full 3 days for There is no need to perform emergency appendec-
waiting. Appendicitis is either diagnosed within 1 day or tomy in the middle of the night. It has been shown re-
another diagnosis is considered. peatedly that the procedure may safely be performed
The white blood count is of little value in acute ap- within regular working hours.
pendicitis. It seems that interleukin (IL)-6 or CRP levels The question remains open whether laparoscopic or
are correlated with acute appendicitis, but not the leuko- open appendectomy is better. A Cochrane review sup-
cytes. There is no significant difference in diagnostic use ports the view that in those clinical settings where surgi-
between IL-6 and CRP. Therefore, in clinical practice, cal expertise and equipment are available and used often,
the CRP is a relatively reliable indicator. laparoscopy seems to have various advantages. In con-
Clinically, there is no big difference between appen- trast, in settings where laparoscopy is not very common,
dicitis in children and adults. The typical, migrating pain a laparoscopic appendectomy may be an uncomfortable
is observed in all age groups. The negative appendec- adventure. Still, in young female patients or in any other
tomy rate is higher in adults, possibly because the diag- circumstances in which the diagnosis is not yet defini-
nosis is even easier to establish in children. Otherwise, tively established, laparoscopy is the superior technique.
statistically, mortality and morbidity, including wound The intraoperative view and visualization are better with
infection rate and intra-abdominal abscess rate, are sim- laparoscopy, and irrigation is also easier. Clinically, how-
ilar in adults and children. Contrary to traditional teach- ever, especially with respect to postoperative recovery,
ing, diagnosing acute appendicitis in children is similar there is no obvious advantage of laparoscopy over the
to that in adults. The outcome of acute appendicitis in open approach. The degree of peritonitis and local in-
children is not associated with a delay in presentation or flammation or even perforation determines postopera-
a delay in diagnosis. tive recovery, not the technical approach.
The ultrasonographic picture (Fig. 1) is a transverse
section of the right lower quadrant. Within hyperechoic

Suggested Reading
1. Ceydeli A, Lavotshkin S, Yu J, Wise L. When should we or- 4. Sauerland S, Lefering R, Neugebauer EA. Laparoscopic
der a CT scan and when should we rely on the results to di- versus open surgery for suspected appendicitis. Cochrane
agnose an acute appendicitis? Curr Surg 2006; 63:464–468, Database Syst Rev 2004; 18:CD001546
2. Lee SL, Ho HS. Acute appendicitis: is there a difference be- 5. Taylor M, Emil S, Nguyen N, Ndiforchu F. Emergent vs
tween children and adults? Am Surg 2006; 72:409–413 urgent appendectomy in children: a study of outcomes. J
3. Sack U, Biereder B, Elouahidi T, Bauer K, Keller T, Trobs Pediatr Surg 2005; 40:1912–1915
RB. Diagnostic value of blood inflammatory markers for 6. Wu HP, Huang CY, Chang YJ, Chou CC, Lin CY. Use of
detection of acute appendicitis in children. BMC Surg changes over time in serum inflammatory parameters
2006; 28:15 in patients with equivocal appendicitis. Surgery 2006;
139:789–796
7 Emergency and Trauma 439

Q 206
Felix Schier

Fig. 1 Fig. 2

A 12-year-old boy had recurrent, right-sided, lower ab- The laboratory values showed a normal leukocyte
dominal pain for at least 2 weeks. He had no fever. There count and C-reactive protein (CRP) level.
was tenderness on palpation of the right lower abdo- Ultrasound yielded the images shown in Figs. 1 and 2.
men. • What is the most likely diagnosis?
440 Emergency and Trauma

A 206

Figure 1 is a sonogram of the right lower abdomen. The on the scout localizer scans, despite windowing modi-
appendix appears extended, wall-thickened, and con- fications. Surgery would cure the problem if the patient
tains a hyperechoic oval structure at the tip, including a and parents are not ready to tolerate the finding and the
dorsal acoustic shadow. symptoms.
Figure 2 is a color-coded Doppler sonogram demon- When retained within the abdominal cavity, they
strating marked hypervascularity within the appendi- may cause intra-abdominal or intrapelvic abscesses. Fe-
ceal wall. caliths missed during laparoscopic appendectomy have
The diagnosis is fecalith and appendicitis. been published repeatedly as a cause of subsequent ab-
It has been found that a fecalith of the appendix may scess formation. They definitively are a risk factor for the
be a cause of chronic, right lower quadrant abdominal development of postoperative complications.
pain, at least in adult patients. This may well be the case Retained fecaliths have been removed laparoscopi-
also in children. In these patients, the classical signs of cally. It appears logical to attempt to localize and remove
acute appendicitis may be absent, such as fever, localized them laparoscopically after a previous laparoscopic in-
pain on palpation, or leukocytosis. On CT, a fecalith and tervention.
appendiceal thickening may be demonstrated, without If a perforated appendicitis is treated nonoperatively,
mesenteric infiltration, abscess, or collection. Thus, the the presence of a fecalith on radiologic imaging is re-
fecalith would serve as a marker rather than an actual lated to a recurrence rate of appendicitis of almost three
cause of appendicitis. It was also shown that focused CT times higher than if there is no fecalith. Thus, an abscess
scans with additional lung and bone windows proved with fecalith has a far lower rate to heal—with antibiot-
optimal in detecting fecaliths which were not visible ics—than an abscess without a fecalith.

Suggested Reading
1. Ein SH, Langer JC, Daneman A. Nonoperative management 3. Guillem P, Mulliez E, Proye C, Pattou F. Retained appendi-
of pediatric ruptured appendix with inflammatory mass or colith after laparoscopic appendectomy: the need for sys-
abscess: presence of an appendicolith predicts recurrent ap- tematic double ligature of the appendiceal base. Surg En-
pendicitis. J Pediatr Surg 2005; 40:1612–1615 dosc 2004; 18:717–718
2. Giuliano V, Giuliano C, Pinto F, Scaglione M. Chronic ap- 4. Nitecki S, Karmeli R, Sarr MG. Appendiceal calculi and fe-
pendicitis “syndrome” manifested by an appendicolith and caliths as indications for appendectomy. Surg Gynecol Ob-
thickened appendix presenting as chronic right lower ab- stet 1990; 171:185–188
dominal pain in adults. Emerg Radiol 2006; 12:96–98 5. Smith AG, Ripepi A, Stahlfeld KR. Retained fecalith: lapa-
roscopic removal. Surg Laparosc Endosc Percutan Tech
2002; 12:441–442
7 Emergency and Trauma 441

Q 207
Felix Schier

Fig. 2

Fig. 1

A 2-year-old boy, previously healthy, suddenly displayed Laboratory values were normal except for a slight
colicky attacks of abdominal pain. The child vomited leukopenia.
several times and passed bloody stool 8 h later. Imaging studies are shown in Figs. 1 and 2.
There was a mass palpable at the right lower abdo- • What is the most likely diagnosis?
men.
442 Emergency and Trauma

A 207

Figure 1 is a sonogram of the right lower abdomen. A and the success rate has also risen from 45% to more
target-shaped structure with a diameter of 3.3 cm is than 80%.
identified. The bowel walls are thickened. Otherwise, the basic techniques in intussusception
During pneumatic reduction, the head of the intus- are uniform. An attempt is made to reduce the intussus-
susceptum is still visible within the cecum (Fig. 2, ar- ceptum by air or liquid. Approaches vary with respect
row). to the conventional “open” approach versus the laparo-
The diagnosis is ileocolic intussusception. scopic approach.
In Europe, intussusception is the most common Laparoscopy is indicated if the reduction is incom-
cause of acute intestinal obstruction in children younger plete (as in this case) or uncertain. In a distended abdo-
than 5 years. The classic triad of symptoms (abdominal men, however, an open approach may be safer. There is
pain, abdominal mass, bloody stools) is present in 30% an approximately 10% chance that the intussusception
of patients. Etiologically, it has been linked to rotavirus has spontaneously reduced during induction of anesthe-
vaccine, without proof so far. The peak incidence is in sia. This is why Chinese pediatric surgeons perform a
children of 3–9 months of age. last reduction attempt with the patient on the table; with
Is it commonly believed that in school-age children, an overall success rate of more than 90%. These 10%
intussusception is more commonly caused by some or- spontaneous reductions are also an aspect which princi-
ganic abnormality than in younger children. This is not pally favors laparoscopy in intussusception.
uniformly confirmed, however. It has been found that over the last 25 years, the du-
Interestingly, the management of pediatric intussus- ration of symptoms and signs prior to diagnosis is now
ception outside tertiary centers is not uniform or stan- three times longer than earlier. In contrast, however, the
dardized. This probably applies to all European countries. incidence of pain, vomiting, abdominal mass, and rec-
Outside tertiary care centers the approach to intussus- tal blood has decreased. The classical symptoms seem
ception varies widely. In England for example, pediatri- less dramatic than they were 25 years ago. Furthermore,
cians favor air or saline, surgeons prefer water-soluble fewer lead points are found today than earlier. The rea-
contrast media, and radiologists still use barium. Today, son for this is unclear.
possibly stimulated by the success rate of our Asian col- Along the same lines, today fewer children require
leagues, air is far more frequently used than 25 years ago, surgery than earlier, but more children need resections.

Suggested Reading
1. Calder FR, Tan S, Kitteringham L, Dykes EH. Patterns of 3. Huppertz HI, Soriano-Gabarro M, Grimprel E, Franco E,
management of intussusception outside tertiary centres. J Mezner Z, Desselberger U, Smit Y, Wolleswinkel-van den
Pediatr Surg 2001; 36:312–315 Bosch J, De Vos B, Giaquinto C. Intussusception among
2. Grimprel E, de La Rocque F, Romain O, Minodier P, Dom- young children in Europe. Pediatr Infect Dis J 25, Suppl
mergues MA, Laporte-Turpin E, Lorrot M, Parez N, Caulin 2006; S22–29
E, Robert M, Lehors H, Cheron G, Levy C, Haas H; Groupe 4. Ikeda T, Koshinaga T, Inoue M, Goto H, Sugitou K, Hagi-
de Pathologie Infectieuse Pediatrique; Groupe Franco- wara N. Intussusception in children of school age. Pediatr
phone d’Urgences et de Reanimation Pediatrique; Societe Int 2007; 49:58–63
Francaise de Chirurgie Pediatrique: Management of intus- 5. Schier F. Experience with laparoscopy in the treatment of
susception in France in 2004: investigation of the Paediatric intussusception. J Pediatr Surg 1997; 32:1713–1714
Infectious Diseases Group, the French Group of Paediatric
Emergency and Reanimation, and the French Society of
Paediatric Surgery Arch Pediatr 2006; 13:1581–1588
7 Emergency and Trauma 443

Q 208
Felix Schier

Fig. 1 Fig. 2

Fig. 3

A 2-year-old boy had abdominal pain, joint pain, a pur- Laboratory values were normal except for a leuko-
puric rash at the lower extremities, hematuria, and occult penia.
gastrointestinal blood loss. A nonspecific viral infection Imaging studies are shown in Figs. 1,2.
had preceded these symptoms several days earlier. • What is the most likely diagnosis?
444 Emergency and Trauma

A 208

Figure 1 is an ultrasonogram of the left upper abdomen. because of the presence of pathologic lead points and/
There is a “target sign” with a diameter of 2.2 cm. The or bowel complications. Some surgeons had the idea to
bowel walls are enlarged. wait for spontaneous reduction because they had seen
Figure 2 shows a plain abdominal view. There is an cases of spontaneous reduction upon performing lapa-
irregular air distribution pattern in the proximal small rotomy. Today it would appear more appropriate to per-
bowel. Also, there is decreased gas within the distal form laparoscopy in these cases, with the option either
bowel, a sign of incomplete obstruction. to confirm a spontaneous reduction or to encounter
The diagnosis is jejunojejunal intussusception with an intussusception amenable to laparoscopic reduction
abdominal wall bleeding in Henoch–Schönlein purpura. (which is technically easier than an ileocolic intussus-
Figure 3 is shown for comparison: This is a tar- ception) or to proceed to a laparoscopy-assisted proce-
get sign of 1.7 cm diameter at the left upper abdomen, dure with exteriorizing the involved bowel segment via
caused by jejunojejunal intussusception in a 7-year old the umbilicus. A conventional “open” approach would
boy without Henoch–Schönlein. In contrast to the He- appear exaggerated today.
noch–Schönlein purpura intussusception there is no Do all small bowel intussusceptions require surgery?
bowel wall thickening. There are no uniform observations. It has been found
Enema reduction is considered ineffective in small that an intussusception length greater than 3.5 cm dur-
bowel intussusception. There are, however, reports that ing sonography is a strong independent predictor of the
enemas can be safely performed in intussusception from need for surgical intervention.
Henoch–Schönlein. Intussusception from bowel wall hematoma may also
Literature reports differ on the rate of observed spon- be a consequence of trauma, not only from Henoch–
taneous reductions of small bowel intussusception. In Schönlein purpura.
contrast, others report a surgery rate of virtually 100%

Suggested Reading
1. Erichsen D, Sellstrom H, Andersson H. Small bowel intus- 4. Schwab J, Benya E, Lin R, Majd K. Contrast enema in chil-
susception after blunt abdominal trauma in a 6-year-old dren with Henoch-Schonlein purpura. J Pediatr Surg 2005;
boy: case report and review of 6 cases reported in the litera- 40:1221–1223
ture. J Pediatr Surg 2006; 41:1930–1932 5. Sonmez K, Turkyilmaz Z, Demirogullari B, Karabulut
2. Koh EP, Chua JH, Chui CH, Jacobsen AS. A report of 6 R, Aral YZ, Konus O, Basaklar AC, Kale N. Conserva-
children with small bowel intussusception that required tive treatment for small intestinal intussusception associ-
surgical intervention. J Pediatr Surg 2006; 41:817–820 ated with Henoch-Schonlein‘s purpura. Surg Today 2002;
3. Munden MM, Bruzzi JF, Coley BD, Munden RF. Sonogra- 32:1031–1034
phy of pediatric small-bowel intussusception: differentiat-
ing surgical from nonsurgical cases. AJR Am J Roentgenol
2007; 188:275–279
7 Emergency and Trauma 445

Q 209
Felix Schier

Fig. 3
Fig. 1

Fig. 2 Fig. 4

During a routine interuterine screening, distended Laboratory values showed signs of infection.
bowel loops and intra-abdominal calcifications were Imaging studies are shown in Figs. 1–4.
seen. Shortly after birth, the newborn had a distended • What is the most likely diagnosis?
abdomen and vomited bile. Both scrota appeared mod-
erately inflamed, were edematous, and had a slightly
brownish discoloration.
446 Emergency and Trauma

A 209

Figure 1 is a prenatal ultrasound showing distended scan in order to define persistent intestinal perforation
bowel loops and signs of calcification. invisible with prenatal ultrasound.
Figure 2 is a plain abdominal radiograph taken on Meconium periorchitis is a rarity. Clinically, a soft
the first day of life. The bowel loops are distended as in swelling of the scrota is seen. The swelling may be as-
ileus. The arrow points to an oval-shaped grainy calcifi- sociated with bluish discoloration of the scrotum. There
cation. is a risk of unnecessary surgery of this condition. It has
Figure 3 is a transsection of the right lower abdomen. been stated that the “mass” and the calcifications will re-
Again, dilated bowel loops are seen as in ileus. There is solve spontaneously without compromising the testicle.
ascites between the bowel loops. The arrow points to a Sonographic features together with an abdominal plain
hyperechoic structure like a calcification (with acoustic film radiograph are diagnostic, and visualization of the
shadow). normal testicle may be helpful in differentiating this tu-
Figure 4 is a radiograph of the scrotum demonstrat- mor-like lesion from scrotal tumors.
ing calcifications of the scrotum. Even meconium thorax has been reported. In these
The diagnosis is intrauterine meconium peritonitis cases, meconium peritonitis extends through muscular
after bowel perforation. defects in the diaphragm up into the thorax. Meconium
The perforation is caused by distal obstruction. Me- thorax has also been identified with prenatal ultrasound.
conium acts as an aseptic chemical. Not all intra-abdominal calcifications are caused by
Prenatal ultrasonography allows one to suspect meco- meconium peritonitis. Patients with anorectal anoma-
nium peritonitis. This in turn allows transfer of mother lies and rectourethral fistulae are also reported to show
and child to a tertiary center for delivery and appropri- intraluminal calcified meconium. It has been postulated
ate management. It has been found that the chances of that intestinal stasis and mixing of urine and meconium
survival of these babies then are excellent if they are not may be predisposing factors for the calcification of me-
associated with cystic fibrosis. conium. In these cases, intraluminal calcifications may
Prenatal MR imaging has recently been described as appear as discrete punctate flecks within the distribution
an add-on in order to improve the low diagnostic yield of the bowel, in contrast to meconium peritonitis, where
of prenatal ultrasound for meconium peritonitis. It has the calcifications are linear and plaque-like, occurring
been suggested to also perform a postnatal contrast CT anywhere in the abdominal cavity and scrotum.

Suggested Reading
1. Chan KL, Tang MH, Tse HY, Tang RY, Tam PK. Meconium 4. Simonovsky V, Lisy J. Meconium pseudocyst secondary to
peritonitis: prenatal diagnosis, postnatal management and ileal atresia complicated by volvulus: antenatal MR demon-
outcome. Prenat Diagn 2005; 25:676–682 stration. Pediatr Radiol 6, 2007
2. Miller JP, Smith SD, Newman B, Sukarochana K. Neonatal 5. Varkonyi I, Fliegel C, Rosslein R, Jenny P, Ohnacker H. Me-
abdominal calcification: is it always meconium peritonitis? conium periorchitis: case report and literature review. Eur J
J Pediatr Surg 1988; 23:555–556 Pediatr Surg 2000; 10:404–407
3. Patole S, Whitehall J, Almonte R, Stalewski H, Lee-Tannock
A, Murphy A. Meconium thorax: a case report and review
of literature. Am J Perinatol 1998; 15:53–56
7 Emergency and Trauma 447

Q 210
Felix Schier

Fig. 1 Fig. 2

A premature infant weighing 1,500 g developed a dis- was passed perianally. Laboratory values yield a septic
tended abdomen and feeding intolerance. Biliary vomit- picture.
ing set in and the child became septic within a few hours. Imaging studies yielded the figures shown here
Eventually, the child was close to shock. The right lower (Figs. 1, 2).
abdomen was distended, reddened, and tender. Blood • What is the most likely diagnosis?
448 Emergency and Trauma

A 210

Figure 1 is an abdominal plain film radiograph in up- ear pneumatosis, submucosal air demonstrates a bubbly
right position. Gas is distributed unequally within the appearance. In premature infants younger than 2 weeks
abdomen. Bowel loops in the middle and left upper ab- the radiographic appearance of colonic stool is rare.
domen are massively dilated. Some air–fluid levels are As in the above case, neonates most commonly pres-
identified. The arrow points to a dilated bowel loop with ent with feeding intolerance, delayed gastric emptying,
foamy stool/gas content in the right abdomen. In addi- abdominal distention or tenderness (or both), occult or
tion there is a double contour of the bowel wall, the so- gross blood in the stool, lethargy, apnea, respiratory dis-
called pneumatosis intestinalis. tress, or poor perfusion.
Figure 2 is an abdominal plain film radiograph of a Some indications for surgery are obvious: pneumo-
different patient with the child supine. On the right liver peritoneum, paracentesis positive for brown fluid or bac-
lobe a radial “pneumoportogram” is seen, demonstrat- teria, portal venous gas, severe pneumatosis intestinalis;
ing air-filled portal veins. Bowel loops are massively di- numerous others are less evident, such as erythema, di-
lated as in ileus. lated loops on serial radiographs, fixed abdominal mass
The diagnosis is necrotizing enterocolitis with pneu- etc.
matosis intestinalis and pneumoportogram. Approximately 30% of all children with necrotizing
Portal venous gas is usually associated with extensive enterocolitis will undergo surgery. About 50% of the op-
bowel gangrene and high mortality. erated children die. The smaller and the sicker the child,
With respect to radiologic signs of necrotizing entero- the higher the mortality rate.
colitis, it may be stated that most patients have pneuma- Postoperative complications of surgery are wound
tosis intestinalis, mild to severe, and one-third of patients dehiscences, intra-abdominal abscesses, and intestinal
have pneumoperitoneum. In approximately 15% of pa- strictures. In dramatic cases short bowel syndrome may
tients portal venous gas can be demonstrated; 10% have ensue.
both, portal venous gas and pneumatosis intestinalis. Virtually no progress has been made in the last
Radiologists seem to find it difficult to establish a re- 30 years with respect to the pathophysiology of necro-
liable diagnosis when evaluating radiologic signs of nec- tizing enterocolitis. Several mechanisms are still under
rotizing enterocolitis. It has repeatedly been shown that discussion—immature intestinal motility and digestion,
there is a relatively high observer variability. The indica- immature intestinal circulatory regulation, immature
tion for surgery thus cannot rely solely on radiology. intestinal barrier function, abnormal bacterial coloniza-
Diffuse distention and asymmetric bowel gas pattern tion, immature intestinal innate immunity—without an
are early signs. Pneumatosis intestinalis and portal ve- answer so far. No wonder that surgery does not achieve
nous gas are remarkable signs. Subserosal air makes lin- better results.

Suggested Reading
1. Caplan MS, Jilling T. New concepts in necrotizing entero- 4. Lin PW, Stoll BJ. Necrotising enterocolitis. Lancet 2006;
colitis. Curr Opin Pediatr 2001; 13:111–115 368:1271–1283
2. Kosloske AM, Musemeche CA, Ball WS Jr, Ablin DS, Bhat- 5. Di Napoli A, Di Lallo D, Perucci CA, Schifano P, Orzalesi
tacharyya N. Necrotizing enterocolitis: value of radio- M, Franco F, De Carolis MP. Inter-observer reliability of ra-
graphic findings to predict outcome. AJR Am J Roentgenol diological signs of necrotising enterocolitis in a population
1988; 151:771–774 of high-risk newborns. Paediatr Perinat Epidemiol 2004;
3. Kosloske AM, Necrotizing enterocolitis In: Surgery of In- 18:80–87
fants and Children: Scientific principles and practice. Keith
T. Oldham, Paul M. Colombani, and Robert P. Foglia (eds).
Lippincott-Raven Publishers, Philadelphia, p 1201–1213,
1997
7 Emergency and Trauma 449

Q 211
Felix Schier

Fig. 1

Fig. 2

A 3-week-old girl presented with frequent vomiting after


feedings. The physical examination was normal. Labora-
tory values were also normal. Imaging studies yield the
figures shown here (Figs. 1–2).
• What is the most likely diagnosis?
450 Emergency and Trauma

A 211

The contrast medium remains for an unusually long vein and artery and the “whirlpool sign” were more pre-
period of time within the pars descendens of the duo- dictive for malrotation and volvulus than anterior/poste-
denum. Next, the contrast medium passes into a short rior orientation. Therefore, ultrasonography seems a good
segment of the duodenum running immediately to the screening tool that effectively rules out malrotation at risk
right of the vertebral column (arrow) and not proceed- for volvulus. The conclusion was that children with an ab-
ing to the left upper abdomen as normally. Eventually, normal ultrasound should have an upper GI study or go
the contrast medium passes into small bowel segments to the operating room, depending on clinical findings.
more distally. Studies of patients up to adulthood have demon-
On the sonogram (Fig. 1), a “target sign” with a cen- strated that identification of the clockwise whirlpool
tral vessel is identified within the left abdomen and pre- sign on sonography is an accurate way of diagnosing
vertebrally. volvulus, precluding the need for further investigations
A color-coded Doppler sonogram demonstrates that and allowing for prompt surgical intervention.
the blood vessels run in a spiral in this same area (“whirl- Should a patient with asymptomatic malrotation
pool sign”). The Doppler flow pattern demonstrates that undergo a prophylactic Ladd’s procedure? It has been
barely the end diastolic blood flow is maintained. shown that the gain in life expectancy was highest when
Figure 2, for comparison, shows volvulus in a differ- asymptomatic malrotation was treated at 1 year of age
ent patient: a 5-year old girl with recurrent abdominal and steadily declined until asymptomatic malrotation
pain. Note the typical “corkscrew sign” of the proximal was treated at 20 years of age. An increasing advantage of
jejunum. observation over prophylactic surgery on life expectancy
The diagnosis is malrotation with incomplete volvu- was observed after the second decade of life. A twofold
lus. increase in mortality risk for an elective Ladd’s proce-
Midgut volvulus is provoked by intestinal malrota- dure decreased the age threshold to 14 years, whereas a
tion, a congenital abnormal position of the duodenoje- fourfold increase decreased the threshold to 7 years. The
junal junction, and is a potentially life-threatening con- conclusion was that Ladd’s procedure should be consid-
dition, although chronic conditions have occasionally ered for children diagnosed with asymptomatic malrota-
been seen (see Fig. 2). Radiologically, the diagnosis is tion, particularly those who are younger and with a low
considered straightforward. However, there seems to be risk of postoperative mortality.
a remaining approximately 15% of upper GI tract exam- Surgically, we have perhaps been too aggressive in
inations which are equivocal and lead to a false-positive the past when encountering seemingly nonviable small
or false-negative interpretation. This difficulty seems to bowel in volvulus. A case of a newborn has been de-
increase with increasing patient age. scribed in whom the complete small bowel appeared
In ultrasonography, abnormal orientation of the su- ischemic and nonviable during surgery for volvulus. The
perior mesenteric artery and vein has been described in abdomen was closed and the child left alone. Weeks later,
malrotation. Ultrasonography alone was found to be false subsequent relaparotomy showed that 40 cm of bowel
positive in 21% and false negative in 2%. Among abnor- had unexpectedly survived. And so did the child.
mal ultrasounds, inversion of the superior mesenteric

Suggested Reading
1. Applegate KE, Anderson JM, Klatte EC. Intestinal malrota- 3. Orzech N, Navarro OM, Langer JC. Is ultrasonography a
tion in children: a problem-solving approach to the upper good screening test for intestinal malrotation? J Pediatr
gastrointestinal series. Radiographics 2006; 26:1485–1500 Surg 2006; 41:1005–1009
2. Houben CH, Mitton S, Capps S. Malrotation volvulus in a 4. Taori K, Sanyal R, Attarde V, Bhagat M, Sheorain VS, Jawale
neonate: a novel surgical approach.Pediatr Surg Int 2006; R, Rathod J. Unusual presentations of midgut volvulus with
222:393–394 the whirlpool sign. J Ultrasound Med 2006; 25:99–103
7 Emergency and Trauma 451

Q 212
Felix Schier

A newborn presented with anal atresia. The abdomen


was markedly distended and the child displayed pro-
gressive circulatory decompensation.
Laboratory values showed signs of the beginning
of sepsis. Imaging studies yield the picture shown here
(Fig. 1).
• What is the most likely diagnosis?

Fig. 1
452 Emergency and Trauma

A 212

Figure 1 is an abdominal plain film radiograph in reclin- weight infant represent a clinically distinct phenom-
ing position. The abdomen is distended and bowel loops enon. Very likely, the traditionally accepted diagnostic
are dilated. There is tent-shaped intraperitoneal, subdia- criteria for bowel perforation due to NEC are unreliable
phragmatic free air. A linear shadow (arrow) is created in this condition.
by air around the falciform ligament (the so-called foot- Spontaneous perforations of the small bowel in small
ball sign). In addition, there is free air below the liver children have also been reported. A pathogenetic mecha-
(arrowhead). nism is unknown so far. Again, the prognosis was good
The diagnosis is pneumoperitoneum after bowel per- after surgery. Typical of isolated intestinal perforations
foration. without signs of NEC are isolated perforations on healthy
Spontaneous perforation of the colon in newborns bowel, in contrast to NEC. Interestingly the survival af-
is occasionally reported. Also, a few cases of spontane- ter NEC perforation and after isolated perforation of the
ous colonic perforations in children with anal atresia are small bowel is equally bad (60% and 50%, respectively).
known. Practically all recover completely after surgery. In surgery for perforation a decision has to be made
The cause mostly remains unknown. between resection and anastomosis and stoma. It has
Other parts of the GI tract may also perforate sponta- been found that in preterm neonates, resection and
neously without signs of necrotizing enterocolitis (NEC). anastomosis is an acceptable option in spontaneous id-
Focal GI perforations occurring in the very-low-birth iopathic intestinal perforation as well as in NEC.

Suggested Reading
1. Calisti A, Perrelli L, Nanni L, Vallasciani S, D’Urzo C, Molle 3. Komuro H, Urita Y, Hori T, Hirai M, Kudou S, Gotoh C,
P, Briganti V, Assumma M, De Carolis MP, Maragliano G. Kawakami H, Kaneko M. Perforation of the colon in neo-
Surgical approach to neonatal intestinal perforation. An nates. J Pediatr Surg 2005; 40:1916–1919
analysis on 85 cases (1991–2001). Minerva Pediatr 2004; 4. Mintz AC, Applebaum H: Focal gastrointestinal perfora-
56:335–339 tions not associated with necrotizing enterocolitis in very
2. Harms K, Ludtke FE, Lepsien G, Speer CP. Idiopathic intes- low birth weight neonates. J Pediatr Surg 1993; 28:857–860
tinal perforations in premature infants without evidence of 5. Singh M, Owen A, Gull S, Morabito A, Bianchi A. Surgery
necrotizing enterocolitis. Eur J Pediatr Surg 1995; 5:30–33 for intestinal perforation in preterm neonates: anastomosis
vs stoma. J Pediatr Surg 2006; 41:725–729
7 Emergency and Trauma 453

Q 213
Felix Schier

Fig. 2

Fig. 1

A 4-year-old boy presented with an acutely painful, red-


dened, and enlarged scrotum. The child also suffered
from severe diarrhea and had fever.
Laboratory values pointed to an infectious process.
Imaging studies yield the pictures shown here (Figs. 1–
3).
• What is the most likely diagnosis? Fig. 3
454 Emergency and Trauma

A 213

Figure 1 is a longitudinal section of the left colon. The agnosis. Salmonella enteritidis will be cultured from the
submucosa is enlarged, the mesenterial fat tissue is not intraoperative swab. All other cultures from blood, CSF,
enlarged. The bowel lumen is massively narrowed. and urine may prove sterile. The risk of extraintestinal
Figure 2 is a longitudinal section of the scrotum. The manifestations is the reason why it has been suggested
wall of the scrotum is massively thickened. There is a that children less than 3 months of age with suspected
hydrocele with hyperechoic septae, indicating infection or proven salmonellosis should receive early antibiotic
of the hydrocele. The testicle is found at the back of the treatment.
scrotum, with the epididymis on top, also enlarged and In adult patients, apparently, the most notable echo-
hyperechoic. graphic finding is an enlarged and heterogeneous epi-
Color-coded Doppler sonography (Fig. 3) demon- didymis, predominantly in its body and tail. Testicular
strates good perfusion in the testicle and also the epidid- involvement consisted of a diffusely hypoechoic testis
ymis, thus ruling out testicular torsion. This is reactive or focal intratesticular areas. Thickening of the scrotal
hyperemia, a consequence of infection. wall and tunica albuginea and moderate hydrocele were
Salmonellae were eventually identified in the hydro- also noted occasionally. Follow-up scans occasionally
cele of the same species as in the feces. revealed intratesticular abscesses.
The diagnosis is acute scrotum in septic salmonello- Ultrasonography is not significantly better than the
sis. merely clinical diagnosis of orchitis. Statistically it has
Extraintestinal manifestations of salmonellosis in pe- been found that the presence of dysuria and micturition
diatric patients are found predominantly in infants less disorders points to orchitis. There seems to be a consid-
than 3 months of age. Genital involvement is a compli- erable risk of misdiagnosis. Clinical parameters are still
cation which has been described in a few isolated cases. of importance in surgical decision making when orchitis
The children may erroneously undergo surgery on suspi- is a possibility.
cion of testicular torsion, while orchitis is the correct di-

Suggested Reading
1. Berner R, Schumacher RF, Zimmerhackl LB, Franken- 4. Huang CB, Chuang JH. Acute scrotal inflammation caused
schmidt A, Brandis M. Salmonella enteritidis orchitis in a by Salmonella in young infants. Pediatr Infect Dis J 1997;
10-week-old boy. Acta Paediatr 1994; 83:992–993 16:1091–1092
2. Ciftci AO, Senocak ME, Tanyel FC, Buyukpamukcu N. 5. Salmeron I, Ramirez-Escobar MA, Puertas F, Marcos R,
Clinical predictors for differential diagnosis of acute scro- Garcia-Marcos F, Sanchez R. Granulomatous epididymo-
tum. Eur J Pediatr Surg 2004; 14:333–338 orchitis: sonographic features and clinical outcome in bru-
3. Foster R, Weber TR, Kleiman M, Grosfeld JL. Salmonella cellosis, tuberculosis and idiopathic granulomatous epidid-
enteritidis: testicular abscess in a newborn. J Urol 1983; ymo-orchitis. J Urol 1998; 159:1954–1957
130:790–791
7 Emergency and Trauma 455

Q 214
Brice Antao and Azad Najmaldin

Fig. 1 Fig. 2

A 6-month-old infant presented with severe abdominal


pain, vomiting, and bleeding per rectum.
• What is the possible diagnosis?
• What signs might you find on clinical examination?
• What is the initial management of this condition?
• What features in Fig. 1 are suggestive of this condi-
tion?
• What do Figs. 2 and 3 show?
• What other investigations may be useful in this con-
dition?
• What is the definitive management for this condi-
tion?
• What is the final outcome of this condition?

Fig. 3
456 Emergency and Trauma

A 214

The suspected diagnosis is intussusception. It is a full


thickness invagination of one portion of intestine into
another. The most common intussusception is ileocolic.
It commonly presents between 3 months and 3 years of
age with the peak being between 5 and 9 months. The
classical triad (abdominal pain, vomiting, and bleeding
per rectum) occurs in only one-third of cases. Profound
lethargy, diarrhea, and abdominal distension are also
present, not uncommonly. Bleeding per rectum is often
described as red currant jelly stool.
In early stages, the child usually appears lethargic
between attacks of abdominal pain. As the disease pro-
gresses, signs of dehydration, hypotension, or even sep-
tic shock may set in. The hallmark findings are a right
hypochondrium sausage-shaped mass with emptiness
in the right lower quadrant (Dance sign). In advanced
cases, there may be abdominal distension and signs of Fig. 4
peritonitis. In rare cases, the intussusception may pro-
lapse through the rectum.
Initial management is appropriate intravenous fluid
resuscitation, nasogastric decompression, and antibiotic Contrast or air enema (Fig. 4) is diagnostic and
cover. therapeutic. In the absence of hemodynamic instability
Figure 1 is an abdominal radiograph which shows and/or peritonitis, pneumatic or hydrostatic reduction
features of small-bowel obstruction (dilated small-bowel is the first line of treatment. This is done through a cath-
loops and air–fluid level). The other features suggestive eter placed inside the rectum with the buttocks taped to
of intussusception are an abdominal mass in the up- prevent leak. The maximum pressure of air insufflation
per quadrant, abnormal distribution of bowel gas, and should not exceed 150 mmHg and the barium column
sparse or absent large-bowel gas. should not be higher than 1 m. In cases of perforation
Figures 2 and 3 demonstrate ultrasonographic fea- and peritonitis and in critically ill patients, air and hy-
tures of intussusception. Ultrasonography is a useful drostatic enema reduction are contraindicated. In these
noninvasive diagnostic investigation. The characteristic situations, and if pneumatic or hydrostatic reduction
findings are of a target lesion as seen in transverse sec- has failed, operative reduction is indicated. In cases of
tion (Fig, 2) and a pseudokidney sign on longitudinal perforation, gangrene/necrosis of the bowel, and in the
section (Fig. 3). A target lesion consists of two rings of presence of a lead point, intestinal resection and anasto-
low echogenicity separated by a hyperechoic ring. Simi- mosis are necessary. The patient presented here under-
larly, a pseudokidney sign appears as superimposed hy- went successful pneumatic reduction of the intussuscep-
poechoic and hyperechoic layers. tion and was discharged home 24 h later.
A CT scan of the abdomen gives a high diagnostic Recurrences can occur in 2%–20% of cases and usu-
yield. The classical ying-yang sign of an intussusceptum ally within 6 months of initial presentation. An underly-
inside an intussuscipiens is not very consistent and when ing pathological condition (lead point) such as Meckel’s
present is almost diagnostic of intussusception. Laparos- diverticulum or polyps should be suspected in all recur-
copy and capsule endoscopy are useful diagnostic tools rent cases.
and can provide information regarding the cause in
cases of recurrent intussusception.
7 Emergency and Trauma 457

Q 215
Brice Antao and Azad Najmaldin

Fig. 2

Fig. 1

Fig. 3

A 12-year-old boy with learning difficulties presented • Why did the surgeon perform the examination as
with sudden-onset left-sided chest pain and respiratory shown in Fig. 3
distress. Four weeks prior to this, he had complained of • What is the underlying pathophysiological condition
vague abdominal pains. relating to the child’s current symptoms?
• What features are seen in Fig. 1, suggestive of the di- • What is the best line of treatment of this child?
agnosis? • What was the outcome of treatment?
• What other investigation is useful in assessment of
this child?
458 Emergency and Trauma

A 215

On clinical examination, the child was very distressed abdominal pain 4 weeks prior to admission. The reason
and there was decreased air entry on the left side of for there not being any evidence of pneumoperitoneum
chest. There was a past history of ingestion of various and/or generalized peritonitis is the close proximity of
foreign bodies. the stomach to the left hemidiaphragm aided by the
Figure 1 is a radiograph of the chest and abdomen. It child’s scoliotic posture, gradual migration of the foreign
shows a linear foreign body in the left side of the chest body, and omental adhesions.
with pleural effusion and collapse of the lung around it. Most foreign bodies lodged in the upper GI tract can
A CT scan of the chest as outlined in Fig. 2 is a useful be removed endoscopically. It is generally recommended
examination in order to assess the extent of damage to that foreign bodies greater than 3 cm in length are re-
the underlying structures. It delineated the foreign body moved because they are likely to perforate the intestinal
which looked like a pencil lodged within the left lung tract. The foreign body in this case was not removed
and a surrounding empyema, collapsed lung, and me- endoscopically because it had already migrated into the
diastinal shift. chest, there was an advanced empyema, and the site of
From the CT scan, it was assumed that the pencil had the perforation in both the stomach and diaphragm re-
perforated the esophagus and lodged in the left thoracic quired formal repair. Thoracoscopy may be considered
cavity. Prior to surgery, an upper gastrointestinal (GI) en- in similar circumstances. Through a left lateral thora-
doscopy was carried out to assess the status of the upper cotomy, an eyeliner pencil, 4 cm long, was removed. The
GI tract. At endoscopy (Fig. 3) the blunt end of the pencil surrounding lung and chest wall was decorticated and
was seen lodged in the lateral wall of the stomach. the perforation in the stomach and diaphragm was re-
The foreign body had perforated through the stom- paired. An intercostal drain was left in situ.
ach and diaphragm into the chest, causing empyema and The intercostal drain was removed on the second
collapse of the lung. The child had probably ingested the postoperative day and the child was allowed home 2 days
foreign body at the time of the original complaints of later. At 1-year follow-up, the child remained well.

Suggested Reading
1. Antao B, Foxall G, Guzik I, Vaughan R, Roberts JP. Foreign 3. Lam PY, Marks MK, Fink AM, et al. Delayed presentation
body ingestion causing gastric and diaphragmatic perfora- of an ingested foreign body causing gastric perforation. J
tion in a child. Pediatr Surg Int 2005; 21:326–328 Paediatr Child Health 2001; 37:303–304
2. Arana A, Hauser B, Hachimi-Idrissi S, et al. Management
of ingested foreign bodies in childhood and review of the
literature. Eur J Pediatr 2001; 160:468–472
7 Emergency and Trauma 459

Q 216
Brice Antao and Azad Najmaldin

Fig. 1 Fig. 2

A newborn baby who was delivered at 31 weeks’ gesta-


tion and weighed 1.8 kg developed abdominal disten-
sion and bloody stools following introduction of enteral
feeds at 3 days of age.
• What changes are seen in Fig. 1, suggestive of the di-
agnosis?
• What is the management of this condition?
Two days later, the patient’s general condition deterio-
rated, and there was increased ventilatory requirement, Fig. 3
gross abdominal distension, and bilious nasogastric as-
pirates. A repeat abdominal radiograph was acquired
(Fig. 2). This baby underwent laparotomy the following day.
• Why was additional imaging (Fig. 3) performed? • How else could this condition be managed?
• What are the most common long-term complications
of this condition?
460 Emergency and Trauma

A 216

This is a case of necrotizing enterocolitis (NEC). It typi- loop or multiple loops of bowel that retain the same po-
cally affects premature infants with a low birth weight sition and shape on serial abdominal films for 24–36 h
following oral feeds. Neonates with NEC usually present (the so-called fixed loop sign) are usually suggestive of
with abdominal distension and/or signs of sepsis (apnea, full-thickness bowel necrosis.
bradycardia, unstable temperature, hypotension, or hy- The presence of free intra-abdominal gas may be
poglycemia). As the disease progresses the abdominal excluded by a lateral decubitus plain abdominal radio-
distension increases and the abdominal wall becomes graph (Fig. 3).
edematous and erythematous. Other features include The indications for surgical intervention are: dete-
bilious vomiting, hematemesis, and rectal bleeding. rioration of clinical condition and/or the presence of an
Figure 1 is an abdominal radiograph, which shows abdominal mass, a fixed loop sign, or free intraperito-
dilated loops of bowel. There is evidence of intramural neal gas. This patient underwent laparotomy and a mass
gas (pneumatosis intestinalis), which is characteristic of was identified in the right side of abdomen. A stoma
NEC. was created proximal to the mass and the distal end of
Uncomplicated cases are managed conservatively bowel was brought out as a mucous fistula. Extreme
with broad-spectrum antibiotics, nasogastric decom- premature and low-birth-weight neonates and those
pression, and parenteral nutrition and the outcome is that are critically ill may be managed with an intraperi-
usually good. toneal drain in the first instance and definitive surgery
Deterioration is likely to be due to progression of the at a later date.
disease. Progress can be monitored by daily measure of In this case, the neonate made a good recovery and
inflammatory markers (leukopenia and thrombocyto- the stoma was closed 8 weeks later. Full enteral feeds
penia) and abdominal radiographs. A repeat abdominal were commenced and he was thriving at the 3-month
radiograph (Fig. 2) shows increased bowel distension, follow-up. Single or multiple strictures and short gut
extensive pneumatosis intestinalis, presence of portal syndrome are not uncommon long-term complications
venous gas, and possibly a pneumoperitoneum. A single of advanced NEC.

Suggested Reading
1. DeCurtis M, Paone C, Vetrano G, et al. A case control study 3. Horwitz JR, Lally KP, Cheu HW, et al. Complications after
of necrotizing enterocolitis occurring over 8 years in a neo- surgical intervention for necrotizing enterocolitis: A multi-
natal intensive care unit. Eur J Pediatr 1987; 146:398–400 center review. J Pediatr Surg 1995; 30:994–999
2. Fotter R, Sorantin E. Diagnostic imaging in necrotizing en- 4. Morgan LJ, Shochat SJ, Hartman GE. Peritoneal drainage
terocolitis. Acta Pediatr Suppl 1994; 396:41–44 as primary management of perforated NEC in the very low
birth weight infant. J Pediatr Surg 1994; 29:30–34
7 Emergency and Trauma 461

Q 217
Craig T. Albanese

A pregnant woman was noted to be large for her stage of


pregnancy. An fetal abdominal ultrasound at 23 weeks’
gestation was performed (Fig. 1).
• What are the findings?
• What are the differential diagnoses?
• Are there any other prenatal imaging studies or tests
to be performed?

Fig. 1
462 Emergency and Trauma

A 217

The ultrasound (Fig. 1) reveals polyhydramnios (circle)


and the double-bubble sign (asterisks) corresponding to
the dilated stomach and duodenal bulb (spine, arrow).
The double-bubble sign comes from fluid distention
(prenatally) or gaseous distention (postnatally) of the
stomach (one “bubble”) and the proximal duodenum
(the second “bubble”), with the pylorus separating the
bubbles.
The differential diagnoses include all causes of com-
plete duodenal obstruction such as duodenal atresia, du-
odenal stenosis, duodenal web, annular pancreas, pre-
duodenal portal vein, and Ladd’s bands associated with
incomplete intestinal rotation.
A fetal echocardiogram should be obtained to assess
for congenital heart disease. It should also be repeated
postnatally. Fetal karyotyping should be offered to the
family since this is associated with chromosomal abnor-
malities, most commonly trisomy 21 in 30% of cases.
Figure 2 is a postnatal plain abdominal radiograph Fig. 2
demonstrating the double-bubble sign (asterisks).
This baby had an annular pancreas and trisomy 21.
Figure 3 is showing an associated common atrioventric-
ular canal (arrowheads) (common atria, A; fetal spine,
arrow). He underwent an uneventful duodenoduode-
nostomy (IVS, interventricular septum).

Suggested Reading
1. Bailey PV, Tracy TF Jr, Connors RH, Mooney DP, Lewis JE,
Weber TR. J Congenital duodenal obstruction: a 32-year
review. Pediatr Surg 1993 Jan; 28(1):92–5
2. Bittencourt DG, Barini R, Marba S, Sbragia L. Congenital
duodenal obstruction: does prenatal diagnosis improve the
outcome? Pediatr Surg Int 2004 Aug; 20(8):582–5
3. Jimenez JC, Emil S, Podnos Y, Nguyen N. Annular pancreas Fig. 3
in children: a recent decade’s experience. J Pediatr Surg
2004 Nov; 39(11):1654–7
7 Emergency and Trauma 463

Q 218
Craig T. Albanese

Fig. 1 Fig. 2

A screening ultrasound at 18 weeks’ gestation was nor-


mal. A follow-up examination of the fetal abdomen at
22 weeks (Fig. 1) revealed dilated loops of small bowel
(arrowheads) with intraluminal meconium filling defects
(spine, arrow). An ultrasound examination at 24 weeks
(Fig. 2) demonstrated intra-abdominal calcifications
(arrowheads) (fetal abdomen, A; fetal ribs, arrows). A
subsequent examination at 26 weeks (Fig. 3) showed
formation of a calcified meconium cyst (arrowheads).
• What anatomic/physiologic process led to these find-
ings?
• What systemic disease is associated with these find-
ings?

Fig. 3
464 Emergency and Trauma

A 218

A prenatal bowel perforation has occurred. Fatty acids in


the meconium are saponified, resulting in calcifications.
A pseudocyst can form and peritonitis may be evident at
birth. This may be associated with cystic fibrosis.
At birth, the baby had a distended, erythematous
abdomen with peritonitis. An abdominal radiograph
(Fig. 4) at birth demonstrated a calcified meconium cyst
(arrowheads). A subsequent water-soluble contrast en-
ema (Fig. 5) demonstrated a microcolon (arrowheads)
with filling defects in the distal ileum diagnostic for me-
conium ileus. At surgery, there was a meconium pseu-
docyst with a segmental volvulus and perforation of the
terminal ileum. The pseudocyst was removed, the bowel
irrigated free of meconium (using saline and N-acetyl-
cysteine), and an ileostomy fashioned. The baby tested
positive for cystic fibrosis.

Suggested Reading
1. Chan KL, Tang MH, Tse HY, Tang RY, Tam PK. Meconium
peritonitis: prenatal diagnosis, postnatal management and
outcome. Prenat Diagn 2005 Aug; 25(8):676–82 Fig. 4
2. Dirkes K, Crombleholme TM, Craigo SD, Latchaw LA,
Jacir NN, Harris BH, D’Alton ME. The natural history of
meconium peritonitis diagnosed in utero. J Pediatr Surg
1995 Jul; 30(7):979–82
3. Eckoldt F, Heling KS, Woderich R, Kraft S, Bollmann R,
Mau H. Meconium peritonitis and pseudo-cyst formation:
prenatal diagnosis and post-natal course Prenat Diagn 2003
Nov; 23(11):904–8

Fig. 5
7 Emergency and Trauma 465

Q 219
Craig T. Albanese

A routine prenatal ultrasound at 19 weeks’ gestation re-


vealed an abdominal wall defect.
• What are the findings on this ultrasound (Fig. 1)?
• What other prenatal tests should be done?
• What are some of the associated anomalies?
• What are the two main types of abdominal wall de-
fects?

Fig. 1
466 Emergency and Trauma

A 219

There is a central abdominal wall defect (Fig. 1) in which


the liver (asterisk) is outside the abdominal cavity (fetal
abdomen, arrowheads; spine, arrow). Fetal MR imaging
(Fig. 2) shows the herniated liver (asterisk) and small
bowel (arrowheads) surrounded by the peritoneal mem-
brane (arrows).
This lesion is associated with chromosomal abnormal-
ities, cardiac defects, Beckwith–Wiedemann syndrome
(macroglossia, hemihypertrophy, hyperinsulinism), cloa-
cal exstrophy, and Pentalogy of Cantrell (epigastric om-
phalocele, sternal defect, pericardial defect, congenital
heart disease, anterior diaphragmatic hernia). A fetal
echocardiogram and karyotyping should be offered. An
omphalocele should be distinguished from a gastroschi-
sis. Gastroschisis is a smaller defect, located to the right
of the umbilical cord, and has no investing membrane.
Unlike an omphalocele, the liver is virtually never a part
of the hernia, and the only associated anomaly is an ap-
proximately 7% incidence of intestinal atresia. Fig. 2
This child had a solitary omphalocele with no associ-
ated anomalies. It was closed primarily and she recov-
ered uneventfully.

Suggested Reading
1. Kitchanan S, Patole SK, Muller R, Whitehall JS. Neonatal 3. Maksoud-Filho JG, Tannuri U, da Silva MM, Maksoud JG.
outcome of gastroschisis and exomphalos: a 10-year review. The outcome of newborns with abdominal wall defects ac-
J Paediatr Child Health 2000 Oct; 36(5):428–30 cording to the method of abdominal closure: the experience
2. Ledbetter DJ. Gastroschisis and omphalocele. Surg Clin of a single center. Pediatr Surg Int 2006 Jun; 22(6):503–7
North Am 2006 Apr; 86(2):249–60
7 Emergency and Trauma 467

Q 220
François Becmeur

Fig. 1
Fig. 2

A premature boy, born at 31 weeks of amenorrhea, pre- The doctors carried out a radiological study shown
sented with frankly bloody stools and abdominal disten- in Fig. 1.
sion at 8 days of life. An abdominal radiograph showed • What is your assessment of this study?
intestinal pneumatosis. • Which complementary examination would you re-
• Which diagnosis do you suspect? quire?
• Which treatment do you choose? • How would you describe Fig. 2 and what do you de-
duce for the diagnosis and treatment?
The patient’s evolution under treatment was favorable.
One month later, the infant presented with severe ab-
dominal distension associated with bile-stained vomit-
ing and cessation of intestinal peristalsis. There were no
clinical or biological infectious signs.
468 Emergency and Trauma

A 220

Necrotizing enterocolitis is suspected. The treatment (Fig. 2) we note the existence of a stenosis of the left colic
consists in placing the patient on bowel rest, parenteral angle, right colic narrowing, and dilation of the ileocecal
nutrition is administered, and intravenous broad-spec- loop.
trum antibiotics are given. The diagnosis of a postenterocolitis scar stenosis is
The premature infant presented with an intestinal oc- suggested.
clusive syndrome. Surgical exploration is performed by laparotomy. A
Figure 1 is a radiograph of the abdomen without stenosis of the left colic angle and a bridle of the epiploon
preparation. The radiograph shows a diffuse intestinal on the level of the ileocecal loop are found. A section of
distension due to the occlusion. the adherence and a resection of the colic stenosis with
To supplement the investigation, a rectal injection end-to-end colic anastomosis are performed.
of water-soluble medium is made. On this radiograph

Suggested Reading
1. Chan KL, Ng SP, Chan KW, Wo YH, Tam PK. Pathogenesis 3. Hallstrom M, Koivisto AM, Janas M, Tammela O. Labora-
of neonatal necrotizing enterocolitis: a study of the role of tory parameters predictive of developing necrotizing en-
intraluminal pressure, age and bacterial concentration. Pe- terocolitis in infants born before 33 weeks of gestation. J
diatr Surg Int 2003 Oct; 19(8):573–7 Pediatr Surg 2006 Apr; 41(4):792–8
2. Chardot C, Rochet JS, Lezeau H, Sen N, Brouillard V, Caey- 4. Henry MC, Lawrence Moss R. Surgical therapy for nec-
maex L, Verellen G, Otte JB, Gauthier F, Reding R. Surgical rotizing enterocolitis: bringing evidence to the bedside.
necrotizing enterocolitis: are intestinal lesions more severe Semin Pediatr Surg 2005 Aug; 14(3):181–9
in infants with low birth weight? J Pediatr Surg 2003 Feb; 5. Hofman FN, Bax NM, van der Zee DC, Kramer WL. Sur-
38(2):167–72 gery for necrotising enterocolitis: primary anastomosis or
enterostomy? Pediatr Surg Int 2004 Jul; 20(7):481–3
7 Emergency and Trauma 469

Q 221
Alessandro Settimi and Ciro Esposito

Fig. 1

A 7-month-old boy born after an uncomplicated preg- An abdominal color Doppler US (Fig. 1) was per-
nancy and delivery had normal growth without any formed that indicated the need for other procedures to
pathological condition until a few hours before his hos- define the diagnosis and establish the treatment.
pitalization because of alimentary vomiting accompa- • What did the abdominal US show?
nied by mucous vomiting; the child was transferred to • Which other procedures were performed and what
our institute. was their result?
On physical examination, an abdominal mass was • What was the diagnosis?
found at the epigastric region. On rectal exploration, a • What was the treatment?
small quantity of dark blood was found. • What was the follow-up?
470 Emergency and Trauma

A 221

The abdominal color Doppler US (Fig. 1) shows the typi-


cal feature of an intestinal intussusception with its char-
acteristic aspects.
To define the site of the intestinal invaginated loops, a
barium enema was performed, which also aimed to ob-
tain bloodless reduction of the intussusception (Fig. 2).
The barium enema demonstrated the presence of an
ileocecal invagination, with the minus sign correspond-
ing to the colon.
During the procedure, reduction of the invagination
was obtained and surgical disinvagination was therefore
unnecessary.
Twenty-four hours after the disinvagination, the in-
fant was doing well and was discharged.
In general, after a radiological resolution of the intus-
susception it is preferable to keep the patient hospital-
ized for about 24 h, and after two defecations of normal
stools the patient can be discharged without any par-
ticular therapy.

Fig. 2

Suggested Reading
1. Boehm R, Till H. Recurrent intussusceptions in an infant 4. Sonmez K, Turkyilmaz Z, Demirogullari B, Karabulut
that were terminated by laparoscopic ileocolonic pexie. R, Aral YZ, Konus O, Basaklar AC, Kale N. Conserva-
Surg Endosc 2003 May; 17(5):831–2 tive treatment for small intestinal intussusception associ-
2. Crystal P, Barki Y. Using color Doppler sonography-guided ated with Henoch-Schonlein’s purpura. Surg Today 2002;
reduction of intussusception to differentiate edematous 32(12):1031–4
ileocecal valve and residual intussusception. AJR Am J 5. van der Laan M, Bax NM, van der Zee DC, Ure BM. The
Roentgenol 2004 May; 182(5):1345 role of laparoscopy in the management of childhood intus-
3. Poll LW, Lackmann GM, May P, Willnow U, Braunstein S, susception. Surg Endosc 2001 Apr; 15(4):373–6
Engelbrecht V, Kemperdick H. Follicular lymphoid hyper-
plasia simulating intussusception in a 6-year-old boy: clini-
cal, radiological and histopathological findings. Australas
Radiol 2003 Dec; 47(4):453–6
8
Oncology

Case 222–249
472 Oncology

Introduction

Imaging studies are today the most important tool in Assessment of tumor response to treatment, as a pro-
the diagnosis, initial assessment of the extent, evaluation spective end point in clinical trials or as a guide for the
of the response to treatment, and follow-up of pediatric clinician in decisions regarding a single patient, is an-
malignancies. other important step in the management of pediatric
Modern advances in technology have allowed the de- solid tumors.
velopment of sophisticated imaging modalities such as Criteria to measure tumor lesions were defined by
proton magnetic resonance spectroscopy (MRS) or posi- the World Health Organisation in 1979. Recently, they
tron emission tomography (PET). However, these new have been revised by a large cooperative group in order
techniques entail a greater cost and increased difficulty to provide a simplified and reproducible response evalu-
in diagnostic procedures. ation method based on the use of unidimensional mea-
At the present time it can be accepted that conven- surements. This work has led to the publication of the
tional CT and MR imaging—for their sensitivity, ana- Response Evaluation Criteria in Solid Tumors (RECIST)
tomic delineation, and cost—are the best choice for the Guidelines, to which we refer.
detection of central nervous system (CNS), abdominal, At diagnosis, tumor lesions should be identified and
and pelvic lesions. Moreover, plain film radiographs defined as measurable if they can be accurately measured
must be considered of value as a first screening modal- in at least one dimension, turning out equal to or greater
ity in patients with suspected thoracic or skeletal abnor- than 20 mm with conventional techniques or to 10 mm
malities. with spiral CT scan. These lesions, up to a maximum of
However, it is to be noted that in the diagnosis of five per organ and ten in total, should be recorded at ini-
pediatric tumors, conventional CT and MR imaging tial staging as “target lesions.” Bone lesions, leptomenin-
should be performed with contiguous cuts of 10 mm or geal disease, ascites, pleural and pericardial effusions, and
less in slice thickness in order to completely detect le- cystic lesions are considered a priori nonmeasurable.
sions of at least 20 mm in longest diameter. As a rule, Correct evaluation of response requires that the same
we can correctly evaluate lesions no less than double the method of assessment and the same technique be used at
slice thickness applied. baseline and during follow-up. It is particularly impor-
Moreover, pulmonary lesions can be evaluated with tant, when using MR imaging, that lesions are measured
chest radiography only when they are clearly defined by the same imaging sequences on subsequent examina-
and surrounded by aerated lung. tions.
The above observations concern the baseline evalua- RECIST criteria of response are as follows: complete
tion of a child with a suspected diagnosis of malignancy, response is the disappearance of all target lesions; par-
but the evaluation of the extent of most solid tumors tial response is at least a 30% decrease in the sum of the
needs more accurate staging procedures. In the case of longest diameter of target lesions, taking as reference the
CNS location, MR imaging is considered to be more ac- baseline sum longest diameter; progressive disease is at
curate and to offer better resolutions than CT. CT with least a 20% increase in the sum of the longest diameter
and without contrast represents a far more valuable im- of target lesions, taking as reference the smallest sum
aging modality than plain film radiographs for staging longest diameter recorded since the treatment started or
lesions in the lungs or mediastinum. In such cases, even the appearance of one or more new lesions; stable dis-
more useful are spiral CT scan, also known as volumet- ease is neither sufficient shrinkage to qualify for partial
ric acquisition CT, and high-resolution CT; with both response nor sufficient increase to qualify for progres-
techniques there is little chance of missing small lesions sive disease, taking as reference the smallest sum longest
falling between slices, as can happen with conventional diameter since the treatment started.
CT. For bone lesions, radionuclide studies and MR im- We now believe that we already have optimal tools
aging have proven to be more sensitive than plain films in pediatric tumor imaging, provided that we use them
or CT in detecting early metastatic disease. correctly.
8 Oncology 473

Q 222
Sabine Sarnacki

Fig. 1 Fig. 2

Fig. 3

A 4-year-old girl presented with vaginal bleeding. Clini- Measurements of alpha-fetoprotein and human chori-
cal examination revealed a solid pelvic abdominal mass. onic gonadotropin (HCG) levels were performed, which
The mother noticed the appearance of pubic hair and were within normal ranges.
breast enlargement 2 months earlier. • Which additional biological examinations should be
A US examination was performed (Fig. 1). done?
• What complementary radiological examinations • What treatment should be proposed?
should be performed? • What is the follow-up?
The abdominal CT scan (Figs. 2, 3) shows a heteroge-
neous ovoid lesion with ascites.
• What is the important sign to observe on Fig. 2?
• How do you interpret the ascites?
474 Oncology

A 222

This 4-year-old girl presented with an abdominal mass occur in ruptured forms. Ascites without any rupture,
and signs of precocious pseudopuberty: vaginal bleed- which is ascribed to pseudo-Meigs’ syndrome, is present
ing, abnormal pubic hair development, and breast en- in 10% of cases. There is no prognostic value attached to
largement. This presentation suggests the presence of the ascites.
a secreting ovarian tumor. Juvenile ovarian granulosa The normal HCG level rules out the diagnosis of a
cell tumor (JGCT) and malignant germinal tumor with malignant germ cell tumor comprising choriocarcinoma
a choriocarcinoma component are the most frequently or embryonal carcinoma components. Thus, the diag-
encountered lesions, but embryonal carcinoma and Ser- nosis to retain is JGCT. This tumor usually synthesizes
toli–Leydig cell tumors may also be revealed by preco- estrogen, but an elevation of testosterone and/or dehy-
cious pseudopuberty signs. droepiandrosterone (DHEA) may be observed, probably
The US exam shows a heterogeneous mass with solid linked to a thecal testosterone-secreting area. These hor-
and cystic components. The ovarian origin of the lesion monal levels should be measured and be used as tumor
may be difficult to confirm on this examination, but is markers for the follow-up. Inhibin B and antimüllerian
suggested by the clinical presentation. An abdominal hormone levels are elevated in about 100% of cases and
and thoracic CT scan should be performed to evaluate return to normal after removal of the tumor. They are
the local extension of the lesion and eliminate the pres- good markers for follow-up, as they seem to increase
ence of distant metastasis. JGCT almost always presents several months before recurrence.
as stage-I disease; 3% of these tumors are bilateral and If the tumor is localized, as in the present case and
only 2% have extraovarian spread (peritoneum, lymph in the majority of JGCT observed during childhood
nodes, liver, and more rarely bone, lungs, CNS). and infancy (FIGO stage Ia), treatment consists of an
Figure 2 shows an enlargement of the uterus with en- ovariectomy with ascites analysis. Omentectomy is not
dometrial hyperplasia linked to the estrogen secretion. indicated in localized lesions. Laparoscopy is not recom-
Because of high estrogen secretion, these tumors are mended because of the risk of tumoral spillage during
rather vascular and bloody ascites and peritonitis may extraction and thus of relapse.

Suggested Reading
1. Kalfa N, Patte C, Orbach D, Lecointre C, Pienkowski C, 3. Schmitt R, Weichert W, Schneider W, Luft FC, Kettritz R.
Philippe F, Thibault E, Plantaz D, Brauner R, Rubie H, Pseudo-pseudo Meigs’ syndrome. Lancet 2005; 366:1672
Guedj AM, Ecochard A, Paris F, Jeandel C, Baldet P, Sultan 4. Schneider DT, Calaminus G, Harms D, Gobel U; German
C. A nationwide study of granulosa cell tumors in pre- and Maligne Keimzelltumoren Study Group. Ovarian sex cord-
postpubertal girls: missed diagnosis of endocrine mani- stromal tumors in children and adolescents. J Reprod Med
festations worsens prognosis. J Pediatr Endocrinol Metab 2005; 50:439–446
2005; 18:25–31
2. Lane AH, Lee MM, Fuller AF Jr, Kehas DJ, Donahoe PK,
MacLaughlin DT. Diagnostic utility of Mullerian inhibit-
ing substance determination in patients with primary and
recurrent granulosa cell tumors. Gynecol Oncol 1999;
73:51–55
8 Oncology 475

Q 223
Sabine Sarnacki

Fig. 1 Fig. 2

A 7-year-old girl presented with abdominal distension.


Clinical examination revealed a solid abdominal mass,
and a pelvic CT scan showed bilateral ovarian masses
(Figs. 1, 2) with ascites.
• How would you proceed to make a diagnosis?
• What do you see on the abdominal CT scan that di-
rects the diagnosis?
• As a surgeon, how would you manage this situation?
476 Oncology

A 223

All pediatric ovarian tumors may potentially be bilat- ence of ascites, the diagnosis of lymphoma is more likely
eral, but this is very rare except in pure mature teratoma than the diagnosis of metastatic neuroblastoma. Other
where the bilaterality is about 8%–15% depending on locations such as peripheral, abdominal, or mediastinal
the series. In contrast, the ovaries may be the second- lymph nodes have to be searched for.
ary site of a primary neoplasm such as leukemia, non- The treatment of a lymphoma is an emergency. The
Hodgkin’s lymphoma (mainly Burkitt’s lymphoma), suspicion of this diagnosis in a department of surgery
neuroblastoma, alveolar rhabdomyosarcoma, Wilms’ should lead to contact being made with a team of pedi-
tumor, mucinous colon adenocarcinoma, and rhabdoid atric oncologists to find out how to proceed. If the pa-
tumors. This hypothesis should be raised in the present tient has to be anesthetized for the fine-needle aspiration
case. The best way to make the diagnosis is to perform and/or needle biopsy of the masses, it is better to take
ultrasonography-guided percutaneous fine-needle as- advantage of the anesthesia to complete the evaluation
piration of the ascites and, if negative, ultrasonography- of the disease (medullar biopsies, myelogram, lumbar
guided percutaneous fine-needle aspiration or biopsy of puncture). The question of a central catheter or of a port-
the ovarian masses. a-catheter should be raised because the treatment will
The CT scan shows abundant ascites and a tumoral begin as soon as possible.
syndrome of the left adrenal gland. Because of the pres-

Suggested Reading
1. McCarville MB, Hill DA, Miller BE, Pratt CB. Secondary 3. Young RH, Kozakewich HP, Scully RE. Metastatic ovarian
ovarian neoplasms in children: imaging features with histo- tumors in children: a report of 14 cases and review of the
pathologic correlation. Pediatr Radiol 2001; 31:358–364 literature. Int J Gynecol Pathol 1993; 12:8–19
2. Somjee S, Kurkure PA, Chinoy RF, Deshpande RK, Advani
SH. Metastatic ovarian neuroblastoma: a case report. Pedi-
atr Hematol Oncol 1999; 16:459–462
8 Oncology 477

Q 224
Sabine Sarnacki

Fig. 2

Fig. 1 Fig. 3

A 26-month-old girl presented with a fever (38.5°C) Abdominal and pelvic US examinations and a CT scan
without abdominal pain. Clinical examination revealed were performed (Figs. 2, 3).
a solid abdominal mass. • What information is provided by these examina-
• What is the key sign of the clinical examination tions?
that could be helpful in delineating the origin of the • What biological examinations should be performed
mass? before surgery? Why?
• What type of surgery would you propose for this pa-
A standard abdominal radiograph was acquired (Fig. 1). tient?
• How do you interpret this examination?
• At this stage of the observation, what should you do?
478 Oncology

A 224

The mobility of the mass must be checked, as it clearly solid and cystic components as well as the calcifications
indicates its mesenteric or ovarian origin. Because of the previously found on the standard abdominal radiograph.
higher frequency of ovarian masses in girls, the mobil- These images do not provide any additional information
ity of the mass in this case suggests an ovarian origin. in the present case because: the mobility of the mass at-
Fever in this situation is confusing but can be ascribed tests the ovarian origin, the presence of calcifications
to an ordinary infection, and the discovery of the mass suggests a diagnosis of mature teratoma, and the nega-
is incidental. tivity of the markers eliminates a malignant germ cell
The abdominal radiograph shows multiple calcifica- tumor. This will lead to primary surgery. The side of the
tions in the right part of the abdomen. Although calcifi- lesion could not be identified on these examinations (in
cations are classically found in neuroblastoma and some this case it was a left ovarian teratoma). US and CT scan
Wilms’ tumors, the presence of calcifications in a mobile may be helpful to ensure the normality of the opposite
mass leads to the diagnosis of ovarian tumor. ovary and to evaluate the importance of the remaining
Considering the aspect of the calcifications, the first ovarian parenchyma when conservative surgery can be
diagnosis to propose is a mature teratoma of the ovary. attempted.
Clinical examination does not show any sign of preco- Beside the usual examinations required for any op-
cious pseudopuberty and thus eliminates the diagnosis eration, it is preferable to obtain a blood sample for mea-
of ovarian granulosa cell tumor. Surgery may be pro- surement of alpha-fetoprotein and HCG markers. The
posed without any further investigation except for the elevation of one of these markers shows a malignant
measurement of alpha-fetoprotein and human chorionic component of the lesion and additional chemotherapy
gonadotropin (HCG) levels in order to eliminate the may be required. This information is also crucial for the
possibility of a malignant germinal tumor component. follow-up, which includes measurement of the initially
The imaging pattern of ovarian teratomas is a com- elevated marker.
plex mass in two-thirds of cases. Typical tooth-like or If the markers are negative, conservative surgery (tu-
rim calcifications or ossifications are seen in 56% of morectomy sparing the normal ovarian tissue) should
cases. The characteristic sonographic appearance is that be attempted, because the risk of onset of a contralateral
of a hypoechoic mass with an echogenic mural nodule tumor is about 10% in cases of teratoma. In the pres-
frequently associated with acoustic shadowing linked ent case, the volume of the tumor probably precludes the
to either calcification or a matted mixture of sebum feasibility of a tumorectomy. Although this tumor is cer-
and hair. In all, 10%–15% of benign teratomas appear tainly a mature benign teratoma (markers negative, nu-
as purely anechoic masses. Another 10%–15% contain merous calcifications), laparoscopy is not recommended
primarily hair and fat and a sparse amount of fluid or se- because a malignant nonsecreting component could
baceous material, and they are predominantly echogenic. not be formally eliminated as well as the risk of tumoral
Figures 2 and 3 show a very heterogeneous lesion with spillage during extraction.

Suggested Reading
1. Baranzelli MC, Bouffet E, Quintana E, Portas M, Thyss A, 3. Garel L, Dubois J, Grignon A, Filiatrault D, Van Vliet G. US
Patte C. Non-seminomatous ovarian germ cell tumours in of the pediatric female pelvis: a clinical perspective. Radio-
children. Eur J Cancer 2000; 36:376–383 graphics 2001; 21:1393–1407
2. de Silva KS, Kanumakala S, Grover SR, Chow CW, Warne 4. Templeman CL, Fallat ME, Lam AM, Perlman SE, Hert-
GL. Ovarian lesions in children and adolescents—an 11- weck SP, O’Connor DM. Managing mature cystic teratomas
year review. J Pediatr Endocrinol Metab 2004; 17:951–957 of the ovary. Obstet Gynecol Surv 2000; 55:738–745
8 Oncology 479

Q 225
Nancy Rollins and Korgun Koral

Fig. 1 Fig. 2

Fig. 3 Fig. 4

A 5-month-old boy presented with an abdominal mass • What are the findings?
that was detected by his parents. Abdominal ultrasound • What is the differential diagnosis?
showed a large mass in the liver. Abdominal CT without • What is the most likely diagnosis?
and with intravenous contrast material was performed
(Figs. 1–4). The alpha-fetoprotein level was elevated.
480 Oncology

A 225

Noncontrast CT of the abdomen shows a large, round,


hypodense mass occupying much of the liver. No calci-
fications are present. Contrast-enhanced images show
the solid nature of the mass, which demonstrates hetero-
genous enhancement. The interface with the normal
liver parenchyma is well delineated (Fig. 2, arrows). The
right kidney is displaced by the mass.
The differential diagnosis includes hepatoblastoma,
hemangioendothelioma, neuroblastoma metastasis, mes-
enchymal hamartoma, and hepatocellular carcinoma.
Hepatoblastoma is the most likely diagnosis given
the patient’s age and the elevated alpha-fetoprotein level.
Hepatoblastoma is the most common hepatic malig-
nancy in children. The role of imaging is to define the
anatomy for preoperative planning and to monitor re-
sponse to chemotherapy.

Suggested Reading
1. Donnelly LF. Pocket Radiologist: Pediatrics. Amirsys Ed.
2002
8 Oncology 481

Q 226
Nancy Rollins and Korgun Koral

Fig. 2

Fig. 1

A 5-year-old boy with bone pain underwent a whole-


body bone scan following administration of 5 mCi of
technetium 99m.
• What does the bone scan show (Fig. 1)?
• What are the findings on the CT scan (Figs. 2–4)?
• What is the differential diagnosis?
• What is the most likely diagnosis? Fig. 3
• What is the role of imaging in the staging of the dis-
ease?

Fig. 4
482 Oncology

A 226

A large area of abnormal uptake is seen in the right side


of the abdomen.
There is a large, round, solid lesion with multiple foci
of calcifications. The lesion is primarily retroperitoneal
with encasement of the retroperitoneal vessels. The right
kidney is displaced, but not infiltrated.
Wilms’ tumor is the main differential diagnosis, but
the extrarenal location of the lesion is helpful in exclud-
ing this possibility. In newborns, adrenal hemorrhage is
a consideration. Other rare tumors of the adrenal glands,
such as paraganglioma and adrenal carcinoma, are also
in the differential diagnosis.
The most likely diagnosis is neuroblastoma.
Imaging is helpful in distinguishing stage-1 disease
(confined to organ of origin) from stage 2 (outside the
confines of organ of origin). Stage 3 (tumor crossing
midline) and stage 4 (distant metastasis) can also be
distinguished with imaging. For a particular subset of
patients (stage 4S, age <1 year, metastatic disease con-
fined to skin, liver, and bone marrow) there is an excel-
lent prognosis.

Suggested Reading
1. Donnelly LF. Pocket Radiologist: Pediatrics. Amirsys Ed.
2002.
2. Kuhn JP, Slovis TL, Haller JO. Caffey’s Pediatric Diagnostic
Imaging. Kuhn JP, Slovis TL, Haller JO. Elsevier Ed. 2004
3. Siegel MJ, Coley BD. Pediatric Imaging. Lippincott Wil-
liams & Wilkins Ed. 2006
8 Oncology 483

Q 227
Nancy Rollins and Korgun Koral

Fig. 1 Fig. 2

A 3-year-old girl was found to have an abdominal mass


during routine physical examination at her pediatrician.
CT of the abdomen was performed (Figs. 1–3).
• What is the differential diagnosis prior to the CT ex-
amination?
• What does the scan show?
• What is the most likely diagnosis?
• What is the differential diagnosis?
• Why was a CT scan of the chest performed?
• What is the role of imaging in staging this disease?

Fig. 3
484 Oncology

A 227

In a child, the most common abdominal mass is hydro-


nephrosis. Neoplasms such as neuroblastoma, Wilms’
tumor, and lymphoma are in the differential diagnosis.
An ultrasound examination of the abdomen should be
the first step to distinguish between hydronephrosis and
solid abdominal masses.
The CT scan (Figs. 1–3) shows a large mass arising
from the right kidney. Unlike extrarenal masses, the kid-
ney is not only displaced but is also infiltrated. In Fig. 2,
the right renal vein does not opacify normally and tu-
mor thrombus is visible in the displaced inferior vena
cava. There is normal opacification of the left renal vein.
The differential diagnosis includes neuroblastoma,
multilocular cystic nephroma, clear cell sarcoma, and Fig. 4
rhabdoid tumor of the kidney.
The CT of the chest was performed to exclude lung
metastasis. Several nodules were identified, the largest
of which is in the superior segment of the right lower
lobe (Fig. 4).
Staging of Wilms’ tumor is surgical. Imaging is help-
ful in surgical planning and identifying distant metas-
tasis (stage IV). Bilateral disease (stage V) can also be
diagnosed with imaging. An abdominal ultrasound ex-
amination (Fig. 5) of another child shows bilateral large,
solid masses in the lumbar regions. CT of the abdomen
(Fig. 6) shows the solid, heterogenous masses arising
from the kidneys. In the left-sided mass, foci of fat are
present (arrow) representing entrapment of perirenal fat
in the tumor. Fig. 5

Suggested Reading
1. Beckwith JB. National Wilms’ Tumor Study: an update for
pathologists. Pediatr Dev Pathol 1998; 1:79
2. Breslow NE et al. Clinicopathologic features and progno-
sis for Wilms’ tumor patients with metastases at diagnosis.
Cancer 1986; 58: 2501
3. Ritchey ML et al. Management and outcome of inoperable
Wilms’ tumor. A report of National Wilms’ Tumor Study-3.
Ann Surg 1994; 220:683

Fig. 6
8 Oncology 485

Q 228
Alfonso Papparella, Mercedes Romano, and Pio Parmeggiani

Fig. 1

Fig. 2

A 10-year-old patient was affected by bilateral gyneco- An NMR scan was requested.
mastia: Tanner stage P2, with sparse pubic hair and nor- • What does Fig. 2 show?
mal penis size for his age. No symptoms were reported. • What additional diagnostic tests would you suggest
The mother consulted an endocrinologist, who per- to reach a diagnosis?
formed standard hormone screening, as well as mam- • What is the most appropriate surgical treatment?
mary and testicular ultrasound tests.
• What are the (expected) results of the standard hor-
mone screening?
• What does Fig. 1 (ultrasound) show?
• What is the most likely diagnosis?
• What is the differential diagnosis?
486 Oncology

A 228

The diagnostic suspicion is that of testicular Leydigioma.


Gynecomastia is detected in 20%–30% of these patients.
In most cases, this benign tumor is sometimes diagnosed
by chance, given its small size, and often in pediatric age
(the peak incidence is in 4–5-year-old children).
In this case, on palpation of the right testis, there was
a small intraparenchymal mass with a hard consistency,
closely connected to the surrounding parenchyma, with
no alterations of the epididymis and hydrocele.
Routine hormonal screening showed an increase in
serum testosterone levels (2.0 ng/ml), whereas the levels
of serum alpha-fetoprotein (1.3 ng/ml), carcinoembry-
onic antigen (CEA; 5.2 ng/ml), and beta-human cho-
rionic gonadotropin (HCG; 0.1 mIU/ml) were within
normal ranges.
The ultrasound image of the right side shows a small Fig. 3
hypoechoic polylobular lesion with a small area of col-
liquation and calcifications (1 cm×1 cm; Fig. 1). The
color Doppler showed a richly vascularized didymus, al-
though the mass was poorly vascularized; no abnormali-
ties involving the left didymus were detected.
The differential diagnosis includes early puberty due
to a pituitary lesion, Leydig’s cell hyperplasia, and nodu-
lar testicular hyperplasia associated with pediatric un-
treated congenital adrenal hyperplasia.
When dealing with a patient affected by testicular
Leydigioma, a full hormonal screening is warranted, in-
cluding urinary 17-ketosteroids, abdominal ultrasound,
and MR imaging (Fig. 2). In this case, the NMR scan
confirmed the presence of the small area in the right tes-
tis and no involvement of the satellite lymph node sta-
tions and of other abdominal organs such as the adrenal
glands.
The surgical treatment of choice is orchifunicolec-
tomy (Fig. 3), even though tumor enucleation has been
reported. From the histopathological perspective, the Fig. 4
macroscopic appearance of the testicular tissue is yel-
lowish-brown in color (Fig. 4), given its role in the pro-
duction of hormones. Suggested Reading
1. Ciftci AO et al. Testicular tumors in children. J Pediatr Surg
2001; 36(12):1796–801
2. Carmignani L et al. High incidence of benign testicu-
lar neoplasms diagnosed by ultrasound. J Urol 2003;
170(5):1783–6
3. Henderson CG et al. Enucleation for prepubertal Leydig
cell tumor. J Urol 2006; 176(2):703–5
8 Oncology 487

Q 229
Masayuki Fujioka, Carl Muroi, Nadia Khan, and Yasuhiro Yonekawa

Fig. 1 Fig. 2

A 14-year-old girl presented with seizures accompanied The brain lesion did not appear on T1-weighted MR im-
by loss of consciousness that had started 1 year earlier. aging.
A head CT scan revealed no marked findings at that • What kind of MR imaging examination should be
time and she was diagnosed as having juvenile myo- considered next?
clonic epilepsy (Janz syndrome).
The girl visited our hospital for further investigations Electroencephalography showed epileptiform activity
because the epilepsy became intractable. in the bilateral frontal cerebral regions (more active in
MR imaging was performed. the left). [(18)F]-fluorodeoxyglucose positron emis-
• What do Figs. 1 (T2-weighted image) and 2 (FLAIR) sion tomography (FDG-PET) showed the mild uptake
show? of [(18)F]-FDG in the hippocampal lesion observed on
MR imaging.
• What is the possible diagnosis of the lesion?
• How should the lesion be treated?
488 Oncology

A 229

Fig. 3 Fig. 4 Fig. 5

The MR imaging study reveals a lesion of hyperintensity incision, supracerebellar transtentorial approach). The
in the posterior portion of the left hippocampus on both tumor was removed completely. The histological study
T2-weighted and FLAIR images (Figs. 1, 2). The tumoral showed that the tumor was a ganglioglioma.
lesion appears to be located mainly in the cerebral cortex. Gangliogliomas are glioneuronal tumors of children
A tumor derived from neuronal cells, therefore, should and young adults, and occur more frequently in the su-
be considered as a possible etiology. pratentorial region, especially the temporal lobe. Gan-
A gadolinium-diethylenetriamine pentaacetic acid glioglioma is a basically benign tumor composed of neo-
(Gd-DTPA)-enhanced T1-weighted MR imaging study plastic astrocytes (rarely oligodendrocytes) and ganglion
was performed (Figs. 3–5). These images show homoge- cells. Solid gangliogliomas may be missed on CT scan-
neous enhancement of the tumor. ning but revealed on MR images. Gangliogliomas tend
The MR imaging demonstrated no marked mass ef- to exhibit increased signal intensity on T2-weighted im-
fect and no surrounding edema of the tumor. The homo- ages, whereas they may be difficult to recognize on T1-
geneous enhancement with Gd-DTPA on T1-weighted weighted images. The solid portion is usually enhanced
MR imaging and the slight uptake of 18F-FDG on the on Gd-DTPA T1-weighted MR images.
PET study suggest the tumor is relatively benign. After 6 months, the epilepsy was well controlled with
The tumor that led to the intractable epilepsy was sur- an anti-epilepsy drug and the follow-up MR imaging
gically removed without any perioperative complications showed no recurrence of the tumor. At the time of writ-
(in the sitting position, left paramedian suboccipital skin ing, the girl was going to school without any sequelae.

Suggested Reading
1. Castillo M, Davis PC, Takei Y, Hoffman JC Jr. Intracranial 2. Haddad SF, Moore SA, Menezes AH, VanGilder JC. Gan-
ganglioglioma: MR, CT, and clinical findings in 18 patients. glioglioma: 13 years of experience. Neurosurgery. 1992;
AJR Am J Roentgenol 1990; 155: 899–900. 31:171–178
8 Oncology 489

Q 230
Masayuki Fujioka, Carl Muroi, Nadia Khan, and Yasuhiro Yonekawa

Fig. 1 Fig. 2 Fig. 3

A 12-year-old girl had difficulty in walking (unstable CT scans showed a space-occupying lesion of isoden-
gait) for 2 weeks and had been complaining of a head- sity in the intracranial posterior fossa.
ache, vomiting, and visual disturbance (weak eyesight) The tumoral lesion appeared hypointense on T1-
for 1 week. weighted MR images and hyperintense on T2-weighted
On admission to hospital, she had truncal ataxia, imaging. Gd-DTPA T1-weighted MR imaging was per-
staggering gait, bilateral abducens nerve palsy, intermit- formed.
tent vertical and horizontal nystagmus, oral dyskinesia, • What do Figs. 1–3 (axial, coronal, and sagittal views)
incomplete palsy of the bilateral oculomotor nerves and show?
of the bilateral facial nerves, and weak response of the • What is a possible diagnosis based on the clinical and
right pharyngeal reflex. radiological data?
• Which treatment should be considered?
490 Oncology

A 230

The tumor occupies the fourth ventricle entirely and has


Gd enhancement effects on T1-weighted MR images.
The MR imaging study shows dilatation of the lateral
ventricle, which suggests obstructive hydrocephalus.
The differential diagnosis for pediatric brain tumors
in the posterior fossa includes medulloblastoma, epen-
dymoma, and astrocytoma.
The tumor was totally removed surgically without
any complication (sitting position, occipital midline
skin incision, suboccipital approach). The pathologi-
cal study revealed medulloblastoma. Medulloblastoma
is the most common malignant pediatric brain tumor
and usually arises within the cerebellar vermis, in the
apex of the roof of the fourth ventricle (fastigium). It is
composed of densely packed cells with round to oval or
carrot-shaped nuclei and scanty cytoplasm. The neuro-
blastic Homer Wright rosette is one of the characteristic
histological features.
The patient underwent postoperative chemotherapy
and radiation therapy.
At the time of writing, the girl was doing very well
and studying at school 1 year after the operation. Gd-
DTPA T1-weighted MR imaging (Figs. 4, 5) shows no Fig. 4
evidence of recurrence of the tumor.

Suggested Reading
1. Chawla A, Emmanuel JV, Seow WT, Lou J, Teo HE, Lim
CC. Paediatric PNET: pre-surgical MRI features. Clin Ra-
diol 2007; 62:43–52
2. Han JS, Benson JE, Kaufman B, Rekate HL, Alfidi RJ, Huss
RG, Sacco D, Yoon YS, Morrison SC. MR imaging of pedi-
atric cerebral abnormalities. J Comput Assist Tomogr 1985;
9:103–114
3. Packer RJ, Batnitzky S, Cohen ME. Magnetic resonance im-
aging in the evaluation of intracranial tumors of childhood.
Cancer 1985; 56:1767–1772

Fig. 5
8 Oncology 491

Q 231
Masayuki Fujioka, Carl Muroi, Nadia Khan, and Yasuhiro Yonekawa

Fig. 1 Fig. 2 Fig. 3

A 6-month old boy was found to be hypoactive for his This tumor appeared isointense on T1-weighted imag-
age. He was not able to sit by himself at 11 months. ing and was homogeneously enhanced with Gd-DTPA
The family doctor diagnosed the child with truncal on T1-weighted images.
ataxia and muscle hypotonia. • What pathological condition is affecting this child?
A CT study revealed a brain tumor in the posterior • What is the best way to manage this condition?
cranial fossa as an isodense space-occupying lesion, and
the boy was referred to a neurosurgical unit.
• What do Figs. 1–3 (T2-weighted MR imaging) show?
492 Oncology

A 231

Fig. 4 Fig. 5 Fig. 6

The T2-weighted MR imaging demonstrates that the Total removal of the tumor was performed on the day
large hyperintense tumor occupies the space from the after admission (sitting position, occipital midline skin
left lateral recess of the fourth ventricle through to the incision, suboccipital craniotomy) without any surgical
fourth ventricle, resulting in obstructive hydrocephalus. complication.
This 1-year-old boy has a posterior fossa tumor in the The histological study revealed a choroid plexus pap-
fourth ventricle. The possible diagnosis of the tumor in- illoma.
cludes choroid plexus papilloma, medulloblastoma, and The tumor-removal operation resulted in no ad-
ependymoma. ditional neurological deficits. An MR imaging study
The tumor compresses the brainstem from the left showed no apparent residual tumor 3 months after the
posterior side. Emergent removal of the tumor is con- operation (Figs. 4–6). At the time of writing, the boy’s
sidered necessary. motor activity was improving and he was able to sit by
himself.

Suggested Reading
1. Aksoy FG, Gomori JM. Choroid plexus papilloma of fora- 4. Shin JH, Lee HK, Jeong AK, Park SH, Choi CG, Suh DC.
men of Luschka with multiple recurrences and cystic fea- Choroid plexus papilloma in the posterior cranial fossa:
tures. Neuroradiology 1999; 41:654–656 MR, CT, and angiographic findings. Clin Imaging 2001;
2. Martin N, Pierot L, Sterkers O, Mompoint D, Nahum H. 25:154–162
Primary choroid plexus papilloma of the cerebellopontine 5. Tasdemiroglu E, Awh MH, Walsh JW. MRI of cerebello-
angle: MR imaging. Neuroradiology 1990; 31:541–543 pontine angle choroid plexus papilloma. Neuroradiology
3. Sarkar C, Sharma MC, Gaikwad S, Sharma C, Singh VP. 1996; 38:38–40
Choroid plexus papilloma: a clinicopathological study of 23
cases. Surg Neurol 1999; 52:37–39
8 Oncology 493

Q 232
Masayuki Fujioka, Carl Muroi, Nadia Khan, and Yasuhiro Yonekawa

Fig. 2

Fig. 1

An 11-year-old boy presented with obesity, increased ap-


petite (hyperphasia), and growth failure (short stature)
that had started when he was 7 years old.
He had been suffering from dizziness, general fatigue,
and headache for 2 weeks, and from decreased visual
acuity and diplopia for 1 week. He also suffered from
downward ocular conjugate deviation for 2 days.
Gd-DTPA-enhanced T1-weighted MR imaging was
performed on admission (shown in Figs. 1–3).
• What do these figures reveal?
With Gd enhancement, the lesion appeared hypoin-
Fig. 3
tense on T1-weighted images and hyperintense on T2-
weighted images.
• What does the differential diagnosis include?
• What is the best way to manage this condition?
494 Oncology

A 232

Fig. 5

Fig. 4

The long-term presence of obesity, hyperphasia, and The tumor was surgically removed (supine position,
growth failure indicates hypothalamus-pituitary grand frontal craniotomy, interhemispheric trans-rostrum
system dysfunction. corporis callosi/lamina terminalis approach). The tu-
The Gd-T1-weighted MR imaging shows the tumor mor was arising from the left hypothalamus. The histo-
consisting of solid and cystic portions with the wall pathological study revealed a pilocytic astrocytoma. MR
enhanced in the third ventricle. The optic chiasm and imaging after the operation (Figs. 4, 5) showed that the
pituitary stalk are pressed from the top and deviated to- tumor was removed subtotally.
ward the bottom. The left lateral ventricle is remarkably At the time of writing, the boy was doing well and
dilated compared with the right. going to school everyday without apparent regrowth of
The differential diagnosis for a pediatric suprasellar the tumor.
tumor that occupies the third ventricle includes cranio-
pharyngioma, glioma, and germ cell tumor.

Suggested Reading
1. Bernaerts A, Vanhoenacker F, Debois V, Parizel PM. Juve- 3. Kayama T, Tominaga T, Yoshimoto T. Management of pilo-
nile pilocytic astrocytoma. JBR-BTR 2003; 86:142–143 cytic astrocytoma. Neurosurg Rev 1996; 19:217–220
2. Fernandez C Figarella-Branger D, Girard N, Bouvier-Labit
C, Gouvernet J, Paz Paredes A, Lena G. Pilocytic astrocyto-
mas in children: prognostic factors—a retrospective study
of 80 cases. Neurosurgery 2003; 53:544–553
8 Oncology 495

Q 233
Masayuki Fujioka, Carl Muroi, Nadia Khan, and Yasuhiro Yonekawa

Fig. 1 Fig. 2 Fig. 3

A 9-year-old boy presented with partial epileptic seizure


and secondary generalization leading to unconscious-
ness.
• What do the pre- and postcontrast CT scans (Figs. 1,
2) show?

Figures 3 and 4 represent T1-weighted and T2-weighted


MR images, respectively.
• What are the findings from Figs. 3 and 4?
The boy was treated conservatively with carbamazepine,
and periodical follow-up imaging studies were per-
formed with CT and MR imaging. Fig. 4
After the initiation of medication, the attacks of un-
consciousness disappeared. However, 2 years later, the
patient developed olfactory hallucinations (once every
several months), and consequently developed absence
attacks.
In addition, auditory hallucinations occurred, and
these partial complex seizures became tractable only
with medication.
• What is a possible diagnosis of the lesion on the MR
imaging study?
• What is the best way to manage this condition?
496 Oncology

A 233

Fig. 5 Fig. 6 Fig. 7

The precontrast CT scan (Fig. 1) shows a cortical lesion The lesion is considered very slow growing because
of low density in the right posterior temporal lobe. The the patient has a long history of partial complex seizures
lesion extends toward the trigone of the lateral ventricle. and imaging does not show any edema and/or mass ef-
The lesion has no mass effect and no enhanced portions fects of the lesion. For cortically based tumors with cys-
on the postcontrast CT scan (Fig. 2). tic components in children, the differential diagnosis in-
The lesion appears hypointense on T1-weighted cludes ganglioglioma, dysembryoplastic neuroepithelial
MR imaging and hyperintense on T2-weighted imag- tumor, and pleomorphic xanthoastrocytoma.
ing (Figs. 3, 4). The MR images show a wedge-shaped, The seizures were uncontrollable with medication.
multicystic (bubbly) mass lesion in the temporal cortex Electroencephalography revealed that the tumor was
without apparent surrounding edema. of epileptogenic focus. Therefore, total removal of the
The Gd-enhanced T1-weighted MR image (Fig. 5) tumor was performed without any intra- or postopera-
shows no marked enhancement of the lesion. The T2- tive complications (Fig. 7, Gd-T1-weighted, axial view).
weighted MR image (sagittal view, Fig. 6) shows the Histology revealed a dysembryoplastic neuroepithelial
bubbly mass extending from the posterior temporal lobe tumor.
(superior and middle temporal gyrus) through the an- After the operation, the seizures subsided and dis-
gular gyrus. appeared. At the time of writing, the boy was going to
school without any sequelae.

Suggested Reading
1. Fernandez C, Girard N, Paz Paredes A, Bouvier-Labit C, 3. Ostertun B et al. Dysembryoplastic neuroepithelial tumors:
Lena G, Figarella-Branger D. The usefulness of MR imaging MR and CT evaluation. AJNR 1996; 17:419–430
in the diagnosis of dysembryoplastic neuroepithelial tumor 4. Stanescu Cosson R, Varlet P, Beuvon F, Daumas Duport C,
in children: a study of 14 cases. AJNR Am J Neuroradiol Devaux B, Chassoux F, Fredy D, Meder JF. Dysembryoplas-
2003; 24:829–834 tic neuroepithelial tumors: CT, MR findings and imaging
2. Lee DY et al. Dysembryoplastic neuroepithelial tumor: Ra- follow-up: a study of 53 cases. J Neuoradiol 2001; 28:230–
diological findings (including PET, SPECT, and MRS) and 240
surgical strategy. J Neurooncol 2000; 47:167–174
8 Oncology 497

Q 234
Masayuki Fujioka, Carl Muroi, Nadia Khan, and Yasuhiro Yonekawa

Fig. 1 Fig. 2 Fig. 3 Fig. 4

An 8-year-old girl had been suffering from vomiting in • What do the pre- and postcontrast CT scans show
the morning for 2 months. In addition, she had a 2-week (Figs. 1, 2)?
history of headache, unstable gait, and frequent fall- • What do the T1-weighted MR images with and with-
ing down. She complained of difficulties in studying at out Gd-DTPA show (Figs. 3, 4)?
school due to attention deficit. • What pathological condition is affecting this child?
On admission, a neurological study showed bilateral • What is the best way to manage this condition?
abducens nerve palsy, ataxic gait, abnormal finger-to-
nose test results, and dysphagia.
498 Oncology

A 234

Fig. 5 Fig. 6 Fig. 7

Figure 1 shows a fourth-ventricle solid tumor of rela- The differential diagnosis for pediatric mid-posterior
tively high density on the precontrast CT scan. The dila- fossa tumor includes medulloblastoma, ependymoma,
tation of the inferior horns of the bilateral lateral ventri- glioma (cerebellar pilocytic astrocytoma and dorsally
cles indicates the presence of obstructive hydrocephalus. exophytic brainstem glioma), choroid plexus papilloma,
Figure 2 shows slight heterogeneous enhancement of and atypical teratoid/rhabdoid tumor (usually off-mid-
the tumor on the postcontrast CT scan. line).
The spherical tumor appears relatively hypointense For the diagnosis and the cure, the tumor was totally
to brain matter on T1-weighted MR imaging (Fig. 3) removed (sitting position, middle suboccipital crani-
and occupies the space of the fourth ventricle and mid- otomy, infra-vermis approach) (Fig. 7, postoperative Gd-
cerebellar area. The signal on the T1-weighted image is T1-weighted MR image). The histological diagnosis was
slightly heterogeneous, which is considered to be due to medulloblastoma.
the presence of small cysts and clefts inside the tumor. After the operation, no additional neurological defi-
The Gd-T1-weighted image (Fig. 4) shows heteroge- cits developed. The patient underwent postoperative
neous enhancement of the tumor without surrounding chemotherapy and radiation therapy. The patient’s ataxia
brain edema. improved and her gait returned to normal 3 months
Figures 5 and 6 (Gd-T1-weighted sagittal and coro- after the operation. The bilateral abducens palsy also
nal views, respectively) show a fourth-ventricle tumor improved. However, at the time of writing, the patient’s
with a cystic portion associated with the hydrocephalus. dysphagia continued and she was undergoing rehabilita-
The solid portion of the tumor shows an enhancement tion therapy.
effect. The tumor pushes the brainstem forward and the
cerebellum backward.
8 Oncology 499

Q 235
Masayuki Fujioka, Carl Muroi, Nadia Khan, and Yasuhiro Yonekawa

Fig. 1 Fig. 2 Fig. 3

A 15-year-old boy complained of deterioration in his poglossal nerve palsy; however, there was no apparent
visual acuity, which had started 1 week earlier. His fam- motor weakness of the upper and lower extremities. His
ily consulted an ophthalmologist. The ophthalmologic consciousness level was clear and alert.
study revealed papilledema in the bilateral ocular fun- • What do Figs. 1 and 2 (T1- and T2-weighted MR im-
dus and right papillary dilatation. ages, axial views) show?
A CT study was performed immediately afterward, • What does Fig. 3 (Gd-T1-weighted MR images, axial
revealing an intracranial tumor, and the boy was re- and sagittal views, respectively) show?
ferred to our neurosurgical unit. • What pathological condition is affecting this child?
The neurological examination showed right oculo- • What is the best way to manage this condition?
motor nerve palsy, right facial nerve palsy, and right hy-
500 Oncology

A 235

Fig. 4 Fig. 5 Fig. 6

Figures 1 and 2 show a dorsal brainstem tumor with neuroimaging study. Therefore, this tumor seems to be a
solid and cystic portions. The solid portion of the tu- slow-growing cystic brainstem tumor with mural nod-
mor appears iso- to hypointense on T1-weighted im- ules such as a low-grade astrocytoma that extends in an
ages and moderately to heterogeneously hyperintense extra-axial direction.
on T2-weighted images. There seems to be no apparent This tectal tumor was removed subtotally (sitting
evidence of edema in the brain matter. The dilatations position, occipital and suboccipital craniotomy, supra-
of the inferior horns (lateral ventricles) and of the third cerebellar paraculmen approach, and occipital transten-
ventricle (sagging/extending downward) suggest ob- torial approach). The histopathological study showed a
structive hydrocephalus. pilocytic astrocytoma (WHO grade1) characterized by
The solid portion—but not the cyst wall—of the a biphasic pattern with a dense fibrillary matrix com-
tumor is heterogeneously enhanced on the Gd-T1- posed of bipolar cells and Rosenthal fibers and with
weighted images (Fig. 3). The tumor is located in the loosely structured microcystic areas of cells resembling
dorsal brainstem (from the midbrain to the pons levels) protoplasmic astrocytes.
and the supracerebellar space. The roof of the fourth Figures 4–6 (Gd-T1-weighted axial, Gd-T1-weighted
ventricle and the cerebellum is pushed downward by the sagittal, and T2-weighted sagittal, respectively) show
tumor. Compression of the sylvian aqueduct by the tu- that the solid portion of the tumor was totally removed
mor results in obstructive hydrocephalus. and the cyst wall subtotally removed. The boy had no
The clinical symptoms of this boy are relatively mild operative complications and no additional neurological
compared to the size of the tumor and there is no marked deficits postoperatively. At the 6-month follow-up, he
edematous change in the peritumoral brain tissue on the was in good condition, doing well, and going to school.

Suggested Reading
1. Bovdston WR, Sanford RA, Muhlbauer MS, Kun LE, Kirk 4. Fernandez C Figarella-Branger D, Girard N, Bouvier-Labit
E, Dohan FC Jr, Schweitzer JB. Gliomas of the tectum and C, Gouvernet J, Paz Paredes A, Lena G. Pilocytic astrocyto-
periaqueductal region of the mesencephalon. Pediatr Neu- mas in children: prognostic factors—a retrospective study
rosurg 1991–1992; 17:234–238 of 80 cases. Neurosurgery 2003; 53:544–553
2. Bowers DC et al. Tectal gliomas: Natural history of an in- 5. Kestle J, Townsend JJ, Brockmeyer DL, Walker ML. Juve-
dolent lesion in pediatric patients. Pediatr Neurosurg 2000; nile pilocytic astrocytoma of the brainstem in children.
32:24–29 J Neurosurg 2004; 101(1 Suppl):1–6
3. Daglioglu E et al. Tectal gliomas in children: The implica-
tions for natural history and management strategy. Pediatr
Neurosurg 2003; 38:223–231
8 Oncology 501

Q 236
Masayuki Fujioka, Carl Muroi, Nadia Khan, and Yasuhiro Yonekawa

Fig. 1 Fig. 2 Fig. 3

An 8-year-old girl’s right eye deviated medially for On admission, the patient’s neurological deficits com-
1 week, according to her mother. prised right abducens nerve palsy, left hemi-dysesthesia,
CT scans obtained by an ophthalmologist revealed a left incomplete hemiparesis, cerebellar ataxia, and right
low to isodense change mainly in the lower pons. The auditory disturbance.
girl was referred to a neurological department. • What do Figs. 2, 3 (Gd-T1-weighted MR images, ax-
An MR imaging study showed a pontine–tegmentum ial, sagittal, and coronal views, respectively) show?
lesion as a hypointense change of irregular shape on T1- • What pathological condition is affecting this child?
weighted images and as a larger area of hyperintense • What is the best way to manage this condition?
change on T2-weighted imaging compared to the area
on the T1-weighted image (Fig. 1).
502 Oncology

A 236

Figures 2, 3 show the tumor localized mainly in the right


lower tegmentum of the pons. The tumor has a ring en-
hancement of irregular shape. The MR imaging study
suggests a high-grade glioma with central necrosis (such
as anaplastic astrocytoma and glioblastoma multiforme)
firstly, and inflammation/infectious change secondarily.
In this case, the pontine tumor has damaged the ab-
ducens nerve fiber and/or its nucleus but not the facial
nerve. The facial nerve fiber runs around the abducens
nerve nucleus and could be damaged together. The com-
bined symptoms of unilateral facial and abducens nerve
palsy and contralateral hemiparesis due to the basal
pontine lesion point to Millard-Gubler syndrome. This
syndrome is usually caused by ischemia, but sometimes
also by a tumor.
This tumor is located in the pontine base near the
floor of the fourth ventricle. Therefore, surgical extirpa-
tion of the tumor is considered feasible.
The tumor was removed subtotally. Histology showed Fig. 4
an anaplastic astrocytoma (WHO grade 3). The opera-
tion was performed without any complication. Figure 4
shows a postoperative T1-weighted image with Gd ad-
ministration.
The girl underwent chemotherapy and radiation after
the operation. Thereafter, she was transferred to a reha-
bilitation institution. At the time of writing, the hemipa-
resis and ataxia were improving.

Suggested Reading
1. Epstein FJ, Farmaer JP. Brain-stem glioma growth patterns. 3. Kwon JW, Kim IO, Cheon JE, Kim WS, Moon SG, Kim TJ,
J Neurosurg 1993; 78:408–412 Chi JG, Wang KC, Chung JK, Yeon KM. Pediatric brain-
2. Kane AG, Robles HA, Smirniotopoulos JG, Heironimus JD, stem gliomas: MRI, FDG-PET and histological grading
Fish MH. Radiologic-pathologic correlation. Diffuse pon- correlation. Pediatr Radiol 2006; 36:959–964
tine astrocytoma. AJNR Am J Neuroradiol 1993; 14:941– 4. Tortosa A et al. Prognostic implication of clinical, radio-
945 logic, and pathologic features in patients with anaplastic
gliomas. Cancer 2003; 97:1063–1071
8 Oncology 503

Q 237
Masayuki Fujioka, Carl Muroi, Nadia Khan, and Yasuhiro Yonekawa

Fig. 1 Fig. 2 Fig. 3

A 5-year-old boy had started complaining of general fa- A T1-weighted MR imaging study showed that the cyst
tigue and visual acuity disturbance 3 weeks earlier. The wall appeared isointense and that the content of the
boy had been relatively lethargic for 1 week and had cyst exhibited hypointensity. T2-weighted MR imaging
been suffering from vomiting for the last 3 days. revealed a cystic lesion (the wall and the contents) and
During the first medical examination, his conscious- surrounding edema with hyperintensity.
ness level was 14 on the Glasgow Coma Scale. The gen- • What does Fig. 3 (Gd-T1-weighted) show?
eral physician performed a CT study. • What pathological condition is affecting this child?
• What do Figs. 1 and 2 (pre- and postcontrast CT • What is the best way to manage this condition?
scans) show?
504 Oncology

A 237

Fig. 4 Fig. 5 Fig. 6

Figures 1 and 2 show the cystic tumor with a solid com- images). Figure 6 [fluid attenuated inversion recovery
ponent in the left prethalamic and hypothalamic areas. (FLAIR) MR imaging] shows that both the solid por-
The tumor pushes the foramen of Monro toward the tion/cyst wall and the cyst contents appear isointense
contralateral side. The precontrast CT scan shows an (partly hyperintense) to the gray matter. The FLAIR
isodense wall with central low density. The postcon- image shows the surrounding edema clearly as hyper-
trast CT scan shows a cystic wall of irregular shape with intense change.
moderate enhancement effect. The tumor has some sur- The differential diagnosis includes glioma (hypothal-
rounding edema. Dilatation of the anterior horns and amus or optic pathway) and craniopharyngioma.
the trigones of the lateral ventricles indicates obstructive The boy underwent surgical removal of the tumor
hydrocephalus. (subtotal expiration, supine position, left frontomedial
Figure 3 shows that the cystic tumor has a mural nod- craniotomy, interhemispheric approach). The tumor
ule enhanced on T1-weighted images. The tumor occu- arose from the left optic tract. The histological diagnosis
pies the premesencephalic cistern and the third ventricle. was pilocytic astrocytoma. A detailed study revealed no
The foramen of Monro is pressed toward the right. The evidence of neurofibromatosis 1 (von Recklinghausen’s
midbrain is compressed and pushed away posteriorly. neurofibromatosis). The boy had no additional deficits
Figure 4 indicates that the solid portion of the tumor after the operation. At the 6-month follow-up, he was in
is located mainly in the optic pathway. The enhancement good condition and was going to school.
effects of the solid component and the wall seem rela-
tively heterogeneous on Figs. 4 and 5 (Gd-T1-weighted

Suggested Reading
1. Coakley KJ, Huston J, Scheithauer BW et al. Pilocytic astro- 3. Komotar RJ et al. Pilocytic and pilomyxoid hypothalamic/
cytomas: well-demarcated magnetic resonance appearance chiasmatic astrocytomas. Neurosurgery 2004; 54:72–80
despite frequent infiltration histologically. Mayo Clin Proc 4. Riccardi VM. von Recklinghausen neurofibromatosis. N
1995; 70:747–751 Engl J Med 1981; 305:1617–1627
2. Fernandez C Figarella-Branger D, Girard N, Bouvier-Labit
C, Gouvernet J, Paz Paredes A, Lena G. Pilocytic astrocyto-
mas in children: prognostic factors—a retrospective study
of 80 cases. Neurosurgery 2003; 53:544–553
8 Oncology 505

Q 238
Masayuki Fujioka, Carl Muroi, Nadia Khan, and Yasuhiro Yonekawa

Fig. 1 Fig. 2 Fig. 3 Fig. 4

An 11-year-old boy had intermittent frontal headaches base). The CT study also showed bone destruction (oste-
for 5 months. The headache had become more severe olysis) in the frontal skull base and the ethmoid.
in the last 6 days. The boy had also been suffering from • What do Figs. 1–3 (Gd-T1-weighted images) and 4
vomiting for 3 days. (FLAIR MR imaging) show?
The precontrast head CT scans showed a space-occu- • What pathological condition is affecting this child?
pying lesion of isodensity to moderate hyperdensity with • What is the best way to manage this condition?
the surrounding edema in the right frontal lobe (frontal
506 Oncology

A 238

The MR images (Figs. 1–4) show a mass lesion in the


frontal cranial base (involving the bilateral olfactory
groove and the right sphenoidal plane) with surround-
ing edema in the right frontal lobe. The Gd-T1-weighted
images demonstrate a heterogeneously enhanced tumor.
The tumor margin is irregular and the tumor seems lob-
ulated. The extending portion of the tumor interdigitates
with the brain. The tumor has an extracranial portion
extending into the frontal sinus and the ethmoid sinus.
The FLAIR image (Fig. 4) shows a tumor of mixed inten-
sities and marked peritumoral edema. The cerebral falx
is also strongly enhanced on the Gd-T1-weighted images
(dural tail sign). This is an intra- and extracranial tumor
associated with bone destruction. This tumor appears as
a dural-based enhancing mass partially associated with
a peritumoral CSF band. The differential diagnosis for
such a tumor includes atypical (WHO grade 2) or ma-
lignant (WHO grade 3) meningioma, dural metastatic Fig. 5
tumor, and sarcoma (ex. osteosarcoma and meningeal
sarcoma).
The patient underwent surgical removal of the tu-
mor (macroscopical total removal, supine position,
frontomedial craniotomy). The histopathological study
showed an atypical meningioma (WHO grade 2). The
postoperative Gd-T1-weighted MR images show that
the tumor was removed by >95% (Figs. 5, 6). No postop-
erative additional neurological deficits developed. At the
3-month postoperative follow-up, the patient was doing
well and going to school.

Suggested Reading
1. Alvernia JE et al. Preoperative neuroimaging findings as
a predictor of the surgical plane of cleavage: prospective
study of 100 consecutive cases of intracranial meningioma.
J Neurosurg 2004; 100:422–430
2. Chen TC, Zee CS, Miller CA et al. Magnetic resonance im-
aging and pathological correlates of meningiomas. Neuro-
surgery 1992; 31:1015–1022
3. Takeguchi T et al. The dural tail of intracranial meningio-
mas on fluid-attenuated inversion-recovery images. Neuro-
radiol 2004; 46:130–135
Fig. 6
8 Oncology 507

Q 239
Jacob C. Langer and Priscilla Chiu

Fig. 1 Fig. 2

A 3-week-old term female infant presented to the local The patient was transferred from the local hospital to the
emergency room with a history of increasing lethargy, tertiary pediatric hospital. Intravenous antibiotics were
fever, and poor oral intake. On physical examination, started.
the infant was febrile at 39°C with normal vital signs and • What investigations would you order for this pa-
appeared well hydrated. She had normal breath sounds tient?
and no evidence of murmurs on auscultation. Her neu-
rological examination was normal. Her birth history and The patient’s fever resolved and she was discharged home
family history were unremarkable. without any symptoms.
She underwent a septic work-up which included a chest After 3 months, the patient was still symptom-free
radiograph, as shown in Figs. 1 and 2. and doing well. She tolerated oral intake well and was
• What do Figs. 1 and 2 show? growing.
• What is the differential diagnosis? • How would you manage this patient?
• What are the considerations in the surgical treatment
of this lesion?
508 Oncology

A 239

Fig. 3 Fig. 4

This infant’s chest radiograph showed a lesion in the lethal solid tumors of childhood. However, spontaneous
apex of the right lung field. The lateral view showed the regression may occur, especially in those identified early
lesion to be in the posterior mediastinum. in infancy. In some cases, the neuroblastoma may ma-
The differential diagnosis for this lesion includes ture into a benign ganglioneuroma.
neurogenic tumor (most common), primitive neuro- Neuroblastomas produce vasoactive substances de-
ectodermal tumor, foregut duplication cyst (including tectable in the serum and urine. Urinary metabolites of
neurenteric cyst), and anterior thoracic meningocele epinephrine, particularly vanillylmandelic acid (VMA)
(extremely rare). and homovanillic acid (HVA), are useful as diagnostic
The radiological modality of choice is MR imaging, tests and for detection of recurrent disease. Elevated se-
which provides excellent multiplanar imaging for neu- rum levels of neuron-specific enolase, lactate dehydro-
rogenic and spinal lesions, and is optimal for accurately genase, and ferritin correlate with the diagnosis of neu-
detecting intraspinal extension. However, this procedure roblastoma and provide prognostic value.
requires sedation or general anesthesia for optimal im- Thoracic neurogenic tumors may infiltrate the spinal
aging resolution. Ultrasound is a less invasive study that canal (“dumbbell” tumors) causing spinal cord compres-
can differentiate between solid and cystic lesions in the sion and paraplegia, which may require emergency lam-
periphery of the chest. CT is also helpful in revealing the inectomy to decompress the spinal cord.
identity and extent of the lesion and is complementary This patient’s urine tests showed elevated HVA and
to MR imaging in both aspects. VMA. The lesion was subsequently imaged with CT
Neurogenic tumors (neuroblastoma and ganglioneu- (Fig. 3) and MR imaging (Fig. 4).
roma) arising from the thoracic sympathetic chain are No other lesions were found, suggesting that this was
the most common lesions of the posterior mediastinum an isolated thoracic neurogenic tumor with no metas-
in infants and children. The chest is the third most com- tases.
mon site for neurogenic tumors after the adrenal gland The definitive management of isolated thoracic neu-
and other retroperitoneal sites. rogenic tumors is surgical resection via thoracoscopy or
Neuroblastomas, arising from neural crest cells with thoracotomy. At 3 months, the lesion was resected tho-
immature neural elements, are the most common and racoscopically without complication.
8 Oncology 509

Q 240
Jacob C. Langer and Priscilla Chiu

Fig. 2

Fig. 1

A 5-year-old child presented to our emergency room


with progressive cough and recent weight loss. He had
no significant past medical history and no travel history.
The initial physical examination by the emergency
physician revealed decreased breath sounds in the right
chest.
Fig. 3
The child was sent for a chest radiograph, shown in
Fig. 1.
• What does Fig. 1 show?
• What is the differential diagnosis for the chest radio- dyspneic requiring supplemental oxygen and admission
graph findings? to the intensive care unit.
The multidisciplinary oncology team was consulted.
The emergency physician ordered the next investigation The surgical team requested the investigation shown in
shown in Fig. 2. Fig. 3.
• What does Fig. 2 show? • What does Fig. 3 show?
• What is the differential diagnosis now?
The patient was referred to the pediatric general surgeon • How would you manage this patient?
after these investigations. He became progressively more
510 Oncology

A 240

This patient’s chest radiograph showed loss of right lung


volume (the “white-out”). The mediastinum was dis-
placed into the left chest.
It was not clear on the radiograph whether fluid or
solid tissue occupied the right chest. Lateral decubitus
positioning may show air–fluid levels suggestive of an
effusion, unless it is multiloculated or extremely large.
Ultrasound is a useful initial study to distinguish
fluid and solid tissue in the chest. The patient’s ultra-
sound (Fig. 2) showed a complex, solid mass with very
little fluid.
The differential diagnosis includes benign and ma-
lignant tumors of the chest. Benign tumors include
myxoma, hemangioma, teratoma, lipoma, and rhab- Fig. 4
domyoma. Malignant tumors include Ewing’s sarcoma,
malignant teratoma, rhabdomyosarcoma, lymphoma,
primitive neuroectodermal tumor, lipoblastoma, and
pleuropulmonary blastoma. Pleuropulmonary blastoma is a rare but aggressive
CT scan can identify features of solid chest tumors tumor of the pleuropulmonary mesenchyme. There has
and aid in their diagnosis. Vascular tumors will enhance been controversy over the association between congeni-
with intravenous contrast medium. The presence of fat tal lung cysts and the risk of malignant degeneration in
and calcification is typical of teratoma. Rib deformity the cyst giving rise to this malignancy.
may suggest Ewing’s sarcoma. Disseminated lymph- Following tissue diagnosis, this patient underwent
adenopathy would suggest lymphoma. Lung nodules in surgical resection. The lesion involved the right middle
children should be investigated with CT imaging of the and lower lobes and was adherent to the pericardium and
head, abdomen, and pelvis, as lung lesions may repre- chest wall. The lesion was completely resected through a
sent metastatic spread from other primary tumors. “clam-shell” incision by complete pneumonectomy and
The chest CT scan showed a large, nonhomogeneous partial resection of the pericardium. The pericardium
mass with minimal intravenous contrast enhancement was then reconstructed using Surgi-sis (Fig. 4).
and a small effusion. No areas of fat or calcification The patient did well postoperatively and was dis-
within the tumor were seen and no lymphadenopathy charged home. The role of postoperative chemotherapy
or rib lesion was noted. The CT appearance suggested a for pleuropulmonary blastoma is unclear and subject to
primary mesenchymal tumor. multidisciplinary oncology evaluation.
Definitive diagnosis of the lesion was obtained with
a core biopsy showing malignant cells consistent with
pleuropulmonary blastoma.

Suggested Reading
1. Dang NC, Siegel SE, Phillips JD. Malignant chest wall tu- 3. Tagge EP, Mulvihill D, Chandler JC, Richardson M, Uflacker
mors in children and young adults. J Pediatr Surg 1999; R, Othersen HD. Childhood pleuropulmonary blastoma:
34:1773–8 caution against nonoperative management of congenital
2. Hill DA. USCAP specialty conference: case 1-type 1 pleu- lung cysts. J Pediatr Surg 1996; 31:187–9
ropulmonary blastoma. Pediatr Dev Pathol 2005; 8:77–84
8 Oncology 511

Q 241
Frédéric Gauthier, Sophie Branchereau, and Chiara Grimaldi

Fig. 1

A 2-year-old girl with no significant medical history pre- At the end of preoperative chemotherapy, the alpha-fe-
sented to the pediatric emergency department with an toprotein levels had decreased to 632. The volume of the
abdominal mass. A US study and an abdominal CT scan lesion also decreased on the CT scan and no extension
were performed; the scan is shown in Fig. 1. A thoracic to the fourth segment was detected.
scan was normal. • What type of resection would you plan?
• What is the initial diagnosis and how can you con-
firm it with blood tests?
• According to the different international protocols
and the localization of the lesion, what are the treat-
ment options?
512 Oncology

A 241

A hepatic lesion is localized in the right hepatic lobe. The


alpha-fetoprotein level is diagnostic measuring 61,200
(normal <15). The diagnosis is nonmetastatic hepato-
blastoma.
We used the SIOP protocol SIOPEL 3 for this patient.
The localization of the lesion allows a right hemi-
hepatectomy to be performed (Fig. 2).

Fig. 2

Suggested Reading
1. Blouin P, Brugieres L, Tabone MD, Leverger G, Rubie H, 2. Czauderna P, Otte JB, Aronson D, Gauthier F , Mackinlay
Branchereau S. Carboplatin-epirubicin regimen for the G, Roebuck D, Plaschkes J, Perilongo G. Guidelines for sur-
treatment of hepatoblastoma. Pediatr Blood Cancer 2004; gical treatment of hepatoblastoma in the modern era—Rec-
42(2):149–54 ommendations from the Childhood Liver Tumour Strategy
Group of the International Society of Paediatric Oncology
(SIOPEL). European Journal of Cancer 2005; 41:1031–
1036
8 Oncology 513

Q 242
Amedeo Fiorillo

Fig. 1

A 6-year-old white girl was admitted to our department ameter—closely connected to the superior vena cava, to
because of severe abdominal pain and fever. On admis- the liver, and to the homolateral kidney; multiple fine
sion, the child was febrile, and her general condition calcifications and one large calcification were also evi-
was compromised. Physical examination showed gen- dent.
eralized lymphadenopathy and an extremely enlarged • What is the diagnosis?
abdomen; a firm mass was palpable 10 cm below the • What differential diagnosis do you take into consid-
right costal margin. Abnormal laboratory test results in- eration?
cluded: WBC, 15,460/µl with 74% neutrophils; platelets, • Which other diagnostic tests do you perform to con-
1,000,000/µl; erythrocyte sedimentation rate, 70 mm/h; firm the suspected diagnosis?
and ferritin, 340 mg/dl. CT scan of the abdomen (Fig. 1) • What is the role of surgery in this condition?
showed a huge retroperitoneal mass—5.2×3.2 cm in di-
514 Oncology

A 242

Fig. 2

Fig. 3

This child was affected by neuroblastoma, arising from 4. Serum markers of predictive value such as neu-
her right adrenal gland. ron-specific enolase, ferritin, lactate dehydrogenase
The primary tumor should be differentiated from (LDH)
a renal abnormality such as a multicystic kidney or a 5. Definitive pathological diagnosis can be made from
Wilms’ tumor. Usually, as in this case, it is possible to tumor tissue obtained by excisional biopsy (in this
demonstrate that the tumor is extrarenal, displacing case it was neuroblastoma, schwannian stroma-
rather than involving the kidney. Moreover, metastatic poor)
sites may be confused with other small, round, blue-cell
tumors such as rhabdomyosarcoma, Ewing’s sarcoma, or The recommended minimum testing to define the clini-
lymphoma. cal stage (from stage 1 that defines the localized tumor
Other tests useful in establishing the diagnosis are: to stage 4 that defines any primary tumor with distant
1. Measurement of urine catecholamines and metabo- dissemination other than skin, liver or bone marrow,
lites such as homovanillic acid (HVA) and vanillyl- according to the International Neuroblastoma Staging
mandelic acid (VMA) (in our case the VMA level System) also comprises bone radiographs and scintigra-
was 15 mg/24 h and the HVA level was 20 mg/24 h, phy by technetium 99m scan, chest radiograph (AP and
both greater than 3.0 SD above the mean for age) lateral), and chest CT or MR imaging if the chest radio-
2. Bone marrow aspirate and trephine biopsy to search graph is abnormal.
for unequivocal tumor cells (in this case bone mar- Surgery plays a pivotal role in the management of
row aspirate was positive; Fig. 2) neuroblastoma in order to establish the diagnosis, to
3. Metaiodobenzylguanidine scan, applicable to all sites provide tissue for biologic studies, and to completely
of disease (Fig. 3) remove the tumor or the residual disease in delayed pri-
mary or second-look surgery.
8 Oncology 515

Q 243
Amedeo Fiorillo

Fig. 1 Fig. 2

A 1-year-old white boy presented with an enlarged, pain- A CT scan of the abdomen (Fig. 1) showed a large,
less abdomen; no other sign or symptom was reported inhomogeneous, mainly solid mass extending from the
apart from occasional episodes of diarrhea. On admis- second lumbar vertebra into the pelvis; the bladder was
sion his general condition was good; on physical exami- displaced to the left (Fig. 2). A radiographic skeletal sur-
nation a large abdominal mass was palpable and arterial vey, standard chest radiograph, CT of the chest, and a
hypertension was registered. brain CT scan were all normal.
Pertinent laboratory test results were found to be in • How would you achieve the diagnosis?
the normal range. • What is the differential diagnosis?
• What are the prognostic factors in this disease?
• What is the role of the surgeon?
516 Oncology

A 243

The diagnosis, made on tumor tissue obtained by ex- e) Stage of disease (group I: localized disease is com-
cisional biopsy, was “embryonal rhabdomyosarcoma” pletely resected; group II: localized tumor, with or
(RMS). without spread to regional lymph nodes, is grossly
Traditionally, histologic classification includes em- resected with microscopic residual disease; group III:
bryonal, botryoid, alveolar, and pleomorphic variants; localized or locally extensive tumor shows gross re-
nearly two-thirds of cases, like our case, are of embryo- sidual disease after resection or biopsy only; group
nal histology. IV: distant metastases are present at onset)
RMS belongs to the category of small, round, blue-
cell tumors of childhood; therefore it must be differenti- RMS may spread by local extension, but metastases via
ated from lymphoma, neuroblastoma, Ewing’s sarcoma, the venous or lymphatic systems are rare at onset, diag-
and primitive neuroectodermal tumor. nosed in no more than 20% of cases.
The major prognosticators are: The goal of surgery should be to achieve complete re-
a) Age (with a relatively poor outlook for infants with section of the primary tumor with an adequate margin
advanced disease) of normal tissue; this will allow local control of the non-
b) Primary site (locations associated with little opportu- metastatic disease and possibly cure. Therefore, treat-
nity for spread, i.e., the orbit, share better prognosis ment should be individualized for each patient, based
compared with deep, poor confined areas, i.e., retro- on clinical stage, site, and resectability of the primary
peritoneum) tumor.
d) Histologic type (the best prognosis is associated with
embryonal, the poorest with alveolar variant)

Suggested Reading
1. Lawrence W Jr et al. Pretreatment TNM staging of child- 2. Rodary C et al. An attempt to use common staging system
hood rhabdomyosarcoma. A report of the Intergroup in rhabdomyosarcoma: a report of an international work-
Rhabdomyosarcoma Study Group. Cancer 1997; 80:1165 shop initiated by the International Society of Pediatric-On-
cology (SIOP). Med Pediatr Oncol 1989; 17:210
8 Oncology 517

Q 244
Amedeo Fiorillo

Fig. 1 Fig. 2

A 5-year-old white girl was admitted to our department


because of abdominal swelling first noticed 1 month
earlier. On admission she was febrile and had severe
abdominal pain. Physical examination showed a large
left flank mass that did not move with respiration. The
initial laboratory evaluation revealed elevation of the
erythrocyte sedimentation rate (56 mm/h), lactate dehy-
drogenase (LDH) (1,967 IU/l), neuron-specific enolase
(68.3 mg/ml), and ferritin (626 µg/l). However, a normal
renal function, including urinalysis, was found and uri-
nary vanillylmandelic and homovanillic acids were in
the normal range for age. Moreover, an abdominal ul-
trasound examination (Fig. 1) showed a large (10×10 cm
in diameter) mainly solid intrarenal mass. Contrast-en-
hanced CT of the abdomen (Fig. 2) confirmed the pres-
ence of a left dishomogeneous renal tumor and a CT of
the chest (Fig. 3) revealed multiple pulmonary densities
also evident on standard chest radiographs.
• What is the diagnosis? Fig. 3
• What is the differential diagnosis?
• Which other tests would you perform in this case?
• What is the role of surgery in this disease?
518 Oncology

A 244

In this case, the final diagnosis was three-phase Wilms’ c. Stage III: gross or microscopic residual tumor re-
tumor without anaplasia, stage IV. mains postoperatively
The chief differential diagnosis is neuroblastoma; d. Stage IV: distant metastases, more commonly to the
however, the distinction is usually made on imaging lung (80% of cases) and to the liver (15% of cases).
studies because nephroblastoma is intrarenal, whereas e. Stage V: bilateral renal tumors
most neuroblastomas arise from the adrenal gland and
only displace the kidney. Other benign and malignant The single most important prognostic factor is the his-
intrarenal neoplasms, such as mesoblastic nephroma, tologic finding of anaplasia. However, lymph node in-
renal cyst, multicystic kidney, or renal rhabdomyosar- volvement and, of course, stage of disease retain their
coma, require the benefit of biopsy to be properly diag- importance as predictors of outcome.
nosed. Immediate and complete removal of the primary
A radionuclide bone scan and radiographic skeletal tumor, if feasible, is mandatory even in the presence of
survey should be obtained in children with pulmonary lung metastases. However, NWTSG recommends de-
or hepatic metastases. layed surgery and preoperative chemotherapy for pa-
All pediatric renal tumors can be staged according tients with tumor extension into the inferior vena cava
to the National Wilms’ Tumor Study Group (NWTSG) above the hepatic veins, for lesions judged to be unre-
Criteria: sectable, and for patients with bilateral tumors.
a. Stage I: the tumor, confined to the kidney, is com-
pletely resected
b. Stage II: there is penetration of the renal capsule or
invasion of the renal sinus vessels, or biopsy of the
tumor before removal or spillage of the tumor during
removal, but the tumor is completely resected

Suggested Reading
1. Beckwith JB. National Wilms’ Tumor Study: an update for 3. Ritchey ML et al. Management and outcome of inoperable
pathologists. Pediatr Dev Pathol 1998; 1:79 Wilms’ tumor. A report of National Wilms’ Tumor Study-3.
2. Breslow NE et al. Clinicopathologic features and progno- Ann Surg 1994; 220:683
sis for Wilms’ tumor patients with metastases at diagnosis.
Cancer 1986; 58: 2501
8 Oncology 519

Q 245
Amedeo Fiorillo

Fig. 2

Fig. 1

A 7-year-old boy was admitted to our department with Histologic examination of tumor tissue, obtained by
a 4-month history of a painless swelling of the lower or- an excisional biopsy, revealed a small cell tumor. Spe-
bital region. On admission, his general condition was cial staining revealed the presence of a large amount of
good. Physical examination showed a painless swelling periodic acid-Schiff-positive material, consistent with
involving the lower right eyelid and the corresponding intracellular glycogen. Electron microscopy showed
malar region; mild proptosis was also evident, but ocular undifferentiated neoplastic cells of a round shape, the
motility was preserved. Pertinent laboratory test results predominant feature being abundant glycogen deposi-
were normal. A CT scan of the head showed a destruc- tion corresponding to the vacuolated cytoplasmic areas
tive lesion extensively involving the right maxillary bone (Fig. 2).
(Fig. 1), without evidence of surrounding soft tissue in- • What is the diagnosis?
volvement. • What evaluation should be performed?
The initial diagnostic imaging evaluation included a • What is the prognosis?
CT scan of the neck region, chest, and abdomen, a skele- • What is the treatment strategy?
tal radiographic survey, a radionuclide 99mTc bone scan,
and a bone marrow biopsy; all were normal.
520 Oncology

A 245

The clinical and histologic features of this case are all Major prognosticators are the localization of the
consistent with a diagnosis of Ewing’s sarcoma. More primary tumor (axial localization implies a very poor
commonly, it is an undifferentiated tumor of the bone, prognosis) and presence of overt metastases at diagnosis.
but it may also arise from soft tissues. A more differenti- However, evaluation of chemotherapy-induced necrosis
ated form of this disease, known as peripheral primitive is also of prognostic value.
neuroectodermal tumor (PPNET), also occurs. It is now According to the histologic response grading method,
widely accepted that all these entities represent a spec- based on the four-tiered semiquantitative grading sys-
trum of a single disease. tem for osteosarcoma treatment effect, grade I is tumor
Ewing’s sarcoma must be differentiated from other necrosis of less than 50% of the tumor, grade II is tumor
small round-cell tumors such as neuroblastoma, rhab- necrosis of more than 50% to less than 90%, grade III is
domyosarcoma, lymphoma, and from inflammatory dis- tumor necrosis of 90% to 99%, and grade IV is tumor
ease, especially if the patient is febrile and an elevated necrosis of 100%. Event-free survival at 5 years was 0%
erythrocyte sedimentation rate is found. Neither a blood for grade I, 37.5% for grade II, and 84% for grade III and
test nor a urine test provides specific markers of Ewing’s IV.
sarcoma; however, immunocytochemistry is of value. It is widely accepted that prompt chemotherapy is
Typical undifferentiated Ewing’s sarcoma is defined as necessary to treat occult micrometastases, present in
a neoplasm that has immunoreactivity only for vimen- over 80% of cases at diagnosis. However, it is also man-
tin and is negative for neuron-specific enolase (NSE); datory, for definitive cure of these patients, to achieve
in contrast, PPNET reacts with vimentin and with a and maintain local control of the primary tumor. This
number of neural markers (NSE, beta2-microglobulin). could be obtained by radical surgery for accessible sites
Rhabdomyosarcoma shows positivity to desmin, myo- and when the patient would not be exposed to the risk of
globin, and actin. Neuroblastoma may be differentiated unacceptable mutilations. Alternatively, as in the present
by using monoclonal antibody NCL-NB84, which is case, radiation therapy can be used, provided that it is
expressed in more than 90% neuroblastomas but not in optimally delivered in terms of total dose and planning
Ewing’s sarcoma. of treatment.

Suggested Reading
1. Dunst J and Schuck A. Role of radiotherapy in Ewing tu- 3. Wunder JS et al. The histological response to chemotherapy
mors. Pediatr Blood Cancer 2004; 42:465 as a predictor of oncological outcome of operative treatment
2. Ginsberg JP et al (2002) Ewing’s sarcoma family of tumors: of Ewing sarcoma. J Bone Joint Surg Am 1998; 80:1020
Ewing’s sarcoma of bone and soft tissue and the peripheral
primitive neuroectodermal tumors. In: Pizzo PA, Poplack
DG (eds) Principles and Practice of Pediatric Oncology.
Lippincott, Philadelphia, pp 973–1016
8 Oncology 521

Q 246
Amedeo Fiorillo

A 3-month-old white boy presented with signs of in-


creased intracranial pressure. MR imaging of the head
showed a contrast-enhanced intraventricular mass and
subdural fluid collection (Fig. 1). Following partial sur-
gical removal of the tumor, microscopic examination
revealed a papillary lesion with columnar epithelium
(Fig. 2); neoplastic cells showed cytologic atypia and in-
creased mitotic activity (Fig. 3).
• What is the diagnosis?
• What is the prognosis of this disease?
• What is the best therapeutic strategy?

Fig. 1

Fig. 2 Fig. 3
522 Oncology

A 246

The intraventricular location together with the histo- The most favorable prognosticator is represented by
logical features point to the diagnosis of choroids plexus the gross total resection of the tumor, usually hampered
carcinoma; this condition is to be differentiated from by its high degree of vascularity and friability.
embryonal tumors, malignant ependymoma, and germ Systemic chemotherapy has been used, with some
cell tumors. However, immunohistochemistry is highly success, before surgical intervention in order to reduce
useful for differential diagnosis. In choroid tumors, the tumor volume, to diminish its vascular response, and
neoplastic cells usually show immunoreactivity for cy- in turn to facilitate its surgical removal. Furthermore,
tokeratins and staining for S-100 protein and vimentin, there is recent evidence that systemic chemotherapy can
whereas glial fibrillary acidic protein (GFAP), carcino- be of value, as adjuvant therapy, even in patients with
embryonic antigen (CEA), and placental alkaline phos- partially resected tumors; alkylating agents, etoposide,
phatase (PLAP) are negative. vincristine, and platinum compounds have been mainly
The disease accounts for less than 1% of childhood used. More recently, excellent results have been obtained
brain tumors and is characterized by a very dismal prog- by the association of doxorubicin and methotrexate with
nosis with few patients surviving more than 6 months carboplatin and cyclophosphamide.
after a partial surgical resection.

Suggested Reading
1. Fiorillo A et al. Efficacy of sequential chemotherapy in- 3. Souweidane MM et al. Volumetric reduction of a choroids
cluding methotrexate and doxorubicin in an infant with plexus carcinoma using preoperative chemotherapy. J Neu-
partially resected choroid plexus carcinoma. Pediatr Neu- rooncol 1999; 43:167
rosurg 2003; 38:21
2. McEvoy AW et al. Management of choroid plexus tumors
in children: 20 years experience at a single neuro-surgical
centre. Pediatr Neurosurg 2000; 32:192
8 Oncology 523

Q 247
Christophe Chardot and Sylviane Hanquinet

Fig. 1 Fig. 2 Fig. 3

Fig. 4 Fig. 5 Fig. 6

A 13-month-old boy presented with increasing abdomi- The diagnosis of hepatoblastoma was confirmed by nee-
nal distension, which had been recently noticed by his dle biopsy.
parents. His general condition was good. Clinical exam- • What are the main rules for the treatment of such a
ination revealed a large mass of the right upper quadrant tumor?
of the abdomen.
• Which diagnosis can be suspected? This child was treated with initial chemotherapy, accord-
ing to the SIOPEL 3 protocol. Chemotherapy was well
Abdominal US revealed a multinodular liver tumor. tolerated, and the oncological response was good, with
• What biological test is likely to confirm the diagnosis significant tumor shrinkage and a decrease of the alpha-
of hepatoblastoma? fetoprotein (AFP) level to 242 UI/l. Preoperative imag-
ing showed residual tumor in the right liver (segments 4
Figures 1–6 represent the CT scan at diagnosis. to 8), and two small nodules in segments 2 and 4, with-
• How would you define the extension of the tumor? out extrahepatic disease.
• How would you stage this tumor according to the • What is the preoperative stage of the tumor?
SIOPEL PRETEXT classification? • What are the surgical options?
524 Oncology

A 247

The abdominal mass in the right upper abdominal quad- In this case, the tumor is multinodular, without extra-
rant is likely to be a tumor. The most common abdomi- hepatic extension. The main tumor invades segments 5
nal tumors in this age group are nephroblastoma, neuro- to 8 (Figs. 2–4), with intra-abdominal protrusion of the
blastoma, and hepatoblastoma; choledochal cyst, benign tumor (Figs. 5, 6). There are also nodules in segments 2
hepatic tumors (cystic mesenchymal hamartoma), duo- and 4 (Figs.1, 2). The right portal vein is compressed by
denal duplication, pancreatic tumor, and lymphoma are the tumor, but there is no radiological evidence of vas-
also possible. cular invasion.
The serum alpha-fetoprotein level is 15,00,000 UI/l SIOPEL (Liver Tumors Study Group of the Interna-
(normal value <5 UI/l): the diagnosis of hepatoblastoma tional Society of Paediatric Oncology) has created the
is almost certain. Another rare cause of multiple liver PRETEXT (Pre-Therapeutic Extension) classification,
nodules and elevated AFP is tyrosinemia. Normal AFP which reflects the intrahepatic extension of the disease,
levels do not exclude malignant liver tumors: mainly and allows stratification of the patients into several ther-
nonsecreting hepatoblastomas or hepatocarcinomas and apeutic and prognostic subgroups. The PRETEXT cat-
sarcomas. egories are based on the number of hepatic sectors (cf.
When describing the extension of the tumor, the fol- supra) free of tumor:
lowing questions should be addressed: – PRETEXT 1: three adjacent hepatic sectors free of tu-
– Uni- or multifocal tumor? mor
– Confined to the liver or extrahepatic extension? – PRETEXT 2: two adjacent hepatic sectors free of tu-
– Intra-abdominal fluid (ruptured tumor?) mor
– Perihepatic extension: diaphragm, liver hilum, – PRETEXT 3: one hepatic sector, or two nonadjacent
other intra-abdominal organs sectors, free of tumor
– Pulmonary metastasis – PRETEXT 4: all four hepatic sectors invaded
– Extra-abdominal and extrathoracic involvement – Extrahepatic growth is indicated by adding one ore
(brain, bone, etc.): imaging according to clinical more of the following characters: V (main hepatic
findings veins or vena cava invaded), P (portal vein or large
– The intrahepatic extension is defined by localization portal branches invaded), E (intra-abdominal ex-
of the tumor as compared to the main vessels, portal trahepatic invasion), M (distant metastases: mostly
bifurcation and hepatic veins: lungs)
– Portal bifurcation delimits
• Couinaud’s segment 1 behind the portal bifur- In the case described here, all hepatic sectors are invaded,
cation without apparent vascular or extrahepatic involvement:
• The rest of the liver (Couinaud’s segments 2 to 8) PRETEXT 4 tumor, V-, P-, E-, M- (Fig. 7).
anterior to the portal bifurcation The main rules for the treatment of hepatoblastomas
– Hepatic veins define four hepatic sectors: are:
• Right lateral sector (Couinaud’s segments 6 – The patient must be treated according to one of the
and 7) on the right of the right hepatic vein current international protocols, using a multidisci-
• Right medial sector (Couinaud’s segments 5 plinary approach.
and 8) between the right and the median he- – Complete excision of the primary liver tumor is nec-
patic veins essary: this can often be achieved by partial hepa-
• Left medial sector (Couinaud’s segment 4) be- tectomy, but may necessitate total hepatectomy (and
tween the median hepatic vein and the umbili- liver transplantation) in some cases. All modern tech-
cal scissure niques and expertise in liver surgery are required. If
• Left lateral sector (Couinaud’s segments 2 and total hepatectomy and liver transplantation are nec-
3) on the left of the umbilical scissure essary (PRETEXT 3 and 4 tumors), the child should
– Patency or invasion/thrombosis of the hepatic veins be referred early on to a pediatric liver transplanta-
and portal vein tion center for assessment, planning of the treatment
8 Oncology 525

Fig. 7 Fig. 8

Fig. 9 Fig. 10

(including possible living related liver donation), and subsequent tumor regrowth while waiting for transplan-
family information. tation, living related donation is proposed to the family
– Hepatoblastomas usually have a good response to che- in order for surgery to be performed at the optimal time
motherapy. Preoperative chemotherapy is therefore after the initial chemotherapy.
recommended in most protocols in order to induce Nevertheless, when this child was treated in 2001,
tumor shrinkage; it allows safer surgery with lower conservative surgery was preferred. Since right hepatec-
risk of incomplete resection and/or tumor spillage. tomy and tumorectomies of the nodules in the left liver
may have exposed the child to postoperative liver insuf-
The preoperative stage of the tumor is a multinodular ficiency (due to potential shortage of remaining liver pa-
nonmetastatic PRETEXT 4 hepatoblastoma. In order to renchyma), a two-stage surgery was chosen: first tumor-
completely remove all residual intrahepatic tumor, to- ectomy of the nodules in segments 2 and 4, and right
tal hepatectomy and liver transplantation are currently portal vein ligation; then 1 month later, after growth of
(2008) recommended in such a case. Due to the risk of the remaining left liver, right hepatectomy. The child
prolonged waiting times for a cadaveric liver graft, and underwent uneventful surgery and postoperative che-
526 Oncology

motherapy. Histology revealed viable hepatoblastoma liver 6 months after the second operation: the remain-
in all three resected tumors, with surgical margins free ing left liver is hypertrophied without tumor recurrence.
of disease. Figure 8 shows tumor shrinkage after initial The child was alive and well 7 years after surgery.
chemotherapy. Figure 9 shows the liver 4 weeks after
the first operation: the left liver has grown, and the right Acknowledgements: To Prof. Claude Le Coultre and
portal branches are not opacified. Figure 10 shows the Gilles Mentha, who operated on this child.

Suggested Reading
1. Aronson DC, Schnater JM, Staalman CR, Weverling GJ, 3. Czauderna P, Otte JB, Aronson DC, Gauthier F, Mackin-
Plaschkes J, Perilongo G et al. Predictive value of the pre- lay G, Roebuck D et al. Guidelines for surgical treatment
treatment extent of disease system in hepatoblastoma: re- of hepatoblastoma in the modern era—recommendations
sults from the International Society of Pediatric Oncology from the Childhood Liver Tumour Strategy Group of the
Liver Tumor Study Group SIOPEL-1 study. J Clin Oncol International Society of Paediatric Oncology (SIOPEL).
2005; 23(6):1245–52 Eur J Cancer 2005; 41(7):1031–6
2. Brown J, Perilongo G, Shafford E, Keeling J, Pritchard J,
Brock P et al. Pretreatment prognostic factors for children
with hepatoblastoma—results from the International Soci-
ety of Paediatric Oncology (SIOP) study SIOPEL 1. Eur J
Cancer 2000; 36(11):1418–25
8 Oncology 527

Q 248
Yves Aigrain and Pascale Philippe-Chomette

Fig. 1 Fig. 2

Fig. 3

A healthy 20-month-old girl presented with vaginal


bleeding.
Clinical examination revealed active vaginal bleeding.
A solid anterior pelvic mass was found at digital rectal Fig. 4
examination prolapsing from the vaginal area, without
any sign of precocious pseudopuberty.
A US examination was performed (Figs. 1, 2). Biological tests were performed, yielding the following
• Can you describe the US findings? results: alpha-fetoprotein (AFP), 6,500 UI; beta human
• What other radiological examinations would you ask chorionic gonadotropin hormone, 0.
for? • What is your diagnosis?
• Can you describe the examination shown in Figs. 3 • What is your strategy?
and 4? What are your hypotheses? • What other explorations could be proposed in this
case?
528 Oncology

A 248

There is a solid vaginal mass with bleeding as the prin-


cipal symptom.
The US examination confirms the vaginal site and the
vascularized character of the tumor. In this age group,
there is the possibility of vaginal rhabdomyosarcoma or
vaginal germ-cell tumor.
A chest radiograph and an abdominal and pelvic MR
imaging examination should be requested.
The pelvic MR imaging examination shows a vaginal
tumor with tissular component and blood inside.
With a high AFP level, the diagnosis is yolk sac tu-
mor, also called endodermal sinus tumor. This is a ma-
lignant germ cell tumor. The elevation of AFP serves as
a diagnostic tumor marker and a method with which to Fig. 5
evaluate response to treatment and monitor postopera-
tive recurrence.
Primary chemotherapy should be started to reduce
the tumoral volume; endoscopy with biopsy could be
useful simply to localize the origin of the tumor before
chemotherapy, but not imperative. The diagnosis is es-
tablished with the high level of AFP (Fig. 5).

Suggested Reading
1. Handel LN, Scott SM, Giller RH, Greffe BS, Lovell MA, 3. Terenziani M, Spreafico F, Collini P, Meazza C, Massimino
Koyle MA. New perspectives on therapy for vaginal endo- M, Piva L. Endodermal sinus tumor of the vagina. Pediatr
dermal sinus tumors. J Urol 2002 Aug; 168(2):687–90 Blood Cancer 2005 Sep 30
2. Lacy J, Capra M, Allen L. Endodermal sinus tumor of the
infant vagina treated exclusively with chemotherapy. J Pe-
diatr Hematol Oncol 2006 Nov; 28(11):768–71
8 Oncology 529

Q 249
Craig T. Albanese

Fig. 1 Fig. 2

An 18-week prenatal ultrasound (Fig. 1) of the lower


fetal spine revealed a large caudal mass (arrowheads;
spine, arrows). A color Doppler sonogram (Fig. 2) dem-
onstrates blood supply to the mass, which is derived
from enlarged branches of the middle sacral artery and
internal iliac arteries (spine, arrows). A fetal MR imag-
ing study (Fig. 3) with a fluid-sensitive sequence demon-
strates that the mass (arrowheads) is largely fluid-con-
taining with a small solid (asterisk) component.
• What are the findings and what is your diagnosis?

Fig. 3
530 Oncology

A 249

This is a sacrococcygeal teratoma. This lesion is complex


(cystic and solid). Predominantly solid lesions have a
very rich blood supply that can cause fetal hydrops due
to high output cardiac failure. Note the bright calcifica-
tions (Fig. 1) in the mass. They are rarely malignant.

Suggested Reading
1. Bittmann S, Bittmann V. Surgical experience and cosmetic
outcomes in children with sacrococcygeal teratoma. Curr
Surg 2006 Jan–Feb; 63(1):51–4
2. Makin EC, Hyett J, Ade-Ajayi N, Patel S, Nicolaides K,
Davenport M. Outcome of antenatally diagnosed sacrococ-
cygeal teratomas: single-center experience (1993–2004). J
Pediatr Surg 2006 Feb; 41(2):388–93
3. Schmidt B, Haberlik A, Uray E, Ratschek M, Lackner H,
Hollwarth ME. Sacrococcygeal teratoma: clinical course
and prognosis with a special view to long-term functional
results. Pediatr Surg Int 1999; 15(8):573–6
Index

Contents Page by Page

1 Head and Neck 1 Case 33 Pulmonary Sequestration 71


Case 1 Congenital Venous Malformation Case 34 Congenital High Airway Obstruction
of the Head 3 Syndrome 73
Case 2 Macrocystic Lymphatic Malformation Case 35 Neonatal Pneumothorax 75
of the Neck 5 Case 36 Diaphragmatic Abscess 77
Case 3 Hemangioma of the Head and Neck 7 Case 37 Mediastinal Mass 79
Case 4 Cervical Teratoma 9 Case 38 Pyopneumothorax 81
Case 5 Lipomatosis of the Parotid 11 Case 39 Thoracic Ganglioneuroblastoma 83
Case 6 Parotid Acinic Neoplasm 13 Case 40 Mediastinal Neuroblastoma 85
Case 7 Fibrous Dysplasia of the Mandible 15 Case 41 Bronchogenic Cyst 87
Case 8 Hemimandibular Hyperplasia 17 Case 42 Mediastinal Lymphangioma 89
Case 9 Juvenile Aggressive Ossifying Fibroma 19 Case 43 Pulmonary Hydatidosis 91
Case 10 Epignathus 21 Case 44 Arteriovenous Pulmonary Fistula 93
Case 11 Craniopharyngioma 23
Case 12 Cervical Teratoma 25
Case 13 Thyroglossal Duct Cyst 27 3 Abdomen 95
Case 45 Intestinal Malrotation 97
Case 46 Meconium Ileus 99
2 Thorax 31 Case 47 Intestinal Duplication Cyst 101
Case 14 Congenital Cystic Adenomatoid Case 48 Intestinal Atresia 103
Malformation 33 Case 49 Pyloric Atresia 105
Case 15 Pectus Excavatum 35 Case 50 Diaphragmatic Hernia 107
Case 16 Cystic Congenital Adenomatoid Case 51 Crohn’s Disease 109
Malformation 37 Case 52 Hirschsprung’s Disease 111
Case 17 Pulmonary Sequestration 39 Case 53 Intestinal Malrotation 113
Case 18 Lobar Emphysema 41 Case 54 Hirschsprung’s Disease 115
Case 19 Tracheoesophageal Fistula 43 Case 55 Retrosternal Hernia 117
Case 20 Thoracoabdominal Cystic Case 56 Mesenteric Multicystic
Lymphangioma 45 Lymphangioma 119
Case 21 Esophageal Duplication 47 Case 57 Gastric Trichobezoar 121
Case 22 Congenital Lobar Emphysema 49 Case 58 Intestinal Duplication 123
Case 23 Esophageal Caustic Strictures 51 Case 59 Intestinal Obstruction, Meckel’s
Case 24 Esophageal Duplication 53 Diverticulum 125
Case 25 Esophageal Duplication 55 Case 60 Intra-abdominal Pulmonary
Case 26 Mediastinal Teratoma 57 Sequestration 127
Case 27 Pneumothorax 59 Case 61 Omphalocele 129
Case 28 Pneumomediastinum 61 Case 62 Esophageal Achalasia 131
Case 29 Esophageal Duplication 63 Case 63 Esophageal Foreign Body 133
Case 30 Thoracic Trauma, Lung Rupture 65 Case 64 Hirschsprung’s Disease 135
Case 31 Bronchogenic Cyst 67 Case 65 Currarino Syndrome 137
Case 32 H-Type Tracheoesophageal Fistula 69 Case 66 Esophageal Achalasia 139
532 Index

Case 67 Intestinal Obstruction, Aortomesenteric Case 116 Hiatal Hernia 241


Compression 141 Case 117 Duodenal Stenosis 243
Case 68 Intestinal Malrotation 143 Case 118 Gallbladder Agenesis 245
Case 69 Cholelithiasis 145 Case 119 Morgagni–Larrey Hernia 247
Case 70 Acute Pancreatitis 147 Case 120 Mesenteric Lymphangioma 249
Case 71 Biliary Atresia 149 Case 121 Congenital Diaphragmatic Hernia 251
Case 72 Gastrointestinal Duplication Cyst 151 Case 122 Gastroesophageal Reflux 253
Case 73 Jejunal Obstruction 153
Case 74 Hypertrophic Pyloric Stenosis 155
Case 75 Necrotizing Enterocolitis 157 4 Genitourinary Disorders 255
Case 76 Distal Small Bowel Obstruction, Small Case 123 Retrocaval Ureter 257
Left Colon Syndrome 159 Case 124 Vesicoureteral Reflux 259
Case 77 Gastroesophageal Reflux Disease 161 Case 125 Ureteropelvic Junction Obstruction 261
Case 78 Achalasia 163 Case 126 Ureteropelvic Junction Obstruction 263
Case 79 Gastroesophageal Reflux Recurrence 165 Case 127 Hematocolpos 265
Case 80 Gastroesophageal Reflux, Postoperative Case 128 Müllerian Duplication 267
Normal Control 167 Case 129 Ureterocele 269
Case 81 Pseudomembranous Colitis 169 Case 130 Hydronephrosis 271
Case 82 Low Intestinal Obstruction Syndrome 171 Case 131 Vesicoureteral Reflux 273
Case 83 Hirschsprung’s Disease 173 Case 132 Hydronephrosis 275
Case 84 Dolichocolon 175 Case 133 Hydronephrosis 277
Case 85 Anorectal Malformation in Currarino Case 134 Varicocele 279
Syndrome 177 Case 135 Ovarian Cyst 281
Case 86 Anorectal Malformation 179 Case 136 Ectopic Kidney 283
Case 87 Diaphragmatic Eventration 181 Case 137 Horseshoe Kidney 285
Case 88 Hypertrophic Pyloric Stenosis 183 Case 138 Neuropathic Bladder 287
Case 89 Pyloric Atresia 185 Case 139 Multicystic Kidney 289
Case 90 Biliary Atresia 187 Case 140 Vesicoureteral Reflux 291
Case 91 Bochdalek Diaphragmatic Hernia 189 Case 141 Ovarian Cyst 293
Case 92 Roviralta’s Syndrome 191 Case 142 Ureteropelvic Junction Obstruction 295
Case 93 Diaphragmatic Eventration 193 Case 143 Ureteropelvic Junction Obstruction,
Case 94 Jejunojejunal Intussusception 195 Megaureter 297
Case 95 Ventriculoperitoneal Shunt Case 144 Ovarian Cyst 299
Complications 197 Case 145 Ureteropelvic Junction Obstruction 301
Case 96 Biliary Atresia 199 Case 146 Vesicorenal Reflux 303
Case 97 Choledochal Cyst 201 Case 147 Acute Pyelonephritis 307
Case 98 Biliary Lithiasis 205 Case 148 Vesicoureteral Reflux 309
Case 99 Intestinal Telangiectasia 207 Case 149 Duplex System 311
Case 100 Abdominal Cystic lymphangioma 209 Case 150 Crossed Renal Ectopia 313
Case 101 Jejunal Atresia 211 Case 151 Enlarged Neonatal Bladder 315
Case 102 Hemorrhagic Pancreatitis 213
Case 103 Appendiceal Abscess 215
Case 104 Brachyesophagus and Microgastria 217 5 Cardiovascular Disorders 319
Case 105 Crohn’s Disease 219 Case 152 Humeral Artery Pseudoaneurysm 323
Case 106 Paraesophageal Hiatal Hernia 221 Case 153 Coarctation of the Aorta 325
Case 107 Ileitis Terminalis (Yersiniosis) 223 Case 154 Vascular Ring 327
Case 108 Gastric Duplication 225 Case 155 Patent Ductus Arteriosus 329
Case 109 Meconium Ileus 227 Case 156 Tetralogy of Fallot 331
Case 110 H-Type Tracheoesophageal Fistula 229 Case 157 Double Aortic Arch 333
Case 111 Juvenile Polyp of the Sigmoid 231 Case 158 Intracranial Aneurysms 335
Case 112 Pyloric Hypertrophy 233 Case 159 Intracranial Aneurysms 337
Case 113 Intussusception 235 Case 160 Moyamoya Angiopathy 339
Case 114 Crohn’s Disease 237 Case 161 Moyamoya Syndrome 341
Case 115 Right Colon Atresia 239 Case 162 Portal Hypertension 343
Index 533

Case 163 Portal Cavernoma 345 Case 206 Appendicitis 439


Case 164 Hemopericardium 349 Case 207 Ileocolic Intussusception 441
Case 165 Thrombosis of the Splenic Vein 351 Case 208 Jejunojejunal Intussusception 443
Case 166 Transposition of the Great Arteries 353 Case 209 Meconium Peritonitis 445
Case 167 Cardiac Rhabdomyoma 355 Case 210 Necrotizing Enterocolitis 447
Case 211 Malrotation 449
Case 212 Pneumoperitoneum After Bowel
6 Musculoskeletal Disorder 357 Perforation 451
Case 168 Developmental Dysplasia Case 213 Acute Scrotum in Septic
of the Right Hip 359 Salmonellosis 453
Case 169 Mycotic Septic Arthritis Case 214 Intussusception 455
of the Left Hip and Right Knee 361 Case 215 Ingestion of Various Foreign Bodies 457
Case 170 Slipped Capital Femoral Epiphysis 363 Case 216 Necrotizing Enterocolitis 459
Case 171 Chronic Slipped Capital Femoral Case 217 Duodenal Obstruction 461
Epiphysis of the Left Hip 365 Case 218 Meconium Peritonitis 463
Case 172 Pathological Fracture of the Femoral Case 219 Omphalocele 465
Neck 367 Case 220 Necrotizing Enterocolitis 467
Case 173 Humeral Fracture 369 Case 221 Ileocecal Intussusception 469
Case 174 Multiple Exostosis Disease 371
Case 175 Congenital Diaphragmatic Hernia 373
Case 176 Skull Fracture 375 8 Tumors 471
Case 177 Birth Trauma Clavicle 377 Case 222 Juvenile Ovarian Granulosa
Case 178 Tibial Fracture 379 Cell Tumor 473
Case 179 Radial Fracture 381 Case 223 Lymphoma 475
Case 224 Ovarian Teratoma 477
Case 225 Hepatoblastoma 479
7 Emergency and Trauma 383 Case 226 Neuroblastoma 481
Case 180 Abdominal Trauma, Spleen 385 Case 227 Wilms’ Tumor 483
Case 181 Abdominal Trauma, Liver 387 Case 228 Testicular Tumor 485
Case 182 Pancreatic Pseudocyst 389 Case 229 Ganglioglioma 487
Case 183 Acute Appendicitis 391 Case 230 Medulloblastoma 489
Case 184 Appendicular Peritonitis 393 Case 231 Choroid Plexus Papilloma 491
Case 185 Liver Trauma 395 Case 232 Pilocytic Astrocytomas 493
Case 186 Pancreatic Injury 397 Case 233 Dysembryoplastic Neuroepithelial
Case 187 Acute Appendicitis 399 Tumors 495
Case 188 Omental Torsion with Necrosis 401 Case 234 Medulloblastoma of the Fourth
Case 189 Ovarian Torsion Due to a Teratoma 403 Ventricle 497
Case 190 Wandering Spleen 405 Case 235 Brainstem Pilocytic Astrocytoma 499
Case 191 Necrotizing Enterocolitis 407 Case 236 Anaplastic Astrocytoma 501
Case 192 Testicular Trauma 411 Case 237 Pilocytic Astrocytoma
Case 193 Renal Trauma 413 of the Left Optic Tract 503
Case 194 Renal Trauma in Kidney Disease 415 Case 238 Intracranial Meningioma 505
Case 195 Liver Trauma 417 Case 239 Thoracic Neuroblastoma 507
Case 196 Pancreatic Trauma 419 Case 240 Pleuropulmonary Blastoma 509
Case 197 Splenic Trauma, Splenectomy 421 Case 241 Liver Tumor 511
Case 198 Splenic Trauma, Conservative Case 242 Neuroblastoma 513
Treatment 423 Case 243 Rhabdomyosarcoma 515
Case 199 Midgut Volvulus 425 Case 244 Nephroblastoma 517
Case 200 Iatrogenic Esophageal Perforation 427 Case 245 Ewing’s Sarcoma 519
Case 201 Iatrogenic Injury to the Biliary Tree 429 Case 246 Choroid Plexus Carcinoma 521
Case 202 Duodenal Obstruction 431 Case 247 Hepatoblastoma 523
Case 203 Neonatal Midgut Volvulus 433 Case 248 Yolk Sac Tumor 527
Case 204 Traumatic Diaphragmatic Hernia 435 Case 249 Sacrococcygeal Teratoma 529
Case 205 Gangrenous Appendicitis 437
Subject Index

A crossed renal ectopia (Case 150) 313


abdominal Currarino syndrome (Case 65) 137
– cystic lymphangioma (Case 100) 209 cystic congenital adenomatoid malformation
– trauma (Case 16) 37
– liver (Case 181) 387
– spleen (Case 180) 385 D
achalasia (Case 78) 163 developmental dysplasia of the right hip
acute (Case 168) 359
– appendicitis (Cases 183, 187) 391, 399 diaphragmatic
– pancreatitis (Case 70) 147 – abscess (Case 36) 77
– pyelonephritis (Case 147) 307 – eventration (Cases 87, 93) 181, 193
– scrotum in septic salmonellosis (Case 213) 453 – hernia (Case 50) 107
anaplastic astrocytoma (Case 236) 501 distal small bowel obstruction, small left colon
anorectal malformation (Case 86) 179 syndrome (Case 76) 159
– in Currarino syndrome (Case 85) 177 dolichocolon (Case 84) 175
appendiceal abscess (Case 103) 215 double aortic arch (Case 157) 333
appendicitis (Case 206) 439 duodenal
appendicular peritonitis (Case 184) 393 – obstruction (Cases 202, 217) 431, 461
arteriovenous pulmonary fistula (Case 44) 93 – stenosis (Case 117) 243
duplex system (Case 149) 311
B dysembryoplastic neuroepithelial tumors
biliary (Case 233) 495
– atresia (Cases 71, 90, 96) 149, 187, 199
– lithiasis (Case 98) 205 E
birth trauma clavicle (Case 177) 377 ectopic kidney (Case 136) 283
Bochdalek diaphragmatic hernia (Case 91) 189 enlarged neonatal bladder (Case 151) 315
brachyesophagus and microgastria (Case 104) 217 epignathus (Case 10) 21
brainstem pilocytic astrocytoma (Case 235) 499 esophageal
bronchogenic cyst (Cases 31, 41) 67, 87 – achalasia (Cases 62, 66) 131, 139
– caustic strictures (Case 23) 51
C – duplication (Cases 21, 24, 25, 29) 47, 53, 55, 63
cardiac rhabdomyoma (Case 167) 355 – foreign body (Case 63) 133
cervical teratoma (Case 4, 12) 9, 25 Ewing’s sarcoma (Case 245) 519
choledochal cyst (Case 97) 201
cholelithiasis (Case 69) 145 F
choroid plexus fibrous dysplasia of the mandible (Case 7) 15
– carcinoma (Case 246) 521
– papilloma (Case 231) 491 G
chronic slipped capital femoral epiphysis of the left hip gallbladder agenesis (Case 118) 245
(Case 171) 365 ganglioglioma (Case 229) 487
coarctation of the aorta (Case 153) 325 gangrenous appendicitis (Case 205) 437
congenital gastric
– cystic adenomatoid malformation (Case 14) 33 – duplication (Case 108) 225
– diaphragmatic hernia (Cases 121, 175) 251, 373 – trichobezoar (Case 57) 121
– high airway obstruction syndrome (Case 34) 73 gastroesophageal reflux (Case 122) 253
– lobar emphysema (Case 22) 49 – disease (Case 77) 161
– venous malformation of the head (Case 1) 3 – postoperative normal control (Case 80) 167
craniopharyngioma (Case 11) 23 – recurrence (Case 79) 165
Crohn’s disease (Cases 51, 105, 114) 109, 219, 237 gastrointestinal duplication cyst (Case 72) 151
536 Subject Index

H L
hemangioma of the head and neck (Case 3) 7 lipomatosis of the parotid (Case 5) 11
hematocolpos (Case 127) 265 liver
hemimandibular hyperplasia (Case 8) 17 – trauma (Cases 185, 195) 395, 417
hemopericardium (Case 164) 349 – tumor (Case 241) 511
hemorrhagic pancreatitis (Case 102) 213 lobar emphysema (Case 18) 41
hepatoblastoma (Cases 225, 247) 479, 523 low intestinal obstruction syndrome (Case 82) 171
hiatal hernia (Case 116) 241 lymphoma (Case 223) 475
Hirschsprung’s disease (Cases 52, 54, 64, 83) 111, 115,
135, 173 M
horseshoe kidney (Case 137) 285 macrocystic lymphatic malformation of the neck
H-type tracheoesophageal fistula (Cases 32, 110) 69, (Case 2) 5
229 malrotation (Case 211) 449
humeral meconium
– artery pseudoaneurysm (Case 152) 323 – ileus (Cases 46, 109) 99, 227
– fracture (Case 173) 369 – peritonitis (Cases 209, 218) 445, 463
hydronephrosis (Cases 130, 132, 133) 271, 275, 277 mediastinal
hypertrophic pyloric stenosis (Cases 74, 88) 155, 183 – lymphangioma (Case 42) 89
– mass (Case 37) 79
I – neuroblastoma (Case 40) 85
iatrogenic – teratoma (Case 26) 57
– esophageal perforation (Case 200) 427 medulloblastoma (Case 230) 489
– injury to the biliary tree (Case 201) 429 – of the fourth ventricle (Case 234) 497
ileitis terminalis (yersiniosis) (Case 107) 223 mesenteric lymphangioma (Case 120) 249
ileocecal intussusception (Case 221) 469 – multicystic (Case 56) 119
ileocolic intussusception (Case 207) 441 midgut volvulus (Case 199) 425
ingestion of various foreign bodies (Case 215) 457 Morgagni–Larrey hernia (Case 119) 247
intestinal Moyamoya
– atresia (Case 48) 103 – angiopathy (Case 160) 339
– duplication (Case 58) 123 – syndrome (Case 161) 341
– cyst (Case 47) 101 Müllerian duplication (Case 128) 267
– malrotation (Cases 45, 53, 68) 97, 113, 143 multicystic kidney (Case 139) 289
– obstruction multiple exostosis disease (Case 174) 371
– aortomesenteric compression (Case 67) 141 mycotic septic arthritis of the left hip and right knee
– Meckel’s diverticulum (Case 59) 125 (Case 169) 361
– telangiectasia (Case 99) 207
intra-abdominal pulmonary sequestration N
(Case 60) 127 necrotizing enterocolitis (Cases 75, 191, 210, 216,
intracranial 220) 157, 407, 447, 459, 467
– aneurysms (Cases 158, 159) 335, 337 neonatal
– meningioma (Case 238) 505 – midgut volvulus (Case 203) 433
intussusception (Cases 113, 214) 235, 455 – pneumothorax (Case 35) 75
nephroblastoma (Case 244) 517
J neuroblastoma (Cases 226, 242) 481, 513
jejunal neuropathic bladder (Case 138) 287
– atresia (Case 101) 211
– obstruction (Case 73) 153 O
jejunojejunal intussusception (Cases 94, 208) 95, 443 omental torsion with necrosis (Case 188) 401
juvenile omphalocele (Cases 61, 219) 129, 465
– aggressive ossifying fibroma (Case 9) 19 ovarian
– ovarian granulosa cell tumor (Case 222) 473 – cyst (Cases 135, 141, 144) 281, 293, 299
– polyp of the sigmoid (Case 111) 231 – teratoma (Case 224) 477
– torsion due to a teratoma (Case 189) 403
Subject Index 537

P slipped capital femoral epiphysis (Case 170) 363


pancreatic splenic trauma
– injury (Case 186) 397 – conservative treatment (Case 198) 423
– pseudocyst (Case 182) 389 – splenectomy (Case 197) 421
– trauma (Case 196) 419
paraesophageal hiatal hernia (Case 106) 221 T
parotid acinic neoplasm (Case 6) 13 testicular
patent ductus arteriosus (Case 155) 329 – trauma (Case 192) 411
pathological fracture of the femoral neck – tumor (Case 228) 485
(Case 172) 367 tetralogy of Fallot (Case 156) 331
pectus excavatum (Case 15) 35 thoracic
pilocytic astrocytoma (Case 232) 493 – ganglioneuroblastoma (Case 39) 83
– of the left optic tract (Case 237) 503 – neuroblastoma (Case 239) 507
pleuropulmonary blastoma (Case 240) 509 – trauma, lung rupture (Case 30) 65
pneumomediastinum (Case 28) 61 thoracoabdominal cystic lymphangioma (Case 20) 45
pneumoperitoneum after bowel perforation thrombosis of the splenic vein (Case 165) 351
(Case 212) 451 thyroglossal duct cyst (Case 13) 27
pneumothorax (Case 27) 59 tibial fracture (Case 178) 379
portal tracheoesophageal fistula (Case 19) 43
– cavernoma (Case 163) 345 transposition of the great arteries (Case 166) 353
– hypertension (Case 162) 343 traumatic diaphragmatic hernia (Case 204) 435
pseudomembranous colitis (Case 81) 169
pulmonary U
– hydatidosis (Case 43) 91 ureterocele (Case 129) 269
– sequestration (Cases 17, 33) 39, 71 ureteropelvic junction obstruction
pyloric (Cases 125, 126, 142, 145) 261, 263, 295, 301
– atresia (Cases 49, 89) 105, 185 – megaureter (Case 143) 297
– hypertrophy (Case 112) 233
pyopneumothorax (Case 38) 81 V
varicocele (Case 134) 279
R vascular ring (Case 154) 327
radial fracture (Case 179) 381 ventriculoperitoneal shunt complications (Case
renal trauma (Case 193) 413 95) 197
– in kidney disease (Case 194) 415 vesicorenal reflux (Case 146) 303
retrocaval ureter (Case 123) 257 vesicoureteral reflux (Cases 124, 131, 140, 148) 259,
retrosternal hernia (Case 55) 117 273, 291, 309
rhabdomyosarcoma (Case 243) 515
right colon atresia (Case 115) 239 W
Roviralta’s syndrome (Case 92) 191 wandering spleen (Case 190) 405
Wilms’ tumor (Case 227) 483
S
sacrococcygeal teratoma (Case 249) 529 Y
skull fracture (Case 176) 375 yolk sac tumor (Case 248) 527

You might also like